You are on page 1of 169

Histria Geral 04

Histria do Brasil 12
Literatura 22
Espanhol 32
Comp. Textual 37
Portugus 44
Geografia 51
Qumica 67
Fsica 81
Matemtica 108
Biologia - 154

1) UFPEL VERO 2007 - ROMA) Os animais da Itlia possuem cada um sua toca, De acordo com o mapa, os povos que viviam nas regies identificadas pelas letras
seu abrigo, seu refgio. No entanto, os homens que combatem e morrem pela Itlia A, B e C, so, respectivamente,
esto merc do ar e da luz e nada mais: sem lar, sem casa, erram com suas
mulheres e crianas. Os generais mentem aos soldados quando, na hora do combate,
os exortam a defender contra o inimigo suas tumbas e seus lugares de culto, pois
nenhum destes romanos possui nem altar de famlia, nem sepultura de ancestral.
para o luxo e enriquecimento de outrem que combatem e morrem tais pretensos
senhores do mundo, que no possuem sequer um torro de terra.
PLUTARCO DE QUERONIA, (50-125). In: PINSKY, Jaime. 100 textos de Histria
Antiga. So Paulo: Contexto, 2003.
O documento est associado reforma agrria promovida pela(s)
a) Revolta de Esprtaco.
b) Lei das Doze Tbuas.
c) Lei Canulia.
d) Guerras Pnicas.
e) Leis dos Irmos Graco.
2) UFPEL VERO 2007 IDADE MDIA) Os clrigos devem por todos orar
os cavaleiros sem demora
devem defender e honrar
e os camponeses, sofrer
cavaleiros e clero sem falha
vivem de quem trabalha
tm grande canseira e dor
pagam primcias, corvias, oraes ou talha
e cem coisas costumeiras
e quanto mais pobre viver
mais mrito ter
das faltas que cometeu
se paga a todos o que deve
se cumpre com lealdade a sua f
se suporta paciente o que lhe cabe:
angstias e sofrimentos.
In: COTRIM, Gilberto. Histria global: Brasil e Geral. 6 ed. So Paulo: Saraiva, 2002.
ESTEVO DE FOURGES. In: COTRIM, Gilberto. Histria global: Brasil e Geral. 6 ed. a) astecas, incas e maias.
So Paulo: Saraiva, 2002.
b) incas, maias e astecas.
O poema est diretamente relacionado
c) astecas, maias e incas.
a) Revoluo Francesa, enfatizando as obrigaes servis, como a corvia que era d) maias, astecas e incas.
a entrega da primeira colheita ao senhor.
e) maias, incas e astecas.
b) estratificao social, no feudalismo europeu, justificada pela Igreja, e composta
pelo clero, pela nobreza e pelo povo.
5) UFPEL VERO 2007 REVOLUO INDUSTRIAL)
c) ao final da Idade Mdia, durante a expanso colonial europia na Amrica, com o Um fato saliente chamou a ateno de Adam Smith, ao observar o panorama da
apoio da Igreja.
Inglaterra: o tremendo aumento da produtividade resultante da diviso minuciosa e
d) ideologia burguesa, nas Cruzadas, quando os cavaleiros defenderam os valores da especializao de trabalho. Numa fbrica de alfinetes, um homem puxa o fio,
cristos ocidentais contra os muulmanos.
outro o acerta, um terceiro o corta, um quarto faz-lhe a ponta, um quinto prepara a

extremidade para receber a cabea, cujo preparo exige duas ou trs operaes
diferentes: coloc-la uma ocupao peculiar; prate-la outro trabalho.
Arrumar os alfinetes no papel chega a ser uma tarefa especial; vi uma pequena
e) ao perodo medieval, por referir a explorao dos camponeses atravs de trabalho fbrica desse gnero, com apenas dez empregados, e onde conseqentemente
escravizado, bem como pela talha que era o pagamento pelo uso do moinho.
alguns executavam duas ou trs dessas operaes diferentes. E embora fossem
muito pobres, e portanto mal acomodados com a maquinaria necessria, podiam
3) UFPEL VERO 2007 IDADE MDIA)
fazer entre si 48.000 alfinetes num dia, mas se tivessem trabalhado isolada e
Texto 1:
independentemente, certamente cada um no poderia fazer nem vinte, talvez nem
Durante a Idade Mdia europia, as estratgias matrimoniais organizavam e um alfinete por dia.
sustentavam as relaes sociais. O casamento era antes de tudo um pacto entre FARIA, Ricardo de Moura et all. Histria. Vol. 1. Belo Horizonte: L, 1993. [adapt.].
famlias. Nesse ato, a mulher era ao mesmo tempo doada e recebida, como um ser O documento sobre a Revoluo Industrial, na Inglaterra,
passivo. Sua principal virtude, dentro e fora do casamento, deveria ser a obedincia, a) relaciona a diviso de trabalho com a alta produtividade, situao bem diferente
a submisso. Solteira, era identificada sempre como filia de, soror de. Casada, da produo artesanal caracterstica da Idade Mdia.
passava a ser personificada como uxor de. Filha, irm, esposa: os homens deviam b) enfatiza o trabalho em srie e as condies do trabalhador nas fbricas,
ser sua referncia.
reforando a importncia das leis trabalhistas, no incio da Idade Moderna.
MACEDO, Jos Rivair. A mulher na Idade Mdia. 5ed. So Paulo: Contexto, 2002. c) demonstra que a produtividade est diretamente relacionada ao nmero de
[adapt.].
empregados da fbrica, ao contrrio das Corporaes de Ofcio, em que a produo
Texto 2 :
artesanal dependia do mestre.
O que as revistas femininas aconselhavam s leitoras :
d) destaca a importncia da especializao do trabalho para o aumento da
[...] - A mulher deve fazer o marido descansar nas horas vagas, nada de incomod-lo produtividade, situao semelhante que ocorria nas Corporaes de Ofcio, de que
com servios domsticos. (Jornal das Moas, 1959).
participavam aprendizes, oficiais e mestre.
- Se o seu marido fuma, no arrume brigas pelo simples fato de cair cinzas no tapete. e) evidencia as idias fisiocrticas e mercantilistas, ao realar a diviso do trabalho,
Tenha cinzeiros espalhados por toda a casa. (Jornal das Moas, 1957).
caractersticas marcantes da Revoluo Comercial.
- O lugar de mulher no lar, o trabalho fora de casa masculiniza. (Querida, 1955).
[...].
6) UFPEL VERO 2007 REVOLUO FRANCESA) O Comit de Salvao Pblica,
In: COTRIM, Gilberto. Histria e conscincia do Brasil, 7 ed. So Paulo: Saraiva, quando chefiado por Robespierre, ordenou uma mobilizao em massa, adotou
1999.
rigorosas medidas econmicas, controlando a produo, racionando os vveres,
Com base na leitura comparativa dos textos, constata-se que
fixando os preos mximos dos gneros de primeira necessidade, bem como o nvel
a) o patriarcalismo medieval europeu influenciou, durante muito tempo, a cultura mximo dos salrios, e instituiu ainda tribunais revolucionrios para eliminar todos os
brasileira, tendo em comum a valorizao da submisso feminina.
suspeitos de serem adversrios da Revoluo. Essas medidas do Comit instauraram
b) a ideologia patriarcal da Europa renascentista foi absorvida, sem alteraes, pela na Frana o perodo do Terror.
sociedade brasileira na Ditadura Militar.
HOLLANDA, Srgio Buarque de. Histria da Civilizao. So Paulo: Companhia Editora
c) o Direito Romano, desde a Antigidade, submeteu a mulher uma inferioridade Nacional, 1975. [adapt.]
legal, que vigora na atual legislao brasileira.
Esses acontecimentos correspondem fase da Revoluo Francesa denominada
d) as mulheres no possuam direitos civis, tanto na Europa, na Idade Moderna, a) Assemblia Nacional Constituinte (1789 a 1791).
quanto no Brasil, na segunda metade do sculo XX.
b) Repblica, na Conveno Nacional (1792 a 1795).
e) o papel feminino est subordinado, nas duas conjunturas histricas, coisificando a c) Monarquia Constitucional (1791 a 1792).
mulher, contudo no existe a discriminao de gnero na sociedade brasileira atual. d) Diretrio (1795 a 1799).
4) UFPEL VERO 2007 POVOS PR-COLOMBIANOS)

e) Consulado (1799 a 1802).

7) UFPEL VERO 2007 DESCOLONIZAO LATINO-AMERICANA)

d) Patrice Lumumba, lder nacionalista do Congo Belga.


e) Abdel Nasser, na luta pela libertao do Egito.

10) UFPEL VERO 2007 DESCOLONIZAO AFRO-ASITICA) Durante a Guerra


Fria, a estratgia da Casa Branca tinha como horizonte mudar a aparncia da
dominao colonial para na verdade no mudar nada: as novas naes manteriam o
vnculo de dependncia econmica e sofreriam um certo grau de ingerncia poltica
por parte das antigas metrpoles. Eventualmente, as metrpoles apoiariam a
formao de ditaduras que lhes fossem fiis. Assim, nos anos 50 e 60, a frica
conheceu intensamente um processo hoje chamado descolonizao. Apenas no ano
de 1960, dezessete colnias da Frana e da Inglaterra conquistaram o status de
naes autnomas.
ARBEX JR., Jos. Guerra Fria: terror de Estado, poltica e cultura. So Paulo:
Moderna, 1997 [adapt.].
Contrariando essa estratgia dos Estados Unidos da Amrica, atravs da qual os
processos colonizatrios haviam sido reforados, ocorreram na segunda metade do
sculo XX, na frica, movimentos com caractersticas revolucionrias e emancipaes
polticas com o apoio da Unio Sovitica. Entre os pases que vivenciaram este ltimo
processo transformador no perodo referido, esto
a) Arglia, Congo e frica do Sul.
b) Sudo, Etipia e Marrocos.
c) Sudo, Arglia e Gabo.
CAMPOS, Flvio de e MIRANDA, Renan Garcia. Oficina de Histria. So Paulo: d) Angola, Moambique e Guin Bissau.
Moderna, 2000. [adapt.]
e) Costa do Marfim, Uganda e Senegal.
As emancipaes polticas do Mxico, da Gr-Colmbia, do Peru, do Chile, do
Paraguai e da Argentina, expressas no mapa, esto relacionadas
11) UCPEL 2007 2. GUERRA MUNDIAL) Bush chamou lderes da Coria do Sul,
a) aos interesses ingleses em oposio poltica adotada por Napoleo sobre a Rssia, Japo e China para reafirmarem o compromisso com uma pennsula coreana
Pennsula Ibrica, desde o Bloqueio Continental.
livre de armas nucleares. Na ONU, os EUA propuseram sanes duras, como
b) s determinaes do Congresso de Viena, que restauraram as fronteiras europias proibio do comrcio de artigos militares e de luxo, inspeo de mercadorias
na Amrica, vigorantes antes da Revoluo Francesa.
importadas ou exportadas do pas e congelamento de bens ligados s armas. O
c) s unificaes alem e italiana, e s idias nacionalistas que inspiraram os embaixador americano, John Bolton, ameaou: .A Coria do Norte possui uma
movimentos de independncia na Amrica.
histria de sucesso em intimidar outros pases, mas no ter sucesso conosco.,
d) poltica da Amrica para os americanos, que serviu de fundamento para a acrescentou..
independncia dos Estados Unidos.
Dirio Popular, 11 de outubro de 2006, p. 24.
e) atuao da Santa Aliana na restaurao das possesses na Amrica Latina e O texto acima faz referncia ao suposto teste nuclear realizado pela Coria do Norte.
manuteno do equilbrio poltico entre os pases europeus.
No passado, o Japo foi alvo de duas bombas atmicas. Sobre este assunto todas as
opes esto corretas, com exceo daquela que afirma que
a) a derrota do imperialismo militarista nipnico implicou, tambm, a perda de
8) UFPEL VERO 2007 REVOLUO CHINESA)
mercados consumidores dos seus produtos e fornecedores de matrias-primas.
b) passados alguns anos aps o ataque das duas bombas atmicas, o Japo
vivenciou uma extraordinria e rpida recuperao econmica, fato que ficou
conhecido como o .milagre japons.
c) as bombas atmicas lanadas pelos Estados Unidos em Hiroshima e Nagasaki
levaram os nipnicos a revidarem por meio do ataque japons base americana de
Pearl Harbor.
d) o lanamento das duas bombas atmicas levou o governo japons a solicitar o
armistcio dias depois das exploses ocorridas em Hiroshima e Nagasaki.
e) as bombas de Hiroshima e Nagasaki lanadas pelos Estados Unidos, em agosto de
1945, causaram a curto prazo milhares de mortes e, a longo prazo, a incidncia de
muitas doenas causadas pela radioatividade.
12) UCPEL 2007 REVOLUO INDUSTRIAL)

GARZA, Hedda. Os grandes lderes do sculo XX Mao Ts-Tung. So Paulo: Nova


Cultural,1990. [adapt.].
Pelo documento percebe-se
a) os principais pontos de resistncia das tropas chinesas na Guerra Sino-Japonesa,
no incio do sculo XX.
b) a trajetria dos comunistas com a adoo de tticas na guerrilha camponesa, no Com base na figura acima e nos seus conhecimentos sobre a temtica Revoluo
processo da Revoluo Chinesa, que culmina em 1949.
Industrial, correto afirmar que
c) o caminho da ao guerrilheira contra a forte explorao econmica ocidental, a) os artesos foram os principais articuladores da Revoluo Industrial, afinal,
principalmente de ingleses, franceses e alemes, durante a Guerra dos Boxers.
dispunham de capital suficiente para financiar as fbricas, comprar matria-prima e
d) a ao das tropas chinesas, apoiadas pelos norte-americanos, contra a invaso contratar novos empregados para os seus empreendimentos industriais.
japonesa, durante a 2 Guerra Mundial.
b) devido ao seu forte desenvolvimento econmico, resoluo de problemas
e) o movimento dos comunistas chineses contra os nacionalistas, durante a internos e obteno de matria-prima em larga escala, a Alemanha foi o primeiro
denominada Guerra do pio.
pas a se industrializar.
c) a Alemanha principiou o processo da Revoluo Industrial em virtude de possuir
9) UFPEL VERO 2007 DESCOLONIZAO AFRO-ASITICA) A primeira coisa, grandes reservas de carvo mineral em seu subsolo e esta, na poca, representava a
portanto, dizer-vos a vs mesmos: No aceitarei mais o papel de escravo. No principal fonte de energia para movimentar as mquinas e as locomotivas a vapor.
obedecerei s ordens como tais, mas desobedecerei quando estiverem em conflito d) o crescimento populacional foi um dos fatores condicionantes Revoluo
com minha conscincia. O assim chamado patro poder surrar-vos e tentar forar- Industrial, mas a lei dos cercamentos de terras, implantada na Europa do sculo
vos a servi-lo. Direis: No, no vos servirei por vosso dinheiro ou sob ameaa. Isso XVIII, prejudicou a expanso industrial na medida em que enclausurou importante
poder implicar sofrimentos. Vossa prontido em sofrer acender a tocha da fonte de mo-de-obra.
liberdade, que no pode jamais ser apagada.
e) a Revoluo Industrial teve incio na Inglaterra, em meados do sculo XVIII, e
PAZZINATO, Alceu L. e SENISE, Maria Helena V. Histria Moderna e Contempornea, caracterizou-se, inicialmente, pelo desenvolvimento da mquina a vapor, dos teares
So Paulo: tica, 2002.
mecnicos, da minerao e da metalurgia do ferro.
O texto caracteriza a poltica de Desobedincia Civil defendida por
a) Mahatma Gandhi, como estratgia para a independncia da ndia.
13 UCPEL INVERNO 2006 ABSOLUTISMO MONRQUICO) O Estado sou eu.
b) Nelson Mandela, no processo de descolonizao e independncia da frica do Sul. Com base nessa famosa frase de Lus XIV, considere as afirmativas a seguir.
c) Agostinho Neto, na luta pela independncia de Angola.

I . O Estado Moderno tinha como atribuies, entre outras, a direo da economia e III. At o sculo VIII, a ordem jurdica dos reinos germnicos baseou-se
a nomeao de ministros.
exclusivamente nos cdigos escritos romanos, aos quais geralmente se submetiam
II . O rei era absolutista porque estava acima das leis e nada acontecia sem a sua tanto os habitantes romanizados quanto os de origem germnica.
autorizao.
IV. As invases de novos povos, no sculo IX, incrementaram o processo de
III . O Estado absolutista apoiava os negcios burgueses porque quanto mais lucro desagregao no Ocidente, acentuando as caractersticas essenciais do mundo
eles obtivessem, mais impostos poderiam pagar.
feudal.
Est(o) correta(s):
Esto corretas somente as alternativas
a) Todas.
a) I e II.
b) Somente a II.
b) I e IV.
c) Somente a III.
c) II e III.
d) Somente a I e a II.
d) I, II e IV.
e) Somente a II e a III.
e) II, III e IV.
14) UCPEL INVERNO 2006RENASCIMENTO CULTURAL)

17) PUCRS 2008 FORMAO DOS ESTADOS NACIONAIS ) O final da Guerra dos
Cem Anos (1337-1453), da Guerra das Duas Rosas (1453-1485) e da Guerra da
Reconquista (sculos XII a XV) desencadeou processos diferenciados de
a) fortalecimento da servido clssica.
b) enfraquecimento das burguesias.
c) centralizao poltica.
d) afirmao do poder temporal da Igreja.
e) declnio do racionalismo humanista.
18) PUCRS 2008 UNIFICAES ITALIANA E ALEM) Em 1885, com o objetivo de
fixar regras para a partilha do continente africano entre as potncias europias, o
chanceler alemo Bismarck convocou a Conferncia de Berlim. Relacionando o fato
com o contexto econmico europeu, correto afirmar que a Alemanha ingressava na
corrida neocolonial para atender as novas necessidades de exportao de _________
por parte do capitalismo _________, bem como para justificar os altos investimentos
estatais no setor _________.
a) mo-de-obra monopolista financeiro
b) capitais liberal financeiro
c) capitais monopolista blico
d) mo-de-obra liberal blico
e) capitais monopolista financeiro

A obra intitulada .Davi., de Miquelngelo, est relacionada


a) ao aparecimento de um novo mtodo de produo de esculturas, que se
19) PUCRS 2008 GUERRA FRIA) No comeo da dcada de 1980, o presidente dos
estruturava na tradio medieval.
b) ao surgimento de uma nova filosofia que tem por base o desenvolvimento das Estados Unidos, Ronald Reagan, colocou em prtica uma poltica econmica
conhecida por reaganomics, a qual buscava ser uma resposta conservadora ao
atividades artsticas promovidas pela escolstica.
c) busca dos valores defendidos pelo renascimento, como o dogmatismo e o contexto de crise e de reordenamento da economia norte-americana e mundial. Essa
poltica, que seria exportada como modelo para as economias capitalistas no
misticismo.
d) ao renascimento cultural, que se caracteriza pela retomada dos valores greco-contexto, apresenta vrias caractersticas, EXCETO
a) o corte nos gastos com servios previdencirios.
romanos como, por exemplo, o antropocentrismo e o racionalismo.
e) defesa do hedonismo, prtica que caracteriza a beleza como uma manifestao b) a diminuio gradativa de impostos.
c) a liberao das importaes.
do esprito divino.
d) a adoo de taxas de juros elevadas.
15) PUCRS 2008 - GRCIA) No sculo V aC., com o final das Guerras Mdicas, e) a criao de empresas estatais.
estabeleceu-se um perodo de hegemonia de Atenas sobre o mundo grego, em
contraposio a Esparta. Entre os fatores condicionantes dessa hegemonia, NO se20) UCS 2007 CONCEITOS HISTRICOS) A Pedra da Rosetta, descoberta por uma
expedio napolenica ao Egito no final do sculo XVIII, uma importantssima fonte
pode apontar
histrica para os estudiosos da Antigidade. Analise, quanto sua veracidade (V) ou
a) o incremento do poderio das foras navais atenienses.
falsidade (F), as proposies abaixo, relacionadas ao conceito de fonte histrica.
b) a formao da Confederao de Delos.
( ) Toda fonte histrica necessariamente escrita, as demais so
c) a permanncia das foras terrestres espartanas no Peloponeso.
consideradas pr-histricas.
d) a instituio, por Pricles, de uma tirania aristocrtica imperialista em Atenas.
( ) O historiador deve priorizar as fontes com notria imparcialidade. Nos tempos
e) a concentrao do comrcio do mar Egeu em Atenas.
16) PUCRS 2008 IDADE MDIA) INSTRUO: Para responder questo 22, atuais, esse o caso dos jornais e das revistas.
considere as seguintes afirmativas sobre o processo de formao do Feudalismo na ( ) Nenhuma fonte histrica deve ser encarada como portadora de verdades
incontestveis. Na qualidade de produo humana, ela deve ser analisada
Europa Ocidental durante a Alta Idade Mdia.
I. Os povos germnicos que se estabeleciam no Ocidente ainda estavam organizados criticamente e comparada com outras fontes.
em comunidades primitivas, entretanto no mais constituam sociedades igualitrias e Assinale a alternativa que preenche corretamente os parnteses, de cima para baixo.
a) V V V
j haviam abandonado o nomadismo, praticando uma agricultura rudimentar.
II. Os processos de ruralizao, de decadncia do comrcio, de fragmentao poltica b) F F V
e de dependncia dos pequenos produtores em relao aos latifundirios foram c) F V V
introduzidos no Ocidente pela invaso germnica, j que no se verificaram no Baixo d) V F F
e) F V F
Imprio romano.

21) UCS 2007 IDADE ANTIGA) Egpcios, mesopotmicos, hebreus, fencios e persas apresentaram grandes realizaes, muitas delas influenciando outros povos e outras

culturas. De certa maneira, muito da cultura ocidental atual conta com seus feitos.
(VICENTINO, Cludio; DORIGO, Gianpaolo. Histria para o ensino mdio: histria geral e do Brasil. So Paulo: Scipione, 2005, p. 45 Adaptado.)

Alternativa
a

Civilizao
Egpcia

Localizao
Nordeste da frica

Governo
Fara: centralismo

Cultura
Religio
medicina,
astronomia,Culto dos mortos, mumificao
engenharia e matemtica

Mesopotmica Oriente Mdio, no atual Iraque

Fencia

Persa

Hebraica

Monarquias despticas teocrticas


Astronomia,
arquitetura,
Politesta, ligada de forma mais direta
escultura,
calendrio
busca de benefcios terrenos
dividindo o ano em 12 meses
e a semana em 7 dias,
Cdigo de Hamurbi

No norte da Palestina, na rea atual do Lbano


Cidades-estado
independentes

Leste da Mesopotmia
Corresponderia, aproximadamente,
territrio de Israel

autnomasAstronomia,
e
matemtica
Politesta,
e
sendo os principais deuses
criao do alfabeto, adotado
Baal e Astartia
mais tarde pelos gregos e
difundido para o Ocidente

Democracia
ao Patriarcas,
atual
juzes e reis

Administrao: satrpias
Monotesmo
Arquitetura
(Templo Monotesmo
de
Jerusalm) e fora da religio

Considerando a Antigidade Oriental, assinale a alternativa INCORRETA.

22) UCS 2007 CONCEITOS HISTRICOS) Escravido e servido so duas


formas de trabalho compulsrio, mas com caractersticas prprias. Associe
essas duas formas de trabalho, listadas na Coluna A, s caractersticas que
as identificam, elencadas na Coluna B.
COLUNA A
1 Escravido
2 Servido
COLUNA B
( ) O trabalhador era uma propriedade de seu patro; no tinha direito a
decidir a sua vida, uma vez que esta pertencia a seu dono.
( ) O trabalhador era juridicamente livre, embora sobre ele pesasse a
obrigatoriedade da prestao de servios e do pagamento de tributos
( ) Pode ser considerada a base de sustentao da vida em Atenas, onde
eram muitos os que assumiam essa forma de trabalho.
( ) Predominou no perodo conhecido como Idade Mdia (sculo V at o
sculo XV), quando o feudalismo dominava como modo de produo, na
Europa.
( ) Um trabalhador assumia esta forma de trabalho, caracterstica da
Antigidade Clssica, por dvida, pelo nascimento ou por ser prisioneiro de
guerra.
Assinale a alternativa que preenche corretamente os parnteses, de cima
para baixo.
a) 1 2 2 1 1
b) 2 1 1 2 2
c) 2 1 1 2 1
d) 1 2 2 1 2
e) 1 2 1 2 1
23) UCS 2007 - MERCANTILISMO) O mercantilismo foi um conjunto de
teorias e prticas de interveno econmica, surgido na Europa a partir do
sculo XV, quando da transio do feudalismo para o capitalismo. Analise,
quanto sua veracidade (V) ou falsidade (F), as proposies abaixo, sobre as
caractersticas do mercantilismo.
( ) Metalismo a prtica econmica caracterizada pelo entesouramento de
moedas, proveniente da idia de que quanto maior fosse a quantidade de
metais acumulados mais rico seria o pas.
( ) A defesa da produo nacional tem como caracterstica o incentivo aos
produtos nacionais, sobretudo aos itens que concorriam no mercado
internacional.
( ) Para que as idias mercantilistas fossem colocadas em prtica, era
imprescindvel a liberdade econmica, da a no interveno do Estado na
economia.
Assinale a alternativa que preenche corretamente os parnteses, de cima
para baixo.
a) V V F
d) V F F
b) V V V
e) F V F
c) F F V
24) UCS 2007SCULO XX) Considere os acontecimentos do sculo XX,
referentes histria mundial e brasileira, listados abaixo.
1 Ecloso da Primeira Guerra Mundial
2 Implantao do Estado Novo
3 Revoluo Russa
4 Semana de Arte Moderna
5 Crise na Bolsa de Nova York
Assinale a alternativa em que esses acontecimentos esto colocados na
ordem cronolgica correta.
a) 1, 2, 3, 4 e 5.
d) 5, 4, 3, 2 e 1.
b) 3, 4, 5, 2 e 1.
e) 2, 1, 4, 5 e 3.
c) 1, 3, 4, 5 e 2.
25) UCS 2007 PRIMEIRA GUERRA MUNDIAL) Ao final da I Guerra Mundial,
as naes vencedoras do conflito assinaram o Tratado de Versalhes, que
imps Alemanha condies de paz extremamente duras, envolvendo
questes territoriais, militares e financeiras. As alternativas abaixo
apresentam as imposies feitas Alemanha pelo tratado, EXCETO a
a) perda de todas as suas colnias ultramarinas e a devoluo da AlsciaLorena Frana.
b) obrigao de reconhecimento da independncia da Polnia e a liberao
de uma faixa de terra, permitindo o acesso desse pas ao mar.
c) obrigao de pagar pesadas indenizaes/reparaes de guerra aos pases
da Trplice Entente.
d) reduo do poder militar alemo, atravs da diminuio de seus efetivos
no exrcito e da desmilitarizao da fronteira franco-germnica.
e) obrigao de sediar, em Berlim, a Liga das Naes, encarregada de
preservar a paz mundial.
26) UFPEL 2007 - EGITO) Observe atentamente as colunas a seguir sobre a
Histria do Egito e as relacione:

1 Coluna
( 1 ) Perodo Pr-Dinstico
( 2 ) Antigo Imprio
( 3 ) Mdio Imprio
( 4 ) Novo Imprio
2 Coluna
( ) expanso territorial com anexao da Etipia, Sria e Fencia.
( ) unificao do Alto e do Baixo Egito efetuada pelo fara Mens.
( ) formao dos nomos.
( ) invaso dos hicsos.
A ordem que relaciona corretamente a segunda coluna, em relao
primeira, a seguinte:
a) 1, 2, 3, 4.
d) 4, 2, 1, 3.
b) 3, 1, 4, 2.
e) 4, 3, 2, 1.
c) 2, 4, 1, 3.
27) UFPEL 2007 IDADE MDIA)

Execuo na fogueira por ordem do Tribunal da Inquisio, Inglaterra


1314.
Arrancada a confisso do ru, os inquisidores proferiam a sentena em uma
sesso pblica denominada sermo geral. As sentenas previam trs tipos
bsicos de penas: confiscao de bens, priso e morte. A maioria dos
condenados morte eram queimados vivos numa grande fogueira. Somente
a alguns permitia-se o estrangulamento antes de serem lanados ao fogo.
COTRIM, Gilberto. 6 ed. So Paulo: Saraiva, 2001.
O texto didtico faz uma anlise das aes do Tribunal da Inquisio, criado
pela
a) Igreja Anglicana, durante a Reforma Religiosa.
b) Religio muulmana, no perodo das Cruzadas.
c) Frana dos Huguenotes, no perodo da Contra-Reforma.
d) Reforma Protestante, liderada por Lutero, no fim da Idade Mdia.
e) Igreja Catlica Romana, durante a Idade Mdia.
28) UFPEL 2007 COLONIZAO DA AMRICA LATINA)

Astecas vitimados por varola. Desenho do sculo XVI.


Naquele tempo, no havia doenas, nem febres, nem doenas dos ossos
ou de cabea (...). Naquele tempo, tudo estava em ordem. Os estrangeiros
mudaram tudo quando chegaram. De fato, por mais saudosismo que possa
expressar esse lamento, parece mesmo que as doenas do Velho Mundo
foram mais freqentemente mortais nas Amricas do que na Europa. O

10

missionrio alemo chegou inclusive a escrever no finalzinho do sculo XVIII


que os ndios morrem to facilmente que s a viso ou o cheiro de um
espanhol os fazem passar deste para outro mundo. Umas quinze epidemias
dizimaram a populao do Mxico e do Peru.
FERRO, Marc. Histria das colonizaes das conquistas s independncias
sculos XIII a XIX. So Paulo: Cia. das Letras, 1996.
Os documentos denunciam as doenas provocadas pelos agentes do
a) Colonialismo espanhol que dizimaram populaes nativas na Amrica, na
Idade Moderna.
b) Colonialismo portugus em suas possesses, entre os sculos XVI e XVIII.
c) Imperialismo ibrico e dos Pases Baixos exterminando as populaes
incas, maias e astecas, na Idade Contempornea.
d) Mercantilismo europeu nas colnias anglosaxnicas, desde o final da Idade
Mdia.
e) Colonialismo lusitano no Mxico e no Peru, a partir do sculo XVI.

31) FURG 2007 - ROMA) "[Estes] so homens impiedosamente expostos pelo


destino a toda sorte de ultrajes. Mas so, em ltima anlise, homens de uma
segunda categoria, a quem poderamos at conceder as vantagens da nossa
liberdade. Mas que o poderei dar a essa guerra (...) contra ns? Confesso
que no sei. Porque [nela] vemos [homens] combatendo e gladiadores
comandando. Os primeiros so de origem bem humilde. Os segundos esto
condenados pior de todas as condies sociais".
O texto acima, do historiador Florus, trata de uma das mais famosas revoltas
sociais ocorridas em Roma durante a Repblica. Tal luta estendeu-se entre o
perodo de 73 e 71 a.C. e ficou conhecida como:
a) Revolta dos cristos e ateus, comandada por Anbal.
b) Revolta de escravos, comandada por Esprtaco.
c) Revolta Judaica ou Revolta Hasmoniana.
d) Campanha de massacre aos patres, dirigida por Catilina.
e) Revolta dos plebeus, sob a liderana de Tarqunio.

29) UFPEL 2007 - IMPERIALISMO) Em 1887 o Vietn passou a ser,


oficialmente, uma colnia, situada na Pennsula da Indochina e era
fornecedora de arroz, borracha e madeira para o mercado europeu, nos
moldes do modelo imperialista implantado pelas grandes naes capitalistas.
Durante a Segunda Guerra Mundial, foi fundada a Liga para a Independncia
do Vietn (Vietminh), de orientao socialista e liderada por Ho Chi Minh.
Depoimento do advogado portugus Jorge Santos, In: RODRIGUES, Urbano
Tavares (org.). A Guerra do Vietname. Lisboa: Estampa, 1968.
A colonizao referida foi efetivada no sculo XIX, pelo seguinte pas:
a) China.
b) Japo.
c) Estados Unidos.
d) Inglaterra.
e) Frana.

32) FURG 2007 IDADE MDIA) Analise as proposies abaixo sobre o


Feudalismo.
I. A sociedade feudal era estamental, e o indivduo era classificado segundo a
forma como possua a terra e o seu nascimento, ou origem sangnea.
II. Os servos tinham mobilidade geogrfica e eram soldados e artesos,
pagando a corvia, as banalidades e o vintm, sendo dispensados da talha
sobre a produo.
III. Os escravos eram muito raros na Europa Ocidental, devido condenao
religiosa, sendo mais freqentes entre os muulmanos ibricos.
IV. A nobreza tinha a posse jurdica da terra, prestava Homenagem e
Benefcio, possuindo poderes polticos, militares e jurdicos sobre os demais.
Esto corretas as afirmativas:
a) I, III e IV.
c) I, II e IV.
e) todas.
b) II, III e IV.
d) I, II e III.

30) UFPEL 2007 DITADURAS LATINO-AMERICANAS) Alm do Brasil, outros


pases da Amrica Latina, na dcada de 1970, vivenciaram governos
militares.

33) FURG 2007 REVOLUO INGLESA) A Revoluo Gloriosa (1688-1689),


na Inglaterra, trouxe como uma de suas decorrncias o Bill of Riths, ou
Declarao dos Direitos. Sobre ela, podemos afirmar que:
a) foi causa da Guerra dos Trinta Anos, devido discriminao econmica e
religiosa contra os holandeses.
b) orientou as decises do perodo chamado de Restaurao Stuart,
idealizada por Oliver Cromwell.
c) visava a garantir os interesses dos lords em oposio aos interesses da
nascente burguesia liberal.
d) inspirou-se nas idias utpicas de Thomas Morus, que em seguida foi
decapitado por ordem de Henrique VIII Tudor.
e) teve parte de seus princpios incorporados s dez primeiras emendas
norte-americanas e Declarao dos Direitos do Homem da Frana, em
1789.

34) FURG 2007 - ABSOLUTISMO) Analise as proposies abaixo, sobre o


chamado Antigo Regime, ocorrido na Frana durante a Modernidade.
I. os burgueses, principalmente a alta burguesia, auxiliaram aos monarcas na
luta contra a nobreza, atravs de impostos e financiamento, pois pretendiam
retirar desta ltima os monoplios econmicos que detinha.
II. os Estados Nacionais foram importantes na coordenao da Expanso
Martima e na conquista de colnias, pois com isso conseguiriam o
enriquecimento burgus nacional e seu prprio fortalecimento.
III.visando a justificar a centralizao e a concentrao de poderes nas mos
O regime de Pinochet perseguiu, torturou e assassinou inmeros opositores,
do rei, juristas e legistas apoiaram-se no Direito Romano, na religio e
alm de levar o pas ao retrocesso social e cultural em todas as esferas. Na mesmo na praticidade poltica do Estado Moderno.
foto: militares queimam livros perigosos.
Esto corretas as afirmativas:
a) todas. b) I e II.
c) I e III. d) II e III.
e) nenhuma.
35) FURG 2007 NAPOLEO BONAPARTE) A Santa Aliana, aps o
Congresso de Viena (1814-1815), adotou uma poltica em relao s colnias
hispano-americanas, que visava a:
a) impor os princpios da Doutrina Monroe sobre todos os Estados europeus.
b) restabelecer o antigo sistema colonial espanhol.
c) difundir as idias poltico-sociais da Revoluo Francesa de 1789.
d) obrigar D. Joo VI a entregar a Provncia Cisplatina a Fernando VII da
Espanha.
e) incentivar os movimentos nacionalistas de carter separatista na Amrica
do Sul.
Marcha das mes de desaparecidos, na Praa de Maio. As Mes da Praa
de Maio viraram um smbolo mundial de resistncia contra a violncia
praticada pelos regimes militares.
As fotos e suas legendas referem-se, respectivamente, s ditaduras do(a)
a) Uruguai e do Chile.
b) Argentina e do Paraguai.
c) Paraguai e do Uruguai.
d) Chile e da Argentina.
e) Bolvia e do Peru.

36) FURG 2007 REVOLUO INDUSTRIAL) Com a Revoluo Industrial,


pioraram as condies de vida e de trabalho dos operrios, ocasionando
movimentos de reivindicao, greves, violncia e at mesmo destruio de
mquinas. Na primeira metade do sculo XIX, a Inglaterra, pioneira na
industrializao, tambm conheceu movimentaes operrias, em que se
destacou:
a) a fundao do Partido Comunista Ingls, que alm de alinhar-se com as
ordens de Moscou, passou a dirigir os movimentos sindicais, antes
descentralizados e sem lideranas.

11

b) o movimento chamado de socialismo cientfico, que reconhecia como


necessidade uma fase de ditadura do proletariado para se chegar
organizao de uma sociedade comunista.
c) a criao e o funcionamento da Associao Internacional dos
Trabalhadores, fundada por Karl Marx e Friederich Engels.
d) um movimento jacobinista, que pretendia a igualdade social e poltica,
como o voto universal tambm para as mulheres, assim como o direito a
iguais salrios e oportunidades.
e) o movimento cartista, que desenvolveu um programa vinculado s
reivindicaes trabalhistas, tais como: limitaes da jornada de trabalho,
abolio da lei dos indigentes e questes de garfo e faca.
37) FURG 2007 SEGUNDA GUERRA MUNDIAL) O Brasil tem postulado um
lugar permanente no Conselho de Segurana da Organizao das Naes
Unidas. No entanto, desde a sua fundao, existem apenas cinco pases
permanentes e dez eleitos pela Assemblia Geral. Em sua criao, aps a II
Guerra Mundial, eram permanentes os seguintes pases:
a) China Nacionalista, Frana, Estados Unidos, Unio Sovitica e Reino Unido.
b) Israel, Repblica Federal Alem, Blgica, Unio Sovitica e Espanha.
c) China Comunista, Hungria, Estados Unidos, Tchecoslovquia e Benelux.
d) Repblica Democrtica Alem, Estados Unidos, Rssia e Argentina.
e) Holanda, Repblica Malgache, Taiwan, China e Estados Unidos.
38) Universidade do Estado de Santa Catarina 2007 - IMPERIALISMO) O
avano da industrializao e a consolidao do capitalismo monopolista
levaram as potncias europias, os Estados Unidos e o Japo a iniciarem, nas
ltimas dcadas do sculo XIX, um movimento de expanso e de conquista
que ficou conhecido como Imperialismo.
Sobre esse processo, incorreto afirmar:
a) Esse processo foi acompanhado de uma forte justificativa ideolgica
exercida de vrias formas como, por exemplo, com teorias que asseguravam
a superioridade da raa branca sobre as demais.
b) Inglaterra e Frana ficaram com poucos territrios, de reduzidas
possibilidades econmicas, e reacenderam sua antiga rivalidade.
c) Esse processo promoveu a partilha da frica e da sia e estabeleceu reas
de influncia.
d) As potncias capitalistas buscavam reas para aplicao de capitais, novos
mercados consumidores e regies fornecedoras de matrias-primas para a
indstria.
e) Esse processo gerou descontentamentos e um estado de tenso
permanente entre as potncias, em funo das reparties desiguais das
reas de influncia.
39) FURG 2000 - IMPERIALISMO) A respeito do imperialismo so feitas 6
afirmativas.
I - A constante busca por mercados para colocao de bens industrializados,
investimento de capitais e obteno de matrias-primas foram alguns dos
fatores que levaram corrida imperialista.
II - O racismo e o darwinismo social foram alguns dos argumentos utilizados
pelos
pases imperialistas para legitimar a expanso colonial.
III - A sia, a frica e a Oceania foram os alvos preferenciais do
imperialismo; porm, as potncias imperialistas tambm atuaram em regies
da Amrica e da Europa Meridional e Centro-Oriental, embora no exercendo
um predomnio poltico e colonial direto.
IV - Algumas das modalidades de domnio empregadas na poca da
expanso imperialista foram as colnias propriamente ditas, os protetorados,
as zonas de influncia e as colnias estratgicas.
V - As disputas por melhores pores na partilha do mundo colonizado levou
deflagrao das I e II Guerras Mundiais, conflitos eminentemente
interimperialistas.
VI - O imperialismo viria a produzir profundas conseqncias junto s
sociedades colonizadas, trazendo a atrofia no desenvolvimento econmico, as
distores sociais e a pobreza endmica como algumas de suas heranas.
Quais afirmativas esto corretas?
a) Nenhuma.
b) Apenas I, II, III e IV.
d) Apenas II, III, V e VI.
c) Apenas I, IV, V e VI.
e) Todas.
40 Universidade Estadual de Santa Cruz PRIMEIRA GUERRA
MUNDIAL) "Quando se pensa nas conseqncias da Grande Guerra, que
agora finda, pode-se perguntar se a estrela da Europa no perdeu seu brilho,
e se o conflito do qual ela tanto padeceu no iniciou para ela uma crise vital
que anuncia a decadncia (...)."

c) A crise econmica e a decorrente inflao na Europa do primeiro psguerra comprovam a perda do brilho referida no texto.
d) O conflito relativo Primeira Grande Guerra decorreu do confronto entre
pases socialistas e anarquistas.
e)A economia alem foi grandemente beneficiada com a vitria do pas na
Primeira Grande Guerra
41) UDESC PRIMEIRA GUERRA MUNDIAL) A regio dos Blcs, onde se
desenrolaram nos ltimos anos sangrentas lutas entre bsnios e srvios, foi
tambm o palco de acontecimentos que serviram de estopim para a Primeira
Guerra Mundial, entre 1914 e 1918. Assinale a alternativa que INDICA o
pretexto imediato para este ltimo conflito:
a) independncia da Iugoslvia, que era anteriormente dominada pelos
austracos;
b) as reivindicaes srvias acerca de terras que pertenciam BsniaHerzegovina;
c) o envio de tropas franco-britnicas para a defesa do Governo da Bsnia;
d) a posse do Governo socialista do general Tito, aps uma violenta guerra
civil;
e) o assassinato do arquiduque austraco Francisco Ferdinando.
42) UNISC PERODO ENTRE GUERRAS) E, pior de tudo, o aspecto mais
desolador da Grande Depresso era a impresso ntida de que ela no iria ter
fim (...). Em 1930, a nao assobiava a cano Happy Days Are Here Again
(Os Dias felizes Voltaram), mas o produto nacional caa de US$ 87 bilhes
para US$ 75 bilhes. Em 1931,o pas cantava Ive Got Five Dollars (Eu
ganhei cinco dlares), enquanto o produto caa para US$ 59 bilhes. Em
1932, a cano era mais triste: Brother, can you spare a dime? (Irmo, pode
me emprestar um trocado?) - e o produto nacional diminua para miserveis
US$ 42 bilhes.
(ANVERSA, G. L. A Crise econmica mundial de 1929. In: PADRS, E.et alli.
Segunda Guerra Mundial. Porto Alegre: Folha da Histria/Palamrinca, 2000,
p. 31-32).
Essas canes retratam
a) a crise do capitalismo intervencionista e a ascenso do nazi-fascismo.
b) o sucesso do capitalismo e do liberalismo impulsionado pelas msicas de
Hollywood.
c) a prosperidade do liberalismo e do capitalismo ps crise de 1929.
d) o desencanto dos americanos com o futuro econmico ps-1929.
e) o canto e o desencanto como aparelhos de dominao ideolgica.
43) UNISC PERODO ENTRE GUERRAS) A cultura humana e a civilizao
nesta parte do mundo esto inseparavelmente ligadas existncia dos
arianos. A sua extino ou decadncia faria recair sobre o globo o vu escuro
de uma poca de barbrie (...) Quem ousa pr as mos sobre a mais elevada
semelhana de Deus ofende a essa maravilha do Criador e coopera para a
expulso do paraso.
(HITLER, Adolf. Minha Luta. So Paulo: Moraes, 1983, p. 241).
Essa afirmao se refere
a) ao esprito religioso de Hitler, que lembrava a criao de Ado e Eva e o
Paraso.
b) f na superioridade racial dos mestios arianos da Alemanha nazista.
c) proteo divina do povo eleito por Deus para ocupar a Terra Santa
(Palestina).
d) expulso do Paraso depois de recair sobre o globo o vu escuro da
barbrie.
e) concepo racista de mundo presente no discurso nazista.

1)
2)
3)
4)
5)
6)
7)
8)
9)
10)

E
B
A
C
A
B
A
B
A
D

11)
12)
13)
14)
15)
16)
17)
18)
19)
20)

C
E
A
D
D
B
C
C
E
B

21)
22)
23)
24)
25)
26)
27)
28)
29)
30)

D
E
A
C
E
D
E
A
E

31)
32)
33)
34)
35)
36)
37)
38)
39)
40)

B
A
E
D
B
E
A
B
E
C

41)
42)
43)
44)

E
D
E
C

(Demangeon. In: Aquino et al., p. 243)

De acordo com o texto e com os conhecimentos sobre a Primeira Grande


Guerra, possvel afirmar:
a) A predominncia dos pases democrticos e dos partidos antiautoritrios
destaca-se entre as conseqncias da Grande Guerra.
b) A citada decadncia da Europa decorria, entre outros fatores, da
derrubada de governos monrquicos e republicanos no continente.

12

1) UFPEL VERO 2007 FORMAO DO RS)


Fronteiras Meridionais Do Brasil

b) crise do escravismo e imigrao europia para o Brasil, em perodo de


desenvolvimento da cafeicultura.
c) ao Ciclo da Minerao, quando escravos e imigrantes formaram o principal
contingente de mo-de-obra.
d) ao processo de transio do escravismo colonial brasileiro para o trabalho
assalariado durante o Primeiro Reinado.
e) necessidade de manuteno de uma fora de trabalho em torno de 150 mil
pessoas no processo de industrializao paulista, durante o Perodo Regencial.
3) UFPEL VERO 2007 GETLIO VARGAS)

In: Sampaulo: 30 anos vistos a lpis. 1958.

A charge relaciona-se ao:


a) final do Estado Novo e s aes polticas do Partido Comunista de Lus Carlos
Prestes, dos integralistas e dos movimentos tenentistas.
b) ltimo governo de Vargas e aos movimentos insurrecionais como o Levante dos
Dezoito do Forte de Copacabana, a Coluna Prestes e o Putsch integralista.
c) Governo Provisrio de Vargas, quando ocorreram as Intentonas Comunista e
Integralista, representadas, respectivamente, pelas figuras de Lus Carlos Prestes e
Plnio Salgado.
d) Governo Constitucional de Vargas, momento que coincide com a Segunda Guerra
Mundial, e movimentos militares, como a Repblica do Galeo e o Levante de
Aragaras.
e) Governo Provisrio e ao Constitucional de Getlio Vargas, quando ocorreram a
Ao Integralista, a Revoluo Constitucionalista, a Intentona Comunista e o Plano
Cohen.
4) UFPEL VERO 2007 GETLIO VARGAS) [...] Depois de decnios de domnios e
espoliao dos grupos econmicos e financeiros internacionais, fiz-me chefe de uma
revoluo e venci. [...] A campanha subterrnea dos grupos internacionais aliou-se
dos grupos nacionais revoltados contra o regime de garantia do trabalho. A lei de
lucros extraordinrios foi detida no Congresso. Contra a justia da reviso do salrioFERREIRA FILHO, Arthur. Histria Geral do RS 1503-1957. Porto Alegre:Globo, mnimo se desencadearam os dios. Quis criar a liberdade individual na
1958.potencializao das nossas riquezas atravs da Petrobrs, mal comea esta a
As linhas dos Tratados tracejadas nos mapas representam, respectivamente,
funcionar, a onda de agitao se avoluma. A Eletrobrs foi obstaculada at o
a) o Tratado de Santo Ildefonso quando as misses jesuticas foram entregues aos desespero. No querem que o trabalhador seja livre. No querem que o povo seja
portugueses pelos espanhis e a demarcao do Tratado de Tordesilhas.
independente. Assumi o Governo dentro da espiral inflacionria que destrua os
b) o Tratado de Badajoz (1750) que determinou a criao dos Campos Neutrais evalores do trabalho. Os lucros das empresas estrangeiras alcanavam at 500% ao
o Tratado de Madri (1777), que, ao contrrio, manteve a Colnia do Sacramento ano. [...] Lutei contra a explorao do Brasil. Lutei contra a espoliao do povo. [...]
como portuguesa.
Eu vos dei a minha vida. Agora ofereo a minha morte. Nada receio. Serenamente
c) o Tratado de Madri pelo qual os portugueses entregaram as misses para os dou o primeiro passo no caminho da eternidade e saio da vida para entrar na
espanhis em troca da Colnia do Sacramento e o limite meridional do Tratado de Histria.
Tordesilhas no sul do Brasil.
Carta Testamento de Getlio Vargas 24/08/1954.
d) o Tratado de Santo Ildefonso (1750) que garantiu a posse da Provncia O documento expressa uma poltica de
Cisplatina para o Imprio brasileiro e a representao geogrfica dada pelo Tratado a) liberalismo econmico e nacionalismo, caracterstica do perodo em que governou
de
provisoriamente.
Badajoz (1777), que contrastou com o tratado anterior e antecedeu a Guerra b) estatizao, restrio ao capital externo e financeiro, que corresponde ao perodo
Guarantica.
no qual foi eleito diretamente pelo povo.
e) a fronteira determinada pelo Tratado de Madri (1750) que eliminou os limites do c) abertura ao capital externo, criao de empresas estatais, como a Eletrobrs e a
Tratado de Tordesilhas e a demarcao do Tratado de Santo Ildefonso (1777).
Petrobrs, representando o perodo do Estado Novo.
d) protecionismo estatal e populismo, sintetizando a ditadura legalizada pela
constituio polaca, momento poltico no qual a Carta foi redigida.
e) assistncia aos trabalhadores e liberalismo, que ensejava o Estado mnimo,
2) (UFPEL VERO 2007 ESCRAVIDO)
durante o seu Governo Constitucional.
5) UCPEL 2007 BRASIL REDEMOCRATIZADO)

CAMPOS, Flvio de e MIRANDA, Renan Garcia. Oficina de Histria, So Paulo:


Moderna, 2000. [adapt.]

Com base na anlise da charge acima e nos seus conhecimentos sobre a


O grfico est diretamente relacionado
a) estabilidade do sistema escravista e emigrao europia no sculo XIX, quando Histria do Brasil, considere as afirmativas a seguir.
I . Em 1985, aps um perodo de 20 anos de ditadura no Brasil, o pas elegeu de
iniciou a industrializao paulista.
forma direta Tancredo Neves como presidente da Repblica; a vitria deste sobre o

15

candidato Paulo Maluf foi amplamente festejada porque era o fim do regime militar e c) a charge representa o grande desenvolvimento econmico do pas durante a
o incio do exerccio da cidadania total no pas.
dcada de 70, bem como a satisfao da populao brasileira com a economia e a
II Em 5 de outubro de 1988, foi promulgada a nova Constituio do Brasil, apelidada conquista da Copa do Mundo.
de a .Constituio cidad., tendo em vista o fato de que ela concedeu aos d) em 1970, sob a gide do governo de Ernesto Geisel, o Brasil vence, pela quarta
analfabetos e aos jovens de 16 anos o direito de votar, ampliando, desta forma, a vez, a Copa do Mundo de futebol e isto refora a imagem do governo junto
cidadania poltica.
sociedade.
III . Em maro de 1989, Fernando Collor de Mello, do Partido da Reconstruo e) em 1970, o Brasil vence a Copa do Mundo e, nesse momento, o regime militar
Nacional (PRN), assumiu a presidncia do pas, tendo por base 3 promessas centrais: alcana, a um s tempo, os mais altos nveis de crescimento e de represso poltica.
derrubar a inflao em um s golpe, manter seus ministros at o fim do governo, e
modernizar o pas por meio das privatizaes e da abertura comercial.
8) UCPEL INVERNO 2006 MOVIMENTOS COLONIAIS) Portanto, condenam ao ru
IV. A repblica foi instaurada no Brasil em 1889 e, em tese, deveria ser para todos; Joaquim Jos da Silva Xavier, por alcunha o Tiradentes, alferes que foi da tropa paga
no entanto, com roupagens distintas, conviveu e ainda convive com a excluso de da Capitania de Minas Gerais a que, com embarao e pregao, seja conduzido pelas
muitos cidados, pois, na atualidade, o pas preserva os direitos polticos de seus ruas pblicas ao lugar da forca, e nela morra de morte natural para sempre e que,
cidados, mas a grande maioria da populao permanece excluda da cidadania na depois de morto, lhe seja cortada a cabea e levada a Vila Rica, aonde, em o lugar
sua dimenso social.
mais pblico dela, ser pregada em alto poste, digo poste alto, at que o tempo o
Esto corretas
consuma, e o seu corpo ser dividido em quatro quartos iguais e pregado em postes,
a) somente a I e a III.
e) todas.
pelo caminho de Minas...
b) somente a II, a III e a IV.
(Autos da Devassa da Inconfidncia Mineira. Biblioteca Nacional, 1936, p. 194)
c) somente a II e a IV.
Sobre o texto acima, considere as afirmativas a seguir.
d) somente a I, a II e a III.
I . A Inconfidncia Mineira marca o incio do processo da emancipao poltica do
Brasil, tendo em vista que a primeira vez que o pacto colonial posto em xeque.
6) UCPEL 2007 HISTRIA DO RS)
II. O programa de organizao poltica para o momento posterior ao rompimento
com Portugal, inspirava-se na ilustrao; os inconfidentes advogavam pela Repblica,
liberdade aos escravos e criao de uma universidade em So Joo d. El Rei.
III . Ao final da Inconfidncia Mineira, em razo dos participantes serem numerosos e
alguns importantes, o perdo de Portugal alcanou a muitos deles, alguns foram
exilados mas Tiradentes foi a nica vtima condenada forca.
IV. Apesar de ser uma Inconfidncia que envolveu inmeras pessoas e questionou a
legitimidade do escravismo colonial, a tarefa de constituir-se um movimento
realmente popular s foi desempenhada pela Inconfidncia Baiana, em 1798.
Esto corretas
a) I e III. c) I, II e III.
e) Todas.
b) II e IV. d) II e IV.
9) UFSC REPBLICA VELHA) Sobre a Revolta da Vacina, correto afirmar que:
1) a Revolta da Vacina foi um movimento popular cujo motivo imediato foi a
publicao, em 9 de novembro de 1904, do decreto de regulamentao de aplicao
da vacina obrigatria contra a varola;
2) a Revolta da Vacina teve, como uma de suas causas, a demolio dos cortios que
se localizavam no centro do Rio de Janeiro. Esta ao fazia parte de um projeto mais
Tendo por base a anlise da charge acima e os seus conhecimentos acerca da amplo denominado Regenerao e que possua, como meta, a construo de casas
Histria do Brasil e do Rio Grande do Sul, considere as afirmativas abaixo.
populares, favorecendo a populao desassistida da cidade;
I . Os tempos mudaram porque a primeira vez que uma mulher governa o Rio 4) o decreto que desencadeou o movimento impunha vacinaes e exames, alm de
Grande do Sul, Estado altamente conservador, que tem suas origens aliceradas no punir aqueles que se recusassem a cumprir suas determinaes com multas e
separatismo da Revoluo Farroupilha . movimento chefiado pela elite estancieira e demisses. A populao no foi devidamente esclarecida sobre o efeito da vacina ou
charqueadora gacha e na Revoluo Federalista, a qual, por meio do Pacto de seus possveis benefcios;
Pedras Altas, garantiu uma reforma na Constituio gacha, proibindo a reeleio de 8) dentre as vrias causas da Revolta da Vacina destaca-se o descontentamento da
Borges de Medeiros.
populao diante das reformas urbanas praticadas no centro do Rio de Janeiro e a
II . A mudana referida na charge diz respeito quebra na tradio conservadora do introduo de novos hbitos por meio do Cdigo de Posturas, numa tentativa de
Estado rio-grandense na medida em que este elege uma mulher chefia do tornar a cidade uma capital mais adequada Repblica.
executivo estadual, pois historicamente, o Rio Grande do Sul constituiu-se como uma
fronteira viva do pas, fato que levou assinatura de diversos tratados para a 10) PUC-RS REPBLICA VELHA) Entre as mudanas polticas e institucionais
definio dos limites do Brasil como, por exemplo, o Tratado de Lisboa (1681), que introduzidas no Brasil logo aps a Proclamao da Repblica, durante o Governo
devolvia a Portugal a Colnia do Sacramento (sua antiga fundao).
Provisrio do Marechal Deodoro da Fonseca, no correto citar:
III . A charge ilustra a surpresa causada pela eleio da primeira governadora do Rio a) a separao entre Igreja e Estado;
Grande do Sul, tendo em vista toda a formao tradicional do Estado que, inmerasb) a instituio de um Estado federativo;
vezes, forneceu contingente militar para os conflitos no Prata, como o caso da c) a aprovao de um projeto de naturalizao dos estrangeiros;
participao do Brasil na Guerra do Paraguai, conflito que transformou a repblica d) o banimento da Famlia Real do Brasil;
paraguaia numa nao pobre, arruinada e endividada.
e) a instituio do voto censitrio e fechado.
Est(ao) correta(s)
a) somente a III.
11) PUCRS 2008 BRASIL COLNIA) INSTRUO: Responder questo 26 com
b) somente a II.
base nas afirmativas abaixo, sobre o Barroco brasileiro do Perodo Colonial.
c) I e III somente.
I. A arte do Barroco surgiu, no contexto de reorganizao da Igreja Catlica, pela
d) II e III somente.
necessidade de expandir a f crist no Novo Mundo.
e) todas.
II. O Barroco baiano produziu igrejas de pequeno porte e de decorao austera,
devido decadncia da economia do acar.
7) UCPEL INVERNO 2006 DITADURA MILITAR)
III. A riqueza advinda da minerao possibilitou o florescimento do Barroco mineiro,
cujos mestres e artistas, vrios deles mestios, introduziram elementos originais da
cultura local (negra e indgena).
IV. Antonio Francisco Lisboa, o Aleijadinho de Vila Rica, foi um dos mais destacados
escultores e decoradores do perodo, tendo produzido uma das obras mais
importantes do barroco brasileiro no Santurio de Bom Jesus de Matosinhos, em
Congonhas do Campo (MG).
Esto corretas apenas as alternativas
a) I e II.
c) I, III e IV.
e) II e IV.
b) I e III. d) II e III.
A charge reflete a conquista brasileira da Copa do Mundo de Futebol, em 1970. Sobre 12) PUCRS 2008 REPBLICA VELHA) O modernismo brasileiro tem seus
antecedentes na modernizao da imprensa (jornais e revistas) e na boemia literria
o assunto, correto afirmar que
a) a conquista da Copa do Mundo no satisfaz a fome da populao brasileira, isso carioca no incio do sculo XX, mas foi na dcada de 1920, aps a Semana de Arte
Moderna (1922), em So Paulo, que as duas principais correntes do movimento se
porque o pas vivencia um perodo de baixssimo crescimento econmico.
b) na dcada de 70, o pas vive o seu apogeu econmico e financeiro e essa articularam e publicaram manifestos em defesa de suas idias. Oswald de Andrade e
democratizao poltica reflete-se, tambm, na conquista da Copa de 1970, no Mario de Andrade representaram uma vertente nacionalista __________, enquanto
Cassiano Ricardo e Plnio Salgado tinham uma posio nacionalista mais __________.
Mxico.
As palavras que preenchem correta e respectivamente as lacunas so:

16

a) vanguardista conservadora
b) integralista socialista
c) romntica futurista
d) simbolista realista
e) naturalista concretista

c) F F V V V

14) PUCRS 2008 DITADURA MILITAR) INSTRUO: Responder questo 30 com


base nas afirmativas abaixo, sobre os meios de comunicao durante os Governos
Militares, no Brasil.
I. O final dos anos 1960 caracterizou-se por um abrandamento do controle poltico e
13) PUCRS 2008 GETLIO VARGAS) INSTRUO: Para responder questo 29, da censura ideolgica nos meios de comunicao.
analise as afirmaes referentes poltica, economia e cultura no Estado Novo, II. Nos anos 1970, aps a criao da EMBRATEL, foi possvel interligar o sistema de
preenchendo os parnteses com V (verdadeira) ou F (falsa).
comunicaes, abrangendo todo o territrio brasileiro e permitindo, pela primeira vez
( ) O Estado interveio diretamente na economia, visando a promover ono pas, a integrao e a difuso de informaes
desenvolvimento de uma indstria de base.
em cadeia nacional.
( ) O governo estimulou a liberdade sindical como forma de fortalecer a participao III. Entre os anos 1970 e 1980, a televiso entrou nos lares brasileiros das classes
efetiva do operariado na poltica.
mdia e popular, possibilitando publicidade e s telenovelas difundirem novos
( ) O Estado criou rgos para controlar os meios de comunicao e encampou rdios estilos de vida e de consumo para as massas urbanas.
e jornais, visando tanto a censurar os opositores do regime quanto a difundir IV. Jornais da imprensa alternativa, como o Pasquim e o Coojornal, conseguiram
informaes que legitimassem os projetos e as aes do governo perante a sobreviver censura governamental e foram importantes canais de reflexo e de
sociedade civil.
crtica durante o regime militar, promovendo o debate sobre a necessidade de uma
( ) O governo manteve-se neutro ao longo de toda a II Guerra Mundial, visando aabertura poltica.
beneficiar-se de uma poltica de negociaes tanto com os EUA quanto com a Esto corretas apenas as afirmativas
Alemanha.
a) I e II.
( ) Sob a alegao da defesa das instituies, Getlio Vargas ordenou a dissoluo b) I e III.
dos partidos polticos e da Assemblia Nacional, passando a nomear interventores c) II e IV.
federais para os estados e a governar por decretos-lei.
d) II, III e IV.
A seqncia correta de preenchimento dos parnteses
e) III e IV.
a) V F V F V d) V V F V F
b) F V F V F
e) V
VFFV
15) UCS 2007 REVOLTAS BRASILEIRAS) Assinale a alternativa que estabelece uma
comparao INCORRETA entre as revoltas brasileiras ocorridas no Perodo
Regencial (1831-1840).
Alternativa

Revolta

Provncia

Data

Cabanagem

Par

1835-1840

Farroupilha

Revolta dos Mals

Bahia

1835

Pretendia acabar com a escravido Matar os brancos e conseguir a


africana.
liberdade.

Sabinada

Bahia

1837-1838

Contra a centralizao do governo


Instituir uma repblica na provncia
regencial e manuteno da autonomia
da Bahia, enquanto o prncipe
provincial conseguida com o Ato
herdeiro fosse menor de idade.
Adicional.

Balaiada

Maranho

1838-1841

Crise decorrente do declnio da


Lutar contra a misria, a fome, a
exportao de algodo; pobreza da
escravido e os maus-tratos.
maioria da populao da provncia.

Rio Grande do Sul

1835-1845

Causa

Objetivo

Contra a misria e contra as oligarquias


Acabar com a escravido e distribuir
locais.
terras para os lavradores.

Concorrncia do charque uruguaio e


Proclamar uma Repblica autnoma e
argentino, devido aos baixos impostos
acabar com a escravido.
de importao.

17

16) UCS 2007 - ESCRAVIDO) Analise a veracidade (V) ou falsidade (F) das
proposies abaixo, sobre o escravismo no Rio Grande do Sul.
( ) O escravo negro foi elemento de pouca importncia no desenvolvimento
da economia gacha no sculo XIX.
( ) A abolio da escravido, com a transformao dos negros em cidados,
j estava prevista no iderio republicano dos revolucionrios farroupilhas.
( ) Ao contrrio do restante do Pas, no territrio gacho, os escravos, de
uma forma geral, eram bem tratados e no sofreram castigos e torturas.
Assinale a alternativa que preenche corretamente os parnteses, de cima
para baixo.
a) V V V
b) V V F
c) V F F
d) F F V
e) F F F
17) UCS 2007 CICLO DO CAF) Analise a veracidade (V) ou falsidade (F)
das proposies abaixo, sobre a economia brasileira no sculo XIX.
( ) A partir da segunda metade do sculo XIX, o centro econmico do Pas
deslocou-se das antigas reas agrcolas do nordeste para o centro-sul, em
funo dos cafezais que se expandiam.
( ) Nas fazendas de caf em So Paulo, o trabalho do escravo foi sendo
substitudo lentamente pelo trabalho assalariado, com predomnio de
imigrantes europeus (italianos, alemes, etc).
( ) O caf foi introduzido no Brasil por volta de 1727. A princpio, era um
produto sem grande valor comercial, utilizado apenas para consumo local.
Entretanto, a partir do incio do sculo XIX, o hbito de beber caf alcanou
grande popularidade na Europa e nos Estados Unidos.
Assinale a alternativa que preenche corretamente os parnteses, de cima
para baixo.
a) V F F
b) V V F
c) V V V
d) F F V
e) F V F
18) UCS 2007 DITADURA MILITAR) Analise a veracidade (V) ou falsidade
(F) das proposies abaixo, sobre o perodo da histria brasileira conhecido
como Ditadura Militar (1964-1985).
( ) Os militares estavam divididos em duas tendncias: a linha moderada,
representada pelos liberais conservadores, e a linha dura, que agrupava os
defensores da represso pura e simples oposio.
( ) O general Castelo Branco, um moderado, foi o primeiro presidente militar.
A linha dura chegou ao poder com a ascenso de Costa e Silva, em 1967. O
Ato Institucional n 5, editado em 1968, institucionalizou a ditadura no Pas.
( ) Os moderados voltaram ao poder com o general Garrastazu Mdici, que
deu incio abertura poltica. Extinguiu o AI-5, reinstituiu o pluripartidarismo
e indicou Ernesto Geisel como seu sucessor.
Assinale a alternativa que preenche corretamente os parnteses, de cima
para baixo.
a) V V F
c) V F F
e) F V F
b) V V V
d) F F V
19) UFPEL 2001 COLONIZAO DO BRASIL) Uma das experincias mais
difceis para os grupos humanos aprender a relacionar-se com o outro: ver,
conviver e respeitar seres, grupos diferentes. Quando os portugueses
chegaram ao Brasil, a partir de 1500, encontraram populaes com hbitos
diferentes dos seus. Essas populaes, poca estimadas em
aproximadamente 9 milhes de ndios, hoje, no somam mais de 500 mil,
dos quais poucos vivem excludos nas periferias urbanas, ao passo que a
maior parte encontra-se em reservas.
Com base no texto, e em seus conhecimentos, assinale a alternativa coerente
que contm UMA DAS CAUSAS que contribuiu para a queda demogrfica das
populaes indgenas.
a) os indgenas brasileiros viviam no Paleoltico, desenvolvendo a horticultura,
de forma sedentria, o que no os deixava vulnerveis a vrus da gripe e
bactria da varola.
b) os indgenas viviam no estado Neoltico, como nmades suas fontes de
subsistncia eram a caa e a coleta o que os fazia contrair doenas em
contato com os animais selvagens.
c) a colonizao do continente americano deflagrou um confronto de
culturas, pois os europeus possuam uma cultura superior, que os deixava
imunes s doenas transmitidas pelo contato com os indgenas.
d) as populaes indgenas sofreram com a ocupao territorial por parte do
colonizador, devido ao contato com doenas contagiosas trazidas pelos
europeus, contra as quais os ndios no portavam anticorpos.
e) a colonizao do Brasil trouxe um enorme avano para os amerndios, que
se beneficiaram com a chegada dos civilizados, pois passaram a usufruir do
uso de vacinas e soros.
20) UFPEL 2008 MOVIMENTOS NATIVISTAS) No decorrer do perodo
colonial no Brasil os interesses entre metropolitanos e colonos foram se
ampliando.

O descontentamento se agravou quando, a 1 de abril de 1680, a Coroa


estabeleceu a liberdade incondicional dos indgenas, proibindo taxativamente
que fossem escravizados. Alm disso confiou-os aos jesutas, que passaram a
ter a jurisdio espiritual e temporal das aldeias indgenas.
Visando solucionar o problema da mo-de-obra para as atividades agrcolas
do Maranho, o governo criou a Companhia do Comrcio do Estado do
Maranho (1682).
Durante vinte anos, a Companhia teria o monoplio do comrcio importador
e exportador do Estado do Maranho e do Gro-Par. Cabia-lhe fornecer dez
mil escravos africanos negros, razo de quinhentos por ano, durante o
perodo da concesso outorgada.
AQUINO, Rubim Santos Leo de [et al.]. Sociedade Brasileira: uma histria
atravs dos movimentos sociais. 3 ed., Rio de Janeiro: Record, 2000.
Pelos elementos mercantilistas, geogrficos e cronolgicos, o conflito inferido
do texto foi a Revolta
a) dos Emboabas.
b) dos Mascates.
c) de Amador Bueno.
d) de Filipe dos Santos.
e) de Beckman.
21) UFPEL 2003 - ESCRAVIDO)
A CANO DO AFRICANO
L na mida senzala,
Sentado na estreita sala,
Junto ao braseiro, no cho,
Entoa o escravo o seu canto,
E ao cantar correm-lhe em pranto,
Saudades do seu torro...
(...)
Minha terra l bem longe,
Das bandas de onde o sol vem;
Esta terra mais bonita,
Mas a outra eu quero bem!
(...)
Aquelas terras to grandes,
To compridas como o mar,
Com suas poucas palmeiras
Do vontade de pensar...
L todos vivem felizes,
Todos danam no terreiro;
A gente l no se vende
Como aqui s por dinheiro.
(...)

(Castro Alves, 1863)

De acordo com o poema e com base em seus conhecimento, analise as


afirmaes abaixo:
I) O poema foi publicado no Segundo Reinado; a terra do africano
oriental em relao ao lugar em que o eu-lrico canta, e expressa uma
idealizao romntica, pois l tambm houve escravido.
II) Ao explorar o tema escravido, atravs da figura do negro saudoso de sua
terra, Castro Alves recria romanticamente o lugar natal do africano, com o
que contesta a tese escravocrata.
III)A explorao da mo-de-obra escrava e o trfico do negro africano no
perodo colonial foram extintos com a independncia do Brasil, devido aos
interesses ingleses em ampliar o mercado de consumo.
IV) Castro Alves manifesta o desejo do negro de estar em outro lugar,
considerando a anttese entre os diferentes espaos (mida senzalas, estreita
sala, terras to grandes). Assim, expressa correo histrica quando exalta a
frica como o lugar onde todos viviam felizes pelo fato de o colonialismo
europeu ser ali ainda muito incipiente.
Est (o) correta (s) apenas a (s) afirmao (es)
a) I e II.
b) III
c) I
d) II e IV
e) III e IV

22) UFPEL 2007 INVASES ESTRANGEIRAS) (...) da amizade dos ndios


depende em parte o sossego e a conservao da colnia do Brasil e que se
tendo isto em vista deve-se-lhe permitir conservar a sua natural liberdade,
mesmo aos que no tempo do rei de Espanha caram ou por qualquer meio
foram constrangidos escravido, como eu prprio fiz libertando alguns.
Devem-se dar ordens, tambm, para que no sejam ultrajados pelos seus
capites, ou alugados a dinheiro ou obrigados contra sua vontade a
trabalhar nos engenhos; ao contrrio deve-se permitir a cada um viver do
modo que entender e trabalhar onde quiser, como os da nossa nao (...).
Fragmento do relatrio de Maurcio de Nassau aos diretores da Companhia
das ndias Ocidentais, em 1644.
O documento demonstra que, durante

20

a) a Insurreio Pernambucana, a Companhia das ndias Ocidentais era


contrria a qualquer trabalho escravo na produo aucareira.
b) a Unio Ibrica, os holandeses proibiram o trfico de escravos para o
Brasil e promoveram a liberdade aos indgenas.
c) o perodo Colonial, a escravizao indgena foi inexistente, devido aos
interesses estratgicos e comerciais dos europeus.
d) as ocupaes francesas, no nordeste do Brasil, ocorreram transformaes
nas relaes dos europeus com as populaes nativas, no que se refere ao
trabalho cativo.
e) a ocupao holandesa, no nordeste brasileiro, foi combatida a
escravizao indgena promovida pelos ibricos.

27) FURG 2007 PERODO REGENCIAL) O clamor pela antecipao da


Maioridade, expresso na quadrinha popular, reafirma as expectativas de
superao da crise vigente durante o perodo regencial. Dentre os fatores
constitutivos desta crise, podemos destacar:
a) a disputa envolvendo os conservadores e os liberais, diferentes faces
oligrquicas, e a irrupo de movimentos regionais.
b) o declnio da lavoura cafeeira e a interveno da Guarda Nacional.
c) o nacionalismo e a defesa do liberalismo.
d) o prestgio poltico dos liberais exaltados, que contestavam a Regncia.
e) a unificao dos movimentos sociais regionais de rebelio, promovendo
uma forte resistncia Regncia.

23) UFPEL 2007 - ESCRAVIDO) Observe o seguinte esquema:


a) Macaco: centro poltico e administrativo;
b) Subupira: campo de treinamento militar;
c) Amaro, Andalaquitude, Aqualtune, Acetirene, Tabocas, Zumbi, Ozenga,
Danibraganga e outras menores unidades autnomas de produo.
Nessas povoaes, a maioria era de negros, antigos escravos que buscavam
a liberdade e a reconquista da sua dignidade como seres humanos. Tambm
havia muitos no negros: ndios, mulatos e mamelucos. Os documentos
indicam ainda a presena de muitos brancos. Era um verdadeiro ncleo de
deserdados da sociedade colonial nordestina dos sculos XVI e XVII.
AQUINO, Rubim et al. Sociedade Brasileira: uma histria atravs dos
movimentos sociais. 3 ed. Rio de Janeiro: Record, 2000.
O esquema descreve o(a)
a) Revolta da Chibata.
b) Quilombo de Palmares.
c) Revolta dos Mals.
d) Conjurao Baiana.
e) Revolta da Cabanagem.

28) FURG 2007 REPBLICA VELHA) Cabo de enxada engrossa as mos


o lao de couro cru, machado e foice tambm. Caneta e lpis so
ferramentas muito delicadas. A lida outra: labuta pesada, de sol a sol, nos
campos e nos currais (...) Ler o qu? Escrever o qu? Mas agora preciso: a
eleio vem a, e o alistamento rende a estima do patro, a gente vira
pessoa.
(Mrio Palmrio Vila dos Confins. Rio de Janeiro: Editora Jos Olympio.)
A partir da leitura do texto acima, podemos afirmar que na Repblica Velha:
a) a ampliao do poder poltico dos grandes proprietrios era a forma de
fortalecer a organizao dos trabalhadores rurais.
b) a campanha eleitoral visava a atingir os trabalhadores urbanos.
c) o predomnio oligrquico pode ser representado pela troca de favores.
d) os coronis, em poca de eleio, preocupados com o analfabetismo,
estimulavam a melhoria da educao.
e) o perodo eleitoral era o nico momento em que os chefes locais
dispensavam os trabalhadores do trabalho rduo para garantirlhes o direito
de cidadania.

24) FURG 2007 COLONIZAO DO BRASIL) Quando vocs falam que


foram mortos aproximadamente seis milhes de pessoas nos campos de
concentrao, das quais grande parte se sabe o nome e o dia da morte, ns,
indgenas, lembramos os quase seis milhes de irmos nossos exterminados
sem que se tenha, na maioria dos casos, qualquer informao sobre esses
massacres. Foi um extermnio silencioso e contnuo, que continua at hoje.
(Nailton Patax, numa visita a um campo de concentrao nazista na
Alemanha em 2000).
Considerando esse posicionamento sobre a populao indgena ocupante do
atual territrio brasileiro desde, pelo menos, 12.000 anos atrs, possvel
afirmar que:
I. dezenas de milhares de pessoas morreram em conseqncia do contato
direto e indireto com os europeus e as doenas por eles trazidas, a ponto de
podermos falar na existncia de um verdadeiro genocdio.
II. as sociedades indgenas, especialmente as que habitaram os espaos de
floresta, eram igualitrias e pacficas. Tais atributos eram indispensveis, pois
sem cooperao e com o baixo nvel tecnolgico que caracterizava essas
populaes, no seria possvel sobreviver nesses ambientes agressivos.
III.o espao ocupado pelo atual territrio brasileiro possuiu, e ainda possui,
uma imensa diversidade tnica. Esta diversidade refletiu-se, na poca da prconquista, numa grande variedade de formas de organizao social, que vem
sendo conhecida atravs dos estudos arqueolgicos.
Esto corretas as afirmativas:
a) II e III.
b) I e II.
c) I e III.
d) I, II e III.
e) nenhuma.
25) FURG 2007 COLONIZAO DO BRASIL) Na conquista e ocupao das
terras brasileiras, o estabelecimento de feitorias constituiu:
a) limites geogrficos das capitanias hereditrias.
b) entrepostos de trocas feitos pelos primeiros invasores, na primeira metade
do sculo XV.
c) necessidades policiadoras da metrpole na explorao do pau-brasil.
d) primeiras grandes fazendas de cultivo da cana-de-acar.
e) regies delimitadoras no serto, para pesquisa aurfera.

29) FURG 2007 REPBLICA VELHA) Sobre a industrializao no Brasil,


possvel afirmar que:
I. o Brasil faz parte de um grupo de pases de industrializao relativamente
tardia, que se inicia no final do sculo XIX.
II. durante a Repblica Velha, ocorreu o desenvolvimento das indstrias
txteis, com a utilizao da mo-de-obra imigrante, expandida, sobretudo,
com o fim da escravido em 1888.
III. durante a Primeira Guerra Mundial, houve um aumento da produo
industrial no Brasil, que resultou no processo de substituio de bens
importados.
Esto corretas as afirmativas:
a) I e II.
b) I e III.
c) II e III.
d) I, II e III.
e) nenhuma.
30) FURG 2007 GETLIO VARGAS) A grande preocupao do Estado Novo
foi a organizao dos trabalhadores brasileiros em sindicatos, que eram
caracterizados pela:
a) participao ativa do proletariado no contexto poltico.
b) rejeio do modelo fascista italiano.
c) atitude reivindicatria junto ao patronato.
d) oposio ditadura de Getlio Vargas.
e) superviso e controle governamental.
31) FURG 2007 DITADURA MILITAR) O Ato Institucional no 5, durante o
governo do General Costa e Silva, permitiu a esse presidente da Repblica,
entre outras medidas:
a) criar novos ministrios e empresas estatais.
b) decretar o recesso parlamentar e promover cassaes de mandatos e de
direitos polticos.
c) promover uma ampliao do sistema partidrio.
d) controlar os emprstimos no exterior.
e) convocar uma Assemblia Nacional

32) UFRGS GETLIO VARGAS) A respeito dos antecedentes da


denominada Revoluo de 1930, correto afirmar que
I - o governador Getlio Vargas, no Rio Grande do Sul, manteve o apoio ao
26) FURG 2007 BRASIL COLNIA) A casa-grande venceu no Brasil a governo federal, embora estivesse envolvido nas articulaes polticas que
Igreja, nos impulsos que esta a princpio manifestou para ser a dona da levaram criao da Aliana Liberal.
terra. Vencido o jesuta, o senhor de engenho ficou dominando a colnia II - existia na poltica nacional uma tradio de rotatividade na ocupao dos
quase sozinho. O verdadeiro dono do Brasil. Mais do que os vice-reis e os cargos do Executivo federal, o que permitia a representao de todas as
bispos. (Gilberto Freyre Casa Grande e Senzala. Rio de Janeiro: Editora regies do pas, sem favorecimento dos Estados mais fortes, como So Paulo
Jos Olympio.)
ou Minas Gerais.
No texto acima, so citadas caractersticas da:
III - o assassinato do poltico paraibano Joo Pessoa foi o fato catalisador
a) sociedade mineira.
que precipitou as articulaes dos golpistas, visando deposio do
b) sociedade cafeeira.
presidente Washington Lus.
c) sociedade urbana.
Quais esto corretas?
d) sociedade aucareira.
a) Apenas II.
d) Apenas II e III.
e) sociedade algodoeira.
b) Apenas I e II.
e) I, 11 e III.
c) Apenas I e III.

21

33) UFRGS REPBLICA POPULISTA) Assinale com V (verdadeiro) ou F


(falso) as seguintes afirmaes, relativas ao governo Kubitschek.
( ) O golpe preventivo do general Lott garantiu a sua posse ao abafar o
descontentamento de setores militares e polticos contrariados com os
resultados finais da eleio de 1955.
( ) Sua administrao foi marcada pelo lema "Cinqenta anos de progresso
em cinco de governo" e pela interiorizao do governo central com a
construo de Braslia.
( ) Sua poltica econmica foi delineada no Plano de Metas, ambicioso
programa que apontava para o desenvolvimento das indstrias de base e dos
setores energtico, de transporte, alimentar e educacional.
( ) Ele promoveu ampla atividade do Estado no setor de infra-estrutura e no
incentivo industrializao, mas tambm desenvolveu aes para atrair
capitais estrangeiros.
( ) Ele criou o Grupo Executivo da Indstria Automobilstica (GEIA), com o
intuito estratgico de alavancar a produo de automveis e caminhes com
capitais privados.
A seqncia correta de preenchimento dos parnteses, de cima para baixo,
a) V-F-V-F-F.
c) V-F-F-V-V.
e) V-V-V-V-V.
b) F-V-F-V-F.
d) F-V-V-F-V.
34) UFRGS REPBLICA POPULISTA) Considere as seguintes afirmaes,
referentes ao perodo de governo de Leonel Brizola no Rio Grande do Sul
(1959-1963).
I - Durante a crise de 1961, deflagrada com a renncia de Jnio Quadros, o
governador Brizola liderou o chamado "Movimento da Legalidade", que
acabou assegurando a posse de Joo Goulart na presidncia da Repblica.
II - Brizola promoveu um ciclo de privatizaes na economia gacha,
vendendo empresas estatais ineficientes, em especial nos setores de energia
eltrica e telefonia.
III- Brizola realizou a primeira iniciativa de reforma agrria no Estado,
quando assentou camponeses ligados ao MASTER (Movimento dos
Agricultores Sem Terra), nas terras do Banhado do Colgio, em Camaqu.
Quais esto corretas?
a) Apenas I.
b) Apenas I e II.
c) Apenas I e III.
d) Apenas II e III.
e) I, II e III.
35) FURG DITADURA MILITAR) Foram caractersticas da formao histrica
brasileira poca dos Governos Militares:
I a prtica de uma poltica externa independente, como alternativa ao
atrelamento aos Estados Unidos;
II o emprego da doutrina de segurana nacional para legitimar os atos dos
governantes;
III a concentrao de poderes nas mos do Executivo;
IV a inexistncia de uma Constituio Federal
V o ideal de desenvolvimento econmico expresso no chamado milagre
brasileiro;
VI a utilizao do Ministrio do Trabalho para cooptao do operariado
atravs de prticas populistas;
Esto corretas:
a) I, II e III.
c) II, III e V.
e) IV, V e VI.
b) II, III e IV.
d) III, IV e VI.
36) UNIFRA DITADURA MILITAR) O ano de 1970 marcou a histria do
Brasil com dois momentos muito diferentes. De um lado, o pas rejubilava-se
com a conquista do tri-campeonato mundial de futebol. De outro, sofria com
o perodo mais implacvel e terrvel da represso do regime militar. Assinale
a alternativa em que esto presentes as caractersticas principais deste
perodo.
a) O governo do Presidente Mdici foi marcado pela violncia, pela censura e
pela tortura. O pretexto era a forte luta armada contra o regime e que
contava com amplo apoio da sociedade.

b) A guerrilha teve sucesso no interior do pas e nas reas urbanas, pois o


processo de urbanizao do pas integrava-se, naquele momento, ao mundo
rural.
c) A resistncia ao regime encontrou pouco ou nenhum respaldo nos diversos
setores da sociedade que se encontravam seduzidos pelo chamado milagre
brasileiro na economia, e eram constantemente alarmados com a
propaganda anticomunista que fazia do regime um defensor das
instituies democrticas.
d) As universidades nada sofreram com a represso, apenas alguns
professores foram cassados, mas, no geral, estas prosseguiram livremente
um ensino cada vez mais democrtico.
e) Em comparao com os outros regimes militares da Amrica Latina, os
chamados Anos de Chumbo, no Brasil, foram bem amenos. Houve poucos
desaparecidos e o regime nunca chegou a matar.
37) UNIFRA DITADURA MILITAR)
"H soldados armados.
Amados ou no.,
Quase todos perdidos,
De armas na mo.
Nos quartis lhes ensinam uma antiga lio.
De morrer pela ptria
E viver sem razo..." (Geraldo Vandr)
O trecho acima, parte da composio Para no dizer que no falei das flores,
um representante do movimento conhecido como tropicalismo que integra
o conjunto de manifestaes contra o regime militar, das dcadas de 60 e 70,
no Brasil. Considerando este perodo, assinale (V) para verdadeiro e (F) para
falso.
( ) A partir de maro de 1964, os interesses polticos brasileiros fundiram-se
em dois blocos, representados por dois partidos, a Arena e o MDB, o que
demonstrou uma forte conciliao entre sociedade civil e governo militar.
( ) O governo de Castelo Branco comprometeu-se com o crescimento
econmico do pas, para isto colocou em prtica um projeto de fomento
indstria nacional e de controle rgido das importaes.
( ) O governo Costa e Silva, atravs do Ato Institucional de nmero 5 - o AI-5
- concedeu amplos poderes aos presidentes militares, entre estes, o de
fechar o Congresso nacional.
( ) Com o General Emlio Garrastazu Mdici, a tendncia assumida pelo
governo foi de assegurar para o Estado o controle dos setores bsicos da
economia, como a produo de energia, enquanto que as exportaes eram
o atrativo principal para os setores privados brasileiros.
A seqncia correta :
a) F V F V.
d) F F V V.
b) F F V F.
e) V V F F.
c) V V V F.

1) E
2) B
3) E
4) B
5) B
6) D
7) E
8) ANLDA
9) 13
10) E

11)
12)
13)
14)
15)
16)
17)
18)
19)
20)

C
A
A
D
B
E
C
A
D
E

21)
22)
23)
24)
25)
26)
27)
28)
29)
30)

C
E
B
C
C
D
A
C
D
E

31) B
32) C
33) E
34) C
35) C
36) A
37) D

22

1) Toms

Antnio Gonzaga escreveu Marlia de Dirceu, um dos mais


conhecidos poemas de nosso Arcadismo. Leia duas estrofes da Lira I, da
primeira parte do poema.
Eu, Marlia, no sou algum vaqueiro
Que viva de guardar alheio gado,
De tosco trato, d`expresses grosseiro,
Dos frios gelos e dos sis queimado.
Tenho prprio casal e nele assisto;
D-me vinho, legume, fruta, azeite;
Das brancas ovelhinhas tiro o leite
E mais as finas ls, de que me visto.
Graas, Marlia bela,
Graas minha estrela!
-----------------------------------------Irs a divertir-te na floresta,
Sustentada, Marlia, no meu brao;
Aqui descansarei a quente sesta,
Dormindo um leve sono em teu regao;
Enquanto a luta jogam os pastores,
E emparelhados correm nas campinas,
Toucarei teus cabelos de boninas,
Nos troncos gravarei os teus louvores.
Graas, Marlia bela,
Graas minha estrela!
(GONZAGA, Toms Antnio. Marlia de Dirceu.
In: Literatura comentada. So Paulo: Abril
Cultural, 1980).
correto afirmar sobre as estrofes:
a) Ilustram no s preferncias temticas do Arcadismo, como o ideal de vida
simples, o heri que se faz pela honradez e pelo trabalho, mas tambm o
sentimento de transitoriedade da vida, que arrasta o poeta ao carpe diem
(aproveitar a vida) horaciano.
b) Representam os temas do bucolismo, do fugere urbem (fuga da cidade),
da urea mediocritas (existncia dentro da mediania), alm de se
caracterizarem pela linguagem elevada, repleta de inverses sintticas e
adornada por figuras de estilo como antteses e paradoxos.
c) Apresentam a fina ironia de Gonzaga, o que liga o poema s Cartas
Chilenas, escritas pelo autor, depois que estava preso, para ridicularizar o
Visconde de Barbacena, feroz inimigo dos Inconfidentes.
d) Reiteram o nome de Marlia nos estribilhos, mas, ao contrrio do que pode
parecer, esse recurso no imprime musicalidade ao texto, serve apenas para
enaltecer a mulher, vista, naquele momento, como elemento anglico,
divinal, e inatingvel, o que faz a potica de Gonzaga preceder o Romantismo.
e) Ilustram tpicos preferenciais do Arcadismo, como o locus amenus (lugar
ameno), o ideal de vida elevada materialmente, a pintura de cenas pastoris,
e revelam os valores da nobreza de ento, favorvel ao despotismo da Coroa
Portuguesa.

2)

Marlia de Dirceu Toms Antonio Gonzaga


Primeira Parte
Lira I
-----------------------------------------Os teus olhos espalham luz divina,
A quem a luz do sol em vo se atreve;
Papoila ou rosa delicada e fina
Te cobre as faces, que so cor da neve.
Os teus cabelos so uns fios, douro;
Teu lindo corpo blsamos vapora.
Ah! no, no fez o Cu, gentil Pastora,
Para glria de amor igual Tesouro!
Graas, Marlia bela,
Graas minha Estrela!
(blsamos vapora = exala perfumes)
Segunda Parte
Lira XV
------------------------------------------Se o rio levantado me causava,
Levando a sementeira, prejuzo,
Eu alegre ficava, apenas via
Na tua breve boca um ar de riso.
Tudo agora perdi; nem tenho o gosto
De ver-te ao menos compassivo o rosto.
Em relao compreenso dos dois fragmentos e compreenso global da
obra "Marlia de Dirceu", assinale a alternativa correta.
a) A descrio de Marlia real, o retrato fiel da mulher daquela poca.
b) O tom de felicidade e otimismo permeia os dois fragmentos e constitui-se
caracterstica da obra toda.

c) O "eu-lrico", na primeira parte da obra, mostra-se fragilizado pela


incerteza de seu destino.
d) Na segunda parte da obra, relacionada priso do poeta, o eu-lrico revela
um sentimento de perda.
e) Embora seja considerado Romntico, Gonzaga explora imagens rcades
como o bucolismo em uma poesia metrificada, porm, sem rimas.

3)

Texto I
a vaidade, Fbio, nesta vida Rosa que, da manh lisonjeada, Prpuras
mil, com ambio dourada, Airosa rompe, arrasta presumida.
Texto II
Fatigado de calma, se acolhia Junto o rebanho sombra dos salgueiros, E o
sol, queimando os speros oiteiros, Com violncia maior no campo ardia.
A natureza, para os poetas ___________era fonte de smbolos (rosa, cristal,
gua), que transcendiam do material para o espiritual (Texto I); para os
poetas ___________era sobretudo o cenrio idealizado, dentro do qual se
podia ser feliz (Texto II).
a) romnticos parnasianos
b) parnaianos simbolistas
c) rcades romnticos
d) simbolistas barrocos
e) barrocos rcades

4) A

pastora Manilia, conforme nos apresentada nas liras de Toms


Antnio Gonzaga, carece de unidade de enfoques; por isso muito difcil
precisar, por exemplo, seu tipo fsico. Esta impreciso da pastora:
a) suficiente para seu autor ser apontado como pr-romntico.
b) fundamental para situar o leitor dentro do drama amoroso do autor.
c) reflete o carter genrico e impessoal que a poesia neoclssica deveria
assumir.
d) responsvel pela atmosfera de mistrio, essencial para a poesia
neoclssica.
e) mostra a inteno do autor em no revelar o objeto do seu amor.

5) Que

diversas que so, Manilha, as horas Que passo na masmorra


imunda, e feia, Dessas horas felizes j passadas Na tua ptria aldeia!.
Esses versos de Marlia de Dirceu caracterizam:
a) a primeira parte da obra, o estado de paixo
b) a felicidade da futura famlia
c) a segunda parte da obra, a priso
d) o sonho de realizar a Inconfidncia
e) o sonho de um futuro feliz ao lado da amada

6) Assinale o que no se refere ao Arcadismo:

a) Volta aos princpios clssicos greco-romanos e renascentistas (o belo, o


bem, a verdade, a perfeio, a imitao da natureza)
b) poca do Iluminismo (sculo XVIII) Racionalismo, clareza, simplicidade
c) Apia-se em temas clssicos e tem como lema: inutilia truncat (corta o
que intil)
d) Ornamentao estilstica, predomnio da ordem inversa, excesso de figuras
e) Pastoralismo, bucolismo suaves idlios campestres

7) Considerando

a obra Marlia de Dirceu, de Toms Antnio Gonzaga,


assinale a opo INCORRETA.
a) As liras que compem essa obra recontam a paixo do poeta pela jovem
Maria Dorotia Joaquina de Seixas Brando, a Marlia.
b) Dirceu (pseudnimo rcade do poeta) descreve a amada Marlia e fala
sobre a vida futura que tero quando casados, fato que se concretiza no final
da histria. Em seus contos, faz uma crtica violenta aos preconceitos sociais
e decadncia dos costumes da burguesia.
c) Lembrado e celebrado at hoje pela publicao de Marlia de Dirceu,
Toms Antnio Gonzaga um dos mais conhecidos entre os poetas do
arcadismo brasileiro.
d) Na obra, Gonzaga se posiciona como um abastado pastor que cultiva o
ideal da vida campestre, vive intensamente o momento (carpe diem) e pinta,
por meio de palavras, a natureza e Marlia, a mulher amada.
e) Antnio Cndido, crtico literrio, diz que o melhor ttulo para a obra seria
Dirceu de Marlia, mas o patriarcalismo de Gonzaga jamais lhe permitiria
colocar-se como a coisa possuda

8) Assinale a alternativa correta.

A poesia brasileira do Romantismo do sculo XIX pode ser dividida em:


a) duas fases: a histrica e indianista, e a fase subjetiva e individualista.
b) trs fases: a subjetiva, a nacionalista e a experimental.
c) trs fases: a poesia da natureza e indianista, a poesia individualista e
subjetiva, e a poesia liberal e social.
d) quatro fases: a histrica, a de crtica nacionalista, a experimental e a
subjetiva.

23

e) duas fases: a amorosa e sentimental e a fase nacionalista.

9) Em

termos didticos, o ano de 1836 marcou o inicio do Romantismo


Brasileiro. Qual a primeira obra brasileira tipicamente romntica publicada na
ocasio:
a) Suspiros poticos e saudades, de Gonalves de Magalhes.
b) Primeiros cantos de Gonalves Dias.
c) Lira dos vinte anos de lvares de Azevedo.
d) Espumas flutuantes de Castro Alves.
e) Vozes da Amrica de Fagundes Varela.

10) No prlogo de Primeiros Cantos, ao dizer, sobre as poesias, Escrevi-as

para mim, e no para os outros, Gonalves Dias incorre um paradoxo. Que


trao romntico se evidencia por essa contradio?
a) O individualismo, a exposio do eu numa perspectiva essencialmente
romntica.
b) A grandiloqncia to evidenciada pelos romnticos.
c) A liberdade de expresso, smbolo marcante dessa gerao.
d) O universo como forma de perpetuao do mais exuberante e imortal
e) A preferncia pelo nacionalismo como forma de sentir-se parte da
coletividade.

11) Gonalves

Dias consolidou o romantismo no Brasil. Sua Cano do


exlio pode ser considerada tipicamente romntica porque:
a) apia-se nos cnones formais da poesia clssica grecoromana; emprega
figuras de ornamento, at com certo exagero; evidencia a musicalidade do
verso pelo uso de aliteraes.
b) exalta terra natal; nostlgica e saudosista; o tema tratado de modo
sentimental, emotivo.
c) utiliza-se do verso livre, como ideal de liberdade criativa; sua linguagem
hermtica, erudita; glorifica o canto dos pssaros e a vida selvagem.
d) poesia e msica se confundem, como artifcio simblico; a natureza e o
tema buclico so tratados com objetividade; usa com parcimnia as formas
pronominais de primeira pessoa.
e) refere-se vida com descrena e tristeza; expe o tema na ordem
sucessiva, cronolgica; utiliza-se do exlio como o meio adequado de referirse evaso da realidade.

12) Considerando-se a obra Primeiros Cantos, de Gonalves Dias, assinale a

alternativa INCORRETA:
a) O poeta chamou de Poesias americanas todo o conjunto de seus poemas
indianistas e nacionalistas, dentre os quais destacam-se Cano do Exlio e O
Canto do Guerreiro.
b) Seus versos indianistas por vezes desenham um ndio de sentimentos e
atitudes artificiais, extremamente europeizado.
c) De modo geral, os versos de Gonalves Dias caracterizam-se pela perfeita
utilizao dos vrios recursos da mtrica, da musicalidade e do ritmo.
d) Os poemas amorosos so marcados pela dor e sofrimento, chegando a
beirar o ultra-romantismo em alguns momentos.
e) O nacionalismo do autor faz com que traos comuns ao Romantismo,
como o medievalismo e a religiosidade sejam negaceados em sua obra.

13) Leia as afirmaes que seguem, sobre poemas constantes em Primeiros

Cantos, e assinale a nica alternativa correta:


a) As figuras monstruosas que aparecem no Canto do Piaga simbolizam sinais
que predizem o extermnio das naes selvagens com a chegada de
estrangeiros (que vm pelo mar) para matar guerreiros, roubar as mulheres
e escravizar os heris.
b) Em Deprecao temos a revolta de um ndio contra o deus Tup, que teria
permitido a vitria do invasor branco contra os indgenas, constituindo-se
uma crtica ao processo de colonizao.
c) Em Cano do Exlio, num ritmo marcado pelo uso da redondilha menor, o
poeta traa uma viso nacionalista que privilegia o tipo humano brasileiro.
d) Em O Canto do Guerreiro o eu-lrico indgena dirige-se a uma platia (ouvime, Guerreiros) para falar de sua bravura, de seu trabalho (caa e guerra) e
de sua superioridade sobre os inimigos.
e) No poema Caxias, constante em Poesias Americanas, o poeta exalta sua
cidade natal, no Maranho

14)

Ossian o bardo triste como a sombra


Que seus cantos povoa. O Lamartine
montono e belo como a noite,
Como a lua no mar e o som das ondas...
Mas pranteia uma eterna monodia,
Tem na lira do gnio uma s corda;
Fibra de amor e Deus que um sopro agita:
Se desmaia de amor a Deus se volta,
Se pranteia por Deus de amor suspira.
Basta de Shakespeare. Vem tu agora,
Fantstico alemo, poeta ardente

Que ilumina o claro das gotas plidas


Do nobre Johannisberg! Nos teus romances
Meu corao deleita-se... Contudo,
Parece-me que vou perdendo o gosto.
(...)
(lvares de Azevedo, Lira dos vinte anos)
Considerando-se este excerto no contexto do poema a que pertence (Idias
ntimas), correto afirmar que, nele,
a) o eu-lrico manifesta tanto seu apreo quanto sua insatisfao em relao
aos escritores que evoca.
b) a disperso do eu-lrico, prpria da ironia romntica, exprime-se na
mtrica irregular dos versos.
c) o eu-lrico rejeita a literatura e os demais poetas porque se identifica
inteiramente com a natureza.
d) a recusa dos autores estrangeiros manifesta o projeto nacionalista tpico
da segunda gerao romntica brasileira.
e) Lamartine criticado por sua irreverncia para com Deus e a religio,
muito respeitados pela segunda gerao romntica.

15)

Oh! ter vinte anos sem gozar de leve


A ventura de uma alma de donzela!
E sem na vida ter sentido nunca
Na suave atrao de um rseo corpo
Meus olhos turvos se fechar de gozo!
Oh! nos meus sonhos, pelas noites minhas
Passam tantas vises sobre meu peito!
Palor de febre meu semblante cobre,
Bate meu corao com tanto fogo!
Um doce nome os lbios meus suspiram,
Um nome de mulher... e vejo lnguida
No vu suave de amorosas sombras
Seminua, abatida, a mo no seio,
Perfumada viso romper a nuvem,
Sentar-se junto a mim, nas minhas plpebras
O alento fresco e leve como a vida
Passar delicioso... Que delrios!
Acordo palpitante... inda a procuro;
Embalde a chamo, embalde as minhas lgrimas
Banham meus olhos, e suspiro e gemo...
Imploro uma iluso... tudo silncio!
S o leito deserto, a sala muda!
Amorosa viso, mulher dos sonhos,
Eu sou to infeliz, eu sofro tanto!
Nunca virs iluminar meu peito
Com um raio de luz desses teus olhos?
Os versos acima integram a obra Lira dos Vinte Anos, de lvares de Azevedo.
Da leitura deles podemos depreender que o poema:
a) ilustra a dificuldade de conciliar a idia de amor com a de posse fsica.
b) manisfesta o desejo de amar e a realizao amorosa se d concretamente
em imagens de sonho.
c) concilia sonho e realidade e ambos se alimentam da presena sensual da
mulher amada.
d) espiritualiza a mulher e a apresenta em recatado pudor sob vu suave de
amorosas sombras.
e) revela sentimento de frustrao provocado pelo medo de amar e pela
recusa doentia e deliberada entrega amorosa.

16) O

fragmento do poema abaixo pertence segunda parte da obra Lira


dos vinte anos, de lvares de Azevedo. Leia-o, analise as afirmativas que o
seguem e assinale a alternativa correta.
ela! ela! ela! ela!
ela! ela! murmurei tremendo,
E o eco ao longe murmurou ela!
Eu a vi minha fada area e pura
A minha lavadeira na janela!
[...]
Esta noite eu ousei mais atrevido
Nas telhas que estalavam nos meus passos
Ir espiar seu venturoso sono,
V-la mais bela de Morfeu nos braos!
[...]
Afastei a janela, entrei medroso:
Palpitava-lhe o seio adormecido...
Fui beij-la... roubei do seio dela
Um bilhete que estava ali metido...
Oh! Decerto... (pensei) doce pgina
Onde a alma derramou gentis amores;
So versos dela... que amanh decerto

24

Ela me enviar cheios de flores...


[...]
ela! ela! repeti tremendo;
Mas cantou nesse instante uma coruja...
Abri cioso a pgina secreta...
Oh! Meu Deus! era um rol de roupa suja!
a) O tema da mulher idealizada constante na obra de lvares de Azevedo.
No poema em questo, a imagem da virgem sonhadora simbolizada pela
lavadeira, uma forma de denunciar os problemas sociais e, ao mesmo tempo,
reportar a imagem feminina ao modelo materno.
b) No poema " ela! ela! ela! ela!", a musa eleita uma lavadeira.
Dizendo-se apaixonado, o eu-lrico a observa enquanto dorme e retira do seio
da amada uma lista de roupa, que imaginara ser um bilhete de amor. Tratase de uma forma melanclica de expressar a grandeza das relaes humanas
e representar a concretizao do amor.
c) O emprego de termos elevados em referncia lavadeira, tais como "fada
area e pura", um fator que refora o riso por associar a lavadeira a uma
musa inspiradora e exaltadora da paixo. Trata-se, portanto, de um poema
de linha irnica e prosaica, que revela os valores morais daquela poca.
d) O poema, no conjunto das estrofes acima transcritas, revela tdio e
melancolia. Esses sentimentos so reforados pelo murmrio do eu-lrico, "
ela! ela!", ao visualizar sua amada.
e) A figura da lavadeira no poema a de uma mulher que no se pode
possuir. Dessa maneira, o poema afasta a possibilidade de concretizao do
ato sexual, confirmando a idealizao da mulher no perodo romntico.

a) V, V, V, V
b) V, F, F, V
c) F, F, V, F
d) F, V, V, F
e) F, F, V, V

19) Dentre as descries, a seguir, a partir da trama romntica de Noite na

taverna, de lvares de Azevedo, assinale V ou F:


( ) A primeira parte da narrativa apresenta a cena que emoldura todas as
demais partes:
embriagados, vrios amigos, em uma taverna, discutem filosofia e poesia, at
que passam a contar histrias uma lembrana do passado de cada um
que, na definio que apresentam, semelham aos contos fantsticos de
Hoffmann.
( ) A saciedade um tdio terrvel, afirma Bertram, que entre diversas
aventuras amorosas rocambolescas, narra ter sido recebido no palcio de
um nobre velho vivo e uma beleza peregrina de dezoito anos; depois de
desonr-la, foge com a moa, vende-a para o pirata
Siegfried, a quem a moa envenena antes de se afogar.
( ) Nauza a jovem mulher de Godofredo, mestre de Gennaro; ela e o
aluno se apaixonam, mas Gennaro se envolve com Laura, virginal filha de
Godofredo; Laura, grvida, morre, e enquanto Godofredo chora, Gennaro e
Nauza se amam; Godofredo atrai Gennaro para uma cilada, atira-o em um
despenhadeiro, mas ele, por milagre, se salva; retornando casa do mestre,
encontra Godofredo e Nauza envenenados.
( ) Claudius Hermann narrador de sua prpria histria rouba ao Duque
Maffio a mulher, Eleonora; certo dia, ao voltar para casa, Hermann depara-se
17) Leia o texto:
com o leito ensopado de sangue e num recanto escuro da alcova um doido
Nessa torrente negra que se chama a vida, e que corre para o passado abraado com um cadver: o Duque os encontrara e matara Eleonora.
enquanto ns caminhamos para o futuro, tambm desfolhei muitas crenas, ( ) Johann, em duelo, acerta Arthur com um tiro queima-roupa, e vai a um
e lancei despidas as minhas roupas mais perfumadas, para trajar a tnica da encontro amoroso no lugar do oponente. Descoberto no idlio, mata um
Saturnal! O passado o que foi, a flor que
vulto que veio proteger a moa.
murchou, o sol que se apagou, o cadver que apodreceu. Lgrimas a ele? Descobre que matou o prprio irmo e que a virgem que lhe propiciara
fora loucura! Que durma, e que durma com suas lembranas negras! uma noite deliciosa era sua irm. Termina sua histria e todos dormem.
revivam: acordem apenas os miostis abertos, naquele pntano! Sobreage Chega taverna sua irm, Giorgia, que se vinga, matando-o com uma
naquele no-ser o eflvio de alguma lembrana pura!
punhalada. Descobrimos ento que o jovem Arthur continua vivo e Arnold,
Bravo! Bravssimo! Claudius, ests completamente bbedo! bof que ests que participava da orgia na taverna. Giorgia se mata e Arnold, beijando-a,
romntico!
crava um
Silncio, Bertram! certo que esta no uma lenda para inscrever-se aps punhal no prprio peito.
das vossas:
A seqncia correta :
uma dessas coisas que se contem com os cotovelos na toalha vermelha, e os a) V, V, V, V
b) V, F, F, V
lbios borrifados de vinho e saciados de beijos... Mas que importa?
c) F, F, V, F
d) F, V, V, F
Vs todos, que amais o jogo, que vistes um dia correr naquele abismo uma e) F, F, V, V
onda de ouro redemoinhar-lhe no fundo, como um mar de esperanas que
se embate na ressaca do acaso, sabeis melhor que vertigem nos tonteia 20)
ento: ideais melhor a loucura que nos delira naqueles jogos de milhares de Senhor Deus dos desgraados!
homens, onde fortuna, aspiraes, a vida mesma vo-se na rapidez, de uma Dizei-me vs, Senhor Deus!
corrida, onde todo esse complexo de misrias e desejos, de crimes e virtudes Se loucura... se verdade
que se chama a existncia se joga numa parelha de cavalos!
Tanto horror perante os cus...
AZEVEDO, lvares de. Noite na taverna. Porto Alegre: L&PM, 2006. p. 57. mar! Por que no apagas
(Coleo L&PM Pocket) Coa esponja de tuas vagas
Analise as afirmaes que seguem:
Do teu manto este borro?
I - No Romantismo, o artista traz tona o seu mundo interior, com plena Astros! Noite! Tempestades!
liberdade. Isso se faz presente na obra Noite na Taverna, em que cada Rolai das imensidades!
narrador-personagem revela a sua trajetria de vida marcada por amores Varrei os mares, tufo!
frustrados ou proibidos, pelo sofrimento e pela presena freqente da morte.
Castro Alves, Navio Negreiro, in: Os Escravos
II - Claudius Hermann tem uma viso espiritualizada do amor, o que no lhe Tendo em vista seu autor, o perodo literrio a que pertence o fragmento
permite toc-la, apenas contemplar sua beleza ao longo de toda a narrativa.
reproduzido, assinale a opo que contenha informao errada:
III - Na viso de Hermann, a vida e o amor so indissociveis e devem ser a) poema com caractersticas condoreiras, com imagens grandiosas da
vividos intensamente, mesmo que de forma passageira como uma corrida de natureza e linguagem grandiloqente.
cavalos.
b) As exclamaes e utilizao de figuras de linguagem so tpicas da poesia
Esto corretas:
social do autor.
a) apenas I
c) A indignao com a escravido fazem do poema um paradigma
b) I e II
abolicionista do sc. XIX.
c) II e III
d) O lirismo subjetivo, abordando questes coletivas, lembra a
d) I e III
sentimentalidade amena tpica do Romantismo.
e) todas
e) A invocao de elementos da natureza confirma o fundamento romntico
da identificao do eu-lrico com a paisagem que o cerca.
18) Os relatos emoldurados em Noite na taverna, de lvares de Azevedo,
so, todos, construdos a partir de uma oposio entre amor e morte. 21) Leia os versos de Navio Negreiro:
Assinale V ou F.
So os filhos do deserto
( ) No relato de Bertram, o protagonista se apaixona por Claudius Hermann, Onde a terra esposa a luz.
o amor homossexual terminando com os amantes condenados, pela Onde voa em campo aberto
Inquisio, morte na fogueira.
A tribo dos homens nus...
( ) No relato de Johann, o heri que narra a sua histria informa aos So os guerreiros ousados,
ouvintes de como, em um acesso de cimes, matou a sua amante, ngela.
Que com os tigres mosqueados
( ) Giorgia, que surge como amante virgem, em relato narrado por um dos Combatem na solido...
convivas que desfiam aventuras na taverna, ressurge ao final para vingar-se Homens simples, fortes, bravos...
da desonra, suicidando-se em seguida.
Hoje mseros escravos
( ) O relato de Solfieri mescla aventuras amorosas com fuga de piratas e Sem ar, sem luz, sem razo...
expedio ao extremo Oriente.
Assinale a alternativa correta com relao ao sentido expresso pelos versos
A seqncia correta :
transcritos:

25

a) Descreve a vida dos escravos nas fazendas.


b) Sada a liberdade decorrente da abolio da escravatura.
c) Salienta a integrao dos negros com os ndios.
d) Compara o negro livre, na frica, com o negro escravizado no Brasil.
e) Prope que os homens se tornem escravos por quererem fugir do deserto.

22) Leia

o trecho a seguir, extrado do poema Navio Negreiro, de Castro


Alves, e marque o que CORRETO sobre o poema:
Era um sonho dantesco... o tombadilho
E ri-se a orquestra irnica, estridente...
Que das luzernas avermelha o brilho.
E da ronda fantstica a serpente
Em sangue a se banhar.
Faz doudas espirais ...
Tinir de ferros... estalar de aoite...
Se o velho arqueja, se no cho resvala,
Legies de homens negros como a noite,
Ouvem-se gritos... o chicote estala.
Horrendos a danar...
E voam mais e mais...
Negras mulheres, suspendendo s tetas
Presa nos elos de uma s cadeia,
Magras crianas, cujas bocas pretas
A multido faminta cambaleia,
Rega o sangue das mes:
E chora e dana ali!
Outras moas, mas nuas e espantadas,
Um de raiva delira, outro enlouquece,
No turbilho de espectros arrastadas,
Outro, que martrios embrutece,
Em nsia e mgoa vs!
Cantando, geme e ri!
a) Navio Negreiro um poema barroco, o que pode ser percebido pelo
tema da morte.
b) Trata-se de um poema da primeira fase do Romantismo, quando prevalece
um nacionalismo ingnuo.
c) Percebe-se claramente que o poema pertence segunda fase do
Romantismo, quando se pratica uma poesia pessimista e voltada para a
morte.
d) Este poema, escrito na terceira fase do romantismo, faz crtica social,
porque se manifesta contra a escravido.
e) Navio Negreiro no pode ser classificado como um poema do
Romantismo, porque no idealiza, mostra a realidade nua e crua.

23) Com relao obra Iracema, de Jos de Alencar, assinale a alternativa

INCORRETA.
a) A personagem Martim aproxima-se dos princpios considerados corretos no
modelo colonizador.
b) Iracema valorizada e caracterizada com atributos fsicos concebidos pela
natureza selvagem.
c) A herona mistura da idia de pureza e de sensualidade.
d) O sentido fabular estabelece contrastes entre a civilizao e o novo
mundo.
e) A ndia, por ser recriao do nacional, apresenta traos da realidade vivida
no sculo XVIII

24) Os comentrios abaixo referem-se Iracema, exceto:

a) A histria de amor foi tragada pelo tempo (Tudo passa sobre a terra.),
enquanto o discurso pede passagem para debater a questo da
nacionalidade. (Eliana Yunes)
b) Neste romance, Alencar tenta reconstruir a vida e os costumes dos ndios
brasileiros, antes do contato com o colonizador branco. (Ulisses Infante)
c) H de viver este livro, tem em si as foras que resistem ao tempo e do
plena fiana do futuro... Espera-se dele outros poemas em prosa. Poema lhe
chamamos a este, sem curar de saber se antes uma lenda, se um romance:
o futuro chamar-lhe- obra-prima. (Machado de Assis)
d) Um crtico j observou que Iracema anagrama de Amrica, isto ,
Iracema tem exatamente as mesmas letras de Amrica, s que em outra
ordem. Assim, simbolicamente, a morte da ndia, no final da estria, pode
representar a aniquilao da cultura nativa pela invasora que conquista e
domina a terra. (Literatura Comentada, Editora Abril)

25) O

trecho abaixo foi extrado de Iracema. Ele reproduz a reao e as


ltimas palavras de Batuiret antes de morrer:
"O velho soabriu as pesadas plpebras, e passou do neto ao estrangeiro um
olhar bao.
Depois o peito arquejou e os lbios murmuraram:
Tup quis que estes olhos vissem antes de se apagarem, o gavio branco
junto da narceja. O abaet derrubou a fronte aos peitos, e no falou mais,
nem mais se moveu."

(Jos de Alencar, Iracema: lenda do Cear. Rio de Janeiro: MEC/INL, 1965,


p. 171-172.)
Analise as afirmaes:
I- Batuiret era o lder dos potiguaras (pitiguaras). Fora grande guerreiro,
mas, nessa cena do romance, j se mostrava impossibilitado de continuar a
guerrear, devido idade avanada.
II- Batuiret se dirige ao neto e ao estrangeiro, sendo o primeiro Poti e o
segundo, Martim Soares Moreno. Poti um dos valorosos guerreiros da tribo
potiguara, aliada dos portugueses. Essa aliana aparece bem representada
pela amizade que une Poti e Martim (guerreiro j no nome, derivado de
Marte), por quem se apaixona a herona da histria, a virgem de Tup.
III- A metfora gavio branco junto da narceja empregada de modo a
profetizar a destruio dos ndios que ser promovida pelo estrangeiro,
colonizador. Em nota ao romance, o prprio autor-narrador trata de
esclarecer que o gavio se refere ao homem branco, Martim, e a narceja
(espcie de ave tpica do continente sulamericano), ao ndio Poti. Por meio da
relao predatria entre a ave de rapina e sua presa, a imagem metafrica
busca simbolizar a dominao e posterior destruio da populao indgena
pelo colonizador.
Esto corretas:
a) apenas I
b) I e II
c) II e III
d) I e III
e) todas

26) A

prxima questo refere-se ao texto a seguir, extrado do sexto


captulo de Quincas Borba (1892), de Machado de Assis (1839-1908).
Supe tu um campo de batatas e duas tribos famintas. As batatas apenas
chegam para alimentar uma das tribos, que assim adquire foras para
transpor a montanha e ir outra vertente, onde h batatas em abundncia;
mas, se as duas tribos dividem em paz as batatas do campo, no chegam a
nutrir-se suficientemente e morrem de inanio. A paz, nesse caso, a
destruio; a guerra a conservao. Uma das tribos extermina a outra e
recolhe os despojos. Da a alegria da vitria, os hinos, aclamaes,
recompensas pblicas e todos os demais efeitos das aes blicas. Se a
guerra no fosse isso, tais demonstraes no chegariam a dar-se, pelo
motivo real de que o homem s comemora e ama o que lhe aprazvel ou
vantajoso, e pelo motivo racional de que nenhuma pessoa canoniza uma
ao que virtualmente a destri. Ao vencido, dio ou compaixo; ao
vencedor, as batatas.
(ASSIS, Joaquim Maria Machado de. Quincas Borba. Rio de Janeiro: Nova
Aguilar, 1997. p. 648-649.)
O Humanitismo, filosofia criada por Quincas Borba, revelador:
a) Do posicionamento crtico de Machado de Assis aos muitos ismos
surgidos no sculo XIX: darwinismo, positivismo, evolucionismo.
b) Da admirao de Machado de Assis pelos muitos ismos surgidos no incio
do sculo XX: futurismo, impressionismo, dadasmo.
c) Da capacidade de Machado de Assis em antever os muitos ismos que
surgiriam no sculo XIX: darwinismo, positivismo, evolucionismo.
d) Da preocupao didtica de Machado de Assis com a transmisso de
conhecimentos filosficos consolidados na poca.
e) Da competncia de Machado de Assis em antecipar a esttica surrealista
surgida no sculo XX.

27) Leia o texto para responder prxima questo.

Captulo CC
Poucos dias depois, [Rubio] morreu... No morreu sbdito nem vencido.
Antes de principiar a agonia, que foi curta, ps a coroa na cabea, uma
coroa que no era, ao menos, um chapu velho ou uma bacia, onde os
espectadores palpassem a iluso. No, senhor; ele pegou em nada, levantou
nada e cingiu nada; s ele via a insgnia imperial, pesada de ouro, rtila de
brilhantes e outras pedras preciosas. O esforo que fizera para erguer meio
corpo no durou muito; o corpo caiu outra vez; o rosto conservou porventura
uma expresso gloriosa.
Guardem a minha coroa, murmurou. Ao vencedor...
A cara ficou sria porque a morte sria; dous minutos de agonia, um
trejeito horrvel, e estava assinada a abdicao.
Capitulo CCI
Queria dizer aqui o fim do Quincas Borba, que adoeceu tambm, ganiu
infinitamente, fugiu desvairado em busca do dono, e amanheceu morto na
rua, trs dias depois. Mas, vendo a morte do co narrada em captulo
especial, provvel que me perguntes se ele, se o seu defunto homnimo
que d titulo ao livro, e por que antes um que outro, questo prenhe de
questes, que nos levariam longe... Eia! chora os dous recentes mortos, se
tens lgrimas. Se s tens riso, ri-te! a mesma cousa. O Cruzeiro que a linda
Sofia no quis fitar, como lhe pedia Rubio, est assaz alto para no discernir
os risos e as lgrimas dos homens.
(Machado de Assis. Quincas Borba.)
Depreende-se do texto que:
a) ao narrar a agonia de Rubio, o narrador deixa implcito que aquele
merecia as honrarias de um rei.

26

b) a ambigidade no ttulo do romance, Quincas Borba, justifica-se pelo fato


de o autor no conseguir definir-se por homenagear o filsofo ou seu co.
c) a afirmao que encerra o captulo CC revela um trao machadiano
caracterstico: a ironia.
d) a declarao de que Sofia no quis fitar o Cruzeiro revela a indiferena
como matriz do estilo do autor.
e) a linguagem empregada para descrever a morte de Quincas Borba revela
tendncia do narrador a dar mais importncia ao co do que a Rubio.

28) Este

34) (UFPel-6/2007)

Quincas Borba, se acaso me fizeste o favor de ler as Memrias


pstumas de Brs Cubas, aquele mesmo nufrago da existncia, que ali
aparece, mendigo, herdeiro inopinado, e inventor de uma filosofia. Aqui o
tens agora em Barbacena.
MACHADO DE ASSIS, J. M. Quincas Borba. In: Obras completas.
Rio de Janeiro: Nova Aguilar, 2004. v. I, p. 644.
A partir da leitura desse trecho, CORRETO afirmar que a obra Quincas
Borba:
a) aborda a filosofia de Quincas Borba como algo inventado por Rubio.
b) dissimula o personagem principal quando lhe d o nome de um co.
c) se constitui em um romance escrito por um narrador que j tinha morrido.
d) utiliza a intertextualidade, pois remete a outra narrativa do mesmo autor.

29) Em

relao frase "ao vencedor, as batatas", do livro Quincas Borba,


de Machado de Assis, vencedor e batatas so, respectivamente:
a) Rubio / rei da Prssia
b) Aparncia de riqueza / Rubio
c) Exploradores (Cristiano e Sofia) / bens materiais
d) Herana de Quincas Borba / o co
e) Amigos / os empregados da casa

30) Com

base na leitura de Quincas Borba, de Machado de Assis,


CORRETO afirmar que o narrador do romance
a) adere ao ponto de vista do filsofo, pois professa a teoria do
Humanitismo.
b) apela sentimentalidade do leitor no ltimo captulo, em que narra a
morte de Rubio.
c) apresenta os acontecimentos na mesma ordem em que estes se deram no
tempo.
d) narra a histria em terceira pessoa, no participando das aes como
personagem.

31) (UFRGS

2007) Assinale a alternativa correta em relao a Quincas


Borba, de Machado de Assis.
a) O ttulo do livro, como esclarece o narrador, refere-se ao filsofo Quincas
Borba, criador do Humanitismo.
b) Quincas Borba apenas um interiorano milionrio explorado por parasitas
sociais como Palha e Camacho.
c) Rubo objeto de disputa amorosa entre a bela Sofia e Dona Tonica, filha
do major Siqueira.
d) Rubo, scio do marido de Sofia, comete adultrio com ela sem levantar
suspeitas.
e) ao fugir do hospital, Rubio retorna com Quincas Borba sua cidade de
origem, Barbacena.

32) No

conto Uns Braos, de Machado de Assis, o adolescente Incio vive


durante algum tempo na casa do 'solicitador' Borges e de sua mulher D.
Severina. O jovem, inicialmente atrado pelos braos da mulher de Borges, os
quais observa durante as refeies, acaba se apaixonando. Severina percebe
os sentimentos do rapaz e tomada por emoes ambguas. Num domingo
em que Incio adormecera na rede do quarto, e Borges se ausentara de
casa, D. Severina no resiste tentao e vai ao quarto do jovem.
Assinale a alternativa que d seguimento correto aos fatos referidos acima.
a) O jovem acorda com a aproximao de D. Severina e os dois se beijam.
b) O jovem permanece dormindo, D. Severina chega junto rede com a
inteno de beij-lo, mas se arrepende ltima hora.
c) Incio sonha que D. Severina pega-lhe as mos e d-lhe um beijo na boca;
aturdido, acorda repentinamente e v que tudo no passou de um sonho.
d) Quando est no quarto de Incio, D. Severina surpreendida pelo marido
que retorna casa e lhe exige explicaes.
e) D. Severina observa Incio dormindo, aproxima-se da rede e beija-lhe a
boca, no mesmo instante em que o jovem sonha que est beijando a mulher.

33) Considerando

os contos de Machado de Assis indicados para leitura,


numere a coluna de acordo com os ttulos relacionados abaixo e, a seguir,
marque a alternativa CORRETA.
1. A Carteira
2. Uns Braos
3. Um Esqueleto
( ) Incio Borges D. Severina
( ) Alberto Dr. Belm D. Marcelina

) Honrio Gustavo Amlia

A alternativa que preenche as lacunas adequadamente :


a) 1, 2, 3
b) 1, 3, 2
c) 2, 3, 1
d) 2, 1, 3
e) 3, 2, 1
Sobre o conto Uns braos, de Machado de Assis,
correto afirmar que
a) a idia de que os braos de D. Severina acorrentavam Incio casa de
Borges metfora explorada pelo narrador era inverossmil, pois, embora o
rapaz se sentisse seduzido pela jovem senhora, tudo no passava de uma
iluso pueril. D. Severina jamais esboou qualquer tipo de atitude capaz de
contrariar os costumes da poca, o que revelava a honestidade de seu
carter.
b) o pai do protagonista, por desejar o labor do filho em uma atividade que
lhe garantisse futuramente recursos maiores do que aqueles que ele mesmo
obtinha na barbearia, entregou o rapaz aos cuidados de Borges. O barbeiro
entendia o trabalho no foro como uma forma de ascenso social.
c) D. Severina, por ser um personagem plano, viu-se dividida entre dois
amores. De um lado, a dedicao quase paternal de seu marido, que a cobria
de carinhos e cuidados. De outro, o afeto ingnuo de um rapaz de 15 anos,
que demonstrava no serem seus sentimentos simplesmente fraternais. Essa
situao gerava angstia no esprito da pobre senhora.
d) Incio via D. Severina como uma tentao diablica. Cada gesto, olhar,
atitude da jovem senhora eram entendidos como um jogo de seduo. Por
respeito ao Sr. Borges, Incio esforava-se para no retribuir os apelos da
amante, mas acabou sendo vencido pelo desejo. No clmax da narrativa, o
sentimento de remorso levou o jovem rapaz morte.
e) os empregados do Sr. Borges descobrem logo que a desateno de Incio
em relao s suas tarefas resultava, na verdade, do amor que o rapaz sentia
por D. Severina e que o impedia de pensar em algo, a no ser na jovem
senhora. Ao relatarem a descoberta ao patro, os empregados, os quais se
constituram, assim, nos antagonistas do heri, demonstram a importncia da
fidelidade nos relacionamentos humanos.

35) Sobre o conto Uns Braos, assinale a alternativa correta:

a) O narrador-protagonista descobre a paixo por D. Severina, mulher casada


com Borges, constituindo-se, portanto, a temtica realista do adultrio.
b) O desfecho do conto revela a fria de Borges ao descobrir o envolvimento
de Severina com Incio, o que motiva a despedida do rapaz e o
confinamento da mulher em seu quarto.
c) Incio pode ser compreendido como a personificao do Romantismo, uma
vez que vive em sonhos, desligando-se da realidade.
d) D. Severina uma mulher idealizada, incapaz de aes que contrariem a
moral da poca, por isso, manda Incio embora de sua casa.
e) D. Severina usava sempre um xale negro que lhe cobria os belos braos,
fato que desperta o fetiche do jovem Incio.

36) Sobre A Carteira correto afirmar:

a) Honrio representa a falncia moral e econmica da burguesia paulistana


do incio do sculo XX.
b) Gustavo pode ser compreendido como antagonista de Honrio, uma vez
que contraria as virtudes morais do protagonista, traindo sua amizade.
c) Amlia personagem que revela a submisso e a fragilidade feminina,
uma vez que no ousa contrariar Honrio.
d) O desfecho do conto aponta a idia de que Honrio sabia de toda a
traio, mas aceitava a situao em troca do dinheiro de Gustavo.
e) Nenhuma das alternativa acima est correta.

37) Sobre A Carteira fazem-se as seguintes afirmaes:

I - O narrador-protagonista Honrio
II - A narrativa explora o conflito moral vivido pelo protagonista.
III - O conto demonstra a crtica realista aos valores da sociedade capitalista
Esto corretas:
a) apenas I
b) I e II
c) II e III
d) I e III
e) todas

38) Sobre Um Esqueleto, assinale a alternativa correta:

a) Trata-se de um conto de horror, situado na fase romntica do autor.


b) O conto destoa da produo machadiana, por no conter nenhuma
temtica realista.
c) Pode-se perceber o carter metaficcional na narrativa, uma vez que
Alberto produz uma histria manipulando as emoes de seu pblico.

27

d) A narrativa desenvolve-se dentro dos padres tradicionais do realismo,


ensejando a abordagem de questes como o adultrio e o triunfo da razo
sobre as paixes.
e) O conto est centrado na verossimilhana da histria inventada pelo
protagonista, fato que lhe confere credibilidade frente aos rapazes que
ouviam atentos.

39) Sobre Um Esqueleto fazem-se as seguintes afirmaes:

I - O conto se constri a partir da sobreposio de narrativas: em uma praia


rapazes discutiam assuntos variados, quando um deles comea a contar uma
histria.
II - A histria narrada por Dr. Belm envolve ainda a viva Marcelina e seu
pretendente, o Tenente Soares.
III - O conto pode ser entendido como uma stira ao Romantismo, uma vez
que o narrador se utiliza de expedientes como o sentimentalismo, a morbidez
e o suposto tringulo amoroso.
Esto corretas:
a) apenas I
b) I e II
c) II e III
d) I e III
e) todas

40) Leia as proposies acerca de O Cortio.

I. Constantemente, as personagens sofrem zoomorfizao, isto , a


animalizao do comportamento humano, respeitando os preceitos da
literatura naturalista.
II. A viso patolgica do comportamento sexual trabalhada por meio do
rebaixamento das relaes, do adultrio, do lesbianismo, da prostituio etc.
III. O meio adquire enorme importncia no enredo, uma vez que determina o
comportamento de todas as personagens, anulando o livre-arbtrio.
IV. O estilo de Alusio Azevedo, dentro de O Cortio, confirma o que se
percebe tambm no conjunto de sua obra: o talento para retratar
agrupamentos humanos.
Est(o) correta(s)
a) todas.
b) apenas I.
c) apenas I e II.
d) apenas I, II e III.
e) apenas III e IV.

41) A

questo a seguir baseia-se no seguinte fragmento do romance O


cortio (1890), de Alusio Azevedo (1857-1913):
O cortio
Fechou-se um entra-e-sai de marimbondos defronte daquelas cem casinhas
ameaadas pelo fogo. Homens e mulheres corriam de c para l com os
tarecos ao ombro, numa balbrdia de doidos. O ptio e a rua enchiam-se
agora de camas velhas e colches espocados. Ningum se conhecia naquela
zumba de gritos sem nexo, e choro de crianas esmagadas, e pragas
arrancadas pela dor e pelo desespero. Da casa do Baro saam clamores
apoplticos; ouviam-se os guinchos de Zulmira que se espolinhava com um
ataque. E comeou a aparecer gua. Quem a trouxe? Ningum sabia diz-lo;
mas viam-se baldes e baldes que se despejavam sobre as chamas.
Os sinos da vizinhana comearam a badalar.
E tudo era um clamor.
A Bruxa surgiu janela da sua casa, como boca de uma fornalha acesa.
Estava horrvel; nunca fora to bruxa. O seu moreno trigueiro, de cabocla
velha, reluzia que nem metal em brasa; a sua crina preta, desgrenhada,
escorrida e abundante como as das guas selvagens, dava-lhe um carter
fantstico de fria sada do inferno. E ela ria-se, bria de satisfao, sem
sentir as queimaduras e as feridas, vitoriosa no meio daquela orgia de fogo,
com que ultimamente vivia a sonhar em segredo a sua alma extravagante de
maluca.
Ia atirar-se c para fora, quando se ouviu estalar o madeiramento da casa
incendiada, que abateu rapidamente, sepultando a louca num monto de
brasas.
(Alusio Azevedo. O cortio)
O carter naturalista nessa obra de Alusio Azevedo oferece, de maneira
figurada, um retrato de nosso pas, no final do sculo XIX. Pe em evidncia
a competio dos mais fortes, entre si, e estes, esmagando as camadas de
baixo, compostas de brancos pobres, mestios e escravos africanos. No
ambiente de degradao de um cortio, o autor expe um quadro tenso de
misrias materiais e humanas. No fragmento, h vrias outras caractersticas
do Naturalismo. Aponte a alternativa em que as duas caractersticas
apresentadas so corretas:
a) Explorao do comportamento anormal e dos instintos baixos; enfoque da
vida e dos fatos sociais contemporneos ao escritor.
b) Viso subjetivista dada pelo foco narrativo; tenso conflitiva entre o ser
humano e o meio ambiente.

c) Preferncia pelos temas do passado, propiciando uma viso objetiva dos


fatos; crtica aos valores burgueses e predileo pelos mais pobres.
d) A oniscincia do narrador imprime-lhe o papel de criador, e se confunde
com a idia de Deus; utilizao de preciosismos vocabulares, para enfatizar o
distanciamento entre a enunciao e os fatos enunciados.
e) Explorao de um tema em que o ser humano aviltado pelo mais forte;
predominncia de elementos anticientficos, para ajustar a narrao ao
ambiente degradante dos personagens.

42) Relacione

os trechos da obra O Cortio, de Alusio de Azevedo, s


caractersticas realistas/naturalistas seguintes que predominam nesses
trechos e, a seguir, marque a alternativa CORRETA:
1. Detalhismo.
2. Crtica ao capitalismo selvagem.
3. Fora do sexo.
( ) (...) possuindo-se de tal delrio de enriquecer, que afrontava resignado as
mais duras privaes. Dormia sobre o balco da prpria venda, em cima de
uma esteira, fazendo travesseiro de um saco de estepe cheio de palha.
( ) (...) era a luz ardente do meio-dia; ela era o calor vermelho das sestas de
fazenda; era o aroma quente dos trevos e das baunilhas, que o atordoara nas
matas brasileiras.
( ) E seu tipo baixote, socado, de cabelos escovinha, a barba sempre por
fazer (...) Era um pobre diabo caminhando para os setenta anos, antiptico,
muito macilento.
a) 2, 1, 3
b) 1, 3, 2
c) 3, 2, 1
d) 2, 3, 1
e) 1, 2, 3

43) Texto 1

De cada casulo espipavam homens armados de pau, achas de lenha, varais


de ferro. Um empenho coletivo os agitava agora, a todos, numa solidariedade
briosa, como se ficassem desonrados para sempre se a polcia entrasse ali
pela primeira vez. Enquanto se tratava de uma simples luta entre dois rivais,
estava direito! Jogassem l as cristas, que o mais homem ficaria com a
mulher! mas agora tratava-se de defender a estalagem, a comuna, onde
cada um tinha a zelar por algum ou alguma coisa querida.
(AZEVEDO, Alusio, O cortio. 26. ed. So Paulo: Martins, 1974. p. 139.)
Texto 2
O cortio um romance de muitas personagens. A inteno evidente a de
mostrar que todas, com suas particularidades, fazem parte de uma grande
coletividade, de um grande corpo social que se corri e se constri
simultaneamente.
(FERREIRA, Luiz Antnio. Roteiro de leitura: O cortio de Alusio Azevedo.
So Paulo: tica, 1997. p. 42.)
Sobre os textos, assinale a alternativa correta.
a) No Texto 1, por ser ele uma construo literria realista, h o predomnio
da linguagem referencial, direta e objetiva; no Texto 2, por ser ele um estudo
analtico do romance, h o predomnio da linguagem esttica, permeada de
subentendidos.
b) A afirmao contida no Texto 2 explicita o modo coletivo de agir do
cortio, algo que tambm se observa no Texto 1, o que justifica o
prevalecimento de um termo coletivo como ttulo do romance.
c) Tanto no Texto 1 quanto no Texto 2 h uma viso exacerbada e idealizada
do cortio, sendo este considerado um lugar de harmonia e justia.
d) No Texto 1 prevalece a desagregao e corroso da grande coletividade a
que se refere o Texto 2.
e) O que se afirma no Texto 2 vai contra a idia contida no Texto 1, visto que
no cortio jamais existe unio entre os seus moradores.

44) A personagem Jernimo de O Cortio:

a) Representa a tirania do capitalismo selvagem e da explorao do homem


pelo homem.
b) Representa a idealizao da brasilidade.
c) Representa a busca obsessiva pela ascenso social e a falta de escrpulos.
d) Representa a religiosidade e o misticismo do povo brasileiro.
e) Representa o determinismo do meio sobre o homem.

45) (FURG

2008 inv) Leia atentamente as trs afirmaes abaixo, todas


formuladas a partir do livro Saras de fogo, de Olavo Bilac.
I. Pertencente ao Simbolismo, observam-se composies, em sua maioria, de
difcil compreenso.
II. Composto de poemas que apresentam grande apuro formal,
especialmente na mtrica e na presena das rimas.

28

III. Apresenta alguns poemas de inspirao ertica, tais como O julgamento


de Frinia, Satnia, Beijo eterno e A tentao de Xencrates.
Com base nas afirmativas acima, podemos dizer que, apenas,
a) II est correta.
b) I e III esto corretas.
c) I e II esto corretas.
d) II e III esto corretas.
e) III est correta

46) Sobre o poema Satnia, inscrito em Saras de Fogo, correto afirmar:

a) Trata-se de um poema gtico, em que h a representao de uma cortes


da Grcia Antiga.
b) Revela um romantismo tardio ao idealizar a figura feminina, associando-a
a uma figura angelical e doce.
c) Nega os padres parnasianos de mtrica e rima, inscrevendo-se na fase
pr-modernista do autor.
d) Reveste-se de um carter descritivo, explorando a nudez feminina em um
jogo de imagens no qual a luz do sol percorre as formas de Satnia.
e) Comunica-se com a poesia de lvares de Azevedo, no instante em que o
eu-lrico se evidencia sentimental e emotivo.

47) A Poesia de Olavo Bilac:

51) (FURG

2008 inv) Em Lendas do Sul, Joo Simes Lopes Neto d


tratamento literrio a trs histrias mticas que povoam o imaginrio do Rio
Grande do Sul. So elas, na ordem em que aparecem no livro:
a) O Angera, A salamanca do Jarau e O negrinho do pastoreio.
b) Me Mulita, A salamanca do Jarau e So Sep.
c) A mboitat, O Curupira e A Me do Ouro.
d) O Saci, So Sep e O negrinho do pastoreio.
e) A mboitat, A salamanca do Jarau e O negrinho do pastoreio.

INSTRUO: Para responder s questes 52 e 53, ler o texto que segue.


E quando descerrei a janela e andei para a canastra a tirar a
guampa e libertar a teiniagu* para comer o mel (...)
Bonita, linda e bela, na minha frente estava uma moa!...
Que disse:
- Eu sou a princesa moura encantada, trazida de outras terras por
sobre um mar que os meus nunca sulcaram... Vim, e Anhang-pit
transformou-me em teiniagu de cabea luminosa (...)
A teiniagu que sabe dos tesouros, sou eu, mas sou tambm
princesa moura...
Sou jovem... sou formosa, o meu corpo rijo e no tocado!...
E estava escrito que tu serias meu par.

a) canta o trabalho humano e outros grandes temas universais como a


liberdade e a fraternidade entre os povos.
b) rica em metforas, em emoes evanescentes, em inefveis estados
* lagartixa
dalma.
c) vive da contradio entre as exigncias da alma e as solicitaes do corpo.
d) valoriza aspectos da vida cotidiana, numa linguagem simples e despojada. 52) Trata-se de trecho do conto_________ da obra de Simes Lopes
e) de grande perfeio tcnica, gira em torno do universo greco-latino e do Neto________ que recupera as narrativas determinantes___________ do Rio
amor sensual.
Grande do Sul.
a) A Salamanca do Jarau; Lendas do sul; da mitologia
48) Cruz e Souza foi o principal nome do Simbolismo brasileiro. Sobre sua b) Boitat; Lendas do sul; da histria
c) O Negrinho do Pastoreio; Cancioneiro guasca; da mitologia
poesia, assinale o nico item incorreto.
d) O Boi Velho;
Contos gauchescos; do tradicionalismo
a) Percebe-se a tendncia espiritual e religiosa.
e) Duelo de Farrapos; Contos gauchescos; da histria
b) H musicalidade em seus versos.
c) Nota-se o esprito extremamente racional.
d) Emprega vocabulrio com sentido nebuloso, impreciso, sugestivo.
53) Na recuperao de narrativas regionalistas, Simes Lopes Neto
e) Destaca-se pelo uso da letra maiscula
emprega uma linguagem que
a) est de acordo com a norma culta padro.
49) Sobre ltimos Sonetos fazem-se as seguintes afirmaes:
a) H a predominncia da temtica amorosa, com o resgate do platonismo b) desfigura o falar do gacho da fronteira.
c) aponta para o pitoresco do falar gauchesco.
romntico.
b) O livro gravita em torno de experincias vividas pelo poeta, que revelam d) associa temas e vocbulos com naturalidade.
e) inviabiliza a revelao do universo gauchesco.
explicitamente a dor oriunda do preconceito racial.
c) A obra constitui-se exclusivamente de sonetos, caracterizando-se pela
temtica da transcendncia espiritual
54) Leia as afirmaes abaixo sobre o conto O Negro Bonifcio, inscrito
d) Todos os poemas contidos na obra remetem cor branca, o que revela o em Contos Gauchescos
inconformismo do poeta em relao sua etnia.
I - As aes do conto se passam durante uma carreira de cancha reta,
e) Nenhuma das alternativas acima est correta.
acontecimento tradicional e festivo na campanha
gacha.
II - Tudinha, a personagem central, era a filha mais velha de uma famlia de
50)
estancieiros e estava apaixonada por um
CRCERE DAS ALMAS
rapaz da cidade que lhe dedicara alguns versos.
III - Ao final do conto, o narrador Blau Nunes manifesta sua descrena nas
Ah ! Toda a alma num crcere anda presa,
mulheres, pois no confia em nenhuma delas,
Soluando nas trevas, entre as grades
quer sejam ricas, quer pobres.
Do calabouo olhando imensidades,
Quais esto corretas?
Mares, estrelas, tardes, natureza.
a) Apenas I.
b) Apenas II.
Tudo se veste de uma igual grandeza
c) Apenas III.
Quando a alma entre grilhes as liberdades
d) Apenas I e III.
Sonha e sonhando, as imortalidades
e) I, II e III.
Rasga no etreo Espao da Pureza.
almas presas, mudas e fechadas
Nas prises colossais e abandonadas,
Da Dor no calabouo, atroz, funreo !
Nesses silncios solitrios, graves,
Que chaveiro do Cu possui as chaves
Para abrir-vos as portas do Mistrio ?!
(Cruz e Sousa. Poesia. Rio de Janeiro: Agir, 1957)
Assinale a alternativa que NO traduz entendimento do texto.
a) As palavras, em sua maioria, apresentam um valor simblico
b) O soneto se coloca todo num plano abstrato, traduzindo uma nsia de
captar o mundo em sua potencialidade maior.
c) Para o poeta, a condio material da vida humana no permite que as
almas se realizem plenamente.
d) No soneto, a liberdade da alma possvel atravs do sonho.
e) Evidencia-se no poema o gosto pela plasticidade e a tendncia a uma
concepo materialista da vida.

55) Considerando-se

a obra Contos Gauchescos, assinale a alternativa


INCORRETA.
a) A reunio festiva, o jogo e o final trgico so pontos em comum dos
contos O Negro Bonifcio e Jogo do Osso.
b) Em Melancia-Coco Verde e em Contrabandista, h um desfecho
tradicional nas histrias, com o casamento dos pares enamorados.
c) Em O Negro Bonifcio, o narrador Blau Nunes expressa, em vrios
momentos, a sua opinio negativa em relao personagem que d ttulo ao
conto, qualificando-o de maleva, excomungado ou malvado.
d) Em Melancia-Coco Verde, o casal de namorados cria uma senha para se
comunicar entre si sem o conhecimento dos outros.
e) Em O Negro Bonifcio, o narrador refere-se aos olhos de Tudinha como
a modo de veado-vir, face cor de pssego maduro e,aos dentes
brancos como de cachorro novo, evidenciando assim um modo de qualificar
tpico da linguagem regionalista do autor

56) Sobre Simes Lopes Neto so feitas as seguintes afirmaes.

I - Seus Contos Gauchescos e Lendas do Sul so narrativas que exploram


predominantemente enredos polticos de carter libertrio.

29

II - Suas narrativas fazem perdurar a tradio e o linguajar do gacho do


campo, louvando ao mesmo tempo a modernizao urbana, implantada
rapidamente na capital.
III - Blau Nunes, Jango Jorge, Maria Altina, Tudinha e o Negro Bonifcio
integram um conjunto de figuras humanas que enriquece a galeria de
personagens regionais da literatura brasileira.
Quais esto corretas?
a) Apenas I.
b) Apenas II.
c) Apenas III.
d) Apenas II e III.
e) I, II e III.

lamenta a morte de heris como Peri, bruto convertido em cavalheiro


portugus.
c) No segundo pargrafo, o autor afirma que o novo heri brasileiro fruto
da cruel etnologia dos sertanistas modernos. Isto significa que Lobato no
compactua com os estudos antropolgicos, pois esses estudos esto em
busca de verdades e as verdades so cruis e no fazem bem literatura.
d) No ltimo pargrafo, Lobato sugere um ritual antropofgico entre o ndio e
o portugus (Peri e Ceci), caso Alencar no tivesse feito de nosso ndio
truculento um bom selvagem. Mas o ritual no aconteceu, por felicidade
nossa e de D. Antnio de Mariz. Nessa afirmao, temos um Lobato
pacifista.
e) Nenhuma das alternativas acima est correta.

57) Assinale

60) A

a alternativa que comenta corretamente o enredo dos contos


destacados da obra Contos Gauchescos.
a) Observa-se no conto Trezentas onas que a personagem experimenta em
um curto espao de tempo um complexo estado interior em que os
sentimentos, as emoes travam uma luta com sua conscincia, estado
prprio ao ser humano quando em situaes limites. Ao deparar-se frente a
frente com a ona, pela rapidez da inteligncia que a personagem pode se
livrar da morte fatal.
b) Na obra em questo, as plantas, os animais, os fenmenos naturais
manifestam estados de alma prprios do ser humano, por outro lado a
maldade de algumas criaturas ganha espao, no conto Manantial, Chico,
filho de Chico Triste mesmo atolado no tremendal ainda afronta as pessoas
ofendidas por sua ao.
c) Mas perto da pomba rondava o gavio metfora que se contrape
expresso rabo de saia sempre precipcio para homens, ambas porm se
complementam e nos do uma viso da mulher e do papel que cabe no
universo narrado. A maioria dessas mulheres vivem no espao restrito do lar,
em completa obedincia ao seu marido ou pai, entretanto, o narrador
apresenta tambm personagens femininas que so independentes e
sustentam suas famlias.
d) A defesa da honra tema de alguns dos textos. No conto Manantial, o pai
de Ana Altina prefere matar a filha a v-la casada com o seu ofensor; no
conto Cabelos da china, o ndio Juca Picum, na Guerra dos Farrapos,
respeita e obedece o Capito, seu oficial, embora seja o sedutor de sua filha,
mas por fim o mata frente a ameaa do assassinado da moa pelos cimes
do sedutor.
e) Em O mate de Joo Cardoso, segundo o narrador, o protagonista
chalrador como trinta e que dava dois dentes por um dedo de prosa. A
comicidade do texto surge da participao do escravo que no pra de
oferecer comida s pessoas que se propem a ouvir as histrias de Joo
Cardoso.

58) Considere as seguintes afirmaes sobre obras de Monteiro Lobato.

I - Em Urups, ele produz uma literatura que revela os problemas


socioeconmicos do Brasil, em especial, do interior.
II - Em Urups, ele atribui a culpa pelo atraso do Brasil ao caboclo, por ele
ser acomodado e inadaptvel s mudanas necessrias ao desenvolvimento.
III - A publicao de Urups foi estimulada pela repercusso do artigo Velha
praga.
Quais esto corretas?
a) Apenas I.
b) Apenas II.
c) Apenas III.
d) Apenas II e III.
e) I, II e III.

59) Considere os fragmentos seguintes do conto Urups.

(...)
Morreu Peri, incomparvel idealizao dum homem natural como o sonhava
Rousseau, prottipo de tantas perfeies humanas que no romance, ombro a
ombro com altos tipos civilizados, a todos sobreleva em beleza dalma e
corpo.
Contraps-lhe a cruel etnologia dos sertanistas modernos um selvagem real,
feio e brutesco, anguloso e desinteressante, to incapaz, muscularmente, de
arrancar uma palmeira, como incapaz, moralmente, de amar Ceci.
Por felicidade nossa e de D. Antnio de Mariz no os viu Alencar; sonhouos qual Rousseau. Do contrrio l teramos o filho de Arar a moquear a linda
menina num bom braseiro de pau brasil, em vez de acompanh-la em
adorao pelas selvas, como o Ariel benfazejo do Paquequer.
Monteiro Lobato, Urups.
Marque a alternativa correta.
a) Nesses pargrafos introdutrios de Urups, Monteiro Lobato faz uma
crtica ao heri nacional criado pelo Romantismo ainda vivo na memria
literria brasileira do incio do sculo XX. Para Lobato, o verdadeiro homem
representante do Brasil o Jeca Tatu, figura real e desinteressante que se
contrape ao heri idealizado.
b) No primeiro pargrafo, o autor exalta a figura do bom selvagem criada
por Rousseau e largamente cultuada pelos escritores romnticos. Lobato

ocorrncia de finais trgicos um dos traos singulares da obra


Urups. Em todas as alternativas identificou-se corretamente o aspecto
trgico do desfecho da respectiva histria, EXCETO:
a) A vingana da Peroba O filho de Joo Nunes morre esmagado pelo
monjolo construdo pelo prprio pai.
b) O engraado arrependido Pontes enforca-se numa perna de ceroula,
depois de ter causado a morte do major Bentes e ter perdido a oportunidade
de obter um cargo no servio pblico.
c) Buclica A menina Anica, deficiente fsica, morre de sede por
negligncia e maldade de sua me, Nh Veva.
d) Os faroleiros Gerebita e Cabrea, dois faroleiros inimigos, morrem
quando desaba o farol no qual trabalhavam.

61) Sobre Urups, assinale V ou F:

( ) Tratando de tema srio e crucial sobre o Brasil, Urups se abstm de


ironias e de qualquer trao de humor.
( ) O universo citadino, em contraponto ao rural, tambm tem seu espao
nos contos de Lobato.
( ) Alguns contos como "Os faroleiros" fogem da regio do Vale do Paraba
e ganham outras paisagens.
( ) Monteiro Lobato demonstra uma veia extremamente crtica em Urups,
beirando ao preconceito e ao racismo.
A seqncia correta :
a) V, V, V, V
b) V, F, F, V
c) F, F, V, F
d) F, V, V, V
e) F, F, V, V

62) Assinale a alternativa incorreta sobre Urups:

a) Lobato produz uma obra nica, tpica de sua genialidade, no


necessitando do uso freqente que autores fizeram e fazem de relaes de
intertextualidade.
b) Urups nome do artigo homnimo obra, nele o autor disseca os
defeitos do caboclo brasileiro, que representam o atraso da nao.
c) Lobato mescla narrativas em primeira e terceira pessoa.
d) O autor faz uma intensa e grandiosa descrio da natureza do local, numa
clara utilizao expressionista.
e) Em Urups, com seus catorze contos, Monteiro Lobato traa um painel do
Brasil, tratando de vrias regies do pas.

63) (FURG 2008 inv) O romance Triste fim de Policarpo Quaresma, de Lima

Barreto, divide-se em trs partes distintas, que apresentam, cada uma, as


fases as quais a personagem-ttulo vivencia, visando emancipao do
Brasil. Essas trs diferentes fases podem ser assim nomeadas:
a) Primeira fase: cultural; segunda fase: agrria; terceira fase: econmica.
b) Primeira fase: social; segunda fase: esportiva; terceira fase: polticomilitar.
c) Primeira fase: agrria; segunda fase: cultural; terceira fase: econmica.
d) Primeira fase: cultural; segunda fase: agrria; terceira fase: polticomilitar.
e) Primeira fase: cultural; segunda fase: polticomilitar; terceira fase: agrria.

64) Considere

as seguintes afirmaes a respeito do romance O Triste Fim


de Policarpo Quaresma, de Lima Barreto.
I - O protagonista no compreende bem o mundo em que vive, passando da
ingnua crena no idealismo dos homens, vistos como capazes de construir
um Brasil melhor, para o desencanto melanclico em relao s instituies e
s suas escolhas pessoais
I I- Quaresma condenado morte, porque ofendeu o Imperador D. Pedro
II, quando este no analisou suas propostas de reforma rural, nem atendeu
s suas reivindicaes.
III - No romance, so introduzidos personagens representativos do subrbio
do Rio de Janeiro, como Ricardo Corao dos Outros, com a sua fala popular
e expresses tpicas do brasileiro da poca.
Quais esto corretas?
a) Apenas I.

30

b) Apenas I e II.
c) Apenas I e III.
d) Apenas II e III.
e) I, II e III.

65) Assinale

a alternativa correta referente ao romance Triste Fim de


Policarpo Quaresma.
a) A obra representa a soma de todas as tendncias da segunda metade do
sculo XIX e do incio do sculo XX. Seu autor representa o mau gosto.
b) Denuncia a revolta contra o alto custo de vida, o desemprego e os rumos
da Repblica.
c) Retrata a poca urea da economia cafeeira no Sudeste; o momento da
entrada das grandes levas de imigrantes, notadamente os italianos.
d) Retrata a realidade brasileira, negando o Brasil literrio herdado do
Romantismo.
e) O tema central do livro o nacionalismo, o nacionalismo absurdo, porm
honesto da figura quixotesca, que a personagem central.

66) Quando Policarpo preso, Ricardo Corao dos Outros no pode ajudlo, mas vai pedir auxlio a vrias pessoas. Qual a personagem que
efetivamente tentou libertar Quaresma?
a) Olga.
b) Genelcio.
c) General Albernaz.
d) Coronel Bustamante.
e) Armando Borges.

67) Leia

o excerto abaixo, extrado de Amar, Verbo Intransitivo, de Mrio


de Andrade.
Frulein engole quase um remorso porque se apanha a divagar. Queixumes
do deus encarcerado (...): a profisso dela se resume a ensinar primeiros
passos, a abrir olhos, de modo a prevenir os inexperientes da cilada das
mos dos_rapaces. E evitar as doenas, que tanto infelicitam o casal futuro.
Profilaxia. (...). Mas porm dever parolar, quando mais chegadinho o
convvio, sobre essas meretrizes que chupam o sangue do corpo sadio. O
sangue deve ser puro.
Vejam por exemplo a Alemanha, qued raa mais forte? Nenhuma. E
justamente porque mais forte e indestrutvel neles o conceito da famlia. Os
filhos nascem robustos. As mulheres so grandes e claras. So fecundas. O
nobre destino do_homem se conservar sadio e procurar esposa
prodigiosamente sadia. De raa superior, como ela, Frulein. Os negros so
de raa inferior. Os ndios tambm. Os portugueses tambm.
Mas esta ltima verdade Frulein no fala aos alunos. Foi decreto lido a vez
em que um trabalho de Reimer lhe passou pelas mos: afirmava a
inferioridade dos latinos. Legtima verdade, pois quem Reimer? Reimer
um grande sbio (...)
coisa que se ensine o amor? Creio que no. Ela cr que sim.(...). Quer
mostrar que o dever supera os prazeres da carne, supera.
Com base no fragmento acima, assinale a alternativa INCORRETA:
(ANDRADE, Mrio de. Amar, Verbo Intransitivo Idlio. 16. ed. Belo
Horizonte : Villa Rica, 1995.)
a) A narrativa, nesse fragmento, feita na terceira pessoa, por um narrador
onisciente (sabe de tudo) e onipresente (no perde uma).
b) Percebe-se claramente, nesse fragmento, que o autor no se coloca
dentro do livro para fazer suas numerosas observaes marginais (comentar,
criticar, expor idias, concordar ou discordar).
c) Andrade usa uma linguagem cheia de elipses que obrigam o leitor a ligar e
completar os pensamentos. Ao invs de dizer e de explicar tudo, apenas
sugere em frases curtas, mnimas.
d) H uma referncia ao racismo alemo, principalmente depois que Frulein
leu um trabalho de Reimer, em que se afirmava a inferioridade da raa latina.
e) Frulein ir dirigir a sexualidade do rapaz contra os perigos da cidade
(corporificado no preconceito e na forma estereotipada com que Elza se
refere s prostitutas) e salvaguardar a pureza moral do corpo fsico e social
da nova gerao da burguesia.

68) Marque a alternativa CORRETA a respeito da personagem Elza:


a) Frulein, a governanta alem, professora de amar.
b) Signorita, a governanta italiana, professora de amar.
c) Miss Elza, a governanta inglesa, professora de amar.
d) Mademoiselle, a governanta francesa, professora de amar.
e) Senhorita, a governanta paulista, professora de amar

69) Sobre

o Romance Amar, Verbo Intransitivo assinale a alternativa


INCORRETA:
a) O romance se insere na gerao herica do modernismo, caracterizandose por inovaes na linguagem e por experimentalismos como a palavra fim
antes do trmino do livro.

b) A metalinguagem caracterstica importante da obra, uma vez que o


narrador-autor muitas vezes centraliza as atenes para os seus comentrios
e juzos de valor.
c) A obra aborda de forma caricatural personagens representativas da
burguesia paulistana, como Felisberto Souza Costa.
d) Elza, contratada como governanta pelos Souza Costa, apaixona-se por
Carlos, dando incio ao Idlio referido no subttulo da obra.
e) O ttulo da obra refere-se aos fins e aos resultados da insero de Elza na
casa dos Souza Costa.

70) Leia o poema e, depois, marque a proposio incorreta.


D-me um cigarro
Diz a gramtica
Do professor e do aluno
E do mulato sabido
Mas o bom negro e o bom branco Da
Nao Brasileira
Dizem todos os dias
Deixa disso camarada
Me d um cigarro

(Oswald de Andrade)

a) Com "E do mulato sabido", o autor refere-se miscigenao de raas no


Brasil.
b) O poema de Oswald de Andrade no tem rimas, nem pontuao, o que
desvia da poesia Modernista.
c) O poeta compara a lngua ensinada na escola, que artificial e elitista,
com a lngua falada pelo povo comum.
d) O poema critica a norma gramatical que probe o uso de pronomes
oblquos tonos no incio de frases.
e) Atravs do adjetivo bom, que antecede as palavras negro e branco, o
poeta valoriza o homem iletrado, autntico representante da cultura nacional
brasileira.

71) O modernismo de 1922 quisera-se atual (aberto ao mundo) e nacional

(ficando no solo ptrio), porm, na prtica, levou algum tempo at


concretizar-se plenamente esse sonho bicfalo. O fruto maduro da semente
ento plantada foi a Antropofagia oswaldiana, para a formulao da qual a
pintura de Tarsila, sua companheira, contribui em primeirssima linha,
sobretudo a partir de 1924. Para Oswald, o Brasil, rico de sua prpria seiva
(...), necessitava assumir a urgncia de uma estratgia regeneradora.
(PONTUAL, Roberto. In.: Modernidade: arte brasileira do sculo XX. So
Paulo: MEC/MAM, 1988. p. 26.)
O texto acima aponta uma estratgia regeneradora para o movimento
modernista. Assinale a alternativa que indica essa estratgia regeneradora
proposta por Oswald de Andrade.
a) Absorver as novidades da vanguarda europia, porm expressando a
realidade brasileira.
b) Romper com os padres de pensamento dos modernistas europeus.
c) Valorizar o pensamento racional e o carter cientfico na estrutura da
pintura.
d) Importar passivamente os modelos surrealista e cubista das vanguardas
europias.
e) Apropriar-se da esttica naturalista e da concepo positivista da cultura.

72) No poema Auto-retrato, Manuel Bandeira define-se como:

Provinciano que nunca soube escolher bem uma gravata; / Pernambucano a


quem repugna / A faca do pernambucano; / E em matria de profisso /
Um tsico profissional.
Assinale a alternativa em que o prprio poeta retoma os sentidos dos versos
acima.
a) Irene preta
Irene boa
Irene sempre de bom humor.
b) Quando ontem adormeci
Na noite de So Joo
Havia alegria e rumor.
c) Quero antes o lirismo dos loucos
O lirismo dos bbados
O lirismo difcil e pungente dos bbados.
d) Eu quero a estrela da manh
Onde est a estrela da manh?
Meus amigos meus inimigos
Procurem a estrela da manh.
e) Febre, hemoptise, dispnia e suores noturnos
A vida inteira, que podia ter sido e que no foi
tosse, tosse, tosse.

73) Leia o poema abaixo e assinale a alternativa correta.


CONSOADA
Quando a indesejada das gentes chegar

31

(No sei se dura ou carovel),


Talvez eu tenha medo.
Talvez sorria, ou diga:
Al, iniludvel!
O meu dia foi bom, pode a noite descer.
(A Noite com seus sortilgios.)
Encontrar lavrado o campo, a casa limpa,
A mesa posta,
Com cada coisa em seu lugar.
Manuel Bandeira. Estrela da vida inteira
a) Predomina, no poema, uma relao serena do sujeito potico com a a
indesejada das gentes, a morte, tema
comum na obra do autor.
b) O poema est escrito em uma linguagem rebuscada, com muito rigor
formal, e versa sobre o tema do fazer potico.
c) um soneto escrito nos moldes rcades, com muitas metforas, sobre o
tema do amor impossvel.
d) O sujeito potico aborda o tema da morte de modo muito exagerado, da
a presena de hiprboles, em vrios versos.
e) Trata-se de um trecho de um romance de Manuel Bandeira, autor do
Modernismo que se notabilizou por suas narrativas
sobre morte.

74) Libertinagem,

uma das obras mais expressivas de Manuel Bandeira,


apresenta temtica variada. Indique a alternativa em que no h
correspondncia entre o tema e o poema.
a) cotidiano Poema tirado de uma notcia de jornal
b) recordaes da infncia Profundamente
c) teor metalingstico Potica
d) evaso e exlio Vou-me embora pra Pasrgada
e) amor ertico Irene no cu

75) Procura da Poesia

No faas versos sobre acontecimentos.


No h criao nem morte perante a poesia.
Diante dela, a vida um sol esttico,
no aquece nem ilumina.
(...)
Penetra surdamente no reino das palavras.
L esto os poemas que esperam ser escritos.
Esto paralisados, mas no h desespero,
h calma e frescura na superfcie intata.
Ei-los ss e mudos, em estado de dicionrio.
(...)
Carlos Drummond de Andrade, A rosa do povo.

a) praticar a arte pela arte a maneira mais eficaz de se opor ao mundo


capitalista.
b) a procura da boa poesia comea pela estrita observncia da variedade
padro da linguagem.
c) fazer poesia produzir enigmas verbais que no podem nem devem ser
interpretados.
d) as intenes sociais da poesia no a dispensam de ter em conta o que
prprio da linguagem.
e) os poemas metalingsticos, nos quais a poesia fala apenas de si mesma,
so superiores aos poemas que falam tambm de outros assuntos

1) A

2) D

3) E

4) C

5) C

6) D

7) B

8) C

9) A

10) A

11) B

12) E

13) -

14) A

15) A

16) C

17) D

18) C

19) A

20) D

21) D

22) D

23) E

24) B

25) E

26) A

27) C

28) D

29) C

30) D

31) E

32) E

33) C

34) B

35) C

36) B

37) C

38) C

39) D

40) A

41) A

42) D

43) B

44) E

45) D

46) D

47) E

48) C

49) C

50)

51) E

52) A

53)

54) D

55) B

56) C

57) B

58) E

59) A

60) D

61) D

62) E

63) D

64) C

65) E

66) A

67) B

68)

69) D

70) B

71) A

72) E

73) A

74) E

75) D

76) E

77) C

No contexto do livro, a afirmao do carter verbal da poesia e a incitao a


que se penetre no reino das palavras, presentes no excerto, indicam que,
para o poeta de A rosa do povo,

32

TEXTO I
Peinar el viento
A los grandes artistas es mejor verlos que orlos, porque cuando
explican sus obras suelen ser bastante menos convincentes que cuando
pintan o esculpen; algunos, entre los mejores, resultan incluso tan confusos
que, oyndolos o leyndolos, se tiene la impresin de que son apenas
conscientes de lo que han logrado, o de que estn garrafalmente
equivocados sobre las maravillas que producen sus manos y sus instintos, o
de que su genio pasa casi excluyentemente por su sensibilidad y su intuicin,
sin tocar su inteligencia.
No es el caso de Eduardo Chillida, desde luego, a quien, hace
unos diez aos, dialogando con un crtico en el auditorio de la Tate Gallery de
Londres, o describir con claridad luminosa su trayectoria artstica, desde sus
inicios, cuando esa vocacin fue imponindose al estudiante de arquitectura y
al portero de ftbol de la Real Sociedad que era entonces y precipitndolo en
una aventura creadora que ha marcado como pocas el arte de su tiempo.
Tengo un recuerdo muy vivo de esa conferencia que, a m, me enriqueci
todava ms el alto aprecio que tena por la obra del escultor. A la sencillez
de sus explicaciones sobre su relacin con los materiales por qu lo
fascinaban el granito, la greda y el hierro, por ejemplo, y por qu siempre
desconfi del bronce, con el que nunca pudo amigarse , acompaaba una
franqueza inusitada para revelar sus admiraciones y sus distancias con otros
artistas contemporneos, y una modestia para hablar de s mismo que yo no
he conocido en ninguna otra persona. La insensible manera omo su obra fue
deslizndose, en sus aos veintiaeros de Pars, de los yesos figurativos que
representaban desnudos a las formas abstractas que forjara en hierro en los
aos siguientes, la ilustraba Chillida con ancdotas divertidas, como una lenta
maduracin en la que el azar desempeaba un papel tan importante como la
experiencia. Y, en relacin con sus esculturas, hablaba de entidades tan
escurridizas como el espacio, el tiempo, la luz y el aire ni ms ni menos que
si se tratara de personas de carne y hueso, amigos con los que se ha andado
un largo trecho de camino, hacia un destino todava lejano de alcanzar.
LLOSA, Mario Vargas. El Pas, 8 jul. 2001. Encontrado el 5 jun.
2003, en: http://www.elpais.es (Fragmento)

1) Segn el texto, es INCORRECTO afirmar que los grandes artistas


a) poseen sensibilidad, sin embargo no son para nada listos.
b) parecen a veces estar totalmente confundidos sobre su obra.
c) tienen la costumbre de no ser convincentes, por esto es mejor verlos.
d) son mejores produciendo sus obras que hablando sobre ellas.

2) ... tan escurridizas como el espacio, el tiempo, la luz y el aire...

En esta frase, la palabra destacada slo NO puede ser reemplazada por


a) estimulantes.
b) deslizantes.
c) resbaladizas.
d) huidizas.

3) Segn el texto, es INCORRECTO afirmar que

a) Eduardo Chillida ha trabajado ms con hierro, granito y greda que con


bronce.
b) Eduardo Chillida ha trabajado con el espacio, el tiempo, la luz y el aire.
c) todos los grandes artistas son totalmente conscientes de lo que han
conseguido.
d) la obra de Eduardo Chillida ocupa un lugar importante en el arte de su
tiempo.

4) Y, en relacin con sus esculturas, hablaba de entidades tan escurridizas


como el
espacio, el tiempo...

En esta frase se observa una relacin de


a) conclusin.
b) comparacin.
c) explicacin.
d) consecuencia.

5) Segn el texto, es CORRECTO afirmar que Eduardo Chillida,

a) antes de dedicarse a su vocacin, fue estudiante de arquitectura y arquero


de un equipo de ftbol.
b) a lo largo de su carrera, siempre tuvo mucha confianza y afinidad con el
bronce como material para su obra.
c) entre sus obras, cuenta yesos de formas abstractas y, ms tarde,
desnudos forjados en metal.
d) en sus conferencias, le gusta hablar de sus admiraciones y de las
insensibles maneras de su obra.

6) Segn el texto, las siguientes equivalencias estn correctas, MENOS:


a) hierro = metal
b) desde luego = desde ahora

c) todava = an
d) Tengo = conservo

7) El autor del texto afirma tener un alto aprecio por la obra de Chillida,

enriquecido por la conferencia que dio el artista, principalmente porque el


escultor
a) hizo los yesos de desnudos en sus aos veintiaeros, en Pars.
b) mostr modestia y simplicidad al hablar de s mismo.
c) siempre mantuvo distancias con los dems artistas contemporneos.
d) nunca utiliz con placer el bronce para realizar sus esculturas.
TEXTO II
De Ftbol
Siempre controvertido, brillante e intransigente, Johan Cruyff ha
sido una de las figuras ms singulares del deporte mundial. De un futbolista
como l pueden aprenderse muchas cosas, como lo demuestra Me gusta el
ftbol, libro en el que comparte sus puntos de vista e ideas sobre el deporte
que ama y que conoce como pocos. Hay en ese texto ameno y, sobre todo,
sutil, ideas especficas sobre el juego y la estrategia (la presin debe
ejercerse sobre el baln, no sobre el jugador) y consejos que pueden servir
mas all de las canchas (El mejor mtodo para ensear a un nio a jugar no
es prohibir, sino guiar). Cruyff cierra el libro con un declogo que resume
sus opiniones sobre el ftbol, diez apreciaciones sencillas y contundentes que
deberan ser ledas y memorizadas por directivos, entrenadores y jugadores
para hacer menos mezquino y pragmtico este juego en la actualidad.
A los que juegan ftbol en la calle, en el llano o en la cancha del
colegio este libro les har ver su deporte de otra manera. Sobre todo, les
ayudara a descubrir la belleza inexplicable y esplndida del ftbol, hoy tan
rara y huidiza, pero que a veces aparece por ah para recordarnos que, a
pesar de los directivos, la mercadotecnia y la televisin, la magia persiste.
DURN GRACIA, Miguel. La tempestad, Mxico DF, ao 5, n 29,
mar.-abr. 2003, p. 78. (Adaptado)

8) A los que juegan ftbol en la calle, en el llano o en la cancha...


En esta frase, la palabra destacada significa
a) espacio para deportes.
b) corrales abiertos.
c) recodo del ro.
d) patio de un colegio.

9) Segn el texto, es CORRECTO afirmar que

a) el libro Me gusta el ftbol va a proporcionar una mirada distinta a aqullos


que juegan al ftbol en espacios diversificados.
b) el libro Me gusta el ftbol, a pesar de presentar comentarios especficos
sobre el ftbol, es de lectura poco agradable.
c) la tcnica coercitiva se vuelve la mejor para hacer que un nio aprenda a
jugar al ftbol.
d) Johan Cruyff, adems de haber sido uno de los personajes ms
extraordinarios del deporte mundial, fue un hombre de carcter muy simple.

10) El texto slo NO plantea que el libro de Johan Cruyff es


a) inexplicable.
b) ameno.
c) til.
d) sutil.

11) El mejor mtodo para ensear a un nio a jugar no es prohibir, sino


guiar.

En esta frase, la palabra destacada expresa una idea de


a) contraposicin.
b) consecuencia.
c) inclusin.
b) exclusin.
TEXTO III
Rquiem por las lenguas indgenas
MADRID La mayora de las 6.000 lenguas que existen en todo el mundo se
encuentran en peligro de extincin. La internacionalizacin de los mercados
financieros, la difusin de la informacin por medios electrnicos y las
corrientes migratorias amenazan sobre todo a las lenguas indgenas segn la
UNESCO.
El nmero de parlantes es el principal indicador de la esperanza
de vida de un idioma. Para que una lengua sobreviva es necesario que sea
hablada por 100.000 personas. Pero, si tomamos como referencia esta cifra,
el panorama es bastante desolador, ya que ms de la mitad de las lenguas
son habladas por menos de 10.000 personas.
La situacin es especialmente alarmante en Latinoamrica, cuna
de 400 de las 5.000 lenguas indgenas que existen actualmente. En

33

Guatemala alrededor del 60% de la poblacin es indgena. Sin embargo, el


desuso de las lenguas maternas ha provocado que los 21 grupos
etnolingsticos mayas del pas sumen tan slo tres millones de hablantes.
FERNNDEZ LARRINAGA, Mara. (Fragmento)

c) Jordi y sus amigos trabajan en el reparto de un sper, sobre todo de


cervezas y patatas.
d) slo adolescentes sin dinero participan del botelln, lo que no es aceptable
del todo.

12) Sin embargo, el desuso de las lenguas maternas ha provocado que los 17) ... Jordi ha quedado sobre las ocho de la tarde con unos cuantos
21 grupos etnolingsticos mayas del pas sumen tan slo tres millones de amigos en la esquina de casa.
hablantes.
Si se quiere mantener el sentido de las palabras destacadas en este
En esta frase, la palabra destacada puede ser sustituida por
a) No obstante...
b) Adems...
c) Por tanto...
d) Por ello...

fragmento, NO es posible reemplazarlas por


a) a eso de las ocho.
b) a las ocho en punto.
c) alrededor de las ocho.
d) hacia las ocho.

13) Segn el texto, es INCORRECTO afirmar que

18) ... est socialmente aceptado tomarse unas cervezas...

14) ...

19) Seale

a) el nmero de hablantes es el indicio fundamental para la vida de una La partcula destacada en este fragmento significa que uno
lengua.
a) aprovecha las cervezas.
b) casi todas las lenguas existentes en el mundo pueden ser extinguidas.
b) bebe sin picar algo.
c) una lengua sobrevive si es hablada por 100.000 personas.
c) toma ms de la cuenta.
d) 400 lenguas indgenas se extinguieron en Amrica Latina.
d) toma muchas cervezas.
las corrientes migratorias amenazan sobre todo a las lenguas
indgenas...
En esta frase, la expresin destacada slo NO puede ser sustituida por la
palabra
a) fundamentalmente.
b) bsicamente.
c) solamente.
d) principalmente.

la opcin en la que la correspondencia entre los trminos


destacados y las palabras entre corchetes est INCORRECTA.
a) ... apenas podamos hablar. [= casi no]
b) ... en cambio, est socialmente aceptado... [= por el contrario]
c) ... mientras que Jordi se encarga... [= en tanto que]
d) Tras saludarse, se dirigen... [= Adems de]

20) Segn el texto, es INCORRECTO afirmar que el grupo de amigos

a) lleva a El Paio diversas cosas como copas, manteles y bolsas de residuos.


el texto, es CORRECTO afirmar que estn en peligro de b) se apura para llegar al sper, cuando se encuentra, los viernes.
extincin
c) se distribuye las actividades que tiene que hacer en el sper.
a) la mayora de las 6.000 lenguas indgenas que existen en el mundo.
d) se encuentra con Jordi, todo ltimo da hbil de la semana, para ir al
b) algunas de las 6.000 lenguas indianas que existen en el mundo.
sper.
c) muchas de las 6.000 lenguas que existen en el mundo.
TEXTO V
d) ms de 6.000 lenguas que existen en el mundo.

15) Segn

Volver En palabras de Pedro Almodvar


TEXTO IV
La generacin del botelln
Como cada viernes, Jordi ha quedado sobre las ocho de la tarde
con unos cuantos amigos en la esquina de casa. Tras saludarse, se dirigen al
sper para llegar antes de que cierre y, lista en mano, se reparten el trabajo:
uno va a por las cervezas y las patatas; otro, a por refrescos y alguna botella
de licor, mientras que Jordi se encarga de los vasos de plstico, las servilletas
y las bolsas de basura. Con la compra hecha, se dirigen a El Paio, un local
que comparten desde hace un ao doce colegas. Un par de sillones, una tele
y un reproductor de DVD, una nevera y una gran mesa; en las paredes,
grafitis que han ido pintando ellos mismos en los que se leen lemas
reivindicativos, y un pster del Bara. Vamos casi cada tarde para vernos,
charlar, tomar algo o ver una peli, explica Jordi, que tiene 20 aos y es
estudiante de publicidad. Decidimos alquilar el local porque era muy caro
salir de bares y porque, adems, con el ruido y la msica que inundan estos
locales, apenas podamos hablar.
El fenmeno del botelln no se reduce a los adolescentes y
veinteaeros, ni tampoco a una cuestin econmica. Adems, no siempre
est mal visto: si bien tiene connotaciones negativas que un grupo de
chavales de entre 16 y 24 compartan una botella en el maletero de un coche
o en un parque, en cambio, est socialmente aceptado tomarse unas
cervezas en una terraza, en un bar o en una discoteca, sobre todo si se
rebasa cierta edad, por lo general los 25 aos. Tambin la calle es un
escenario lcito en muchas ocasiones; la bondad del clima de las ciudades
mediterrneas propicia que el espacio pblico se convierta en uno de los
principales puntos de encuentro y relacin, en donde el botelln, entendido
como una reunin de un grupo reducido de amigos que charlan mientras se
toman una cerveza o una copa, es algo frecuente. [...]
El fenmeno del botelln es social y masivo considera la
psicloga Nuria Camps pero no todos los botelloneros son iguales. Cada
joven es diferente y forma parte de un gran puzzle que es reflejo de la
sociedad y en el que las generalizaciones conducen a errores.
SEZ, Cristina. Acceso en 14 abr. 2006. (Texto adaptado)

16) Segn el texto, es CORRECTO afirmar que

a) amigos peleando por causa de una cerveza o una copa tambin se pueden
ver en la calle.
b) el grupo de amigos se rene en un local desde hace un ao, casi todas las
tardes, para divertirse.

Volver es un ttulo que incluye varias vueltas, para m. He


vuelto, un poco ms, a la comedia. He vuelto al universo femenino, a La
Mancha (sin duda es mi pelcula ms estrictamente manchega, el lenguaje,
las costumbres, los patios, la sobriedad de las fachadas, las calles
empedradas). He vuelto a trabajar con Carmen Maura (hace diecisiete aos
que no lo hacamos), con Penlope Cruz, Lola Dueas y Chus Lampreave. He
vuelto a la maternidad, como origen de la vida y de la ficcin. Y,
naturalmente, he vuelto a mi madre. Volver a La Mancha es siempre volver al
seno materno.
Tengo la impresin, y espero que no sea un sentimiento pasajero,
de que he conseguido encajar una pieza (cuyo desajuste, a lo largo de mi
vida, me ha provocado mucho dolor y mucha ansiedad, dira incluso que en
los ltimos aos haba deteriorado mi existencia, dramatizndola ms de la
cuenta). La pieza a la que me refiero es la muerte, no slo la ma y la de
mis seres queridos sino la desaparicin implacable de todo lo que est vivo.
Nunca lo he aceptado, ni lo he entendido. Y eso te pone en una situacin
angustiosa ante el cada vez ms rpido paso del tiempo.
La principal vuelta de Volver es la del fantasma de una madre,
que se aparece a sus hijas. En mi pueblo estas cosas pasan (me he criado
oyendo historias de aparecidos), sin embargo yo no creo en las apariciones.
Slo cuando les ocurren a los dems, o cuando ocurren en la ficcin. Y esta
ficcin, la de mi pelcula (y aqu viene mi confesin), ha provocado en m una
serenidad como hace tiempo no senta (realmente, serenidad es un trmino
cuyo significado es un misterio para m).
En los aos que llevo de vida, nunca he sido una persona serena
(ni me ha importado lo ms mnimo). Mi innata inquietud junto a una
galopante insatisfaccin me han servido generalmente de estmulo. Ha sido
en los ltimos aos, en los que mi vida se ha ido deteriorando, consumida
por una terrible ansiedad. Y eso no era bueno ni para vivir, ni para trabajar.
Para dirigir una pelcula es ms importante tener paciencia que talento.
http://www.clubcultura.com/clubcine/clubcineastas/almodovar/vol
verlapelicula/enpalabras02.htm - Acceso en 14 abr. 2006. (Texto adaptado)

21) Segn el texto, Almodvar, en su pelcula Volver, ha vuelto a

a) considerar la maternidad tanto como origen de la vida como de la ficcin.


b) hacer rer al pblico cinematogrfico con el lenguaje de La Mancha.
c) trabajar el universo de la mujer, sus costumbres y sobriedad, como antes.
d) ver el fantasma de una madre, la suya, que se les aparece a sus hijas.

22) Tengo

la impresin, y espero que no sea un sentimiento pasajero, de


que he conseguido encajar una pieza...
La frase destacada en este fragmento se refiere a

34

a) el paso del tiempo.


b) la muerte de todo lo humano.
c) la muerte en s misma.
d) los fantasmas de su pueblo.

23) Segn el texto, es INCORRECTO afirmar que Almodvar


a) confiesa que cree en las apariciones ficcionales.
b) considera que dirigir una pelcula requiere parsimonia.
c) ha dado el ttulo Volver a una de sus ltimas pelculas.
d) se ha vuelto inquieto en los ltimos aos.

24) En el texto, la palabra desajuste (lnea 11) puede ser sustituida, sin
comprometer el sentido original de la frase, por
a) desnivel.
b) desconcierto.
c) desaprensin.
d) desacomodacin.

Las dos expresiones destacadas en estos fragmentos pueden ser


reemplazadas sin comprometer el sentido original de las frases por,
respectivamente,
a) como mximo / en tanto.
b) cuando ms / aunque.
c) como mximo / pero.
d) por lo menos / no obstante.

29) ... antro, retrete, ergstula pestilente...

Es INCORRECTO afirmar que, en este fragmento, la enumeracin se refiere


a) a la celda.
b) al cuartucho.
c) al cubculo.
d) al presidio.

30) Segn el texto, es INCORRECTO afirmar que

25) Segn el texto, para Almodvar, volver al seno materno significa


a) encontrarse una vez ms a solas con su madre.
b) enfrentarse a sus recuerdos y fantasmas.
c) poder ver el fantasma de su madre nuevamente.
d) volver a la regin que lo conecta con el origen.

a) diecisiete presos ya se haban muerto, de manera estpida, pero dejaron


las herramientas para la apertura del tnel.
b) la guerra civil haba concluido pero la excavacin del tnel todava no
terminaba aunque ya haca cuatro meses que haba comenzado.
c) media docena de presos cavaban la tierra con el plato de metal y hacan
desaparecer la tierra en la lata de orinar.
d) ni el primer desprendimiento de tierra, ni las tinieblas, ni la creciente
humedad asustaron enseguida al preso.

TEXTO VI

TEXTO VII
La excavacin

Aumenta la familia de planetas extrasolares

El primer desprendimiento de tierra se produjo a unos tres


metros, a sus espaldas. No le pareci al principio nada alarmante. Sera
solamente una veta blanda del terreno de arriba. Las tinieblas apenas se
pusieron un poco ms densas en el angosto agujero por el que nicamente
arrastrndose sobre el vientre un hombre poda avanzar o retroceder. No
poda detenerse ahora. Sigui avanzando con el plato de hojalata que le
serva de perforador. La creciente humedad que iba impregnando la tosca
dura lo alentaba. La barranca ya no estara lejos; a lo sumo, unos cuatro o
cinco metros, lo que representaba unos veinticinco das ms de trabajo hasta
el boquete liberador sobre el ro.
Alternndose en turnos seguidos de cuatro horas, seis presos
hacan avanzar la excavacin veinte centmetros diariamente. Hubieran
podido avanzar ms rpido, pero la capacidad de trabajo estaba limitada por
la posibilidad de desalojar la tierra en el tacho de desperdicios sin que fuera
notada. Se haban abstenido de orinar en la lata que entraba y sala dos
veces al da. Lo hacan en los rincones de la celda hmeda y agrietada, con lo
que si bien aumentaban el hedor siniestro de la reclusin, ganaban tambin
unos cuantos centmetros ms de bodega para el contrabando de la tierra
excavada.
La guerra civil haba concluido seis meses atrs. La perforacin
del tnel duraba cuatro. Entre tanto, haban fallecido, por diversas causas, no
del todo apacibles, diecisiete de los ochenta y nueve presos polticos que se
hallaban amontonados en esa inhspita celda, antro, retrete, ergstula
pestilente, donde en tiempos de calma no haban entrado nunca ms de ocho
o diez presos comunes.
De los diecisiete presos que haban tenido la estpida ocurrencia
de morirse, a nueve se haban llevado distintas enfermedades contradas
antes o despus de la prisin; a cuatro, los apremios urgentes de la cmara
de torturas; a dos, la rauda ventosa de la tisis galopante. Otros dos se haban
suicidado abrindose las venas, uno con la pa de la hebilla del cinto; el otro,
con el plato, cuyo borde afil en la pared, y que ahora serva de herramienta
para la apertura del tnel.
ROA BASTOS, Augusto. Cuento.

Ya son 200 los detectados en otras galaxias. Anunciada esta


semana por cientficos europeos, la deteccin de un planeta a 20 aos luz de
la Tierra, que podra tener una temperatura de entre 0 y 40 grados, sum un
nuevo integrante a una familia creciente: la de los planetas extrasolares, cuya
bsqueda constituye uno de los campos ms animados de la astronoma
moderna. Encontrar algunos semejantes a la Tierra y a una distancia
adecuada para la vida es una de las metas ms ambiciosas de los
especialistas.
Por eso, detectar en la constelacin de Libra a este primo lejano
de la Tierra orbitando una dbil estrella conocida como Gliese 581 encendi
las esperanzas de los investigadores, ya que es el ms pequeo entre los
otros integrantes de la familia que existen fuera del sistema solar. Adems,
su rbita est dentro de lo que se considera la zona habitable, en la que el
agua podra existir si las dems condiciones son las correctas, segn dijo
Stephane Udry, del Observatorio de Ginebra. Sin embargo, l y otros
astrnomos advirtieron que era muy pronto para concluir que este nuevo
planeta tena agua lquida sin ms observaciones. Por ejemplo, si el planeta
tuviera una atmsfera ms masiva que la de Venus, entonces la superficie
sera demasiado caliente, explic al The New York Times Sara Seager, del
Massachusetts Institute of Technology.
De todos modos el descubrimiento en la rbita de Gliese 581
donde ya se haba descubierto otro planeta del tamao de Neptuno, y se
sospecha que hay otro ocho veces mayor que la Tierra catapult a este
sistema al tope de la lista de candidatos para futuras generaciones de
misiones espaciales.
http://www.lanacion.com.ar/903819. Acceso: 27 abr. 2007. (Adaptado)

26) Segn el texto, es CORRECTO afirmar que

32) A partir de las informaciones que trae el texto acerca del planeta recin

27) Seale

33) Segn

a) aumentaba el hedor en la celda porque los presos oinaban en el tnel.


b) quedaban unos veinticinco das ms de trabajo porque excavaban en
turnos.
c) sobrevivi solamente uno de los diecisiete presos que quisieron morirse.
d) tena ya por lo menos unos tres metros de largo el tnel que excavaban.
la opcin en que la correspondencia entre la palabra y su
sinnimo entre corchetes est INCORRECTA.
a) angosto [= estrecho]
b) alentaba [= animaba]
c) apacibles [= tranquilas]
d) veta [= roca]

31) Ya son 200 los detectados en otras galaxias.

A partir de esta frase, es CORRECTO afirmar que


a) es preocupante que existan planetas fuera del sistema solar.
b) no se dieron cuenta los cientficos de que haba planetas en otras galaxias.
c) se sabe hoy que hay muchos planetas extrasolares.
d) tard muchsimo el descubrimiento de ese pequeo planeta.

detectado, es INCORRECTO afirmar que


a) el reciente hallazgo impulsa las investigaciones sobre el sistema Gliese.
b) ste es el planeta ms pequeo de los detectados en otras galaxias.
c) este planeta cuya temperatura no alcanza los 40 grados tiene agua.
d) este planeta extrasolar gira alrededor de una estrella llamada Gliese 581.

el texto, es CORRECTO afirmar que la bsqueda de planetas


extrasolares es interesante para la humanidad porque
a) aumenta la esperanza de encontrar condiciones de vida en otro planeta.
b) evidenci una zona con todas las condiciones necesarias para la vida.
c) facilita la investigacin de fenmenos solares que favorecen la vida.
d) quita importancia a futuras misiones espaciales fuera del sistema solar.

28) La barranca ya no estara lejos; a lo sumo, unos cuatro o cinco metros 34) Seale
...
Entre tanto, haban fallecido, por diversas causas...

la opcin en la que la expresin destacada NO puede ser


sustituida por las palabras entre corchetes sin comprometer el sentido de la
frase en el texto.
a) Adems, su rbita est dentro [Tambin]
b) De todos modos el descubrimiento [De esa manera]

35

c) Por eso, detectar [] a este primo [Por lo tanto]


d) Sin embargo, l y otros astrnomos [No obstante]
TEXTO VIII
Malinche, de Laura Esquivel, se enfila para ser un xito editorial
La novela Malinche sirve a Laura Esquivel (Mxico, 1950) para construir
una ventana en la que el lector se asoma a la forma de ver el mundo de los
pueblos indgenas. En la novela intenta comprender mucho mejor el proceso
de la conquista y por qu no hemos podido superarlo. Y es que, como
nacin, tenemos un nudo atorado que tiene que ver con la integracin de esa
visin del mundo que acepta un orden csmico. Una parte de la profunda
espiritualidad de los pueblos indgenas se basa en ello, en presencias y
deidades, donde el agua, el fuego y el aire hablan, nos dicen cosas. Esa
creencia absoluta de formar parte del todo, de estar en la intemporalidad, se
contrapone a la visin de la temporalidad que lleg con los espaoles. Es la
idea del imperio, donde ste no forma parte de nada ni de nadie, sino que
todo le pertenece. La tierra deja de ser la madre para convertirse en algo
apropiable, expropiable, al igual que el espacio areo, las
telecomunicaciones, el agua, el petrleo. [] Cuando escriba Malinche me
preguntaba, por qu el afn de culpar a una sola mujer de un proceso que
fue tan complejo? Si bien ella tena el control de la informacin, no hubiera
hecho nada si Moctezuma no hubiera entregado, de entrada, el reino, o si los
tlaxcaltecas y todos los dems no se hubieran unido contra Tenochtitln. La
Malinche no fue una traidora ni exista el concepto de nacin, es un absurdo.
Pero, qu nos lleva a querer culparla? Qu no nos perdonamos para
culparla a ella? Creo que todava no nos perdonamos el olvido y la falta de
respeto en la que tenemos a nuestros indgenas. [] Aade Laura Esquivel:
quise que el lector sintiera ese momento histrico mediante actos muy
cotidianos y lo que significaba para ella, por ejemplo, encender el fuego. Lo
poquito que hay de la Malinche narrado por los cronistas me sirvi de
estructura dramtica y a partir de ah hago todo un marco histrico que
arrope esos hechos.
http://www.jornada.unam.mx/2006/04/06/a04n1cul.php.

a) Ella tena el control de la informacin conque no hubiera hecho


Moctezuma no hubiera entregado, de entrada, el reino.
b) Ella tena el control de la informacin, pero no hubiera hecho
Moctezuma no hubiera entregado, de entrada, el reino.
c) Ella tena el control de la informacin por eso no hubiera hecho
Moctezuma no hubiera entregado, de entrada, el reino.
d) Ella tena el control de la informacin porque no hubiera hecho
Moctezuma no hubiera entregado, de entrada, el reino.

nada si
nada si
nada si
nada si

37) Es

CORRECTO afirmar que Laura Esquivel sostiene que su novela


Malinche
a) aade a la Historia mexicana aspectos de la conquista que no han sido
superados hasta el presente momento.
b) defiende la idea de falta de salidas para los pueblos indgenas y para los
mexicanos, en manos de malos gobiernos.
c) exalta una visin del mundo intemporal y csmica en contraposicin a otra
temporalidad que lleg a Mxico con los espaoles.
d) problematiza la tendencia constante de culpar a una sola mujer, Malinche,
de todo lo acontecido en la conquista.

38) Segn las informaciones del texto, es INCORRECTO afirmar que

a) an hay una serie de conflictos en Mxico debido a la permanencia del


imperio espaol.
b) quizs el pueblo mexicano no se perdone el hecho de haberse olvidado de
los indgenas.
c) segn la visin del imperio, la tierra, el agua, el petrleo son apropiables y
expropiables.
d) todava la nacin mexicana no logr incorporar la aceptacin de un orden
csmico intemporal.

35) hago todo un marco histrico que arrope esos hechos.

Es CORRECTO afirmar que este fragmento del texto se refiere a que


Esquivel
a) desmiente los hechos narrados por los cronistas utilizando elementos
histricos.
b) envuelve los hechos y los dimensiona a partir de una construccin
histrica.
c) inventa un marco histrico para omitir los hechos de la vida de Malinche.
d) llega a construir un marco histrico que contradice los hechos
comprobados.

1) A

2) A

3) C

4) B

5) A

6) B

7) B

8) A

9) A

10) A

11) A

12) A

13) D

14) C

15) C

16) B

17) B

18) A

19) D

20) A

21) A

22) C

23) D

24) D

36) Si bien ella tena el control de la informacin, no hubiera hecho nada si 25) D
Moctezuma no hubiera entregado, de entrada, el reino...

Seale la opcin en la que, en el perodo transformado, se mantiene la


misma relacin entre las informaciones.

37) D

26) D

27) D

28) A

29) D

30) A

31) C

32) C

33) A

34) B

35) B

36) B

38) A

36

FURG COPERVE PROCESSO SELETIVO 2009


AS QUESTES DE 1 A 4 REFEREM-SE AO TEXTO 1.
Texto 1
O Google no quer que voc veja isto aqui
Quartis, fronteiras, zonas de guerra; conhea os locais censurados pelo site
1. Com o Google Maps, voc pode ver o mundo inteiro
2. de cima. Ou melhor: quase. Pouca gente sabe, mas o
3. servio de mapas fortemente censurado dezenas
4. de lugares, de bases militares a instalaes governa5. mentais, tm suas imagens bloqueadas. Pode parecer
6. teoria da conspirao, mas no : a empresa admite
7. que h censura. Mas diz que no tem culpa. O Goo8. gle no distorce as imagens. Ns usamos as fotos
9. que recebemos dos nossos fornecedores. E algumas
10. delas vm, sim, alteradas, disse um porta-voz do
11. Google ao jornal San Francisco Chronicle. As res12.ponsveis pela censura seriam as empresas que
13. operam os satlites fotogrficos e vendem suas
14. imagens ao Google. Elas assumem a responsabili15.dade. Ns restringimos imagens que possam colo16.car os EUA em risco, diz Chuck Herring, da empre17.sa DigitalGlobe. E a censura no pra a; as fotos de
18. vrios outros pases tambm so manipuladas.
(Texto de Bruno Garattoni. Superinteressante Edio 256 Agosto 2008)
1) Observe as sugestes de reescrita de construes do texto 1.
I que operam (linhas 12 e 13) por operadoras
II que h censura (linha 7) por haver censura
III que possam colocar em risco (linhas 15 e 16) por de risco
IV que no tem culpa (linha 7) por desculpado
V que recebemos (linha 9) por recebidas
Assinale a opo que contm reescritas adequadas ao texto e que dispensem
qualquer outra alterao.
a) I III
b) II V
c) III IV
d) I V
e) I II
2) Analise as seguintes afirmativas sobre ocorrncias no texto 1.
I Quase (linha 2) refere-se a ver o mundo inteiro de cima.
II O subttulo sugere que o Google no publica somente fotos originais.
III Ou melhor (linha 2) introduz uma possibilidade mais restrita do que a
mencionada anteriormente.
IV Substitui-se voc pode (linha 1) por possvel sem prejuzo do sentido
inicial.
V Com (linha 1) expressa companhia.
Escolha a alternativa que contm afirmativas corretas.
a) I II V
b) I II IV
c) I III IV
d) II IV V
e) II III IV
3) Leia as seguintes afirmativas sobre ocorrncias no texto 1
I Ns (linha 8) refere-se a Google (linhas 7 e 8).
II A pontuao est inadequada perante a norma gramatical na linha 5.
III O uso de seriam (linha 12) sugere incerteza quanto ao contedo da
afirmao.
IV A (linha 17) refere-se censura imposta ao Google.
V Manipuladas (linha 18) implica adulterao de fotos.
Assinale a alternativa que indica as afirmativas verdadeiras
a) I III V
b) III IV V
c) II III IV
d) I IV V
e) I II IV
4) Escolha a alternativa que contm afirmativa correta sobre o texto 1.
a) Na construo E a censura no pra a (linha 17), a palavra sublinhada
denota idias consecutivas.
b) Na construo E algumas delas vm, sim, alteradas (linhas 9 e 10), a
palavra sublinhada denota idias em oposio.
c) A palavra que empregada duas vezes, na linha 7, apresenta a mesma
classe gramatical do que utilizado na linha 9.

d) Observa-se a mesma referncia no emprego de elas (linha 14) e de suas


(linha 13).
e) A palavra ns tem a mesma referncia nas linhas 8 e 15.
AS QUESTES 5 A 11 REFEREM-SE AO TEXTO 2.
Texto 2
Voc ainda vai sentir saudades do EUA
A China superpotncia comea a mostrar sua cara. E ela no das mais
bonitas
1. O ex-lder chins Deng Xiaoping tinha um lema para
2. definir sua poltica exterior: taoguang yanghui
3. ou esconda sua capacidade, espere a opor4. tunidade. Dito e feito. H 50 anos, a China Ra5. ramente viajava no carro dos lderes que guia6. vam o planeta. H 10 anos, de vez em quando
7. conquistava o direito de sentar no banco de
8. trs. Hoje, a oportunidade de que Deng Xiao9. ping falava parece ter aparecido. A China d
10. toda pinta de estar interessada em assumir o volan11.te do carro: j disputa com a Alemanha o posto de
12. 3 economia mundial e, se continuar nesse ritmo,
13. em 3 ou 4 dcadas ser capaz de rivalizar com os
14. EUA.
15.Parece uma grande notcia para quem est cansado
16. de ver os americanos dar sozinhos as cartas no
17. planeta (e algum no est?). Mas, a julgar por
18. suas primeiras aes, a superpotncia China no
19. est l muito preocupada em entrar para a histria
20. moderna pela porta da frente. Os chineses no tm
21. grande apreo pela opinio pblica. No parecem
22. se importar com as conseqncias do que fazem. E
23. adoram esconder seus objetivos.
24.Por isso mesmo, ningum sabe ao certo como a
25. China se comportar quando chegar liderana. A
26. nica certeza de que o pas tem duas prioridades
27. internacionais: evitar que a independncia de Tai28.wan seja reconhecida e suprir suas (enormes) ne29.cessidades energticas. A primeira prioridade tem
30. sido moleza: em busca de acordos comerciais ou
31. investimentos em infra-estrutura, muitos pases
32. africanos e latinos (Brasil inclusive) tm concordado
33. em colocar a ilha rebelde no gelo. Para resolver a
34. segunda, a China enfia o p na lama. Em busca de
35. minrio de ferro, leo ou madeira, os chineses
36. saram s compras, em todos os cantos do planeta,
37. sem se preocupar com os detalhes bobagens
38. como poluio, aquecimento global ou a ficha do
39. vendedor.
(Texto de Eduardo Szklarz e Srgio Gwercman. Superinteressante Edio
235 Janeiro 2007)
5) A referncia a carro dos lderes (1 pargrafo) uma metfora para
a) posio secundria na economia.
b) indstria automobilstica de ponta.
c) ascendncia internacional.
d) avano tecnolgico.
e) lugar sem destaque nas relaes internacionais.
6) Quanto ao desenvolvimento da China, o texto afirma que
a) a economia chinesa a terceira no mundo hoje.
b) h 50 anos, o lder chins tinha que utilizar seu prprio carro.
c) a poltica exterior chinesa tem metas explcitas.
d) os carros chineses eram de pssima qualidade.
e) em menos de meio sculo o pas poder ser a principal economia do
mundo.
7) Ao afirmar que a China enfia o p na lama para resolver seus problemas
energticos (linha 34), o autor
a) constata que os chineses compram matria prima de muitos pases.
b) informa que os chineses esto extraindo matria prima de seu solo.
c) avalia que o governo chins no leva em conta a tica das relaes
internacionais.
d) reconhece que a China est determinada a desenvolver sua economia.
e) associa o desenvolvimento chins poluio descontrolada.
8) Assinale a alternativa que contm a reescrita de quando chegar
liderana (linha 25) que causa ALTERAO do sentido inicial da construo.
a) Por chegar liderana.
b) Na liderana.

37

c) Chegando liderana.
d) Ao chegar liderana.
e) Uma vez na liderana.
9) Analise as seguintes afirmativas sobre as seguintes ocorrncias no texto 2.
I Ainda (no ttulo) sugere certeza sobre um evento tido como pouco
provvel.
II Suas (linha 28) refere-se a Taiwan (linha 27).
III Quando (linha 25) marca uma condio para o que a China far.
IV E (no subttulo) marca oposio entre eventos.
V Isso (linha 24) refere-se a E adoram esconder seus objetivos (linhas 22 e
23).
Escolha a alternativa que contm afirmativas corretas.
a) II IV
b) I V
c) II III
d) III IV
e) I IV
10) Escolha a opo que contm a avaliao do autor do texto 2 sobre a
poltica externa da China.
a) E algum no est? (linha 17).
b) A ilha rebelde (linha 33).
c) Enormes (linha 28).
d) Entrar... pela porta da frente (linhas 19 e 20).
e) Bobagens (linha 37).
11) Assinale a opo que d ao texto 2 um tom irnico.
a) Colocar a ilha rebelde no gelo (linha 33).
b) Enfia o p na lama (linha 34).
c) E adoram esconder seus objetivos (linhas 22 e 23).
d) Os chineses no tm grande apreo pela opinio pblica (linhas 20 e 21).
e) Enormes (linha 28).
AS QUESTES 12 A 15 REFEREM-SE AO TEXTO 3.
TEXTO 3
Bula de um medicamento
1. Determinado medicamento tem como substncias
2. ativas a metoclopramida, a dimeticona e apepsina.
3. Elas agem em colaborao entre si de modo a melho4. rar algumas etapas importantes do processo digesti5. vo. A ao do produto se inicia cerca de meia hora
6. aps sua tomada, durando por 4 a 6 horas. A meto7. clopramida uma substncia til no tratamento e
8. preveno de nuseas (enjos) e vmitos. Ela me9. lhora a fora com que o estmago se contrai durante
10. a digesto, acelera o seu tempo de esvaziamento
11. (reduzindo a sensao de peso no estmago aps
12. as refeies) e impede que o alimento volte ao
13. esfago, o que em geral causa a sensao de reflu14. xo (azia, regurgitao). Ela age muito pouco nos
15. intestinos, no sendo causadora de diarria. A dime16. ticona tem como principal ao a destruio de
17. bolhas de gs que se formam dentro do estmago
18. e dos intestinos, reduzindo a sensao de estufa19. mento e peso e os rudos percebidos aps as refei20. es. A pepsina uma substncia capaz de quebrar
21. as protenas da dieta (geralmente vindas de produ22. tos de origem animal e de gros como soja e fei23. jo), ajudando a acelerar o processo de digesto,
24. reduzindo a sensao de desconforto que pode
25. durar vrias horas aps uma refeio rica em prote26. nas.
12) A construo agem em colaborao entre si (linha 3), no texto,
a) significa que as trs substncias tm funo comum.
b) explicita a finalidade das trs substncias.
c) implica que as substncias so incuas quando agem separadamente.
d) deixa dvidas sobre o papel das trs substncias.
e) significa que as trs substncias somam suas funes.
13) Escolha a alternativa que identifica corretamente a relao introduzida
pelas formas verbais terminadas em -ando no texto 3.
a) Sendo (linha 15) e reduzindo (linha 24) conseqncia.
b) Reduzindo expressam relaes diferentes nas linhas 11 e 24.
c) Durando (linha 6) e sendo (linha 15) adio de eventos.
d) Reduzindo (linha 11) e ajudando (linha 23) conseqncia.
e) Ajudando (linha 23) e reduzindo (linha 24) adio de eventos.

14) Assinale a alternativa contendo reescrita que ALTERA o sentido original


de algum trecho do texto.
a) Origem animal e de gros como soja e feijo (linhas 22 e 23) como origem
animal e vegetal.
b) A destruio de bolhas de gs (linhas 16 e 17) como destruir bolhas de
gs.
c) Os rudos percebidos (linha 19) como os rudos que se percebem.
d) Com que o estmago se contrai (linha 9) como de contratibilidade do
estmago.
e) De desconforto (linha 24) como desconfortvel.
15) Assinale a alternativa que identifica corretamente relaes no texto.
a) Os trechos em parnteses traduzem trechos que os antecedem.
b) Ela tem a mesma referncia nas linhas 8 e 14.
c) Aps as refeies (linhas 11e 12) expressa circunstncia de tempo em
relao a reduzindo (na mesma linha).
d) O que empregado duas vezes, na linhas 12 e 13, refere-se ao termo
esfago (linha 13).
e) Ajudando a acelerar o processo de digesto (linha 23) refere-se natureza
da dieta.
UFPel/CES Processo Seletivo UFPel Vero 2009
O texto a seguir servir de base para as questes de 16 a 19.
Mulheres no crcere e a terapia do aplauso
Elas esto no crcere. O crcere no est preparado para elas.
Idealizado para o macho, o crcere no leva em considerao as
especificidades da fmea. Faltam absorventes. No existem creches.
Excluem-se afetividades. Celas apertadas para mulheres que convivem com a
superposio de TPMs, ansiedades, alegrias e depresses.
A distncia da famlia e a falta de recursos fazem com que
mulheres fiquem sem ver suas crianas. Crianas privadas do direito
fundamental de estar com suas mes.
Crianas que perdem o contato com as mes para no crescerem no crcere.
Uma presa, em Garanhuns, Pernambuco, luta para recuperar a
guarda de sua criana, que foi encaminhada para adoo por ela no ter
familiares prximos. Uma criana com cerca de 2 anos de idade, em
Teresina, Piau, nasceu e vive no crcere, no fala e pouco sorri; a me tem
pavor de perd-la para a adoo, sua famlia de Minas Gerais.
Essas mulheres so vtimas do machismo, da necessidade
econmica e do desejo de consumir. So flagradas nas portas dos presdios
com drogas para os companheiros; so seduzidas por traficantes que se
especializaram em abordar mulheres chefes de famlia com dificuldades
econmicas; tambm so vaidosas e, apesar de pobres, querem consumir o
que a televiso ordena que bom.
Um tratamento ofensivo as afeta emocionalmente. A tristeza
facilmente se transforma em fria. Muitas escondem de suas crianas que
esto presas. Sentem vergonha da condio de presas. Na maioria dos casos,
esto convencidas de que so culpadas e que merecem o castigo recebido.
Choram, gritam e se comovem. O crcere despreparado e pequeno demais
para comportar a complexidade das mulheres.
Apesar do aumento do nmero de mulheres presas no Brasil,
especialmente nas rotas do trfico, o sistema penitencirio no se prepara
nem para receb-las, nem para as ressocializar. Faltam presdios femininos,
assim como capacitao especfica para servidores penitencirios que
trabalham com mulheres no crcere. Falta estrutura que considere a
maternidade e que garanta os direitos fundamentais das crianas.
Assim como na sociedade, no crcere o espao da mulher ainda
precrio. O sistema masculino na sua concepo e essncia. Em cidades
como Caic, Rio Grande do Norte, no existe penitenciria feminina. As
mulheres presas so alojadas numa rea improvisada dentro da unidade
masculina. Em Mossor, no mesmo estado, mulheres presas, ainda sem
sentena, aguardam
julgamento numa rea minscula dentro da cadeia pblica masculina. A
presena improvisada das mulheres cria problemas legais e acarreta
insegurana para servidores penitencirios quanto garantia da segurana
geral e da integridade fsica das mulheres.
(Santos,
Brbara.
(Adapt.)
http://carosamigos.terra.com.br/do_site/reportagem_barbara.asp.)
16) As expresses Apesar do aumento () e ...nem para as
ressocializar, no sexto pargrafo,
estabelecem relaes lgico-semnticas, respectivamente, de
a) condio e contraste.
b) alternncia e dvida.
c) concesso e adio.
d) oposio e concluso.
e) explicao e excluso.
17) A personificao uma figura de linguagem que consiste na atribuio
de caractersticas humanas a seres inanimados ou animais, ou simplesmente
na atribuio de caractersticas de seres vivos a coisas inanimadas. Essa
figura aparece em todas as frases, EXCETO em:

38

a) (...) querem consumir o que a televiso ordena que bom. (4


pargrafo)
b) Falta estrutura que considere a maternidade (...) (6 pargrafo)
c) (...) o crcere no leva em considerao as especificidades da fmea. (1
pargrafo)
d) Elas esto no crcere (1 pargrafo)
e) (...) o sistema penitencirio no se prepara nem para receb-las (...) (6
pargrafo)
18) A autora do texto menciona todos estes fatores como problemas
que afetam as prisioneiras, EXCETO a
a) terapia do aplauso.
b) difcil convivncia com filhos.
c) inexistncia de absorventes.
d) precria estrutura.
e) falta de creches.
19) Essas mulheres (...) (4 pargrafo) refere-se s
a) mulheres que so traficantes.
b) mulheres encarceradas que do seus filhos para adoo.
c) mes que so obrigadas a deixar seus filhos com parentes.
d) prisioneiras, que lidam com a falta de estrutura relativa maternidade.
e) presidirias, que escondem sua real situao de suas famlias.
O texto a seguir, retirado do jornal Zero Hora edio do dia 13 de setembro
de 2008 , servir de base para as questes de 20 a 23.
Corpo, doena, cultura
O corpo um dos meios atravs dos quais nossa cultura se
manifesta. Roupas, alimentos, gestos, rituais no lazer e no trabalho, se
expressam na superfcie simblica de nosso corpo, onde tambm as regras,
as hierarquias, as ideologias e as crenas esto inscritas. O corpo, pois, tem
sua linguagem e se constitui numa metfora da cultura na qual vivemos.
Mas ele no somente um texto da cultura, tambm objeto de
controle social.
Rotinas e regras convertidas em atividades habituais constroem
nosso corpo e, reflexamente, o robotizam. Quais dceis ces pavlovianos,
reagimos sempre da mesma maneira diante dos mesmos estmulos.
Por meio da organizao e regulao do tempo, espao e da vida
diria, somos treinados, modelados e marcados para desempenhar um papel
conforme o momento histrico em que vivemos.
Especialmente com as mulheres, sempre foram dispensados mais
tempo e cuidados no manejo de seus corpos, os quais, disciplinados e
normatizados, tornaram-se presa fcil de uma estratgia de controle social.
Nos tempos atuais, de um narcisismo exacerbado e de uma cultura de
imagens, a obsessiva preocupao com a aparncia fsica afeta mais
fortemente o sexo feminino, acentua e mantm o domnio falocrtico e
cristaliza as relaes de poder entre os sexos.
Mas, assim como o corpo, tambm as doenas tm sua histria.
Influenciadas pelos modismos, so intensamente medicalizadas pelos
progressos tcnicos e comerciais da indstria farmacutica, alterados seus
diagnstico e a teraputica. Novas patologias foram criadas, sendo, algumas,
puras elucubraes, elaboradas com propsitos diversos que no mdicos. E,
novamente, as mulheres so as mais envolvidas.
Como a bruxaria e as possesses demonacas na Idade Mdia, a
neurastenia e a histeria no sculo 19, em nosso sculo, a adolescncia, a
gestao, a menstruao, a menopausa, foram adoecidas e, portanto,
tornaram-se passveis de tratamentos.
Assim, vemos que, no passado como no presente, o corpo
sofredor da mulher se inscreve numa construo ideolgica da feminilidade,
caracterstica de cada perodo histrico. Portanto no apenas a tica social, a
aparncia fsica, a sexualidade, o vesturio, a alimentao, so poltica e
ideologicamente conceitualizadas, mas as doenas e seus enfoques
teraputicos tambm o so.
A busca intensa de uma imagem corporal estereotipada e da
perene juventude atormenta mais as mulheres. E esses objetivos femininos
so estabelecidos por especialistas que muscularizaram e masculinizaram o
corpo feminino a tal ponto, que at os suaves e arredondados perfis
corporais e certas manifestaes fisiolgicas prprias do gnero foram
includas na lista de patologias.
As mulheres so as maiores consumidoras de produtos dietticos
e medicamentos, freqentadoras de spas e academias de ginstica, objetos
de tcnicas e experincias de contracepo, de esterilizao, de cirurgias
plsticas e outras. s quais, docilmente, se submetem.
Talvez no seja este o caminho para a autonomia feminina. Pois
remodelar o corpo segundo padres e desejos androcrticos no possibilita
acesso ao poder e aos privilgios masculinos. Desejar sentir-se autnoma e
livre, submetendo corpo e alma s escravizantes prticas
corporais e sociais, resulta em servido, no em rebelio e transformao de
uma cultura que sempre tratou a mulher como um mero pssaro cativo e
decorativo.

(Franklin Cunha (Adapt.))


20) No incio do texto, Cunha refere-se a linguagens culturalmente
imbudas e inscritas na superfcie simblica de nosso corpo.
Analisa possveis causas para essa manifestao.
I. pelas rugas e/ou traos fisionmicos.
II. pela hipertrofia muscular.
III. pelas tatuagens, conotativas dos valores individuais e coletivos.
IV. pela manifestao de tecido adiposo.
Esto corretamente elencadas
a) I, III e IV apenas.
b) II, III e IV apenas.
c) I e II apenas.
d) II e III apenas.
e) I, II, III e IV.
21) Percebemos, ao longo do texto, o uso de alguns recursos coesivos.
Quanto a isso, analisa as asseres a seguir.
I. O nexo pois, no primeiro pargrafo, poderia ser substitudo por
destarte, sem prejuzo ao sentido global do texto.
II. As duas primeiras frases do ltimo pargrafo, no tocante aos preceitos da
gramtica padro, poderiam ser reescritas como: Talvez no seja esse o
caminho para a autonomia feminina, em virtude de remodelar o corpo
segundo padres e desejos androcrticos no possibilitar acesso ao poder e
aos privilgios masculinos.
III. O pronome o no final do 8 pargrafo retoma vesturio.
IV. O relativo na qual no final do primeiro pargrafo retoma metfora.
Esto corretas apenas as afirmaes
a) III e IV. b) I e III. c) I e II.
d) II e IV. e) I, II e III.
22) Ao longo do texto, o autor manifesta vrias opinies. Assinala a
alternativa que corrobore o exarado pelo texto.
a) Muitas mulheres tomam determinadas decises sobre o prprio corpo e
acreditam que isso seja de forma autnoma. O autor rechaa tal idia.
b) Os homens so menos afeitos imagem corporal ditada pela
sociedade/mdia.
c) Certas manifestaes fisiolgicas do gnero foram acertadamente
estereotipadas como patologias.
d) Ainda se vivssemos em outro tempo e cultura, estaramos submetidos s
mesmas influncias hodiernas.
e) Por sua maior propenso a acreditar em fenmenos msticos, a mulher
assaz medicalizada em nossa cultura.
23) A aluso ao corpo como uma metfora da cultura na qual vivemos uma
das ocorrncias dessa figura de linguagem que presenciamos no texto.
Assinala a alternativa que apresenta tanto uma metfora quanto
uma correta interpretao do texto.
a) Todas as patologias do corpo tm longa historicidade. Entend-las
essencial para o tratamento.
b) Atividades espordicas robotizam o corpo, tornando-o mais propenso a
problemas.
c) Diversas etapas da vida da mulher foram adoecidas, em virtude de sua
insero no mercado de trabalho.
d) Malgrado os aparentes avanos da mulher, as relaes de poder entre os
gneros encontram-se cristalizadas.
e) Na viso da sociedade, antigamente falocrtica, a mulher um pssaro
preso, cuja virtude reside precipuamente em seu efeito de ornamentao.
(UCPel) Leia o texto a seguir para responder s questes 24 e 25.
AQUELE RANCHINHO
Ao inspirado poeta rio-grandense Bernardo Taveira
(ROMANCE)
Tu me perguntas a histria
Daquele triste ranchinho,
Que abandonado encontramos,
Coberto por negros ramos
De pessegueiro maninho;
Aquele rancho de palha,
Aquele triste ranchinho? ...
Ai foi um drama de sangue
Que ali se deu ... pois no vs?
Repara para as janelas ...
O fogo passou por elas!
. H quantos anos? . H trs.
Contou-me o velho moleiro
H pouco menos de um ms.

39

Ali morava um velhinho


E mais, um anjo de amor;
Criana bela e morena
Mais formosa que a aucena...
Maria, . a plida flor,
Cujo perfume recende
Ainda aos ps do Senhor.
Maria e Vito se amaram
Iam seus fados unir,
Quando a trombeta da guerra
Plangente ecoou na serra
Convocando a reunir:
Parte o audaz cavalariano,
Porm antes de partir...
Porm antes... entre beijos
Juraram constncia enfim:
Se eu morrer numa batalha,
Nesta casinha de palha
Tu vivers s por mim? ....
A moa beijou-lhe a fronte
E respondeu-lhe: Pois sim..
Os anos voam! H tempo
Que Ela no ri como si...
Chora a triste sertaneja,
Quando por fim lhe negreja
Uma notcia que di!
Morrera Vito em combate...
Morrera como um heri.
Vestiu luto a pobrezinha
O velho tambm vestiu...
Cede por fim a ternura,
E pouco a pouco a tristura
No peito se lhe extinguiu;
Se Ele morreu, foi destino
Foi a sorte que o feriu!
Depois correu pela riba
Uma nova singular:
Que a bela flor do posteiro
Co filho de um fazendeiro
Ia de pronto casar;
Causou abalo a notcia,
Sem que ousassem duvidar.
Uma noite a tempestade
Batia pelos cips,
Gemia o vento nos montes
E as guas frias das fontes
Desciam com rouca voz...
E no rancho do posteiro
Dois noivos dormiam ss.
De repente pela encosta
Um cavaleiro desceu;
Molhado o poncho brilhava
Ao resplendor da saraiva
Que resvalava do cu...
Era um vulto negro... negro...
Trazia enorme chapu!
Soltando a rdea ao cavalo,
Ao rancho foi espreitar...
O vento rugia ao longe
E o bosque - sombrio monge Estava como a rezar...
luz de um raio se abre
A porta de par em par!
Sobre o leito precipita-se
O campons sem temor!
No punho a adaga flutua
E nas mos aperta a sua
Primeira sombra de amor!
Uma luta ento se trava,
Sendo Vito o vencedor.
Prfida! Brada o gacho,
Vs o teu noivo? Morreu!

Morrers tambm, ingrata!


E a fria adaga de prata
Bem nos ares suspendeu.
Baixou a mo e trs vezes
No alvo seio a embebeu.
No outro dia os destroos
De um rancho via-se ento.
O incndio levara tudo
E fora cmplice mudo
Fora cmplice o trovo!
- A tens a histria que pedes,
Do ranchinho do serto!
(LOBO DA COSTA, Francisco. Auras do sul. Rio Grande: Pinto & Cia, 1914.)
24) No verso Prfida! brada o gacho, (v.85), a palavra prfida s NO
pode ser entendida como
a) enganadora.
b) falsa.
c) proba.
d) aleivosa.
e) traidora.
25) Leia as afirmativas a seguir.
I . O poema transmite ao leitor uma sensao de letargia e inrcia.
II . As trs ltimas estrofes contam como Maria incendeia o rancho.
III . A quarta estrofe do poema afirma que Maria jurou constncia e
fidelidade a Vito.
a) Todas as afirmativas esto corretas.
b) Todas as afirmativas esto erradas.
c) A primeira e a segunda esto corretas.
d) A segunda e a terceira esto corretas.
e) Apenas a terceira est correta.
(UCPel) Leia o texto a seguir para as questes 26 e 27.
Consolo na praia
Vamos no chores...
A infncia est perdida
A mocidade est perdida
Mas a vida no se perdeu
O primeiro amor passou.
O segundo amor passou.
O terceiro amor passou.
Mas o corao continua.
Perdeste o melhor amigo.
No tentaste qualquer viagem.
No possuis casa, navio, terra.
Mas tens um co.
Algumas palavras duras,
em voz mansa, te golpearam.
Nunca, nunca cicatrizam.
Mas, e o humour?
A injustia no se resolve
sombra do mundo errado
murmuraste um protesto tmido.
Mas viro outros.
Tudo somado, devias
precipitar-te - de vez - nas guas.
Ests nu na areia, no vento...
Dorme, meu filho.
(DRUMMOND DE ANDRADE, Carlos. Poemas. Rio de Janeiro: Jos Olympio,
1959.)
26) No verso A mocidade est perdida, a palavra perdida s no pode ser
entendida como:
a) sumida.
b) irrecupervel.
c) sem esperana.
d) longnqua.
e) desvairada.
27) Leia as frases a seguir e marque a opo correta:
I . As cinco primeiras estrofes do poema terminam com uma mensagem
positiva.
II . O poema transmite ao leitor tristeza e esperana.
III. A ltima estrofe do poema afirma que ir praia estimula o sono.

40

a) Todas as afirmativas esto corretas.


b) Todas as afirmativas esto erradas.
c) A primeira e a segunda esto corretas.
d) A segunda e a terceira esto corretas.
e) Apenas a terceira est correta.

Os indcios arqueolgicos mais remotos de uso da bola foram


encontrados em runas egpcias e datam de 3000 a.C. Isso no quer dizer
que tenha surgido nesse momento, o que, certamente, aconteceu bem
antes, explica Flvio Campos, historiador da Universidade de So Paulo.
Entre os egpcios, alguns tipos de jogos foram retratados em monumentos, e
uma bola foi encontrada ao lado do corpo mumificado de uma menina, o que
CONCURSO VESTIBULAR-UFSC/2009
sinaliza para o fato de terem sido um dos primeiros brinquedos infantis da
humanidade.
A skrita na internet
No que fossem coisas exclusivas de criana entre nossos
antepassados. At porque, desde sempre, os melhores jogadores de bola
O internets conhecido como o portugus digitado na internet, desfrutaram privilgios. No que diz respeito ao futebol, ele se constituiu num
caracterizado por simplificaes de palavras que levariam em considerao, jogo de smbolos, gestos e ritos que so compreensveis em todo o mundo,
principalmente, uma suposta interferncia da fala na escrita. O vocbulo expe o historiador da USP. Ir ao estdio, de certa forma, uma procisso.
aponta ainda para a prtica de escrita tomada como registro divergente da As pessoas rumam ao templo, ao campo, no apenas para ver os jogadores,
norma culta padro.
mas tambm para apreciar a bola, a deusa. Para v-la rolar, girar,
Os avessos a essa prtica de escrita consideram que os adeptos do internets movimentar-se e mudar tudo sua volta. Inclusive suas vidas.
so assassinos da lngua portuguesa. Nesse contexto, perguntas como H
Em outras pocas, essa simbologia implcita na redonda esteve
um processo de transformao da escrita com o uso da internet? ou H mais ntida. Alguns relatos dos primeiros sculos da Era Crist do conta de
degradao da escrita com a introduo da internet na vida das pessoas? clrigos praticando o harpastum um jogo de bola romano que um dos
so cada vez mais freqentes.
precursores do futebol em cemitrios. Eles o viam como uma espcie de
, pois, com base nesse critrio de pureza projetada como ideal da escrita ponte entre o mundo dos vivos e o dos mortos.
que muitos indivduos fazem a crtica ao internets, tomando-o como a noEmbora os romanos praticassem, alm do harpastum, uma
lngua portuguesa. A imagem de degradao da escrita (e, por extenso, da grande variedade de jogos com a bola, como o follis (espcie de vlei) e a
lngua) pelo uso da tecnologia digital resultado da idia de que h uma pillatrigonalis (ancestral do tnis), o elo mais importante estabelecido por
modalidade de escrita pura, associada seja norma culta padro, seja eles foi o de propagar esses jogos pela Europa. Durante a Idade Mdia,
gramtica, seja imagem de seu uso por autores literrios consagrados. apesar da insistncia das autoridades medievais em proibi-la, a bola se
Haveria, assim, um tipo de escrita sem interferncias da fala, que deveria tornou extremamente popular. Ela exercia grande atrao sobre as pessoas
ser seguido por todos, em quaisquer circunstncias.
e preocupava os poderosos, que temiam a violncia, que sempre fez parte da
As idias correntes de pureza da escrita e de empobrecimento do portugus natureza desses jogos, diz Flvio Campos.
podem ser encontradas em inmeros materiais que circulam na sociedade,
Entretanto, esse no era o nico temor. Muitos jovens, no pice
includos comentrios dos prprios usurios da internet. Na rede de da virilidade, preferiam bater uma bolinha a praticar o arco-e-flecha. Em
relacionamentos Orkut, h quase uma centena de comunidades com ttulos tempos de guerras constantes e da fragilizao dos exrcitos pelas mortes de
como Odeiu gnti ki ixcrevi axim!!!, em referncia s prticas de escrita na soldados por meio da peste negra, bons arqueiros no poderiam ser
internet. Para os que participam dessas comunidades, a escrita na internet dispensados. Por esses motivos, somente entre os anos de 1324 e 1667, na
seria uma forma rude de comunicao, algo parecido com os grunhidos que o Gr-Bretanha, os boleiros foram proibidos de atuar por mais de 30 decretos
ser humano fazia nos tempos da caverna.
reais.
Assim concebida, a escrita da/na internet vista como empobrecimento do
No fim, no passaram de tentativas frustradas de sobrepujar o
idioma. Esse mesmo conceito o que, muitas vezes, se atribui aos usos que caso de amor entre o homem e a pelota, que, entre um impedimento e
fazem os indivduos no dotados da tecnologia da escrita alfabtica, ditos outro, continuou a rolar. Na Bretanha, jogos com bola aconteciam no meio da
analfabetos ou no letrados.
rua, envolvendo multides e, via de regra, resultavam em casas destrudas,
(Extrado de: KOMESU, Fabiana C. A skrita na internet. Discutindo Lngua pessoas feridas e at mortas.
Portuguesa [especial]: ano 1, n. 1, p. 56-57, 2008.)
Tanta violncia certamente acabaria por conduzir a bola
obscuridade. Contudo, a redonda, ciente do perigo, mudou de estratgia e
28) Considerando o Texto 1, assinale a(s) proposio(es) CORRETA(S).
mirou suas baterias nos palcios. J no sculo 14, o rei francs Carlos V
01. Trata-se de um texto expositivo, em que a autora contrape argumentos introduziu uma sala no Palcio do Louvre para a prtica do jeudepaume
favorveis e argumentos desfavorveis prtica do internets.
(jogado com raquetes). Nem mesmo a Igreja resistiu aos seus encantos. Os
02. O nmero crescente de indivduos analfabetos e de usurios da internet papas Clemente VII e Urbano VII foram algumas das celebridades do
responsvel pelo processo de degradao da lngua portuguesa que se passado adeptas do calcio italiano.
verifica atualmente.
O fato que a cada rei ou eclesistico arrebanhado pela bola, os
04. A idia principal do texto que o desenvolvimento da tecnologia digital jogos ganhavam sobrevida. Essa foi a ttica durante os sculos 16 e 17, at
deve ser contido, caso contrrio a lngua portuguesa estar fadada a que, no comeo do 18, a pelota foi parar nos colgios ingleses. Uma jogada
desaparecer num futuro relativamente prximo.
de mestre, diga-se de passagem. Adotadas como atividades fsicas dos filhos
08. Do quarto pargrafo do texto, deduz-se que os crticos do internets que ricos da nao, as modalidades receberam normas de conduta para controlar
fazem parte da rede de relacionamentos Orkut acreditam que as prticas de a dureza, permitindo que regras, materiais e, por fim, os esportes fossem
escrita na internet sejam semelhantes s formas primitivas de comunicao padronizados.
entre os seres humanos.
A aristocracia, contudo, no contava com as voltas que a bola
16. Tanto a palavra destacada no ttulo do texto quanto a frase-ttulo daria. Aprisionada nas fbricas em jornadas de trabalho que antes no
destacada no quarto pargrafo podem exemplificar, adequadamente, a existiam, a recente classe operria precisava de alguma distrao para digerir
definio de internets presente no primeiro pargrafo.
a nova ordem. E escolheram o futebol. Os industriais, antenados nas
32. Apesar de o texto abordar um tema polmico, a autora no se posiciona mudanas, no perderam tempo. Na dcada de 1920, na Frana, as direes
claramente em relao ao internets, limitando-se a defini-lo e a expor da Peugeot e da Michelin, entre outras, instalaram campos de futebol para
algumas crticas feitas bem como razes para tais posicionamentos.
seus funcionrios nas fbricas. A bola, triunfal, voltava ao povo.
64. A autora do texto defende a idia de que fala e escrita so modalidades
Revista Terra, maio de 2006.[Adapt.]
completamente independentes uma da outra, e de que nenhum tipo de
escrita deve apresentar interferncias da fala.
29) A pontuao um recurso formal imprescindvel para a construo do
Esto corretas:
sentido do texto. Em alguns casos, a mudana da pontuao pode implicar
a) 01 e 02
alterao do contedo veiculado. Assinala a alternativa cuja proposta de
b) 02, 04 e 16
interpretao das frases esteja INCORRETA.
c) 01, 02, 16 e 32
a) Os industriais, antenados nas mudanas, no perderam tempo. (10
d) 02, 08 e 64
pargrafo) significa que todos os industriais estavam antenados nas
e) 08, 16 e 32
mudanas e no perderam tempo. / Os industriais antenados nas mudanas
no perderam tempo. significa que apenas os industriais antenados em
UFPel PAVE 2007/2009 1 etapa (Aplicao: 15/12/2007)
mudanas no perderam tempo.
b) A bola, triunfal, voltava ao povo. (10 pargrafo) significa que a bola
Texto para as questes de 29 a 32.
era triunfal e, assim, voltava ao povo. / A bola triunfal voltava ao povo.
significa que um determinado tipo de bola a triunfal voltava ao povo.
A dona do mundo
c) Muitos jovens, no pice da virilidade, preferiam bater uma bolinha (...)
(6 pargrafo) significa que, por serem jovens, as pessoas esto no mais alto
No princpio no era o verbo. Era a bola. Quer dizer, antes mesmo grau de vigor e essas pessoas preferiam jogar futebol. / Muitos jovens no
de aprimorar a cultura, o homem provavelmente j praticava jogos. H pice da virilidade preferiam bater uma bolinha significa que unicamente os
milnios a bola arrebata seguidores, sem se ocupar de distncias fsicas, jovens com alto grau de vigor preferiam jogar futebol.
fronteiras polticas ou divises tnicas. Isso tudo muito quadrado para ela.

41

d) Ela (...) preocupava os poderosos, que temiam a violncia, ... (5


pargrafo) significa que a bola preocupava os poderosos, pois todos esses
temiam a violncia. / Ela (...) preocupava os poderosos que temiam a
violncia ... significa que a bola preocupava apenas os poderosos que
temiam a violncia.
e) Ir ao estdio, de certa forma, uma procisso. (3 pargrafo) significa
que ir ao estdio , sob determinado aspecto, o mesmo que participar de
uma procisso. / Ir ao estdio de certa forma uma procisso significa que
ir ao estdio de uma determinada maneira o mesmo que participar de uma
procisso.
30) No texto, o autor cria uma metfora, caracterizando a bola como uma
deusa. Se a bola uma deusa, os espectadores seriam
a) gladiadores.
b) reis.
c) adoradores.
d) soldados.
e) prncipes.
31) Marca a alternativa cuja leitura seja autorizada pelo texto.
a) Na antiguidade, a bola era um objeto restrito ao universo feminino.
b) A prtica de esportes pelos poderosos foi fundamental para a
sobrevivncia dos jogos.
c) Desde o incio dos tempos at hoje os jogos necessariamente envolvem
violncia.
d) Os jogos precederam o desenvolvimento cultural, salvo o surgimento do
homem.
e) Invariavelmente, as pessoas morrem por causa dos jogos.
32) Os diferentes nveis de linguagem so identificados tanto nos textos
falados quanto nos textos escritos. Certas construes e vocbulos so
apropriados para um determinado tipo de texto e imprprios para outro tipo.
Considerando o objetivo do texto acima, h certas palavras que, devido
carga semntica que carregam, causam uma certa estranheza ao leitor.
Marca a alternativa abaixo em que aparea um desses elementos e a razo
da estranheza esteja corretamente registrada.
a) bolinha (preferiam bater uma bolinha) implica tom pejorativo referente
pouca habilidade dos jogadores de futebol.
b) procisso ( uma procisso) supe uma ridiculizao dos hbitos
religiosos adotados por certas pessoas.
c) encantos (Nem mesmo a Igreja resistiu aos seus encantos) supe
zombaria da fragilidade do carter de certos homens que se deixam seduzir
por um objeto to banal.
d) povo (voltava ao povo) supe certo preconceito social com relao s
pessoas de baixo poder aquisitivo.
e) mestre (Uma jogada de mestre) implica exaltao da inteligncia mpar
do autor da nova estratgia utilizada pela bola.
UFPel PAVE 2006/2008 2 etapa (Aplicao: 15/12/2007)
Texto para as questes 33 e 34.
No blog da psicloga Rosely Sayo, encontramos o seguinte texto, postado
no dia 21 de outubro de 2007.
Do inqurito ao dilogo
Os pais querem saber de tudo da vida dos filhos, principalmente o que
se passa com eles na escola. Quando vo busc-los ou noite, quando se
encontram, o momento do inqurito: "Como foi na escola hoje?"; "O que
fez na hora do recreio?"; "O que aprendeu?", "Quem so seus amigos?" etc.
Nessa hora, o que incomoda mesmo os pais, em geral, so as respostas
evasivas e monossilbicas que recebem dos filhos, como "normal" ou "tudo
bem".
Essa nsia de saber tudo o que ocorre com o filho, principalmente no
espao escolar, tem suas razes. Vivemos na era do controle, e o aparelho de
telefone celular nas mos de crianas um sinal emblemtico desse anseio
paterno. Mas, no final das contas, creio que esse recurso faz com que os pais
que acabem controlados pelos filhos. To importante quanto esse fato a
relao de perda dos filhos que os pais experimentam quando os levam para
a escola.
Esta representa o mundo para os mais novos, como bem diz Hannah
Arendt e, portanto, representa tambm para os pais o incio da trajetria que
os separar dos filhos. E, numa poca de laos afetivos to efmeros, os pais
se ressentem dessa perda. Saber de tudo o que se passa com eles , em
certa medida, uma maneira de resistir ao afastamento. Curioso tentar
integrar a essa ltima idia os resultados de uma pesquisa feita pela Ipsos
Public Affairs, comentados numa reportagem da Folha no ltimo dia 12.

Surpresa: a pesquisa constata que os pais tm mais prazer de assistir TV


do que de passear ou brincar com os filhos.
Como podemos entender essa contradio? Afinal, os pais querem os
filhos bem perto, mas no sabem o que fazer quando esto com eles!
que, ao tentarem controlar a vida dos filhos para terem a sensao de
estarem prximos a eles, os pais se esqueceram de como se relacionar de
modo humanizado com eles. Os pais da atualidade sabem muito bem xeretar
a vida dos filhos, mas no sabem como interagir com eles.
Quando o interrogatrio com o filho no funciona, e costuma mesmo
no funcionar, apelam para a mesma estratgia com os colegas e amigos dos
filhos. Tambm na busca por pistas informativas, vasculham o computador,
os dirios, ouvem conversas telefnicas etc.
Outro dia ouvi um depoimento simples de um pai sobre como conseguiu
ter da filha mais do que respostas lacnicas. Contou ele que, cansado de
perguntar sobre a escola e no ter respostas, decidiu entabular com ela uma
conversa. Disps-se a contar a respeito de seu dia, de seu trabalho, das
dificuldades e dos bons momentos que vivenciara.
A partir de ento, no precisou mais perguntar sobre a escola: no
dilogo e espontaneamente, a filha passou a falar das vivncias escolares
significativas para ela.
O que esse pai descobriu, longe de ser uma estratgia para ter as
informaes que queria, foi uma forma de interagir com a filha, de se colocar
na relao, de dialogar e de coloc-la num lugar privilegiado: o de autora de
sua narrativa, e no o de mera respondente de perguntas. Talvez muitos pais
prefiram ver TV ou fazer questionrios aos filhos porque estas so atividades
que no exigem interao.
http://blogdaroselysayao.blog.uol.com.br/index.html (acessado em 22 de
outubro de 2007)
33) Observa o seguinte trecho:
Nessa hora, o que incomoda mesmo os pais, em geral, so as respostas
evasivas e monossilbicas que recebem dos filhos, como "normal" ou "tudo
bem".
Evidentemente ambas as palavras grafadas, que corresponderiam s
respostas dos filhos, possuem mais de uma slaba. Com base nisso, a autora
t-las chamado de monossilbicas
a) revela um desconhecimento do significado desse termo.
b) constitui-se um jogo semntico, um trocadilho com o sentido de
evasivas.
c) apresenta a mesma confuso semntica de outros termos da gria, como
pior no sentido de mesmo.
d) apropriado, visto que o termo foi utilizado em seu sentido conotativo.
e) tem sua razo de ser no fato de que o blog utiliza a linguagem informal
passvel de erros.
34) Conforme a convenincia, a autora se valeu do discurso tanto direto
quanto indireto. Com base nisso, rel o seguinte trecho.
Contou ele que, cansado de perguntar sobre a escola e no ter respostas,
decidiu entabular com ela uma conversa. Disps-se a contar a respeito de
seu dia, de seu trabalho, das dificuldades e dos bons momentos que
vivenciara.
Supondo que, por uma razo estilstica, a autora quisesse transformar esse
trecho todo em discurso direto, assinala a alternativa que contm a correta
transformao.
a) Cansado de perguntar sobre a escola, ele contou: decidi entabular com
ela uma conversa. Dispus-me a contar a respeito do meu dia, do meu
trabalho, das dificuldades e dos bons momentos que vivenciara.
b) Ele contou que: como estava cansado de perguntar sobre a escola, decidi
entabular com ela uma conversa. Dispus-me a contar a respeito de seu dia,
de seu trabalho, das dificuldades e dos bons momentos que vivenciara.
c) Ele contou: como estava cansado de perguntar sobre a escola, decidi
entabular com ela uma conversa. Disps-se a contar a respeito de seu dia,
de seu trabalho, das dificuldades e dos bons momentos que vivenciara.
d) Ele contou: pra no perguntar sobre a escola, decidi entabular com ela
uma conversa. Dispus-me a contar a respeito do meu dia, do meu trabalho,
das dificuldades e dos bons momentos por ns vivenciados.
e) Ele contou: como estava cansado de perguntar sobre a escola, decidi
entabular com ela uma conversa. Dispus-me a contar a respeito do meu dia,
do meu trabalho, das dificuldades e dos bons momentos que vivenciara.
UFPel PAVE 2005/2007 3 etapa (Aplicao: 15/12/2007)
Texto para as questes 35 a
O crime de Luciano Huck
No conheo Luciano Huck. No costumo ver seus programas na
TV. Sei que ele um apresentador de sucesso. Sei que rico. Sei que
casado com uma linda mulher chamada Anglica. Sei tambm que, no incio
do ms, ele foi assaltado em So Paulo e levaram-lhe um relgio da marca
Rolex. Movido por sua experincia pessoal, Huck escreveu um artigo cheio de
revolta para protestar contra a violncia urbana. verdade que o valor e o

42

teor de seu artigo so discutveis. verdade que Huck pode ser considerado
um mauricinho mimado, um alienado em sua gaiola de ouro, que
demorou a perceber o alcance do problema na sociedade brasileira. Tudo
isso pode ser verdade. Mas tudo isso tambm direito dele. Luciano Huck
tem direito a ser mimado, tem direito a ser mauricinho, tem direito a ser
alienado e tem direito a escrever o que quiser e a publicar onde bem
entender. Luciano Huck tambm tem direito a ser rico e a fazer sucesso.
Luciano Huck, pelo que consta, no cometeu nenhum crime. Foi o assaltante
que levou seu relgio que o fez. Por que, ento, tamanha revolta e tamanho
dio contra o artigo de Huck em blogs, cartas de leitores e manifestaes
populares e to pouca revolta contra o criminoso?
Eis um mistrio da alma brasileira.
A alma que v um criminoso no rico que faz sucesso e um
coitado, ou at uma vtima, no pobre que cometeu um assalto. A alma que
condena um filme como Tropa de Elite um retrato da guerra entre polcia e
trfico no Rio de Janeiro - apenas porque ele no transforma os policiais em
viles, mas trata-os como gente cruel, mas de carne e osso. A alma que
incapaz de enxergar uma contradio flagrante na juventude que consome as
drogas que financiam a violncia e, ao mesmo tempo, vai s ruas protestar
pela paz. Se quisermos ter esperana de, algum dia, viver numa sociedade
menos violenta, essa alma precisa mudar. Que o Brasil no perca a
oportunidade oferecida por Tropa de Elite e pelo caso de Luciano Huck. o
momento de deixar claro o que crime e o que no , quem culpado e
quem, como Huck, inocente.
Hlio Gurovitz - Revista poca 15/10/2007
35) Aps a leitura do texto, correto fazermos todas as afirmaes
seguintes, EXCETO
a) O autor apia o direito de Huck dizer o que quiser.
b) O texto faz uma crtica inverso de valores que a sociedade brasileira
vive no momento atual.
c) Gurovitz culpa a juventude viciada pela violncia que existe na sociedade.
d) Hlio Gurovitz no v possibilidade de mudana na sociedade brasileira.
e) O autor considera Luciano Huck uma pessoa que ignora o contexto social
em que vive.
36) Por que, ento, tamanha revolta e tamanho dio contra o artigo de Huck
em blogs, cartas de leitores e manifestaes populares e to pouca
revolta contra o criminoso?
O trecho entre travesses indica
a) uma exemplificao.
b) uma contradio.
c) uma explicitao.
d) uma explicao.
e) um questionamento.

37) Analisa as afirmativas.


I) Luciano Huck, pelo que consta, no cometeu nenhum crime. (1
pargrafo) a ressalva que o autor faz (pelo que consta) deixa aberta a
possibilidade de que o apresentador tenha cometido algum delito que no
seja de conhecimento pblico.
II) Em Foi o assaltante que levou seu relgio que o fez (1 pargrafo), o
final da frase refere-se a cometer crime.
III) A repetio da palavra alma, no ltimo pargrafo do texto, um
recurso de estilstica.
IV) ... e levaram-lhe um relgio da marca Rolex. (1 pargrafo). Nessa
frase, o verbo levar no pode ser considerado sinnimo de tomar.
V) O mistrio da alma brasileira, segundo o autor, acreditar que
celebridades, como Huck, esto acima do bem e do mal.
Esto corretas
a) II, III e IV.
b) I, II e III.
c) I, IV e V.
d) I, II e V.
e) III, IV e V.

1)

2)

3)

4)

5)

6)

7)

8)

9)

10) d

11) c

12) e

13) d

14) a

15) b

16) c

17) d

18) a

19) d

20) e

21) c

22) a

23) d

24) c

25) b

26) e

27) c

28) e

29) c

30) c

31) b

32) d

33) d

34) e

35) d

36) a

37) b

43

Fundao Faculdade Federal de Cincias Mdicas de Porto Alegre


(2008)
Instruo: As questes de nmeros 01 a 20 referem-se ao texto abaixo.
A (re)humanizao da Medicina
1. Em sua origem, a Medicina Ocidental era
2. uma cincia essencialmente humanstica. Suas razes
3. e seu sistema terico partiam de uma viso holstica
4. que entendia o homem como ser dotado de corpo e
5. esprito. As causas das doenas, portanto, deveriam
6. ser buscadas no apenas no organismo enfermo,
7. mas tambm e principalmente no que h de essenci8. almente humano no homem: a alma, esse componen9. te espiritual que distingue o homem dos outros orga10. nismos vivos do planeta.
11. Este foi o modelo, a concepo de mdico e
12. de medicina que se perpetuou historicamente
13. obviamente no sem mudanas, rupturas e trans14. formaes - no Ocidente at bem pouco tempo
15. atrs. A Antiguidade Latina e a Idade Mdia herda16. riam essa concepo e pouco lhe acrescentariam.
17. certo que no Renascimento, no alvorecer da Mo18. dernidade, muitos dos postulados clssicos comea19. ram a ser revistos, graas ao esprito investigativo
20. que caracterizou o perodo ento. Isso, entretanto,
21. no afetou a concepo filosfica da medicina, que,
22. muito mais agora do que na Idade Mdia, se afir23. mava como uma cincia essencialmente humansti24. ca.
25. Durante o sculo XIX forjou-se a imagem
26. romntica do mdico sbio, conhecedor dos avanos
27. cientficos no campo da clnica, da patologia, da
28. farmacologia, mas tambm amante da literatura,
29. da filosofia, da histria. Homem culto, o mdico
30. romntico aliava seus conhecimentos cientficos
31. .........humansticos e utilizava a ambos na formula32. o dos seus diagnsticos e prognsticos. Conhece33. dor da alma humana e da cultura .......... se inseria,
34. j que invariavelmente andava muito prximo
35. .......... seus pacientes, como mdico de famlia que
36. era, este respeitvel doutor sabia que curar era,
37. fundamentalmente, uma operao que envolvia
38. elementos de carter cultural e psicolgico.
39. Por outro lado, essa insero do mdico em
40. seu meio sociocultural, fazia com que seu papel no
41. se restringisse ao de simplesmente atuar na cura
42. das enfermidades. Ele era tambm aquele que,
43. frente aos limites e s impossibilidades mdicas,
44. sabia acompanhar o enfermo e seus familiares no
45. sofrimento e na preparao para a morte, alm de
46. ser aquele que intervinha como orientador nos
47. assuntos mais diversos, tais como o despertar da
48. sexualidade nos adolescentes, os problemas de
49. relacionamento do casal e inmeras outras ques50. tes da vida familiar.
51. Paradoxalmente, o mesmo sculo XIX, que
52. assistiu consagrao da moderna medicina huma53. nstica em sua verso romntica, marcou tambm o
54. incio da sua crise. Principalmente a partir da se55. gunda metade desse sculo, as importantes desco56. bertas em campos, como o da microbiologia, desen57. cadearam uma verdadeira revoluo no terreno da
58. patologia, gerando profundas transformaes na
59. cincia mdica como um todo. Assistia-se a um
60. milagre mpar e, ao se iniciar o sculo XX, tudo
61. dava a entender que a medicina estava prestes a
62. atingir a sua idade de ouro, o seu estgio de cin63. cia exata.
64. Entretanto, medida que o prestgio das
65. cincias experimentais foi crescendo, o das cincias
66. humanas esvanecia-se no meio mdico, e a medici67. na, ento, passava a se sustentar essencialmente
68. nas cincias exatas e biolgicas. As descobertas
69. ainda mais surpreendentes que ocorreram nas
70. ltimas dcadas, principalmente no mbito da bio71. logia celular e molecular, que ultimamente tm
72. culminado nas pesquisas do genoma, parecem ter
73. definitivamente confirmado a idia de que a chave
74. de todo o conhecimento mdico est nas cincias
75. experimentais. Vistas desta forma, as cincias hu-

76. manas a histria, a filosofia, a literatura no tm


77. mais nada a dizer medicina, a no ser louvar as
78. suas lutas e conquistas e relatar a sua tremenda
79. evoluo.
80. De fato, todo esse processo de supervalori81. zao das cincias biolgicas e dos meios tecnolgi82. cos, que acompanharam o desenvolvimento sem
83. precedentes da medicina nestas ltimas dcadas,
84. trouxe como conseqncia mais visvel a desuma85. nizao do mdico, que foi se transformando cada
86. vez mais em um tcnico, um especialista. E isso,
87. no apenas por fora das exigncias de uma for88. mao cada vez mais especializada, mas tambm
89. em funo das transformaes nas condies soci90. ais de trabalho que tendem a proletarizar o mdico,
91. restringindo a sua disponibilidade para que haja um
92. contato mais amplo com o paciente que assiste.
93. Certamente, mesmo depois de totalmente
94. desvendado o cdigo gentico e desenvolvidas as
95. mais sofisticadas tcnicas de diagnstico e prog96. nstico clnico, os mdicos continuaro enfrentando
97. limitaes e dificuldades que exigiro mais do que o
98. conhecimento cientfico-tecnolgico para que pos99. sam ser superadas. Sem dvida, h uma grande
100. necessidade de se reumanizar a medicina, de se
101. desenvolver e fornecer recursos humansticos
102. para o processo de formao e de atuao do
103. mdico e dos profissionais da sade em geral 104. no s por ser uma questo de tica, mas porque
105. s se pode falar em verdadeira evoluo do co106. nhecimento biolgico-mdico quando se procura a
107. integrao dos saberes que extrapolam o campo
108. puramente fsico-experimental.
(Dante
Marcello
Claramonte
Gallian
in
http://www.epm.br/reitoria/cehfi/index.htm - texto adaptado)
1) Assinale a alternativa que preenche adequadamente as lacunas do texto,
na ordem em que aparecem.
a) aos em que a
b) com os onde
c) a que dos
d) aos onde com
e) nos na qual a
2) Assinale a alternativa que apresenta a expresso que, se colocada entre
vrgulas imediatamente aps partiam (linha 3), estabelece uma relao de
coerncia com a frase anterior.
a) talvez
b) sobretudo
c) entretanto
d) provavelmente
e) apesar disso
3) Assinale a nica alternativa que encontra suporte no texto.
a) Desde seu surgimento at a atualidade, no mundo todo, a medicina
sempre defendeu o princpio de que o homem deve ser visto de forma
holstica; ou seja, no s seu corpo, mas sua alma deve ser analisada,
diagnosticada e tratada.
b) Ao longo dos sculos, a evoluo do mundo ocidental foi acompanhada de
descobertas cientficas que muito contriburam para a rea mdica, mas isso
at meados do sculo XIX pouco alterou a concepo que se tinha do
ofcio da medicina.
c) A imagem do mdico relacionada de um homem sbio no cabe mais
nos dias de hoje, pois os profissionais da medicina do pouca importncia
cultura e ao conhecimento cientfico.
d) A evoluo do conhecimento cientfico e a precarizao dos servios de
sade transformaram os mdicos em profissionais insensveis e pouco
competentes para cuidar de seres to complexos como o ser humano.
e) A descoberta do cdigo gentico era a ltima limitao que havia no
tocante ao conhecimento sobre as doenas; daqui por diante, pouco h para
ser descoberto sobre o organismo humano e os males que o acometem.
4) A respeito de algumas passagens do texto, so feitas as seguintes
afirmaes.
I Conforme o primeiro pargrafo, os antigos gregos entendiam o homem
como um ser dotado de um corpo fsico e uma essncia espiritual, a qual
chamamos hoje de alma; um mdico devia, portanto, partir desse
pressuposto ao tratar um paciente.
II De acordo com o segundo pargrafo, no Renascimento houve uma srie
de importantes descobertas que influenciaram a medicina em detrimento dos
antigos princpios humansticos que norteavam essa cincia.
III Segundo pode-se inferir na concluso do texto, a medicina deve ser
reumanizada principalmente neste sentido: a formao dos profissionais

44

das cincias mdicas deve voltar a contemplar as cincias humanas, pois isso
os tornar mais capazes de perceberem com maior sensibilidade o ser
humano que h por trs de uma doena.
Quais esto corretas?
a) Apenas I.
b) Apenas II.
c) Apenas III.
d) Apenas I e III.
e) I, II e III.
5) O pronome isso (linha 86) refere-se
a) ao processo de supervalorizao das cincias biolgicas e dos meios
tecnolgicos.
b) ao desenvolvimento sem precedentes da medicina.
c) transformao gradual do mdico humanista em um tcnico, um
especialista.
d) s exigncias de uma formao cada vez mais especializada e
segmentada.
e) proletarizao dos mdicos em funo das condies sociais de trabalho.
6) Considere as seguintes afirmativas acerca de possibilidades de
substituies de termos no texto e suas conseqncias.
I A substituio de acrescentariam (linha 16) por alterariam tornaria
necessria a troca de lhe por a.
II A substituio de esprito investigativo (linha 19) por ndole
investigadora criaria condio para uso da crase.
III A substituio de limites (linha 43) por suas limitaes obrigaria a
troca, indistintamente, de aos por s ou as.
Quais esto corretas?
a) Apenas I.
b) Apenas II.
c) Apenas I e II.
d) Apenas I e III.
e) I, II e III.
7) Considerando o sentido que apresenta no texto, ento (linha 20) poderia
ser corretamente substitudo por
a) na poca.
b) hoje.
c) at agora.
d) enfim.
e) finalmente.
8) Assinale a alternativa cujas palavras podem substituir Paradoxalmente
(linha 51), precedentes (linha 83) e desvendado (linha 94), sem que
ocorra qualquer alterao de sentido ou necessidade de ajuste no perodo em
que se inserem.
a) Contraditoriamente antecedentes revelado
b) Contrariamente sucessores destapado
c) Disparatadamente oponentes descoberto
d) Equivocadamente ancestrais esclarecido
e) Coincidentemente antecessores exposto
9) Considere os seguintes trechos retirados do texto.
I simplesmente atuar na cura das enfermidades (4 pargrafo).
II acompanhar o enfermo e seus familiares (4 pargrafo).
III orientador nos assuntos mais diversos (4 pargrafo).
Quais delas so exemplos da insero do mdico em seu meio sociocultural,
conforme o texto?
a) Apenas I.
b) Apenas II.
c) Apenas III.
d) Apenas I e II.
e) Apenas II e III.
10) Assinale com V (verdadeiro) as ocorrncias em que o QUE substitui uma
palavra ou expresso anteriormente explicitada no texto, e com F (falso) os
casos em que isso no ocorre.
( ) que acompanharam o desenvolvimento (7 pargrafo)
( ) que foi se transformando (7 pargrafo)
( ) que tendem (linhas 7 pargrafo)
( ) que haja (7 pargrafo)
A seqncia correta de preenchimento dos parnteses, de cima para baixo,
:
a) F V V V.
b) F F V V.
c) F F F V.
d) V V V F.
e) V V F F.

a) oposio.
b) adio.
c) explicao.
d) causa.
e) concesso.
12) Em qual das formas verbais, listadas a seguir, o complemento no texto
NO introduzido por preposio obrigatria?
a) restringisse (linha 41).
b) despertar (linha 47).
c) assistiu (linha 52).
d) desencadearam (linhas 56 e 57).
e) assistia-se (linha 59).
13) Se substitussemos a palavra mdico (linha 26) por mdicos, quantas
outras palavras do perodo em que se inserem deveriam ser obrigatoriamente
modificadas para fins de concordncia?
a) Seis.
b) Cinco.
c) Quatro.
d) Trs.
e) Duas.
14) As regras que determinam o uso do acento em mpar (linha 60) e
sade (linha 103) so as mesmas que prescrevem a acentuao,
respectivamente, de
a) razes e bilogo.
b) carter e construram.
c) incio e caracterstica.
d) visvel e mido.
e) cincia e perodo.
15) Considere as seguintes afirmaes acerca da formao de palavras
retiradas do texto.
I As palavras reumanizao e desumanizao apresentam prefixos que
significam, respectivamente, repetio e separao.
II A forma verbal intervinha segue o mesmo modelo de conjugao do
verbo vir; por isso conjuga-se eu intervim, tu intervieste, ele interveio.
III A palavra cura formada por derivao regressiva.
Quais esto corretas?
a) Apenas I.
b) Apenas II.
c) Apenas III.
d) Apenas I e III.
e) I, II e III.
16) Todas as palavras a seguir, retiradas do texto, se tivessem seus acentos
grficos suprimidos, formariam outros vocbulos tambm existentes na
Lngua Portuguesa, exceo de
a) mdico e cientfico.
b) diagnstico e prestgio.
c) sade e inmeras.
d) incio e anlise.
e) clnico e dvida.
17) Considere a reescritura para a frase Entretanto, medida que o

prestgio das cincias experimentais foi crescendo, o das cincias


humanas esvanecia-se no meio mdico (6 pargrafo) e assinale aquela

em que a alterao de tempo e modo verbal acarreta mudana em relao


ao significado original.
a) Porm, ao passo que o prestgio das cincias experimentais crescia, o das
cincias humanas esvanecia-se no meio mdico.
b) Contudo, quando o prestgio das cincias experimentais cresceu, o das
cincias humanas esvaneceu-se no meio mdico.
c) No entanto, quanto mais o prestgio das cincias experimentais ia
crescendo, o das cincias humanas ia esvanecendo-se no meio mdico.
d) Porm, proporo que o prestgio das cincias experimentais foi
crescendo, o das cincias humanas esvanecia-se no meio mdico.
e) Todavia, enquanto o prestgio das cincias experimentais crescia, o das
cincias humanas esvanecia-se no meio mdico.
18) Se a frase Principalmente a partir da segunda metade desse
sculo, as importantes descobertas em campos, como o da
microbiologia, desencadearam uma verdadeira revoluo no terreno
da patologia (5 pargrafo) passasse para a voz passiva, a forma verbal
correta nessa nova frase seria
a) foi desencadeada.
b) foram desencadeadas.
c) desencadeou.
d) tm desencadeado.
e) tem desencadeado.

11) Entre a orao mas porque s se pode falar em verdadeira


evoluo do conhecimento biolgico (8 pargrafo) e a anterior h uma 19) Numere a segunda coluna de acordo com a primeira, associando a
relao de
justificativa correta para o uso da vrgula em relao s ocorrncias no texto.

45

1. Vrgulas intercalam na frase uma informao de tempo.


2. Vrgulas separam elementos de mesmo valor sinttico.
3. Vrgulas separam uma explicao.
( ) Linhas 71 e 72.
( ) Linha 27.
( ) Linha 60.
Assinale a alternativa que apresenta a seqncia correta das respostas, de
cima para baixo.
a) 1 2 3.
b) 1 3 2.
c) 2 1 3.
d) 2 3 1.
e) 3 2 1.
20) Considere as seguintes afirmaes acerca da relao entre letras e
fonemas em palavras do texto.
I Os segmentos sublinhados em componente (linhas 8 e 9),
acrescentariam (linha 16) e Conhecedor (linhas 32 e 33) representam um
s fonema.
II As letras sublinhadas em cincia (linha 2), sociocultural (linha 40) e
adolescentes (linha 48) representam o mesmo fonema.
III As palavras holstica (linha 3), essencialmente (linha 23), e
conquistas (linha 78) tm, cada uma delas, mais letras que fonemas.
Quais esto corretas?
a) Apenas I.
b) Apenas II.
c) Apenas III.
d) Apenas I e III.
e) I, II e III.
(UCPel) Leia o texto a seguir para responder s questes de 21 a 23.
Consolo na praia
1234-

Vamos no chores...
A infncia est perdida
A mocidade est perdida
Mas a vida no se perdeu

5678-

O primeiro amor passou.


O segundo amor passou.
O terceiro amor passou.
Mas o corao continua.

9- Perdeste o melhor amigo.


10- No tentaste qualquer viagem.
11- No possuis casa, navio, terra.
12- Mas tens um co.
13141516-

Algumas palavras duras,


em voz mansa, te golpearam.
Nunca, nunca cicatrizam.
Mas, e o humour?

17181920-

A injustia no se resolve
sombra do mundo errado
murmuraste um protesto tmido.
Mas viro outros.

21- Tudo somado, devias


22- precipitar-te - de vez - nas guas.
23- Ests nu na areia, no vento...
24- Dorme, meu filho.
(DRUMMOND DE ANDRADE, Carlos. Poemas. Rio de Janeiro: Jos Olympio,
1959.)
21) Em .No tentaste qualquer viagem.., a palavra sublinhada
a) conjuno.
b) advrbio.
c) pronome demonstrativo.
d) pronome indefinido.
e) nenhuma das respostas anteriores.
22) A funo sinttica da expresso sublinhada em Dorme, meu filho
(verso 24)
a) sujeito simples.
b) vocativo.
c) aposto.
d) objeto direto.
e) predicativo do sujeito
23) Dorme (verso 24) est flexionado no
a) imperativo afirmativo.
b) presente do indicativo.
c) presente do subjuntivo.
d) pretrito imperfeito do indicativo.
e) pretrito perfeito do indicativo.
(UCPel) Texto para as questes 24 a 26.

DR. FLORISMAL
Florismal!... Para Eugnio, aquele nome tinha um secreto
encanto. Florismal aparecia quase todas as noites, chegava muito calmo,
fumando o seu charuto de tosto e ia logo sentando na cadeira de balano.
Era um homem baixo, de cabelos ralos, quase calvo. No rosto gorducho e
redondo, a barba forte era sempre uma sombra azulada, mesmo quando ele
se escanhoava. Os dentes eram maus e midos. Florismal tinha uma voz
macia e uma certa dignidade de estadista. Era um esprito conciliador e
gabava-se de ter muita lbia. Nasci para advogado - dizia. Se eu tivesse
tido mais um pouco de juzo quando moo .... Calava-se, entortava a cabea,
batia a cinza do charuto e ficava em atitude sonhadora. Decerto a ver
mentalmente o seu passado, os seus erros e uma carreira perdida. Ou ento
pensava apenas no efeito que aquelas palavras e aquela postura podiam
estar produzindo nos interlocutores. A verdade era que amigos e conhecidos
de Florismal sempre o chamavam para dar sentenas, e resolver questes.
Dizia-se que o homenzinho arranjava causas para advogados sem clientela e
ganhava com isso gordas comisses. Muito figuro tirava o chapu ao
cumpriment-lo na rua. Florismal fazia at discursos polticos. Por isso tudo,
os amigos lhe chamavam Dr. Florismal. A princpio, diziam doutor com uma
pontinha de ironia. Florismal aceitava o ttulo sorrindo, entre lisonjeado e
divertido. Acabou ficando mesmo Dr. Florismal. Com o tempo, os amigos que
gostavam dele esqueceram que aquilo era uma brincadeira e acabaram
acreditando no ttulo.
(VERSSIMO, rico. Olhai os lrios do campo. Rio de Janeiro: Editora Jos
Aguilar, 1966.)
24) Em Dizia-se que o homenzinho arranjava causas para advogados sem
clientela ...., a palavra destacada classifica-se gramaticalmente como
a) pronome relativo.
b) conjuno subordinativa integrante.
c) preposio.
d) conjuno subordinativa concessiva.
e) nenhuma das respostas.
25) Leia as afirmativas a seguir.
I . A concordncia verbal no est correta em Fazem anos que Eugnio no
v Florismal.
II . No perodo Com o tempo, os amigos que gostavam dele...., a palavra
sublinhada um pronome relativo.
III . Em A verdade era que amigos e conhecidos de Florismal sempre o
chamavam para dar sentenas...., o termo destacado um objeto direto.
a) Somente a primeira afirmativa est correta.
b) Somente a segunda afirmativa est correta.
c) Somente a terceira afirmativa est correta.
d) Todas as afirmativas esto corretas.
e) Nenhuma das alternativas est correta.
26) Marque a NICA alternativa correta.
a) Haveria de ser meio-dia e meia.
b) No sabemos onde Florismal pretende ir.
c) Lemos esta obra a mais de uma semana.
d) Ela s queria saber porque o chamavam de doutor.
e) Devero haver muitos elogios para ele.
(UNISC) Texto para as questes de 27 a 30.
ENERGIA VERDE DEVE CRIAR 20 MILHOES DE EMPREGOS At 2030 - ONU
Da Reuters, 24/09/2008
NAES UNIDAS - O desenvolvimento de energias alternativas deve
criar mais de 20 milhes de empregos ao redor do mundo nas prximas
dcadas, medida que os governos adotarem polticas de reduo de gases
causadores do efeito estufa, de acordo com um relatrio da ONU.
Cerca de 2,3 milhes de pessoas j trabalham com energia verde, sendo
metade em biocombustveis, diz o relatrio Empregos verdes: rumo ao
trabalho decente em um mundo sustentvel e com baixo carbono, elaborado
e patrocinado pelo programa ambiental da Organizao das Naes Unidas.
A criao de empregos vai depender da implementao e expanso
dessas polticas nos pases, incluindo a reduo de emisses de gases
causadores do efeito estufa e a migrao de subsdios das antigas formas de
energia para as novas, em um esforo para diminuir o aquecimento global,
diz o estudo.
O relatrio foi escrito antes de a crise de crdito dos EUA abalar Wall
Street e repercutir ao redor do mundo, o que pode desacelerar muitos
setores, incluindo o de energias alternativas.
O estudo diz que cerca de 12 milhes de novos empregos podem ser
criados at 2030, na indstria e agricultura voltadas aos biocombustveis.
Os crticos dos biocombustveis dizem que o etanol americano, feito
principalmente de milho, faz pouco para reduzir as emisses de gases
causadores do efeito estufa. Mas as empresas esto correndo para fabricar

46

um etanol mais limpo de fontes como resduos de lavouras e acelerando a


plantao de lavouras no-comestveis.
O relatrio afirma que muitos empregos na indstria de biocombustveis
no so justos com os trabalhadores.
Grande parte dos empregos nas plantaes de cana-de-acar e leo
de palma em pases como Brasil, Colmbia, Malsia e Indonsia so mal
remunerados e feitos em condies insalubres, diz o documento.
Tambm existem preocupaes de que a produo de biocombustveis
em larga escala possa expulsar um grande nmero de pessoas de suas terras
no futuro, afirma o estudo. Um acompanhamento de perto ser necessrio
para determinar que poro dos empregos em biocombustveis pode ser
contadas como decente, acrescenta.
A fabricao, a instalao e a manuteno de painis solares devem
acrescentar outros 6,3 milhes de empregos at 2030, enquanto a energia
elica deve somar mais de dois milhes de empregos. Mais postos de
trabalho podem ser criados nos setores de construo, reciclagem e
fabricao de veculos alternativos, diz o relatrio.
O mundo vai tentar alcanar um acordo que suceda o Protocolo de
Kyoto, para tratar da mudana climtica, em um encontro da ONU em
Kopenhagen, no final do ano de 2010.
27) Observe as formas verbais destacadas em cada uma das passagens a
seguir e assinale (F) para falso e (V) para verdadeiro, considerando seu
emprego no texto e os comentrios a elas atribudos.
( ) Deve criar (1 pargrafo) indica com grande grau de certeza os
acontecimentos decorrentes do desenvolvimento das energias alternativas.
( ) Pode desacelerar (4 pargrafo) considera como provvel, diante do
contexto de instabilidade da economia global, uma retrao inclusive no setor
das energias
alternativas.
( ) Possa expulsar (9 pargrafo) determina que a produo em larga
escala provocar, certamente, a expulso dos trabalhadores do campo.
( ) Devem acrescentar (10 pargrafo) e deve somar (10 pargrafo)
conferem ao enunciado um grau de probabilidade do que s pode ser
prospectado luz de um dado contexto.
A alternativa que preenche os parnteses, adequadamente, de cima para
baixo,
a) V- V - V - V.
b) F - V - F - V.
c) F - F - F - F.
d) V - V - F - F.
e) F - F - V - V.
28) Quanto ao emprego de j (2 pargrafo), podemos dizer que
I- sua retirada no acarreta qualquer alterao de sentido ao enunciado do
ponto de vista discursivo.
II- carrega uma idia de temporalidade, mostrando que a situao
totalmente nova, passando a ser vivida somente a partir do devir.
III- pode ser substitudo por ainda, mantendo o mesmo efeito de sentido.
a) Somente a afirmativa I est correta.
b) Somente a afirmativa II est correta.
c) Somente a afirmativa III est correta.
d) Somente as afirmativas I e II esto corretas.
e) Nenhuma das afirmativas est correta.
29) No texto, so empregadas as construes O estudo diz (5 pargrafo),
Os crticos dos biocombustveis dizem (6 pargrafo), O relatrio afirma
que (7 pargrafo). Quanto ao emprego desse recurso, possvel dizer que
a) como se trata de um texto jornalstico, apresenta problemas na construo
do sujeito discursivo, uma vez que deveria assumir uma perspectiva mais
pessoal.
b) ancora o dizer na voz discursiva de um terceiro, processo que procura,
isentar o enunciador, revestindo de credibilidade a declarao.
c) apresenta problemas quanto ao emprego dos tempos verbais, que
deveriam ser alterados para dizia, diziam e afirmava, respectivamente,
uma vez que se referem s situaes j manifestadas num tempo anterior ao
da divulgao do texto.
d) cria um grau elevado de impessoalidade, distanciando o texto de sua linha
argumentativa.
e) Nenhuma das alternativas anteriores est correta.
30) Os conetivos medida que (1 pargrafo), para (3 pargrafo,
segunda ocorrncia) e Mas (6 pargrafo) estabelecem, nos contextos em
que so empregados, relaes de
a) causa, adio, oposio.
b) proporo, explicao, acrscimo.
c) proporo, finalidade, oposio.
d) tempo, adio, explicao.
e) tempo, finalidade, contrariedade.
(Unisc) Texto para as questes de nmero 31 a 33
Esteretipo influi no sucesso e no fracasso

Um artigo publicado na edio de abril/08 da revista especializada


Scientific American afirma que os esteretipos exercem grande influncia
sobre o sucesso ou o fracasso dos indivduos.
Segundo o artigo, assinado por pesquisadores britnicos, o
fracasso no trabalho, na escola ou em esportes no se deve necessariamente
falta de talento ou incompetncia, mas tambm maneira como cada um
percebe o grupo social ao qual pertence.
Assim, por exemplo, mulheres asiticas que fizeram testes de
matemtica obtiveram melhor desempenho ao serem lembradas de suas
origens asiticas (reforando o esteretipo de que os asiticos so melhores
em matemtica) que ao terem sua identidade feminina destacada (j que,
segundo o esteretipo, mulheres so piores em matemtica que os homens).
Da mesma forma, atletas brancos tiveram pior desempenho em
jogos de golfe quando foram informados de que teriam sua capacidade
atltica natural comparada de jogadores negros. Em compensao, o
grupo melhorou ao acreditar que se tratava de um teste de inteligncia
estratgica esportiva.
Em outros experimentos, pessoas mais velhas tiveram rendimento
pior em testes de memria aps serem lembradas do esteretipo que as
relacionam capacidade cognitiva deteriorada.
Estudos anteriores tentaram vincular esta mudana de
desempenho ao uso de reas da memria que deixariam de ser utilizadas
pelos indivduos submetidos ansiedade da ameaa dos esteretipos.
Entretanto, isto no explicaria por que os esteretipos tambm
podem ajudar a elevar o rendimento de membros de grupos considerados os
melhores - neste caso, esta percepo no altera os recursos de memria
disponveis, disseram os pesquisadores.
Para eles, a explicao que a ameaa dos esteretipos no
tanto uma questo de cognio em si, tambm de imagem pessoal e
identidade.
Embora alguns pesquisadores tenham saltado para a concluso
altamente polmica de que as diferenas de desempenho refletem diferenas
naturais entre os grupos, na verdade a raiz de muitas diferenas repousa
sobre os esteretipos, ou pr-conceitos, que outros tm em relao ao grupo
a que pertencemos, diz o estudo.
Ao mesmo tempo, o artigo afirmou que os esteretipos so
flexveis e podem ser modificados para influenciar o desempenho dos
indivduos.
De muitas maneiras, temos um esteretipo do esteretipo, que
errado. Os esteretipos no so necessariamente ruins e podem inclusive ser
ferramentas de progresso, disse o professor Stephen Reicher, da
Universidade Saint Andrews, na Esccia.
Foi precisamente por desafiar esteretipos que ativistas como
Steve Biko e Emmeline Pankhurst puderam alcanar a emancipao de
negros sul-africanos e de mulheres britnicas.
Para os pesquisadores, os estudos em relao ao tema trazem
duas lies fundamentais. A primeira tomar cuidado para no confundir
desempenho e capacidade, especialmente ao tratar de grupos diferentes
entre si, e compreender a fora que as expectativas dos outros exerce sobre
o que fazemos, dizem os pesquisadores.
A segunda perceber que no estamos fadados a ser vtimas de
esteretipos opressivos, mas que podemos aprender a usar os esteretipos
como ferramentas de nossa liberao.
Fonte: (BBC, Brasil) Folha on-line, 22/04/2008.
31) Observe o emprego de segundo (2 pargrafo), mas tambm (2
pargrafo), assim (3 pargrafo), j que (3 pargrafo) e segundo (3
pargrafo). As relaes de sentido estabelecidas so, respectivamente, de
a) conformidade, adio, sntese, explicao, oposio.
b) proporo, soma, concluso, conseqncia, conformidade.
c) condio, relao, explicao, condio, proporo.
d) conformidade, adio, sntese, explicao,conformidade.
e) proporo, oposio, sntese, explicao, conformidade.
32) Quanto ao recurso de intercalar duas oraes, destacadas no texto pelo
emprego dos parnteses, no terceiro pargrafo, podemos dizer que
I. constitui uma estratgia discursiva manejada com competncia, uma vez
que, no interior da comparao, foram acrescidas duas informaes que
reforam a linha argumentativa assumida no texto.
II. dificulta a produo de sentido, porque o leitor desviado da linha de
raciocnio assumida no texto.
III. poderia ser dispensado sem prejuzo ao sentido,considerando que o
repertrio partilhado entre escritor (enunciador) e leitor possibilita o
preenchimento de implcitos autorizados pelo dizer.
Assinale a alternativa correta.
a) Somente a afirmativa I est correta.
b) Somente a afirmativa II est correta.
c) Somente a afirmativa III est correta.
d) Somente as afirmativas I e II esto corretas.
e) Somente as afirmativas I e III esto corretas.

47

33) Os vocbulos entretanto (7 pargrafo), tambm (8 pargrafo), embora


(9 pargrafo) e inclusive (11 pargrafo) poderiam, sem prejuzo ao sentido
original do texto, ser substitudos, respectivamente, por

a) No entanto - mas tambm - Ainda que - tambm.


b) No entanto - inclusive - Posto que - tambm.
c) Conquanto - mas tambm - Ainda que - tambm.
d) No entanto - mas sim - Posto que - ainda.
e) Porm - mas tambm - Se bem que - ainda.
(CENTRO FEDERAL DE EDUCAO TECNOLGICA DO CEAR) Texto
para as questes 34 a 42
APRECIO A PERSPECTIVA DE DELEITAR-ME NA OBSCURIDADE
Poucos chefes de Estado podem, sem parecer falsos, se dizer
profundamente honrados em receber um visitante annimo. Muito menos
seriam capazes de interromper uma reunio de gabinete para perguntar
sobre a sade de uma jornalista grvida, como Mandela fez em certa ocasio,
dando tapinhas no ventre arredondado da futura me com suas imensas
mos de pugilista.
Raros tambm so aqueles que se atrevem a rir de si mesmos, como
quando Mandela confidenciou, com sua voz lenta e rouca, que deveria
retirar-se da poltica enquanto existiam uma ou duas pessoas que o
admiravam. Ele capaz de se abrir, de dizer coisas que outros no poderiam
dizer e, ao mesmo tempo, guardar sua dignidade, disse sobre Mandela sua
segunda mulher, Graa Machel.
De fato, o chefe de Estado, quando tinha 80 anos, criou um certo
constrangimento quando, em visita Lbia de seu amigo Muammar Khadaffy,
evocou seu amor por Graa Machel ao encerrar uma entrevista em que
acabara de falar sobre a Repblica do Congo, a criminalidade e os direitos
humanos.
Estar apaixonado uma experincia que todo homem deve
experimentar. to maravilhoso para mim... seu amor fez com que eu me
abrisse como uma flor, afirmou. Ao afastar-se dos assuntos polticos
cotidianos, Mandela foi proclamado av da nao e cone de reconciliao,
como definiu o arcebispo Desmond Tutu. Gostaria de descansar. Aprecio a
perspectiva de deleitar-me na obscuridade, disse Mandela ao deixar o cargo.
(O Povo, Fortaleza-Ce, domingo 6 de julho de 2008.)
34) No trecho ... existiam uma ou duas pessoas... (2 pargrafo),
substituindo-se o verbo EXISTIR pelo verbo HAVER, tem-se
a) havia.
b) houvessem.
c) houverem.
d) haviam.
e) hajam.
35) Atente para as afirmaes.
I) As formas verbais acabara (3 pargrafo) e evocou (3 pargrafo)
expressam fatos ocorridos em diferentes momentos.
II) A expresso ... em que (3 pargrafo) traduz uma circunstncia de
lugar.
III) Em um certo constrangimento (3 pargrafo), a presena do artigo
indefinido um reduz o carter de indeterminao expressa pelo pronome
indefinido certo.
IV) Na expresso ... todo homem... (4 pargrafo), o pronome todo foi
usado para expressar que a paixo domina o homem por inteiro.
Esto corretas:
a) I e II
b) II e III
c) III e IV
d) I e IV
e) I e III
36) No trecho Ele capaz... sua dignidade (2 pargrafo),
a) O pronome Ele (2 pargrafo) retoma por antecipao o nome prprio
Mandela.
b) O pronome se (Ele capaz de se abrir) se refere a nenhum nome, vez que
faz parte do verbo abrir.
c) O pronome sua (guardar sua dignidade) refere-se palavra outros (2
pargrafo)
d) O pronome que (que outros no poderiam dizer - 2 pargrafo) refere-se
a um substantivo que est no perodo anterior ao que foi citado no comando
da questo.
e) O pronome outros (2 pargrafo) tem como referente o pronome que,
seu antecedente.
37) O acento indicativo de crase fica mantido, se o topnimo Lbia (3
pargrafo) for trocado por
a) Lisboa
b) Madri
c) Genebra
d) Holanda
e) Roma

38) A palavra tapinhas (1 pargrafo) um exemplo de substantivo de


gnero oscilante, isto , masculino ou feminino. Esto classificadas
corretamente quanto ao gnero:
a) alface (masculino), libido (masculino).
b) mascote (feminino), diabetes (masculino).
c) apndice (feminino), eczema (masculino).
d) cal (masculino), fnix (feminino).
e) sentinela (feminino), gengibre (feminino).
39) H um erro de regncia nominal em
a) Mandela mostrou-se curioso do estado de sade da jornalista.
b) A vida de Mandela, a que se fez aluso no texto, um exemplo de
altrusmo.
c) Nelson Mandela, a despeito de todas as atrocidades que foi vtima, ainda
acredita no amor.
d) Mandela nunca foi negligente em demonstrar amor ao prximo.
e) Mandela, ao comparar-se com uma flor, mostrou-se tambm apaixonado
pela natureza.
40) A palavra arredondado (linha 3) foi formada por parassntese. Pelo
mesmo processo formaram-se
a) descobrir e engessar
b) descampado e enriquecer
c) acarpetar e infelizmente
d) aterrador e ameaar
e) intranqilidade e entardecer
41) A palavra Poucos (1 pargrafo) expressa, no texto, uma quantidade
indeterminada, razo por que se classifica como pronome indefinido. No
aparece esse tipo de pronome em
a) Ningum pode negar que Mandela um exemplo de estadista.
b) Certos lderes permanecem na lembrana do povo por muito tempo.
c) Mandela demonstrou sensibilidade, o que falta a muitos chefes de Estado.
d) Vrios estadistas permanecem na Histria pelo mal que praticaram.
e) Existem pessoas cujos exemplos devem ser seguidos.
42) A orao Ao afastar-se dos assuntos polticos cotidianos (4 pargrafo)
a) exerce funo substantiva
b) equivale a um adjetivo
c) a principal de outras subordinadas
d) exerce a funo sinttica de adjunto adverbial
e) sintaticamente independente
(CENTRO FEDERAL DE EDUCAO TECNOLGICA DO PIAU) Para
responder s questes de 43 a 52, leia o fragmento de conto abaixo,
extrado da obra Primeiras Estrias, de autoria de Joo Guimares
Rosa:
01 -"Vosmec agora me faa a boa obra de querer
me ensinar o que mesmo que : fasmigerado... fazmegerado... famisgeraldo... familhas-gerado. Disse, de
golpe, trazia entre dentes aquela frase. Soava com riso
seco. Mas, o gesto, que se seguiu, imperava-se de toda
a rudez primitiva, de sua presena dilatada.
07 Detinha minha resposta, no queria que eu a
desse de imediato. E j a outro susto vertiginoso
suspendia-me: algum podia ter feito intriga,
invencionice de atribuir-me a palavra de ofensa quele
homem; que muito, pois, que aqui ele se famanasse,
vindo para exigir-me, rosto a rosto, o fatal, a vexatria
satisfao?
14 -"Saiba vosmec que sa 'ind-hoje da Serra, que
vim, sem-parar, essas seis lguas, expresso direto pra
mor de lhe preguntar a pregunta, pelo claro... Se
srio, se era. Transiu-se-me.
18 "L, e por estes meios de caminho, tem nenhum
ningum ciente, nem tem o legitimo - o livro que
aprende as palavras... gente pra informao torta, por
se fingirem de menos ignorncias... S se o padre for
capaz, mas com padres no me dou: eles logo
engambelam... A bem. Agora, se me faz merc,
vosmec me fale, no pau da peroba, no aperfeioado: o
que que , o que j lhe perguntei?"
26 Se simples. Se digo. Transfoi-se-me. Esses
trizes:
28 -Famigerado?
29 -"Sim senhor.. ." - e, alto, repetiu, vezes, o
termo, enfim nos vermelhes da raiva, sua voz fora de
foco. E j me olhava, interpelador, intimativo apertava-me. Tinha eu que descobrir a cara. Famigerado? Habitei prembulos. Bem que eu me

48

carecia noutro nterim, em indcias. Como por socorro,


espiei os trs outros, em seus cavalos, intugidos at
ento, mudos. Mas, Damsio:
37 -"Vosmec declare. Estes a so de nada no.
So da Serra. 56 vieram comigo, pra testemunho. S
tinha de desentalar-me. O homem queria estrito o
caroo: o verivrbio.
41 -Famigerado inxio, "clebre","notrio", "notvel.
43 -"Vosmec mal no veja em minha grossaria no
no entender. Mais me diga: desaforado? caovel?
de arrenegar? Farsncia? Nome de ofensa?"
46 -Vilta nenhuma, nenhum doesto. So expresses
neutras, de outros usos...
48 -"Pois.., e o que que , em fala de pobre,
linguagem de em dia-de-semana?"
50 -Famigerado? Bem. : "importante", que merece
louvor, respeito ...
52 -"Vosmec agarante, pra a paz das mes, mo
na Escritura?"
54 Se certo! Era para se empenhar a barba. Do que
o diabo, ento eu sincero disse:
56 -Olhe: eu, como o senhor me v, com
vantagens, hum, o que eu queria uma hora destas era
ser famigerado - bem famigerado, o mais que
pudesse!...
60 -"Ah, bem! .. ." - soltou, exultante....
43) Nas linhas 16 e 17, o trecho lhe preguntar a pregunta aponta para um
caso tpico de:
a) Pleonasmo;
b) Eufemismo;
c) Metfora;
d) Silepse;
e) Zeugma.
44) Alis, o trecho lhe preguntar a pregunta (linhas 16 e 17) aponta a forte
presena da linguagem:
a) Formal;
b) Erudita;
c) Coloquial;
d) Universal;
e) Gramaticalmente correta.
45) No trecho E j a outro susto vertiginoso suspendia-me: algum podia
ter feito intriga... (linhas 08 e 09), poderia haver a supresso do sinal de
dois-pontos, mantendo-se a coeso e a coerncia de sentido, desde que em
seu lugar se pusesse o articulador:
a) Conquanto;
b) Mas;
c) E;
d) Pois;
e) Enquanto.
46) O pronome demonstrativo quele (linha 10) recebe acento indicativo
de crase devido a:
a) Exigncia da concordncia;
b) Exigncia da regncia;
c) Tratar-se de locuo adverbial feminina;
d) Tratar-se de locuo prepositiva feminina;
e) Tratar-se de locuo conjuntiva feminina.
47) A expresso pra mor de (linha 16) traz ao contexto circunstncia de:
a) Causa;
b) Conseqncia;
c) Finalidade;
d) Concesso;
e) Tempo.
48) Em o que eu queria uma hora destas era ser famigerado (linhas 57 e
58), o termo destacado, no plano morfolgico e observando-se o contexto,
classifica-se como:
a) Artigo;
b) Pronome pessoal;
c) Pronome demonstrativo;
d) Substantivo;
e) Interjeio.
49) Em o que eu queria uma hora destas era ser famigerado (linhas 57 e
58), o termo destacado, no plano sinttico e observando-se o contexto, atua
como:
a) Sujeito;
b) Objeto direto;
c) Objeto indireto;
d) Complemento nominal;

e) Agente da passiva.
50) Na frase soltou, exultante (linha 60), o predicado :
a) Nominal;
b) Verbal;
c) Verbo-nominal, com predicativo do sujeito;
d) Verbo-nominal, com predicativo do objeto;
e) No h predicado, pois se trata de frase nominal.
51) Mas, o gesto, que se seguiu, imperava-se de toda a rudez primitiva, de
sua presena dilatada... (linhas 05 e 06). Sobre esse perodo, julgue as
afirmaes abaixo:
I - A orao que se seguiu tem claramente valor adjetivo no contexto;
II - O trecho mostra que a linguagem utilizada bastante adjetivada, como
se comprova com os termos primitiva e dilatada;
III - O gesto a que se refere o trecho foi deveras amigvel, dir-se-ia
carinhoso.
Est (o) correta (s) a(s) afirmao (es):
a) I, II e III;
b) I e II;
c) I e III;
d) II e III;
e) I.
52) Se na linha 31 o termo intimativo fosse substitudo por intimidativo,
as alteraes mais relevantes seriam de ordem:
a) Semntica;
b) Sinttica;
c) Morfolgica;
d) Morfossinttica;
e) Etimolgica, com mudanas profundas no processo de formao da
palavra.
(CEFET-PI) Para responder s questes de 53 a 58, leia o poema a seguir,
extrado da obra Estrela da Manh, de autoria de Manuel Bandeira:
Trem de Ferro
01
02
03
04
05
06
07
08
09
10
11
12
13
14
15
16
17
18
19
20
21
22
23
24
25
26
27
28
29
30
31
32
33
34
35
36
37
38
39
40
41
42
43
44
45

Caf com po
Caf com po
Caf com po
Virgem Maria que foi isto maquinista?
Agora sim
Caf com po
Agora sim
Voa, fumaa
Corre, cerca
Ai seu foguista
Bota fogo
Na fornalha
Que eu preciso
Muita fora
Muita fora
Muita fora
O...
Foge, bicho
Foge, povo
Passa ponte
Passa poste
Passa pasto
Passa boi
Passa boiada
Passa galho
De ingazeira
Debruada
No riacho
Que vontade
De cantar!
O...
Quando me prendero
No canavi
Cada p de cana
Era um ofici
O...
Menina bonita
Do vestido verde
Me d tua boca
Para mat minha sede
O...
Vou mimbora
vou mimbora
No gosto daqui
Nasci no serto

49

46
47
48
49
50
51
52
53
54

Sou de Ouricuri
O...
Vou depressa
Vou correndo
Vou na toda
Que s levo
Pouca gente
Pouca gente
Pouca gente...

53) Na frase Corre, cerca (verso 09) percebe-se claramente a presena de


um(a):
a) Metonmia;
b) Prosopopia;
c) Pleonasmo;
d) Eufemismo;
e) Antonomsia.
54) Na frase Corre, cerca (verso 09), o termo cerca atua como:
a) Sujeito;
b) Objeto direto;
c) Objeto indireto;
d) Complemento nominal;
e) Vocativo.
55) A respeito do trecho Passa ponte, Passa poste, Passa pasto, Passa boi,
Passa boiada, Passa galho.. (versos 20 a 25) julgue as afirmaes a seguir:
I - A aliterao, com repetio enftica dos fonemas p e s, d noo
imediata dos sons produzidos pelo trem;
ponder s questes de n 59 e 60, leia o fragmento inicial da obra
Auto do Lampio no Alm, de autoria de Gomes Campos:
AUTO DO LAMPIO NO ALM
01
02
03
04
05
06
07
08
09
10
11
12
13
14
15
16
17
18
19
20
21

Bem perto aqui do inferno


Vive um mal-assombrado
Um bando de esqueletos
Num barulho achocalhado
So cabras de Lampio
Que se vo l pro serto
Do inferno escorraados.
O que agora se vai ver
Espanta toda a platia
Pois se preparem os fracos
Vo ter muita diarria
Uma cena no inferno
Lampio briga com co
Muito grito, horror e peia.
Lampio, o cangaceiro,
Expulso dos celestiais,
Vem aqui para o inferno
Pedir asilo aos maiorais
Temos muita guerra vista
Mistrio e assombrao
Muita morte e algo mais.

II - A repetio do verbo passar refora a noo de velocidade e repetio


dos rudos do trem;
III - A repetio do verbo passar no incio dos versos constitui uma figura
de linguagem conhecida como anfora.
Est (o) correta (s) a(s) afirmao (es):
a) I, II e III;
b) I e II;
c) I e III;
d) II e III;
e) I.
56) O que presente no verso 29 sintaticamente exerce funo:
a) Substantiva;
b) Adjetiva;
c) Prepositiva;
d) Conjuntiva;
e) Interjetiva.
57) No plano morfolgico, o que presente no verso 29 atua como:
a) Substantivo;
b) Pronome substantivo;
c) Pronome adjetivo; d) Preposio;
e) Conjuno.
58) No verso 39, percebe-se que, embora o poeta respeite a oralidade, h
uma transgresso norma gramatical culta no que diz respeito sintaxe de:
a) Concordncia nominal;
b) Concordncia verbal;
c) Regncia nominal;
d) Regncia verbal;
e) Colocao pronominal.
(CEFET-PI)
03
04
05
06
07
08
09
10
11
12

Para

res

danada da cachaa
De todas as marcas e cores
No alambique do inferno
Se enriquecem os vendedores.
Dizem que pra matar
Os males da solido
Mas s vezes ela s serve
Pra matar o prprio irmo
Aqui tambm tomam pinga
Fervendo no caldeiro.

3 reprter
(Trazendo o smbolo do jogo)
13 O jogo pra toda gente
14 Um foco de perdio
15 No ajuda, engana e mente
16 Nunca vi tanta iluso
17 Mas s o pobre lascado
18 Se arrasa no furaco.
19 Chega em casa arrasado
20 Pensando s no malogro
21 Amaldioa a mulher
22 Os filhos at o sogro
23 Fica todo macambzio
24 Que s galinha com gogo.

59) O termo celestiais (verso 16) traz ao contexto a circunstncia de:


a) Modo;
b) Causa;
c) Tempo;
d) Intensidade;
e) Lugar.
60) O verbo Pedir (verso 18) atua no contexto como:
a) Verbo de ligao;
b) Verbo transitivo direto;
c) Verbo Transitivo indireto;
d) Verbo Transitivo direto e indireto;
e) Verbo Intransitivo.
(CEFET-PI) Para responder s questes de n 61 a , leia os dois
fragmentos abaixo, extrados de Auto do Lampio no Alm, de
autoria do saudoso professor Gomes Campos, referentes
participao de reprteres na intriga:
2 reprter
(Trazendo a garrafa e o copo de cachaa)
01 O pobre e o rico se agarram
02 Sem vergonha nem temores

1)a
6) C
11) b
16) c
21) d
26) a
31) d
36) a
41) e
46) b
51) b
56) b

2) b
7) a
12) d
17) b
22) b
27) b
32) e
37) d
42) d
47) c
52) a
57) c

3) b
8) a
13) c
18) a
23) a
28) e
33) d
38) b
43) a
48) c
53) b
58) e

4) C
9) e
14) b
19) e
24) b
29) b
34) a
39) c
44) c
49) b
54) e

5) C
10) d
15) e
20) d
25) d
30) c
35) e
40) b
45) d
50) c
55) a

50

1. (Unifesp-SP) Observe a tabela.


Mtodos Contraceptivos de Mulheres por
Tipo de Mtodo, Segundo Grupos de Idade
Grupos Idade (anos)Total* (%)
Plula (%)Esterelizao Feminia (%)
Preservativos (%) Outros (%)
15 a 19
54,1
36,1
0,4
6,4
11,2
20 a 24
66,0
39,5
11,4
5,0
10,1
25 a 29
77,6
32,9
26,9
5,4
12,4
30 a 34
84,3
23,2
42,7
5,0
13,4
35 a 39
83,2
12,4
55,1
3,3
12,4
40 a 44
79,1
7,8
58,3
3,6
9,4
45 a 49
68,6
3,8
51,5
3,2
10,1
* Participao relativa de mulheres que usam mtodos contraceptivos, frente populao total de mulheres
no Brasil.

A tabela permite afirmar que, entre as mulheres,


a) o uso da plula menor entre as que tm at 29 anos.
b) a estelirizao predomina a partir dos 30 anos de idade.
c) o emprego de preservativos aumenta de acordo com a idade.
d) a maioria da faixa de 15 a 19 anos no usa mtodo contraceptivo.
e) as da faixa de 20 a 24 anos so as que mais usam mtodos
contraceptivos.
2. (PUC-SP) Observe com ateno os dados da tabela e sua distribuio
regional:
Porcentagem da populao urbana
servida segundo lugar de domiclio
Abastecimento
Regio e Regio
% Esgoto
de gua p/
metropolitana
Sanitrio
rede geral (%)
NORTE
59,22
13,80
Belm (RM)
76,43
31,29
NORDESTE
77,86
29,24
Salvador (RM)
93,87
68,96
SUDESTE
92,40
82,88
So Paulo (RM)
98,40
85,65
SUL
91,28
51,21
CENTRO-OESTE
73,42
36,41
TOTAL DO PAS
85,52
58,19

% Coleta de
Lixo regular
61,82
82,63
70,72
86,09
91,09
99,84
93,98
83,93
84,49

correto afirmar que:


a) as dificuldades de abastecimento de gua e da organizao de esgoto
sanitrio nas cidades das regies Norte e Nordeste devem-se escassez dos
recursos hdricos.
b) os mais baixos ndices de coleta de lixo ocorrem nas cidades das regies
que produzem mais resduos em funo do dinamismo econmico e dos
elevados padres de consumo.
c) quanto maior a cidade piores sero as condies de abastecimento de
gua e de saneamento em funo das dificuldades naturais de organizar
essas infra-estruturas em grandes aglomeraes.
d) o abastecimento de gua e o saneamento nas cidades dependem muito
mais dos recursos investidos do que do tamanho da cidade e das condies
naturais do stio onde ela se encontra.
e) as grandes cidades das regies mais pobres possuem abastecimento de
gua e saneamento bsico abaixo da mdia nacional, por serem mais
recentes se comparadas s cidades de outras regies.

3. (UFES) Olhar os mapas pode ser esclarecedor. Olhar para eles de ngulos
novos pode ser ainda mais esclarecedor.
(The Economist, 14.01.95. APUD Magnoli, 1993.)
Considerando a afirmao anterior, analise
o emblema oficial da ONU, que utiliza um
mapa com projeo azimutal eqidistante,
e indique a alternativa que explica a
representao do mundo expressa no
emblema.
A centralizao do Plo Sul reunindo os
continentes em torno dele.
a) A Eursia no centro, com os outros
continentes em torno dela.
b) O Plo Norte ao centro e as terras do
Hemisfrio Sul ao longe, distorcidas.
c) Os continentes reunidos, simbolizando a Pangia.
d) Um aglomerado aleatrio dos continentes, a partir da centralizao da
Amrica.

4. (UFMG) Observe os planisfrios, construdos a partir de projees


diferentes.
A partir da anlise e da interpretao dos planisfrios, todas as alternativas
esto corretas, EXCETO:

A representao correspondente ao Planisfrio 1 expressa as reais


propores entre os diferentes continentes que compem a superfcie
terrestre.
a) A representao correspondente ao Planisfrio 2 mostra deformaes de
reas que so tanto maiores quanto mais elevadas as latitudes.
b) A representao correspondente ao Planisfrio 1 possibilita a percepo
correta da configurao das massas continentais, principalmente nas regies
intertropicais.
c) A representao correspondente ao Planisfrio 2 utilizada intensamente,
na navegao area e martima, pela viabilidade de se traarem nela, com
preciso, os rumos de uma rota.
d) A cartografia das reas situadas nas latitudes superiores a 80N e S
invivel, nas duas representaes, devido ao excesso de deformao
decorrente do processo de projeo.
5. (FGV-SP) A distncia real entre so Francisco e Nova York de 4.200 km.
A distncia sobre a carta de 105 mm. Com base nestes dados, assinale a
alternativa que indica corretamente a escala deste mapa.

a) 1 : 400.000
b) 1 : 4200.000
c) 1 : 10.500.000
d) 1 : 40.000.000
e) 1 : 105.000.000
6. (UFRGS) Para cada tipo de representao existe uma escala numrica
apropriada. Assim, os mapas podem ser divididos em trs categorias bsicas:
escala grande, mdia e pequena.
Associe as escalas numricas mais apropriadas para as finalidades dos
mapas.
1 Mapas topogrficos
2 Plantas urbanas
3 Planisfrios
4 Plantas arquitetnicas
( ) 1 : 50 a 1 : 100
( ) 1 : 25.000 a 1 : 250.000
( ) 1 : 500 a 1 : 20.000
A seqncia numrica correta das preenchidas com os nmeros referentes s
mesmas :
a) 4 3 1
b) 4 1 2
c) 2 3 4
d) 4 2 1
e) 3 1 4
7. (Furg-RS) Para obter, em um mapa, informao mais detalhada, qual das
escalas a seguir utilizada?

51

a) 1/100
b) 1/1.000
c) 1/10.000
d) 1/100.000
e) 1/1000.000
8. (UFRGS) Sabendo que a Terra tem um raio mdio de 6371 km e que um
globo que a representa tem 25,4 cm de dimentro, correto afirmar que a
escala desse globo corresponde, aproximadamente, a:
a) 1 : 81.000
b) 1 : 5.000.000
c) 1 : 6.300.000
d) 1 : 50.000.000
e) 1 : 100.000.000
9. (Fatec-SP) Considere os diagramas
esquematicamente o fluxo comercial
desenvolvidos e subdesenvolvidos:

a seguir que representam


predominante entre pases

Os diagramas I e II representam, respectivamente, o comrcio internacional


de produtos:
a) minerais e alimentcios in natura.
b) manufaturados e agropecurios.
c) agropecurios e primrios.
d) manufaturados e minerais.
e) primrios e manufaturados.

11. (UFMG) A Organizao Mundial do Comrcio OMC tem sido o frum de


discusses que envolvem interesses comerciais conflitantes entre pases
pobres e ricos.
Considerando-se esses conflitos comerciais, INCORRETO afirmar que:
a) os pases pobres enfrentam barreiras comerciais, impostas por pases
ricos, sob a acusao de devastarem o meio ambiente, o que reduz entrada
de recursos necessrios modernizao da explorao das riquezas naturais.
b) os pases pobres vm elevando as tarifas alfandegrias impostas aos
produtos industriais dos pases ricos, como forma de estimular o
desenvolvimento da tecnologia nacional.
c) os pases ricos, de modo geral, concedem subsdios a seus produtores
agrcolas, mas rechaam atitudes semelhantes dos pases perifricos em
relao a produtos industriais de exportao.
d) os pases ricos impem restries s exportaes dos pases pobres, como
forma de retaliao contra a suposta explorao da mo-de-obra infantil e do
trabalho em regime de semi-escravido.
12. E(UFMG) Em todas as alternativas, o desempenho da economia de
algumas regies do mundo est corretamente caracterizado, EXCETO em:
a) A maior parte dos pases africanos sofreu, nas ltimas dcadas, um
processo de desestruturao das economias tradicionais, o que contribuiu
para afugentar o capital estrangeiro, excluindo a regio do processo de
globalizao econmica.
b) Os pases da Europa Ocidental tm apresentado, nas ltimas dcadas,
taxas negativas de crescimento econmico, visto que os efeitos da
globalizao comeam a se voltar contra os pases que lideram esse processo
no passado.
c) Os pases denominados Tigres Asiticos integraram-se economia
globalizada, nos anos 70, experimentando um processo de industrializao
baseado no capital e na tecnologia estrangeiros, que os transformou em
verdadeiras plataformas de exportao.
d) Os pases latino-americanos, em grande parte, optaram, na primeira
metade do sculo XX, por uma poltica de substituio de importaes,
protegendo as indstrias nacionais, o que contribuiu para retardar a
competitividade no mercado externo.
13. (PUCC-SP) Os Blocos Econmicos surgiram nas ltimas dcadas, numa
conjuntura internacional marcada:

10. (UFSM-RS) O perodo abaixo retratado representou uma alterao


profunda do comrcio mundial.
Observe o grfico a seguir.

(MAGNOLI, D. e ARAUJO, R. A Nona Geografia Estudos de Geografia


Geral. So Paulo, Moderna, 1995. p. 81.)
Assinale a alternativa que melhor explica o fenmeno representado.
a) A transferncia das multinacionais para a periferia capitalista promoveu o
aumento da participao dos produtos industrializados, energticos e
minerais na pauta de exportao dos pases subdesenvolvidos.
b) A alterao na diviso internacional do trabalho comprova o aumento nas
exportaes primrias; apesar disso, no houve supervit comercial, pois
esse crescimento foi compensado pela reduo nas exportaes de
manufaturas.
c) Os pases subdesenvolvidos, ao se industrializarem, venceram a antiga
dependncia das exportaes primrias, representadas, sobretudo, pelos
produtos alimentcios e minerais, e passaram a responder grande demanda
dos mercados por produtos industrializados.
d) Com a abertura de suas economias ao capital financeiro internacional, os
pases subdesenvolvidos passaram por um processo de alterao de sua base
tcnica de produo, onde a exportao de bens e mercadorias
industrializadas substitui as exportaes tradicionais de alimentos e matriasprimas.
e) A industrializao parcial dos pases subdesenvolvidos alterou suas pautas
de exportaes, diminuindo a participao das commodities (produtos
primrios) e aumentando a dos produtos industrializados.

a) pelo aumento das taxas alfandegrias, forma encontrada pelos diferentes


pases para impedir a desnacionalizao do capital.
b) pelas articulaes realizadas por pases do terceiro mundo, que
precisavam se unir para ter foras para enfrentar os pases industriais
desenvolvidos.
c) pelas tentativas de unificao das culturas, j que os grandes obstculos
para a integrao dos povos residiam nas diferenas nacionais, sobretudo em
razo do idioma e da religio.
d) pela necessidade que os pases capitalistas tinham de impedir o avano
das economias baseadas na planificao e na ditadura do proletariado.
e) pela necessidade da abertura da economia, em um mercado altamente
competitivo, onde as grandes empresas que controlam as tecnologias
acabam adquirindo maior poder de deciso que os Estados nacionais.
14. (Unitau-SP) A globalizao da economia e da sociedade, baseada na
expanso sem precedentes do capitalismo e comandada pelo crescente
domnio das corporaes transnacionais, est levando ao desenvolvimento de
uma nova ordem mundial. Essa nova ordem tem como caractersticas
principais o fim da Guerra Fria, o incremento da guerra comercial entre
empresas e pases e a formao de grandes blocos econmicos regionais.
Baseado no mapa a seguir, identifique alguns desses blocos econmicos,
relacionando-os na seqncia e assinalando a alternativa correta:

52

a) F. Ratzel
c) T. Morus
e) F. Engels.

b) T. Malthus
d) La Blache

19. (Fuvest) Os mapas 1, 2 e 3 a seguir representam respectivamente os


pases:

a) NAFTA, MERCOSUL, Unio Europia, Bloco Asitico


b) Unio Europia, Bloco Asitico, NAFTA, MERCOSUL
c) Unio Europia, NAFTA, MERCOSUL, Bloco Asitico
d) NAFTA, Bloco Asitico, Unio Europia, MERCOSUL
e) NAFTA, MERCOSUL, Bloco Asitico, Unio Europia
15. (UFRGS) Vivemos uma nova revoluo tecnolgica, que une os diversos
lugares do mundo num processo acelerado de globalizao.
A esse respeito, assinale a afirmao INCORRETA.
a) Este processo caracteriza-se pelo desenvolvimento de ramos fundamentais
da tecnologia, como a informtica e a mecatrnica.
b) Uma das caractersticas da globalizao da economia a formao de
megablocos regionais, como a Unio Europia, O MERCOSUL e o Nafta.
c) Os investimentos em Cincia e Tecnologia so fundamentais neste
processo. Os EUA, Japo e frica do Sul, Jamaica, lideram os investimentos
em Cincia e Tecnologia no mundo.
d) Empresas transnacionais como a IBM, SONY e MICROSOFT atuam no setor
da informtica.
e) A comunicao por redes tem possibilitado o armazenamento e a
transmisso de informaes, acelerando e ampliando o conhecimento
cientfico.
16. A dcada de 1990 est sendo marcada pelo recrudescimento das idias
neoliberais entre os governos de muitos pases e pela chamada globalizao
da economia. Esses acontecimentos, embora muito difusos, apresentam
certas caractersticas que os identificam, tais como:
I. A globalizao promove a expanso das multinacionais e a padronizao
dos modelos de consumo, como as cadeias de fast food.
II. O neoliberalismo o prosseguimento econmico da glasnost e da
perestroika, aps o desmantelamento poltico da URSSS.
III. Os neoliberais pretendem que uma das funes do Estado na conduo
da economia manter o controle dos preos atravs de polticas
antiinflacionrias e cambiais, garantindo com isso a estabilidade financeira e
montaria.
IV. Os pases mais prejudicados com a mundializao so aqueles que
constituem o chamado G-7.

a) mais populosos - com predomnio de populao jovem - com altas taxas


de alfabetizao.
b) desenvolvidos - com subalimentao crnica - com altas taxas de
alfabetizao.
c) desenvolvidos -mais populosos - exportadores de alimentos.
d) mais populosos - com alta densidade demogrfica - com predomnio de
populao jovem.
e) exportadores de alimentos - com subalimentao crnica - com alta
densidade demogrfica.
20. O grfico a seguir representa a "Evoluo de Esperana de Vida" entre
1960 e 1985. Agrupando-se os pases representados em trs grandes
conjuntos podemos citar como exemplo, para cada um deles,
respectivamente:
a) I. Etipia e Bangladesh; II. China e Brasil; III. Frana e Sucia.
b) I. Brasil e Egito; II. Bangladesh e lemen; III. Portugal e Reino Unido.
c) I. Portugal e Chile; II. Argentina e ndia;
III. Unio Sovitica e Frana.
d) I. Peru e Brasil; II. Unio Sovitica e
Frana; III. Sua e Alemanha.
e) I. Grcia e Vietn; II. Arglia e Canad;
III. Peru e Brasil.
21. Observe a figura a seguir.

So afirmaes certas apenas:


a) II e III.
b) I e IV.
c) III e IV.
d) I e II.
e) I e III.
17.

O pas ao qual corresponde esta pirmidade de idades :


a) Japo
b) ndia
c) Frana
d) Estados Unidos da Amrica
e) Mxico
22. Interprete o grfico

O esboo I representa os continentes segundo a superfcie. Os esboos II e


III representam cada parte do mundo com uma dimenso proporcional a
diferentes informaes. So elas, respectivamente:
a) renda per capita e volume da produo agrcola.
b) total da populao e renda per capita.
c) valor da produo industrial e percentagem da populao alfabetizada.
d) consumo de energia e renda per capita.
e) total da populao e volume da produo de minrios.
18. Durante a Conferncia Internacional sobre a Populao em
Desenvolvimento, realizada na primeira quinzena de setembro de 1994, no
Egito, foi amplamente discutida a mais famosa teoria sobre populao,
elaborada no final do sculo XVIII, de autoria de:

53

Se o crescimento vegetativo dado pelo espao entre as linhas indicadoras


das taxas de mortalidade e natalidade, ento a populao:
a) tem crescido a taxas menores nos ltimos anos.
b) tem crescido a taxas maiores nos ltimos anos.
c) tem aumentado assustadoramente devido elevada taxa de mortalidade
nos ltimos 10 anos.
d) composta em sua grande maioria de velhos.
e) composta em sua grande maioria de crianas de 1 a 6 anos.

c) As reetrncias na pirmide indicam ter estado essa nao seriamente


envolvida nas duas Guerras Mundiais.
d) A expectativa de vida elevada, embora sejam altos os ndices de
mortalidade infantil.
e) Revela-se um certo desequilbrio entre os sexos nas idades mais elevadas,
confirmando-se a menor longevidade dos homens.
26. (Cesgranrio-RJ) Comparando as pirmides etrias a seguir, pode-se
concluir que:

23. O mapa mostra em destaque parte do continente mais populoso da


Terra. Nesse continente vive mais de 60% da populao mundial.

Assinale a alternativa que contm, pela ordem, o nome correto do continente


e identifica corretamente os pases mais populosos (em ordem crescente).
a) sia, Cambodja e Coria.
b) Europa, China e ndia.
c) sia, ndia e China.
d) sia, China e ndia.
e) Velho Mundo, Coria e Cambodja.
24. Observe atentamente os grficos apresentados a seguir:

a) a pirmide II representa pases com altas densidades demogrficas.


b) a pirmide I representa pases com forte crescimento vegetativo.
c) a pirmide I corresponde a pases de domnio de populao adulta, com
baixa natalidade.
d) a expectativa de vida maior na pirmide II.
e) a pirmide II tpica dos pases que realizaram o controle de natalidade.
27. (UEL-PR) Para responder a esta questo, considere as pirmides etrias
e as afirmaes apresentadas a seguir:

a) Considerando a pirmide 1, descreva o perfil demogrfico previsto na


Zmbia em 2010.
b) Analise, considerando a pirmide 1, o impacto que a epidemia de AIDS e
de outras doenas poder provocar no perfil etrio desse pas.
25. (Cesgranrio-RJ)
I - A pirmide de 1920 se assemelha do Brasil atual.
II - No perodo anterior Segunda Guerra Mundial, era elevado o nmero de
crianas no Japo.
III - A pirmide de 1989 indica uma reduo das taxas de natalidade.
IV - A porcentagem de populao adulta permaneceu estvel entre 1920 e
1989.
V - A expectativa de vida no se alterou entre 1920 e 1989.
Esto corretas SOMENTE:
a) I, II e III.
b) I, II e IV.
c) I, III e V.
d) II, IV e V.
e) III, IV e V.

Observe a pirmide de idade, de meados dos anos 1960, onde esto


indicadas as faixas etrias de homens e mulheres, as datas de nascimento e
respectivos percentuais. Assinale a opo que contm uma interpretao
INCORRETA a respeito das condies demogrficas/econmico-sociais do
pas em questo:
a) Trata-se de um pas desenvolvido economicamente.
b) A nao apresenta tendncia diminuio da natalidade.

28. (UNI-RIO) O crescimento populacional de um pas ou de uma regio


funo de duas variveis: o saldo entre o nmero de imigrantes e o de
emigrantes e o saldo entre o nmero de nascimentos e o de bitos. Sobre o
crescimento populacional do continente africano, correto afirmar-se que
a(o):
a) exploso demogrfica coisa do passado, pois os ndices de crescimento
vegetativo cederam em todo o continente.

54

b) frica no acompanha a transio demogrfica, j que vrios pases


apresentam taxas de incremento populacional superiores a 3,0% anuais.
c) transio demogrfica africana marcha rapidamente em direo a um
crescimento populacional inferior a 1 % ao ano.
d) reduo dos ndices de crescimento est relacionada s grandes taxas de
emigrao resultantes dos grandes focos de tenso existentes nesse
continente.
e) principal fator que sustenta a natalidade explosiva desse continente o
tradicionalismo religioso.

e) no mais possvel atribuir s deficincias culturais as altas taxas de


crescimento vegetativo, pois elas so fruto da necessidade econmica de
reproduo da fora de trabalho em pases pobres.
34. (PUC-MG) Observe atentamente a situao a seguir.

29. (Furg-RS) Primeira afirmativa: Somente o controle rigoroso da natalidade


poder reduzir o excedente populacional.
Segunda afirmativa: A Histria tem demonstrado que o melhor
anticoncepcional a melhoria da qualidade de vida da populao.
a) As duas afirmativas so verdadeiras e a segunda uma justificativa da
primeira.
b) As duas afirmativas so verdadeiras e a segunda no uma justificativa
da primeira.
c) A primeira afirmativa verdadeira e a segunda falsa.
d) A primeira afirmativa falsa e a segunda verdadeira.
e) As duas afirmativas so falsas.
30. As reas destacadas no mapa da frica correspondem s regies:
a) de maior aridez.
b) industrializadas.
c) de maior densidade demogrfica.
d) mais chuvosas.
e) de conflitos tnicos e tribais.
31. (Enem) Qual dos slogans a seguir poderia ser utilizado para defender o
ponto de vista neomalthusiano?
a) Controle populacional - nosso passaporte para o desenvolvimento
b) Sem reformas sociais o pas se reproduz e no produz.
c) Populao abundante, pas forte.
d) O crescimento gera fraternidade e riqueza para todos.
e) Justia social, sinnimo de desenvolvimento.

Analisando os quadros demogrficos dados, assinale a opo INCORRETA:


a) O pas A caracteriza-se por reduzidas taxas de natalidade e mortalidade.
b) O pas B caracteriza-se por apresentar significativo crescimento
demogrfico.
c) A e B representam reflexo da Revoluo Mdico-Sanitria, que reduziu os
ndices de mortalidade.
d) O quadro A o tipo dos pases mais desenvolvidos e com controle
demogrfico.
e) O quadro B exclusivo dos pases emergentes e tem perspectiva de
exploso demogrfica.
35. (UFMG)

32. (Enem) Qual dos slogans a seguir poderia ser utilizado para defender o
ponto de vista dos reformistas?
a) Controle populacional j, ou o pas no resistir!
b) Com sade e educao, o planejamento familiar vir por opo!
c) Populao controlada, pas rico!
d) Basta mais gente, que o pas vai pra frente!
e) Populao menor, educao melhor!
33. (PUCC-SP) Observe o grfico para responder questo.

O fenmeno urbano representado no desenho anterior tem o nome de:


a) stio urbano.
b) hierarquia urbana.
c) conurbao.
d) favelizao.
e) metropolizao.
36. (UFMG) A figura representa um modelo de organizao do espao
geogrfico.

Considerando seus conhecimentos sobre a dinmica da populao mundial e


a relao entre a alfabetizao das mulheres e o crescimento demogrfico,
pode-se afirmar que:
a) nos pases mais pobres, um dos fatores que explicam o crescimento
demogrfico a escolarizao insuficiente, em particular, das mulheres.
b) o forte crescimento demogrfico, tomado como desafio pelos Estados
pobres, pode promover grandes inovaes econmicas e sociais, inclusive
erradicando o analfabetismo.
c) o desenvolvimento econmico dos pases mais pobres est cada vez mais
associado ao papel da mulher na sociedade moderna.
d) inmeros pases do mundo subdesenvolvido tendem a promover, no incio
do sculo XXI, revolues demogrficas com queda do crescimento
vegetativo, como ocorreu na Europa no sculo XIX.

Todas as alternativas contm aspectos


representada nesse modelo, EXCETO:
a) Hierarquia da rede urbana.
b) Relaes interurbanas.
c) Funes urbanas.
d) Interdependncia rural-urbana.

da

organizao

do

espao

55

37. (UFPR)

41. (UFF-RJ) Considerando os termos do acordo de paz firmado entre


israelenses e palestinos em 1993 e as reas assinaladas no mapa, marque a
opo que contm a afirmativa correta.

A megalpole no uma megametrpole. Ela se forma quando h


conurbao de duas ou mais metrpoles ou quando o fluxo de pessoas,
capitais, informaes, mercadorias e servios entre elas est plenamente
integrado por modernas redes de transporte e comunicao.
As megalpoles assinaladas na figura pelos nmeros 1, 2 e 3 so,
respectivamente:
a) Boswash, Chipitts e San-San.
b) Chipitts, Boswash e San-San.
c) Chipitts, San-San e Boswash.
d) San-San, Chipitts e Boswash.
e) San-San, Boswash e Chipitts.
38. (Vunesp-SP) Desde o incio dos anos noventa, a rea assinalada no mapa
adiante tem sido palco de sangrenta guerra civil.

Indique a alternativa que exprime as causas dessa guerra e o pas ao qual


pertenciam estas pores territoriais.
a) Ideal srvio de construir a "Grande Srvia", independente da
Tchecoslovquia.
b) Rivalidades tnicas, religiosas, histricas, culturais e territoriais entre os
povos da antiga Iugoslvia.
c) Antiga oposio poltica entre srvios e croatas na Romnia.
d) Idias separatistas reforadas pela "Glasnost" e pela "Perestroika" entre os
povos das repblicas componentes da antiga Unio Sovitica.
e) Dominao estrangeira entre blgaros da antiga Iugoslvia.
39. (FEI-SP) Muitas vezes chamado de "Vietn da Unio Sovitica", este pas
foi recentemente tomado por fundamentalistas (Taleban) que, ao chegar ao
poder, passaram a realizar reformas extremamente radicais, como a
destruio dos aparelhos de TV e a proibio do trabalho para as mulheres.
Estamos falando do(a):
a) Uzbequisto.
b) Ir.
c) Tchetchnia.
d) Casaquisto.
e) Afeganisto.
40. (UFRGS) A parte sombreada do mapa a seguir mostra a rea ocupada
por uma etnia que luta para conquistar sua autonomia e construir um pas
numa regio rica em petrleo. Recentemente (setembro de 1996), o governo
norte-americano atacou tropas iraquianas que invadiram a rea ocupada por
este povo. Que povo este?
a) Chechenos
b) Palestinos
c) Cipriotas
d) Curdos
e) Bsnios

a) As reas 1 e 2 correspondem, respectivamente, faixa de Gaza e


Cisjordnia, territrios em parte sob o controle da Organizao para a
Libertao da Palestina.
b) As reas 3 e 4 correspondem a territrios anteriormente ocupados por
Israel e que foram cedidos, respectivamente, Sria e ao Lbano.
c) A rea 5 passou a pertencer ao territrio de Israel e inclui colnias
palestinas dotadas de autonomia poltica.
d) A rea 6 corresponde pennsula do Sinai, devolvida integralmente
Jordnia.
e) A rea 7 corresponde a territrios cedidos ao Egito atravs da concesso
de usufruto comum do Canal de Suez.
42. (Enem) A figura apresenta as fronteiras entre os pases envolvidos na
Questo Palestina e um corte, no mapa, da rea indicada.

Com base na anlise dessa figura e considerando o conflito entre rabes e


israelenses, pode-se afirmar que, para Israel, importante manter ocupada a
rea litigiosa por tratar-se de uma regio:
a) de plancie, propcia atividade agropecuria.
b) estratgica, dado que abrange as duas margens do rio Jordo.
c) habitada, majoritariamente, por colnias israelenses.
d) que garante a hegemonia israelense sobre o mar Mediterrneo.
e) estrategicamente situada devido ao relevo e aos recursos hdricos.
Gabarito (Geografia Parte 1)

1) B

2) D

3) C

4) C

5) D

6) B

7) A

8) D

9) E

10) E

11) B

12) B

13) E

14) C

15) C

16) E

17) B

18) B

19) A

20) A

21) E

22) A

23) C

24) a) Predomnio de jovens, altas taxas de natalidade e mortalidade


b) Diminuio nos efetivos populacionais

25) D

26) C

27) A

28) B

29) B

30) C

31) A

32) B

33) A

34) E

35) C

36) C

37) D

38) B

39) E

40) D

41) A

42) E

56

1) (UECE) Sobre a ocupao da Amaznia brasileira correto afirmar que:


a) foram as misses religiosas dos sculos XVII e XVIII que deram incio a
esse processo.
b) o norte do pas foi isento da caa a indgenas por tropas de resgate.
c) o elemento negro, originrio da Amrica Central, teve importncia na
formao do "cabloco amaznico".
d) nos sculos XVIII e XIX ocorreu a fase da borracha, que deu impulso
ocupao da regio.

2) (UFPB)

Sobre a ocupao inicial do territrio paraibano, verdadeira a


afirmativa:
a) Os nativos no ofereceram resistncia ocupao do litoral pelos
portugueses.
b) A criao de gado, enquanto atividade econmica complementar,
desenvolveu-se junto aos campos de cana do litoral.
c) A ocupao do territrio baseou-se na explorao da cana-de-acar
atravs do sistema produtivo dos engenhos.
d) O consumo da lenha para os engenhos no alterou a fisionomia da mata
atlntica litornea.
e) A economia monocultora, latifundiria e escravista deu-se em funo da
necessidade do mercado do Centro-Sul do pas.

3) (UFPE) Em relao aos processos de povoamento do Brasil podemos

afirmar:
( ) O trao marcante do incio do povoamento brasileiro pelo europeu foi o
seu carter perifrico.
( ) O povoamento da Amaznia fez-se principalmente ao longo das reas
mais elevadas, que possuam climas mais amenos e menos chuvosos.
( ) A marca do povoamento no Brasil assemelha-se aos processos que
caracterizaram a penetrao do europeu no continente americano.
( ) Durante todo o sculo XVI, os povoadores estabeleceram-se no litoral
brasileiro, onde formaram seus ncleos.
( ) Os estados do sul do Brasil, por possurem climas pouco favorveis ao
desenvolvimento de lavouras tropicais de carter comercial, ou por estarem
em localizao geogrfica afastada dos maiores centros, ficaram durante a
colonizao margem dos principais roteiros de povoamento.

4) (UERJ)

Os dados abaixo tratam da populao ocupada no Brasil entre o


final do sculo XIX e incio do XX.
BRASIL - POPULAO OCUPADA
(em milhares e em %)
SETORES
1872
1920
agricultura
3671 = 64,1%
6377 = 69,7%
indstria
282 = 4,9%
1264 = 13,8%
servios
1773 = 31,0%
1509 = 16,5%
total
5726 = 100%
9150 = 100%

As reas assinaladas com os nmeros 1, 2, 3 correspondem,


respectivamente, a:
a) pecuria em pastos cultivados, cana-de-acar nas baixadas litorneas,
borracha na Floresta Equatorial Amaznica.
b) pecuria no Planalto Brasileiro, cana-de-acar nas baixadas litorneas,
borracha na Floresta Equatorial Amaznica.
c) pecuria em pastos naturais, cana-de-acar nas baixadas litorneas,
borracha na Floresta Equatorial Amaznica.
d) pecuria em pastos naturais, cana-de-acar nas reas frteis do litoral,
cacau na Floresta Equatorial Amaznica.
e) pecuria em pastos cultivados, cana-de-acar nas reas frteis do litoral,
borracha na Floresta Tropical.

7) (UFMG)

Desde o incio da colonizao, a distribuio de terras no Brasil


tem sido feita de maneira desigual. A forma como a questo vem sendo
conduzida sugere que, a curto prazo, o problema deve permanecer.
A respeito da questo agrria no Brasil, INCORRETO afirmar que:
a) o governo brasileiro, na tentativa de neutralizar as constantes invases de
propriedades agrcolas pelos sem-terra, tem ameaado excluir do processo de
desapropriao as reas invadidas.
b) os programas governamentais de assentamento de colonos pouco
alteraram a estrutura fundiria do Pas, que continua muito marcada pela
concentrao de terras.
c) a invaso de terras um fato muito antigo no Pas, pois, desde os
primrdios de sua histria, as terras que pertenciam aos ndios ou Coroa
foram ocupadas por latifundirios.
d) o movimento dos sem-terra, que comeou de forma muito dispersa -do
ponto de vista geogrfico, tem, hoje, como principal estratgia concentrar
sua ao em reas restritas, para ganhar fora.

8) (UFMG) Observe o mapa abaixo:

A anlise dos leva seguinte caracterstica econmica desse perodo:


a) crescimento do setor de servios
b) dinamizao da atividade industrial
c) protecionismo da agricultura de subsistncia
d) desenvolvimento acelerado dos trs setores econmicos

5) (PUC-PR) As migraes internas, de 1870 a 1912, graas s necessidades


de borracha natural do mercado mundial e ao longo perodo de seca que
assolou a Regio Nordeste, proporcionaram o povoamento de Estados como:
a) Acre.
b) Esprito Santo.
c) Mato Grosso.
d) Mato Grosso do Sul.
e) Tocantins.
6) (Vunesp-SP) Observe o mapa a seguir.
Mapa Econmico do Brasil Sculo XIX

Suponha
a
realizao
de
uma
viagem
de
automvel
de
Belo
Horizonte a Luz, com a partida marcada para as 15h de um dia ensolarado na
vspera do Natal. Nessa viagem, com durao aproximada de duas horas e
trinta minutos, o motorista ir receber mais intensamente os raios solares:
a) de frente e sua esquerda.
b) de frente e sua direita.
c) pelas costas e sua esquerda.
d) pelas costas e sua direita.

9) (PUC-RS) Se em Curitiba so 12 horas, sero 22 horas em:


a) Jacarta
b) Guadalajara
c) Chicago
d) Pretria
e) Moscou

10) (UFMG) O Brasil tem institudo o horrio de vero que, no

entanto, no
abrange todo o territrio nacional.
Os Estados brasileiros excludos do horrio de vero so aqueles
a) situados em reas de altos ndices pluviomtricos e de nebulosidade, que
ocasionam fraca luminosidade em todas as estaes do ano.
b) situados em reas de grande altitude, que registram temperaturas
absolutas inferiores mdia nacional em todas as estaes do ano.

57

c) situados na poro equatorial, onde a variao da durao diria do


perodo claro imperceptvel nas diferentes estaes do ano.
d) situados na poro temperada do pas, onde a durao diria do perodo
escuro maior do que a do perodo claro em todas as estaes do ano.

11) (UFMG) Sobre os fusos horrios, INCORRETO afirmar que,


a) no Brasil, como em qualquer outro pas do Ocidente, a hora legal est
adiantada em relao hora legal dos pases do Oriente.
b) no Brasil, como no mundo inteiro, os fusos horrios so definidos em
relao ao fuso horrio inicial, que, por conveno, o delimitado pelo
meridiano de Greenwich.
c) no Brasil e em muitas outras reas do globo, os limites tericos so
substitudos por limites prticos que levam em conta fronteiras polticas.
d) no Brasil e em muitos outros pases do globo, as diferenas de fusos
horrios devem-se grande extenso longitudinal de seus territrios.

12) (UFMG) A instituio do horrio de vero no Brasil possvel tendo em


vista a existncia de fatores que, conjugados, podem explicar por que grande
parte do territrio brasileiro recebe mais luz e calor em determinado perodo
do ano.
Todas as alternativas apresentam esses fatores, EXCETO
a) A inclinao do eixo terrestre, sempre voltado para a mesma direo.
b) A grande extenso territorial do Brasil no sentido leste/oeste.
c) A posio astronmica da Terra, no seu movimento de translao.
d) A posio austral do pas, em relao ao Equador.

13) (UFMG)

Suponha que sejam 9 horas em Belo Horizonte e que voc,


estando aqui, precisa planejar uma ligao interurbana para uma pessoa em
Rio Branco, Acre, que poder ser encontrada, nessa cidade, s 11 horas,
hora local desse Estado. Nesse caso, voc deve:
a) aguardar duas horas para fazer sua ligao porque no Brasil, embora
sejam reconhecidos os limites tericos dos fusos horrios de 15 de
longitude, consideram-se apenas os limites prticos definidos pelas fronteiras
estaduais.
b) aguardar que sejam 11 horas em Braslia, hora oficial do pas, que se
aplica a toda a poro ocidental do territrio brasileiro.
c) fazer sua ligao, imediatamente, porque o horrio do fuso em que se
encontra o Estado de Minas Gerais de duas horas a menos em relao ao
fuso da poro mais ocidental do Brasil.
d) ligar s 13 horas, no horrio de Belo Horizonte, porque o horrio do fuso
em que se encontra o Estado de Minas Gerais adiantado em duas horas em
relao ao da poro mais ocidental do Brasil.
hemisfrio sul estar entrando no inverno e o hemisfrio norte no vero.
4) O movimento de translao da Terra responsvel pela sucesso dos dias
e noites, cujas duraes no so iguais para uma mesma data em diferentes
latitudes, exceto por ocasio dos equincios.
8) Os solstcios e equincios indicam posies da Terra em seu movimento de
translao, e assinalam o incio das estaes do ano.
16) No territrio brasileiro, a linha do Equador passa por Estados das regies
Norte e Nordeste.

19) (UFRGS)

Selecione a alternativa que completa corretamente a lacuna


da afirmao a seguir. Estudos revelam que a freqncia com que ocorrem
as inverses trmicas de superfcie em Porto Alegre aumenta no perodo de
maio a setembro, fato este que resulta no aumento das concentraes de
poluentes atmosfricos e que est diretamente relacionado com a
..................... da referida cidade.
a) altitude
b) latitude
c) continentalidade
d) longitude
e) topografia

20) (Vunesp-SP) No territrio brasileiro, em sentido norte-sul, em relao


mdia e amplitude trmicas, correto afirmar que
a) as mdias trmicas diminuem e as amplitudes aumentam.
b) as mdias e as amplitudes trmicas diminuem.
c) as mdias trmicas aumentam e as amplitudes diminuem.
d) as mdias e as amplitudes trmicas no apresentam variao.
e) as mdias e as amplitudes trmicas aumentam.

21) A

tabela a seguir representa trs cidades brasileiras, que possuem,


aproximadamente, a mesma altitude.
Temperatura mdiaTemperatura
em
mdia Precipitao
em
total anual
Cidades
janeiro
julho
(em mdia)
I
26,9C
26,4C
2302 mm

14) (UFRGS)

O Sol nasce exatamente no ponto do horizonte local que


corresponde ao leste geogrfico
a) em 21 de maro e 23 de setembro
b) em 21 de junho e 22 de dezembro
c) em 21 de maro e 21 de junho
d) em 21 de junho e 23 de setembro
e) em 23 de setembro e 22 de dezembro

15) (UFRGS)

Porto Alegre encontra-se, aproximadamente, na longitude


51W Gr. e, portanto, est includa no fuso horrio de -3h, que tem como
meridiano central 45W Gr. Com base nessa informao correto afirmar
que o meio-dia verdadeiro (ou solar) ocorre quando os relgios indicam
a) 12h24min
b) 12h 00 min
c) 12h06min
d) 11h 54 min
e) 11h 36 min

16) (UECE) Sobre a posio astronmica do Brasil, correto afirmar:


a) a maior parte do territrio brasileiro est localizada na poro extratropical
e o trpico de Capricrnio atravessa a cidade de So Paulo
b) est totalmente situado entre os paralelos de 516'19" de latitude norte e
3349'09" de latitude sul
c) fica entre os meridianos de 3445'54" e 7359'32" a leste de Greenwich
d) est localizado quase totalmente no Hemisfrio Ocidental e totalmente no
Hemisfrio Sul

17) (Fuvest-SP)

A cidade de So Paulo est situada no fuso horrio 45


Oeste. Quando em So Paulo forem 13 horas, que horas sero numa cidade
localizada no fuso 75 Leste?
a) 5 horas
b) 11 horas
c) 15 horas
d) 19 horas
e) 21 horas
18) (UFPR) Na(s) questo(es) a seguir, escreva no espao apropriado a
soma dos itens corretos. Identifique os itens corretos:
1) A inclinao do eixo de rotao da Terra em sua rbita em torno do Sol
permite que, sazonalmente, um hemisfrio receba mais energia que o outro.
2) Quando os raios solares estiverem incidindo perpendicularmente sobre o
Trpico
de
Capricrnio,
o
II
III

24,2C
25,6C

12,2C
20,6C

1286 mm
830 mm

Em relao aos dados anteriormente apresentados, foram feitas as seguintes


afirmaes:
I - A amplitude trmica da cidade III maior do que a da cidade I e menor
do que a da cidade II. II - Os dados da tabela permitem afirmar que a cidade
II est ao sul da cidade I.
III - A cidade I est, provavelmente, nas proximidades do equador.
Analisando a tabela e as assertivas anteriores, pode-se certificar que
a) est correta somente a afirmativa I.
b) est correta somente a afirmativa II.
c) est correta somente a afirmativa III.
d) esto corretas somente as afirmativas I e II.
e) esto corretas as afirmativas II e III.

22) (UFV-MG)

"Moro num pas tropical, abenoado por Deus e bonito por


natureza." Essa frase, de uma cano de Jorge Ben Jor, exalta as belezas
naturais do Brasil e sugere a influncia do clima na formao desse cenrio.
A influncia tropical no clima brasileiro est associada:
a) ao fato do nordeste do pas situar-se sob o Trpico de Cncer, o que
favorece a manifestao de altas temperaturas durante todo o ano.
b) ao fato do nordeste do pas situar-se sob o Trpico de Capricrnio, o que
favorece a manifestao de altas temperaturas durante todo o ano.
c) ao fato do territrio nacional estar localizado em regies de altas latitudes,
isto , afastadas do Equador.
d) ao fato da maior parte do pas estar localizada numa rea entre o Equador
e o Trpico de Cncer.
e) ao fato da maior parte do pas estar localizada numa rea entre o Equador
e o Trpico de Capricrnio.

23) (UFRGS)

Observe as cidades de mesma altitude, identificadas pelos


nmeros 1, 2, 3, 4 e 5 e localizadas no continente hipottico a seguir.

58

c) l, lll e V;
d) ll, lll e V;
e) II e IV.

27) (UFPE) Observe:

Quais nmeros representam, respectivamente, as cidades com maior e


menor amplitudes trmicas anuais?
a) 1 e 4.
b) 2 e 1.
c) 3 e 5.
d) 5 e2
e) 5 e 3.

24) (UFRGS) No que se

refere sua localizao latitudinal e longitudinal, o


Rio Grande do Sul apresenta a seguinte situao: encontra-se localizado
entre os paralelos de 2703' e 3346' de latitude Sul, e os meridianos de
4943' e 5741' de longitude Oeste.
Considere as seguintes afirmativas.
I - O Estado possui seus pontos extremos latitudinais e longitudinais
localizados nos hemisfrios Meridional (Sul) e Ocidental (Oeste) do globo
terrestre, o que lhe assegura a condio de nico Estado do Brasil com esta
caracterstica territorial.
II - A posio do paralelo 30 Sul, que representa a latitude mdia do Estado,
permite afirmar que o Rio Grande do Sul encontra-se numa posio
eqidistante entre a linha do Equador e o Plo Sul.
III - Em Torres o Sol surge no horizonte com pouco, mais de meia hora de
antecedncia em relao a Uruguaiana, devido amplitude longitudinal do
Estado, que de 759'.
Quais esto corretas?
a) Apenas I.
b) Apenas II.
c) Apenas III.
d) Apenas I e II.
e) Apenas II e III.

25) (PUC-RS)

Responder questo com base no quadro adiante, que


representa uma rede de coordenadas de uma rea do globo terrestre.

1. Pelo sistema de fusos horrios, o globo terrestre foi dividido em 24 fusos,


cada um equivalendo a 15 no sentido das longitudes.
2. O equador o crculo mximo que marca o incio da contagem das horas.
3. Quando o Meridiano de Greenwich marcar 19 horas, hora legal, num
ponto situado a uma longitude de 30W, a hora legal ser de 21 horas.
4. O Brasil possui 4 fusos horrios, todos situados ao oeste de Greenwich.
5. Um avio, ao cruzar a linha internacional (da data), no sentido oesteleste, retrocede 1 (um) dia no calendrio.
Esto corretos apenas os itens:
a) 2, 4 e 5;
b) 1, 3 e 4;
c) 3, 4 e 5;
d) 1, 4 e 5;
e) 1, 2 e 4.

28) (Vunesp-SP)

Verificando o mapa adiante e considerando o Equador e


Greenwich, possvel afirmar que o Brasil tem a maioria de suas terras nos
hemisfrios:

a) Norte e Sul.
c) Sul e Oriental.
e) Ocidental e Norte.

b) Sul e Ocidental.
d) Oriental e Ocidental.

29) (Fuvest-SP) Um

avio viaja, em linha reta, de So Paulo para o Rio de


Janeiro, no dia 21 de junho entre 12 e 13 horas. De que lado do avio est o
sol durante o percurso?
a) direita.
b) esquerda.
c) frente.
d) Atrs.
e) A pino.

30) (UFCE)

Duas cidades, uma gacha e outra amazonense, poderiam estar


representadas pelas coordenadas relativas aos nmeros .......... e ..........,
respectivamente.
a) 3 4
b) 2 5
c) 4 1
d) 5 2
e) 3 1

26) (UFPE) Analise as proposies a seguir:


I) os paralelos so importantes porque permitem avaliar a latitude, que a
distncia em graus a partir do Equador;
II) os paralelos tm dimetros iguais e, logicamente, comprimentos ou
permetros tambm iguais;
III) os meridianos so crculos perpendiculares aos paralelos e passam pelos
plos onde eles se cruzam;
IV) a longitude inicial de 0o e a mxima de 180, podendo ser norte ou sul;
V) as coordenadas geogrficas so valores que determinam a localizao de
um lugar na superfcie do globo.
Esto corretas:
a) I, IV e V;
b) II, III e IV;

Atualmente no existem vulces ativos no Brasil, embora


tenham existido em pocas geolgicas passadas. Assinale a alternativa que
indica corretamente a maior rea territorial onde se encontram evidncias de
vulcanismo no Brasil.
a) Domnio das Caatingas com relevos aplainados e solos rasos e pedregosos
b) Regio do Pantanal e sua intensa rede de drenagem com solos arenosos
c) Zona da Mata e seus ambientes litorneos onde se registram constantes
tremores de terra
d) Planalto Meridional com derrames baslticos e presena de solos
denominados Terras Roxas
e) Domnio do Cerrado com relevos aplainados e solos arenosos denominados
Latossolos

31) (PUC-PR) A expresso "aluvial" ou "aluvio" refere-se normalmente:


a) a mapas hipsomtricos como os da proposta do professor Jurandyr Ross.
b) aos solos criados pela ao das guas correntes, principalmente.
c) desintegrao das rochas pela presso do congelamento das guas nas
regies polares ou montanhosas.
d) ao processo de subduco que teria formado cordilheiras como a andina,
ao longo da orla martima.
e) estrutura da Terra no que diz respeito ao ncleo interno e ncleo
externo na gerao do magnetismo terrestre.
32) (PUC-MG) Responda a esta questo com base nas afirmativas
seguintes:
I. A estabilidade do territrio brasileiro conseqncia de sua posio no
interior da placa Sul-americana, no o colocando em condies de
vulnerabilidade aos movimentos orogenticos tpicos das faixas de contato
entre as placas.
II. As ilhas ocenicas brasileiras, a exemplo do Arquiplago de Fernando de
Noronha, correspondem a picos vulcnicos submersos de uma cordilheira
dorsal meso-ocenica.

59

III. Na paisagem geomorfolgica do relevo brasileiro, verifica-se uma


preponderncia dos processos de sedimentao sobre os erosivos,
justificando o predomnio das plancies sobre os planaltos e depresses.
CORRETO afirmar que:
a) apenas a afirmativa I correta.
b) apenas as afirmativas I e II so corretas.
No perfil, I e II correspondem respectivamente a:
c) apenas as afirmativas I e III so corretas.
a) Depresso Perifrica e Planalto Atlntico.
d) apenas as afirmativas II e III so corretas.
b) Pantanal e Planaltos e Serras do Leste e Sudeste.
e) todas as afirmativas so corretas.
c) Plancie Amaznica e Planalto das Guianas.
d) Depresso do So Francisco e Planalto Nordestino.
33) (Mack-SP) Assinale a alternativa INCORRETA sobre a estrutura e) Campanha Gacha e Serra do Mar.
geolgica e o relevo do Brasil.
a) As verdadeiras plancies so raras.
37) (UEL-PR) Sobre a regio assinalada na figura abaixo, correto afirmar:
b) predominantemente planltico.
c) A maior parte do territrio recoberta por rochas sedimentares.
d) Os dobramentos recentes limitam-se Serra do Mar.
e) As maiores altitudes localizam-se no extremo norte.

34) (FGV-SP)

a) uma das reas de relevo mais acidentado no Brasil, o que torna difcil a
utilizao de agricultura moderna na regio.
b) basicamente constituda por rochas cristalinas, o que justifica o relevo
acidentado.
c) Devido s caractersticas pluviomtricas dessa regio, a rede de drenagem
pouco densa, o que tem acarretado problemas de abastecimento hdrico
para cidades de porte mdio a localizadas.
d) Nessa rea, concentram-se as principais reservas de ferro do Brasil.
e) Nessa rea, esto presentes os basaltos que deram origem a solos
conhecidos como terra roxa.
As reas assinaladas no mapa por X-Y-Z correspondem, respectivamente, s
seguintes unidades do relevo brasileiro:
a) Planaltos Residuais Norte-Amaznicos / Planaltos e Chapadas da Bacia do
Parnaba / Planaltos e Chapadas da Bacia do Paran.
b) Depresses Marginais Amaznicas / Depresso Sertaneja e do So
Francisco / Depresso Perifrica Sul-Rio-grandense.
c) Planaltos Residuais Norte-Amaznicos / Depresso Sertaneja e do So
Francisco / Chapadas da Bacia do Paran.
d) Depresses Marginais Amaznicas / Planaltos e Chapadas da Bacia do
Parnaba / Chapadas da Bacia do Paran.
e) Planaltos Residuais Norte-Amaznicos / Planalto da Borborema /
Depresso Perifrica Sul-Rio-grandense.

38) (UFRGS)

O corte topogrfico abaixo esquematiza o perfil do relevo da


Regio Nordeste do Brasil entre o Planalto da Borborema e a Bacia do
Parnaba.

35) (FGV-SP)
As reas identificadas pelos algarismos I e
respectivamente,
a) Depresso Sertaneja e s falsias cristalinas.
b) Depresso Sertaneja e aos tabuleiros litorneos.
c) Depresso Nordestina e aos tabuleiros litorneos.
d) Depresso Sertaneja e aos "inselbergs".
e) Depresso Nordestina e s falsias sedimentares.

39) (PUCC-SP) Considere

II

correspondem,

os mapas apresentados abaixo, para responder

questo.

No mapa anterior, a parte destacada refere-se


a) rea de vegetao de araucrias.
b) rea de derrames baslticos do mesozico.
c) ao planalto Meridional.
d) bacia hidrogrfica do rio Paraguai.
e) rea de clima tropical de altitude.

36) (Mack-SP)

60

43) (UFPI)

Em relao s bacias hidrogrficas que possuem um maior


aproveitamento hidreltrico no Brasil, correto afirmar que elas se localizam
nas regies:
a) Norte e Nordeste. b) Nordeste e Centro-Oeste.
c) Sul e Norte.
d) Sudeste e Sul.
e) Centro-Oeste e Sul.

44) (UFRJ)

Relacione a estrutura geolgica brasileira e a explorao econmica dos


principais recursos minerais, nos mapas.
a) Mapa I - Rochas cristalinas (ferro, mangans e cassiterita); Mapa II Rochas sedimentares (petrleo e carvo);
b) Mapa I - Rochas cristalinas (petrleo e carvo); Mapa II - Rochas
sedimentares (ferro, mangans e cassiteria);
c) Mapa I - Rochas sedimentares (bauxita, ferro e mangans); Mapa II Rochas cristalinas (petrleo, carvo e ouro);
d) Mapa I - Rochas cristalinas (ferro, mangans e cassiterita); Mapa II Rochas sedimentares (bauxita, ouro e cassiterita);
e) Mapa I - Rochas sedimentares (ferro, ouro e bauxita); Mapa II - Rochas
cristalinas (petrleo, carvo e ouro).

No Brasil, os cursos dos rios so motivo de uma afirmativa


verdadeira e curiosa: todos os rios brasileiros desguam direta ou
indiretamente no Atlntico, mesmo aqueles que correm para o oeste.
A correta explicao para o fato resulta
a) das elevadas altitudes ocidentais da Amrica do Sul.
b) do relevo mais alto e acidentado na poro oriental do Brasil.
c) das terras baixas existentes nas fronteiras ocidentais brasileiras.
d) do relevo mais baixo e plano na poro setentrional do Brasil.
e) das elevadas altitudes da poro meridional da Amrica do Sul.
45) (PUC-PR) O "Velho Chico", como carinhosamente conhecido o Rio So
Francisco, tambm denominado "Rio da Integrao Nacional". Da nascente
at a foz, ele atravessa reas com diferentes climas. Assinale os climas na
ordem da nascente para a foz.
a) Equatorial, tropical e litorneo.
b) Semi-rido, de transio e tropical.
c) Tropical, semi-rido e litorneo.
d) Subtropical, tropical e equatorial.
e) Tropical de altitude, de transio e litorneo.

40) (UFV-MG)

46) (PUC-RJ)

O Brasil, devido sua grande extenso territorial e


predominncia de climas midos, tem uma extensa rede hidrogrfica. Sobre
a hidrografia brasileira, assinale a afirmativa INCORRETA:
a) Todas as bacias hidrogrficas so exorricas, mesmo aquelas que tm rios
que correm para o interior.
b) As bacias hidrogrficas brasileiras oferecem grande possibilidade de
navegao e, em razo disso, o transporte hidrovirio muito utilizado no
pas, apesar de seu alto custo.
c) Todos os rios brasileiros, com exceo do Amazonas, possuem regime
pluvial. Uma pequena quantidade de gua do rio Amazonas provm do
regime nival, caracterizando um regime misto.
d) Na maior parte do pas os rios so perenes, contudo em reas de clima
semi-rido existem rios intermitentes.
e) Predominam rios de planalto, o que possibilita a produo de
hidreletricdade.

41) (UFV-MG) Sobre a hidrografia brasileira CORRETO afirmar que:

Em 1997, foi instituda a Poltica Nacional de Recursos


Hdricos. Em sua implementao, ela considera, como unidade territorial,
para avaliar e monitorar a quantidade de gua disponvel, "a rea de
captao da gua precipitada, demarcada por divisores topogrficos, na qual
toda a gua captada converge para um nico ponto de sada".
A unidade territorial a que o texto se refere :
a) o fluxo basal.
b) a bacia hidrogrfica.
c) o regime fluvial.
d) a rede hidrogrfica.
e) o balano hdrico.

47) (UFSC)

O mapa do Brasil, a seguir, apresenta a classificao climtica


de Strahler. Observe atentamente as regies numeradas e assinale com V
(verdadeiro) ou F (falso) as proposies adiante.

a) apesar da maioria dos rios brasileiros ter a vertente para o oceano


Atlntico, o rio Negro, na bacia amaznica, tem sua vertente para o oceano
Pacfico em virtude de sua nascente se localizar nos Andes peruanos, x b)
todas as grandes bacias hidrogrficas brasileiras tm a sua vertente para o
oceano Atlntico.
c) a grande maioria dos rios brasileiros tem a sua foz em forma de delta,
como o caso do rio So Francisco.
d) o rio So Francisco atravessa vrios estados brasileiros percorrendo uma
trajetria no sentido norte/sul do pas.
e) no Brasil predominam rios de plancie, o que favorece a navegao fluvial
como nos rios Uruguai e na maior parte do So Francisco.

42) (PUC-RS) Responder questo com base no mapa.

Os nmeros 1 e 2, representados no mapa, indicam usinas hidreltricas


localizadas em um rio brasileiro. A alternativa que indica corretamente o
nome do rio e as respectivas usinas
a) Rio Parnaba - Urubupunga e Furnas.
b) Rio Paran - Boa Esperana e Urubupunga.
c) Rio So Francisco - Paulo Afonso e Trs Marias.
d) Rio Parnaba - Furnas e Tucuru.
e) Rio So Francisco - Balbina e Paulo Afonso.

( ) As amplitudes trmicas anuais so maiores na rea assinalada pelo


nmero 1 do que na rea de nmero 2. O mesmo ocorre com os totais
anuais de precipitao, que so maiores na regio de nmero 1, onde
provocam cheias com reflexos sociais.
( ) A poro do territrio brasileiro indicada pelo nmero 2 apresenta o clima
subtropical mido, controlado por massas de ar tropicais e polares que
atingem toda a regio, onde vigoravam formaes florestais e campestres
fortemente alteradas pela ao humana.
( ) A regio de nmero 1 refere-se ao clima tropical tendendo a seco pela
irregularidade de ao das massas de ar. Nela dominam o intemperismo
fsico, uma vegetao xerfila e um processo de desertificao causado pela
ao inadequada do homem.
( ) A regio de nmero 2 apresenta um inverno mais rigoroso no planalto,
comparativamente com outras reas do pas, principalmente pela influncia
conjunta de fatores como a latitude, a altitude e a distncia do mar, alm da
ao das massas polares.
( ) As duas regies numeradas localizam-se em zonas climticas distintas,
sendo que a de nmero 1 est na faixa intertropical e a de nmero 2,
predominantemente, em zona temperada, o que, por si s, determina
caractersticas geogrficas diferenciadas.

48) (UFCE) Observe o mapa a seguir.

61

Indique a alternativa que representa, corretamente, o domnio morfoclimtico


brasileiro em que h uma significativa ocorrncia de clima equatorial
supermido.
a) Caatingas.
b) Cerrado.
c) Amaznico.
d) Araucrias.
e) Pampas.

49) (PUC-RS) No Brasil, a variedade de climas existentes est relacionada a


diferentes fatores. Entre aqueles considerados dinmicos, temos as massas
de ar. Em virtude da grande extenso territorial que o Brasil possui, nosso
pas apresenta cinco massas de ar agindo sobre o territrio, cada uma delas
apresentando caractersticas relativas temperatura e umidade. Por
exemplo: a Massa ......... caracteriza-se como ............ .
a) Equatorial Atlntica - quente e mida
b) Equatorial Continental - fria e mida
c) Tropical Atlntica - fria e seca
d) Tropical Continental - quente e mida
e) Polar Atlntica - fria e seca

Considerando o mapa anterior os fatores climticos que determinam a


ocorrncia do clima subtropical com veres amenos so
a) latitude e altitude.
b) longitude e continentalidade.
c) latitude e maritimidade.
d) pluviosidade e latitude.
e) longitude e altitude.

53) (PUC-MG) Relacione as alternativas a seguir com os itens indicados nos

Encontra-se em uma regio mida que recebe forte


influncia da Massa Tropical Atlntica, recebe muita insolao devido sua
localizao tropical e sofre com enchentes em pocas de chuvas. Trata-se da
cidade de
a) Manaus
b) Salvador.
c) Fortaleza.
d) Braslia.
e) Porto Alegre.

parnteses.
I. O calor do continente aquece as bases das massas de ar que atuam sobre
o territrio brasileiro provocando movimento ascensional da atmosfera com
conseqentes pancadas de chuvas.
II. A penetrao do ar polar sobre o territrio brasileiro produz um
resfriamento da atmosfera que ocasiona a condensao da umidade do ar
com conseqentes precipitaes.
III. A massa Tropical atlntica, avanando sobre as regies costeiras,
encontra uma barreira formada por serras do relevo brasileiro fazendo com
que o ar mido se eleve com conseqente condensao do vapor d'gua,
seguida de chuvas.
( ) Chuvas orogrficas
( ) Chuvas frontais
( ) Chuvas convectivas
A ordem CORRETA encontrada, de cima para baixo, :
a) III - II - I
b) III - I - II
c) II - III - I
d) I - II - III
e) II - I - III

51) (UFV-MG)

54) Na

50) (Unifesp-SP)

Qual das opes abaixo faz uma associao INCORRETA


entre os tipos climticos brasileiros e sua localizao espacial?
a) Clima Tropical Semi-mido - Predomina na poro continental do territrio
nacional, principalmente no Planalto Central e em trechos do Sudeste e do
Nordeste.
b) Clima Tropical mido ou Atlntico - Faixa costeira, do Rio Grande do Sul
at So Paulo, podendo aparecer tambm ao longo da poro oriental da
bacia do rio Doce e em parte da Zona da Mata Mineira.
c) Clima Tropical Semi-rido - Predomina no serto nordestino e em parte do
mdio vale do rio So Francisco.
d) Clima Equatorial -Toda a regio Sudeste e parte norte do Paran.
e) Clima Subtropcal - Predomina no sul do pas, abaixo do Trpico de
Capricrnio.

52) (UFRJ)

lista das 25 reas mais ricas em biodiversidade e mais ameaadas


do planeta, esto dois biomas brasileiros: a ...1..., de cuja cobertura original
restam apenas 7%, e o ...2..., do qual restam 60%. No texto, os biomas 1 e
2 so, respectivamente,
a) caatinga e pantanal.
b) mata dos cocais e cerrado.
c) mata dos pinhais e pantanal.
d) floresta amaznica e campo sulino.
e) mata atlntica e cerrado.

55) (UFSCar-SP)

Caracteriza-se pela presena predominante de rvores de


pequeno porte espalhadas por uma cobertura descontnua de gramneas.
A partir da aplicao dos resultados das pesquisas realizadas para corrigir
seus solos, essa formao vegetal foi sendo devastada, porque seu territrio
tornou-se rea de expanso da produo de gros para exportao.
Assinale a alternativa que contm o nome da formao vegetal qual o texto
se refere.
a) Floresta Subtropical.
b) Caatinga.
c) Mangue.
d) Cerrado.
e) Mata de Araucria.

56) (PUC-PR/2003

- C) Observe o texto: " uma rea com


aproximadamente 100.000 km2, com altitudes de 100 e 200 m, estendendose alm das fronteiras do Brasil. A regio marcada pelo ciclo das guas. No
perodo da cheia, os rios extravazam de seus leitos, alagando grandes
extenses de terra. No perodo da vazante, ocorre a principal contribuio

62

dos rios aos solos da regio, grandes quantidades de sedimentos, nutrientes


e material orgnico e decomposio so arrastados pelos rios e depositados
no solo anteriormente alagado. A vegetao bastante variada em espcies
de cerrado, amaznicas, do chaco e da caatinga."
O texto acima refere-se regio:
a) da Mata de Cocais
b) da Amaznia
c) do Pantanal
d) dos Lenis Maranhenses
e) da Chapada Diamantina

57) (UFRN)

Para se classificar o espao brasileiro em "domnios


morfoclimticos", utilizou-se um mtodo de regionalizao baseado na
presena de elementos naturais.
O domnio classificado como Mares de Morros caracterizado por relevo
ondulado, rede de drenagem desenvolvida e cobertura vegetal primria, do
tipo latifoliada tropical.
No mapa abaixo, esse domnio est representado pelo nmero:

a) 1
b) 2
c) 3
d) 4

58) (UFF-RJ)
HAITI
Quando voc for convidado pra subir no adro da
Fundao Casa de Jorge Amado
Pra ver do alto a fila de soldados, quase todos pretos
Dando porrada na nuca de malandros pretos
De ladres mulatos
outros quase brancos
Tratados como pretos
S pra mostrar aos outros quase pretos
(E so quase todos pretos)
E aos quase brancos pobres como pretos
Como que pretos, pobres e mulatos
E quase brancos quase pretos de to pobres so tratados
E no importa se olhos do mundo inteiro
--------------------------------------------------------------------No importa nada
Nem o trao do sobrado, nem a lente do Fantstico
Nem o disco de Paul Simon
Ningum
Ningum cidado
Se voc for ver a festa do Pelo
E se voc no for
pense no Haiti
Reze pelo Haiti
O Haiti aqui
O Haiti no aqui
--------------------------------------------------------------------(http://www.gilbertogil.com.br/disconew/letras_0.htm)
Em "Haiti", est retratada uma realidade sociopoltica observada nos pases
latino-americanos, que diz respeito questo da Cidadania e dos Direitos
Humanos. Acerca dessa questo, pode-se afirmar:
a) O desrespeito Cidadania e aos Direitos Humanos na Amrica Latina, em
particular no Brasil, atinge de modo indiscriminado negros, ndios e brancos,
homogeneizando as condies sociopolticas.
b) As sociedades latino-americanas no tm sido capazes de superar os
conflitos tnicos, fenmeno inexistente nas democracias raciais vigentes nos
pases anglo-saxes da Amrica, onde os direitos do cidado so respeitados.
c) A globalizao econmica, que possui como pressuposto a democratizao
poltica, tem um importante papel na ampliao dos direitos humanos das
etnias na Amrica Latina.
d) Na Amrica Latina, as desigualdades socioeconmicas se entrelaam com
a discriminao tnica e cultural, fragilizando as possibilidades de instituio
de uma territorialidade democrtica que assegure os Direitos do Homem e do
Cidado.

e) As dificuldades para se ampliar o respeito Cidadania e aos Direitos


Humanos em territrios com forte presena de populaes negras, como as
do Brasil e do Haiti, decorrem da incapacidade de adaptao desse grupo
tnico s exigncias de sociedades democrticas.

59) (PUC-RS)

"Central do Brasil", que concorreu ao Oscar de melhor filme


estrangeiro pelo Brasil, ambientado em um grande espao territorial. Este
cenrio constitui-se basicamente de duas regies, ambas com graves
problemas sociais. Historicamente, uma tem-se caracterizado como receptora
e outra como repulsora de migrantes, principalmente nas dcadas de 1960 a
1980.
Tais regies so, respectivamente,
a) Sudeste e Nordeste.
b) Nordeste e Norte.
c) Centro-Oeste e Nordeste.
d) Sul e Norte.
e) Sudeste e Centro-Oeste.

60) (FGV-SP)

Ns, ianomami, queremos uma rea nica e contnua para


nosso povo, para poder viver em paz (...)
Esse desejo de Ianomami justificvel porque seu territrio est sendo
retalhado por
a) inmeras empresas da pecuria vindas do Sul do pas, que implantam
pastos.
b) enclaves militares formados por motivos geopolticos, dada a vizinhana
com as Guianas.
c) postos de fiscalizao de narcotrfico, porque suas terras se estendem ao
longo da fronteira com a Colmbia.
d) numerosos garimpos e dezenas de empresas de minerao que pediram
alvars de explorao.
e) poos de prospeco de petrleo, dada a contigidade com a regio
petrolfera venezuelana.

61) (PUC-MG)

O Brasil foi um dos pases que mais cresceram


economicamente durante o sculo XX. No entanto, a despeito da evoluo
quantitativa positiva de determinados indicadores socioeconmicos, NO se
pode afirmar que:
a) O sistema de educao de massa no ensino fundamental foi efetivado e
ampliado para os nveis seguintes, tornando seu acesso igualitrio social e
espacialmente.
b) A rede de proteo social frgil e marcada por distores distributivas,
tornando-se menos eficaz para parcelas mais pobres da populao.
c) O sistema de comunicao de massa cuja penetrao se d nas mais
longnquas regies e na maioria dos lares possibilita transformaes
qualitativas no processo educativo da populao.
d) O sistema pblico de sade garante atendimento bsico grande maioria
da populao, embora persistam graves distores regionais e sua qualidade
seja questionvel.

62) Leia atentamente a informao a seguir.

So razes corretas para a divulgao do cartaz acima, durante o Censo,


EXCETO:
a) mostra uma posio dos movimentos negros para uma valorizao da sua
cor e raa.
b) mostra uma preocupao com a afirmao do Brasil, como pas de maioria
de populao branca.
c) faz referncia relutncia de parte substancial dos negros em admitir a
sua cor.
d) revela uma preocupao de identificar melhor a quantidade enorme de
no-brancos brasileiros.
63) (UFF-RJ) O Censo 2000 do IBGE registrou, conforme ilustra o grfico a
seguir, significativa reduo do nmero mdio de pessoas na famlia em todo
o pas.

63

e) III e IV

66) (UFJF-MG) Sobre o desemprego no Brasil, CORRETO afirmar que:


a) no sculo XIX, grande parte da populao no tinha emprego devido
escravido.
b) at a II Guerra Mundial, o desemprego era elevado devido ao predomnio
da agricultura.
c) da II Guerra Mundial at os anos 70, houve um elevado crescimento do
desemprego.
d) na dcada de 1990, ocorreu grande queda do desemprego devido
abertura econmica.
e) atualmente, o desemprego estrutural ocorre devido busca de maior
competitividade.

Assinale a opo que apresenta consideraes adequadas acerca dessa


reduo quantitativa de componentes da famlia brasileira.
a) A reduo do nmero mdio de membros das famlias no pas est
associada, sobretudo nas reas de fronteira agrcola, s pssimas condies
sanitrias e concentrao de terras que impedem o pleno desenvolvimento
das famlias.
b) As polticas demogrficas natalistas nas duas ltimas dcadas do sculo
XX, implementadas pelo governo federal, foram malsucedidas, uma vez que o
Brasil apresenta queda no nmero mdio de pessoas nas famlias em todo o
pas.
c) A grande migrao da populao do campo para as cidades, fenmeno
caracterstico da segunda metade do sculo passado, a principal
responsvel pela reduo das famlias em grande parte do pas, sobretudo
nas periferias e nas favelas das grandes metrpoles.
d) A grande diferena do nmero mdio de membros das famlias rurais e
urbanas resultou do baixo nvel cultural da populao camponesa, incapaz de
adotar um planejamento familiar mais eficaz.
e) A adoo do modo de vida urbano, pelo campo, implicando o estmulo ao
consumo de bens, utilizao de servios e s prticas de lazer, bem como
as mudanas culturais nos relacionamentos interpessoais, contriburam para
a reduo do nmero mdio de pessoas nas famlias em todo o pas.

64) (UFRJ)

"Em 1960 a populao brasileira aumentava 2,9% ao ano. O


censo de 1991 demonstrou um i declnio dessa taxa para 1,9% ao ano."
Anurio Estatstico do Brasil, 1993.
O motivo que leva a populao brasileira a crescer atualmente, em nmeros
reais, mais do que antes pode ser explicado em virtude :
a) das medidas tomadas pelos governos para melhoria das condies de
higiene e saneamento (gua e esgotos tratados).
b) da grande quantidade de estrangeiros (imigrao) que entram no pas
com a finalidade de aqui permanecer.
c) das melhores condies de vida e da evoluo da cincia mdica que vm
permitindo a sobrevivncia de um maior nmero de crianas.
d) da taxa de crescimento vegetativo, que, embora seja mais baixa, incide
sobre uma populao muito maior do que na dcada de 60. e) de o pas
haver atingido um alto grau de qualidade de vida, existindo, em decorrncia
desse fato, um nmero muito maior de pessoas idosas.
65) (Fatec-SP) A questo est relacionada ao grfico e s afirmaes I, II,
III e IV.

I. As altas taxas de mortalidade esto relacionadas taxa de fecundidade da


mulher brasileira.
II. A mortalidade infantil um bom indicador da qualidade das polticas de
sade e da rede de saneamento disponveis para a populao.
III. Dentre as causas das altas taxas de mortalidade infantil, as doenas
infecciosas e a desnutrio esto diretamente ligadas pobreza.
IV. Atualmente, as taxas de mortalidade infantil so homogneas em todo o
Brasil.
A leitura do grfico e seus conhecimentos sobre a populao brasileira
permitem concluir que esto corretas apenas as afirmaes
a) I e II
b) I e III
c) I e IV
d) II e III

67) (Enem) Um dos aspectos utilizados para avaliar a posio ocupada pela
mulher na sociedade a sua participao no mercado de trabalho. O grfico
mostra a evoluo da presena de homens e mulheres no mercado de
trabalho entre os anos de 1940 e 2000.

Da leitura do grfico, pode-se afirmar que a participao percentual do


trabalho feminino no Brasil
a) teve valor mximo em 1950, o que no ocorreu com a participao
masculina.
b) apresentou, tanto quanto o masculino, menor crescimento nas trs ltimas
dcadas.
c) apresentou o mesmo crescimento que a participao masculina no perodo
de 1960 a 1980.
d) teve valor mnimo em 1940, enquanto que a participao masculina teve o
menor valor em 1950.
e) apresentou-se crescente desde 1950 e, se mantida a tendncia, alcanar,
em curto prazo, a participao masculina.

68) (Enem)

O quadro a seguir mostra a taxa de crescimento natural da


populao brasileira no sculo XX.
Perodo

Taxa anual mdia de


crescimento natural (%)

1920-1940

1,90

1940-1950

2,40

1950-1960

2,99

1960-1970

2,89

1970-1980

2,48

1980-1991

1,93

1991-2000

1,64

Fonte: IBGE, Anurios Estatsticos do Brasil.


Analisando os dados podemos caracterizar o perodo entre:
a) 1920 e 1960, como de crescimento do planejamento familiar.
b) 1950 e 1970, como de ntida exploso demogrfica.
c) 1960 e 1980, como de crescimento da taxa de fertilidade.
d) 1970 e 1990, como de decrscimo da densidade demogrfica.
e) 1980 e 2000, como de estabilizao do crescimento demogrfico.

69) (UFV-MG)

No final de junho de 2001 a opinio pblica nacional ficou


abalada com o acidente ocorrido em uma rea de minerao prxima a Belo
Horizonte, em que 5 operrios morreram aps o desmoronamento de uma
barragem de conteno de rejeitos. Essa regio, no interior de Minas Gerais,
vem sendo intensivamente explorada, tendo permanecido por dcadas como
a principal produtora de minrio de ferro do pas. Contudo, a partir do final
da dcada de 70 essa hegemonia ficou comprometida com a entrada em
cena de uma nova regio produtora de minrios que, atualmente, est ligada

64

por estrada de ferro a um moderno porto, direcionando a maior parte de sua


produo para a exportao.

70) Assinale a

alternativa que aponta CORRETAMENTE o nome dessa nova


regio produtora, bem como o nome e a localizao do porto destinado
exportao do minrio de ferro l extrado.
a) Serra dos Carajs e porto Trombetas, em Macap (Amap).
b) Serra do Navio e porto de Tubaro, em Macap (Amap).
c) Serra dos Carajs e porto de Itaqui, em So Lus (Maranho).
d) Macio do Urucum e porto de Santos, em Santos (So Paulo).
e) Serra do Navio e porto Trombetas, em Macap (Amap).

71) (UEL-PR) Considere os grficos apresentados a seguir.

A partir dos grficos e de seus conhecimentos sobre o assunto, pode-se


afirmar que: I. O setor primrio da economia, representado por I, teve forte
diminuio na participao do PIB devido crescente importncia do setor
industrial, representado no grfico por II.
II. O grande dinamismo do setor tercirio da economia, representado por
III, deve-se crescente participao dos setores de servios, principalmente
nas reas de telecomunicaes e turismo.
III. A regresso da participao do setor secundrio da economia entre
1980/90, representado por II, est relacionada concentrao industrial
iniciada na dcada de 1980.
Est correto o que est afirmado em
a) I, somente.
b) I e II, somente.
c) I e III, somente.
d) II e III, somente.
e) I, II e III.

72) (PUCC-SP) Observe o grfico para responder questo.

A leitura do grfico e seus conhecimentos sobre a economia brasileira


permitem concluir que a liderana do Brasil no continente sul-americano
deve-se ao fato de que
a) o Pas utiliza toda a capacidade produtiva na indstria automobilstica, por
isso lidera a produo sul-americana de automveis.
b) ao contrrio de outros ramos industriais, o setor automobilstico cresceu
porque no promoveu dispensa de funcionrios e, portanto, no gerou
desemprego.
c) recentemente, o Estado adotou uma srie de medidas protecionistas,
sobretudo no sentido de controlar a produo e o preo dos automveis.
d) o Pas que tem como atrativo o mercado consumidor recebeu inmeros
investimentos no setor, o que resultou na abertura de novas indstrias e
modernizao das j existentes.
e) os pases que compem o Mercosul tornaram-se consumidores de veculos
brasileiros, sendo eles os responsveis pelo aumento de produo.

73) (Fuvest-SP)

Considere as afirmaes a seguir sobre os plos


tecnolgicos no Brasil.
I. Os plos tecnolgicos concentram as atividades de pesquisa e
desenvolvimento de tecnologias de ponta.

II. Os plos tecnolgicos concentram atividades industriais que independem


de outros setores da economia.
III. O principal plo tecnolgico do pas a Zona Franca de Manaus, devido
presena de vrias incubadoras tecnolgicas.
IV. Os principais plos tecnolgicos do Estado de So Paulo se localizam na
capital, em So Jos dos Campos, Campinas e So Carlos.
Est correto o que se afirma em
a) I e II.
b) I e III.
c) I e IV.
d) II e III.
e) II e IV.

74) (UFPE)

O grande plo industrial do Brasil situa-se no tringulo So


Paulo-Rio-Belo Horizonte. uma rea que abrange o leste do Estado de So
Paulo, o Rio de Janeiro, o sul de Minas Gerais e o sul do Esprito Santo. Sobre
esse assunto, podemos afirmar que:
a) a cidade de So Paulo transformou-se no principal plo industrial do pas,
j no sculo XVIII.
b) a economia cafeeira de exportao, apesar de ter gerado um notvel
crescimento do PIB brasileiro no incio do sculo XX, no gerou recursos
necessrios para o processo de industrializao do Sudeste.
c) o principal fator de localizao das indstrias no Sudeste foram as grandes
reservas de carvo existentes na regio, particularmente em So Paulo.
d) as primeiras reas industriais do Rio de Janeiro situaram-se afastadas dos
eixos ferrovirios e prximas Baa de Guanabara.
e) aps a Revoluo de 30, com a finalizao da chamada "poltica do cafcom-leite", as elites econmicas de Minas Gerais voltaram a ateno para o
desenvolvimento industrial do Estado.

75) (UFPE)

No ltimo quartel do sculo XX, particularmente na dcada de


90, uma nova forma de organizao empresarial tem agregado os centros de
formao de pessoal de alto nvel s unidades de produo e de servios,
empregando os mais modernos recursos de microeletrnica. Em tais centros
esto se implantando atividades de alta tecnologia, como em Campinas e So
Jos dos Campos, na regio Sudeste do Brasil. Qual a denominao dada a
esses centros?
a) centros megalopolitanos.
b) centros-acrpoles
c) regies metropolitanas
d) tecnoplos
e) edifcios empresariais urbanos

76) (UFRGS) Observe os dados apresentados na tabela a seguir.


rea em relao
Grau de
Regies
ao Brasil (%)
urbanizao (%)
1 10,9
90,52
2 18,7
86,73
3 18,2
69,04
4 6,8
80,93
Os nmeros 1, 2, 3 e 4 representam, na tabela, respectivamente, as
seguintes regies brasileiras:
a) Sul, Centro-Oeste, Nordeste e Sudeste.
b) Sul, Nordeste, Centro-Oeste e Sudeste.
c) Sudeste, Centro-Oeste, Nordeste e Sul.
d) Sudeste, Nordeste, Centro-Oeste e Sul.
e) Centro-Oeste, Sudeste, Nordeste e Sul.

77) (UFRGS)

Algumas cidades do Brasil foram previamente projetadas e


outras, planejadas em diferentes perodos de sua expanso urbana. Das
cidades a seguir relacionadas, indique as duas que foram previamente
projetadas.
a) Anpolis - So Lus
b) Anpolis-Curitiba
c) Belm-Curitiba
d) Belo Horizonte - So Lus
e) Belo Horizonte - Goinia

78) (UFSM-RS) Considerando o processo de influncia de uma cidade sobre


a outra, assinale a alternativa que apresenta a ordem hierrquica decrescente
desse processo.
a) da metrpole nacional aos centros locais.
b) dos centros regionais metrpole nacional.
c) dos centros locais metrpole regional.
d) das capitais regionais metrpole regional.
e) dos centros regionais s capitais regionais.

79) A questo est relacionada ao mapa e s afirmaes a seguir.

65

I. As metrpoles 1, 3 e 8 so as mais industrializadas de suas respectivas


regies.
II. As metrpoles 4 e 5 apresentam parques industriais intimamente
relacionados com recursos naturais encontrados nas suas respectivas reas
de influncia.
III. As metrpoles 6 e 7 tm um dinamismo econmico que as diferencia das
demais.
IV. As metrpoles 2 e 6 conseguiram reduzir por meio de polticas sociais as
submoradias, como favelas e cortios.
Esto corretas apenas as asseres:
a) I e II.
b) I e III.
c) I e IV.
d) II e III.
e) II e IV.
Gabarito (Geografia Parte 2)

1) A

2) C

3) VFVVV

4) B

5) A

6) C

7) D

8) A

9) A

10) C

11) A

12) B

13) D

14) A

15) A

16) B

17) E

18) 1+8=9 19) B

20) A

21) E

22) E

23) E

24) C

25) E

26) D

27) C

28) B

29) B

30) D

31) B

32) B

33) D

34) A

35) C

36) B

37) E

38) B

39) A

40) B

41) B

42) C

43) D

44) A

45) C

46) B

47) FVVVV 48) C

49) A

50) B

51) D

52) A

53) A

54) E

55) D

56) C

57) C

58) D

59) A

60) D

61) C

62) B

63) E

64) D

65) D

66) E

67) E

68) A

69) C

70) A

71) D

72) C

73) E

74) D

75) C

76) E

77) A

78) D

79)

80)

66

1.

(MACK-SP/99) Rochas gneas ou magmticas so formadas pela


solidificao do magma (lava). Um exemplo tpico de rocha magmtica
o granito, que usado como revestimento de edifcios, pisos, etc. Sobre
o granito correto afirmar que uma:
a) substncia pura composta.
b) mistura heterognea.
c) substncia pura simples.
d) mistura homognea.
e) substncia solvel em gua.

2.

(UFRGS-RS/RS) Indique o sistema que NO corresponde a uma mistura


homognea.
a) gasolina filtrada
b) ar puro
c) ao inoxidvel
d) granito
e) usque

3.

(UFES/99) Uma determinada substncia apresenta a seguinte curva de


aquecimento:

d)
e)
8.

filtrao e destilao fracionada.


decantao e destilao fracionada.

(UFRGS) Associe as atividades dirias contidas na primeira coluna com


as operaes bsicas de laboratrio e fenmenos contidos na segunda
coluna.
(1) preparar um refresco de caj a partir do suco
( ) sublimao
concentrado
(2) adoar o leite
( ) diluio
(3) prepara ch de canela
( ) filtrao
(4) usar naftalina na gaveta
( ) extrao
(5) coar a nata do leite
( ) dissoluo
Os nmeros da segunda coluna, lidos de cima para baixo, so:
a) 3, 2, 5, 4, 1.
b) 1, 3, 4, 5, 2.
c) 4, 3, 2, 1, 5.
d) 3, 2, 4, 5, 1.
e) 4, 1, 5, 3, 2.

9.

(UFPE/98) Considere a figura abaixo:

Considerando que a substncia no estado slido existe em apenas uma


forma alotrpica, assinale a alternativa CORRETA.
a) A substncia um slido a 200 K.
b) A substncia um gs a 300 K.
c) Entre 5 e 10 minutos de aquecimento, a substncia existe somente
na forma lquida.
d) Entre 5 e 10 minutos de aquecimento, a substncia existe como
uma mistura em equilbrio das fases lquida e gasosa.
e) A substncia um gs a 450 K.
4.

(MACK-SP/99)
Substncia
Ponto de fuso (C)
Ponto de ebulio (C)
Etanol
-117
78
ter etlico
-116
34
Pela anlise dos dados da tabela acima, medidos a 1 atm podemos
afirmar que, temperatura de 40 C e 1 atm:
a) o ter e o etanol encontram-se na fase gasosa.
b) o ter encontra-se na fase gasosa, o etanol na fase lquida.
c) ambos encontram-se na fase lquida.
d) o ter encontra-se na fase lquida, e o etanol na fase gasosa.
e) ambos encontram-se na fase slida.
10.

5.

(UFRGS-RS/98) Qual dos mtodos de separao seguintes se baseia na


diferena de densidades?
a) Decantao.
b) Destilao fracionada.
c) Peneirao.
d) Cristalizao.
e) Sublimao.

6.

(UEL-PR) De uma mistura heterognea de dois lquidos imicveis e de 11.


densidade diferentes, pode-se obter lquidos puros por meio de
I. sublimao.
II. decantao.
III. filtrao.
Dessas afirmaes, APENAS
a) I a correta.
b) II a correta.
c) III a correta.
12.
d) I e II so corretas.
e) II e III so corretas.

7.

(UFRGS-RS/2000) Um sistema heterogneo bifsico formado por trs


lquidos diferentes A, B e C. sabe-se que:
A e B so miscveis entre si.
C imiscvel com A e com B.
A mais voltil que B.
Com base nessas informaes, os mtodos mais adequados para
separar os trs lquidos so:
a) centrifugao e decantao.
b) decantao e fuso fracionada.
c) filtrao e centrifugao.

e as seguintes possibilidades para o lquido existente no interior do copo:


(I)
(II)
+ Glicose
(III)
+ Sal de Cozinha
Qual a alternativa que melhor descreve a condio da lmpada?
a) Acesa em II e a apagada nas demais.
b) Apagada em I e acesa nas demais.
c) Apagada em I e II.
d) Acesa em I, II e III.
e) Acesa em I e apagada nas demais.
(ITA-SP) Colocando-se gros de nitrato de potssio em um frasco com
gua, nota-se que com o passar do tempo o slido desaparece dentro da
gua. Qual das equaes abaixo a mais adequada para representar a
transformao que ocorreu dentro do frasco? (c) = cristal ou slido.
( )
()
a)
( )
()
( ) (
) ( )
b)
( )
( )
( )
c)
( )
( )
d)
()
( )
()
( )
( )
e)
(UFES) Os cidos com as frmulas moleculares
respectivamente:
a) metafosfrico, ortofosfrico e pirofosfrico.
b) ortofosfrico, metafosfrico e pirofosfrico.
c) ortofosfrico, pirofosfrico e metafosfrico.
d) pirofosfrico, metafosfrico e ortofosfrico.
e) pirofosfrico, ortofosfrico e metafosfrico.

so,

(MACK-SP/2001) Na reao entre os gases N2 e H2, obtm-se


unicamente gs amnia. A soluo aquosa de amnia recebe o nome de
amonaco (hidrxido de amnio), que o componente ativo de produtos
de limpeza usados na remoo de gorduras.
A partir dessas informaes, considere as seguintes afirmaes:
I. O hidrxido de amnio tem frmula NH 3.
II. Na formao do gs amnia, a reao ocorrida de sntese.
III. O amonaco tem frmula NH4OH.
IV. A amnia tem frmula NH4OH.
V. O cheiro irritante e forte que se sente quando se usa amonaco,
proveniente do gs nitrognio.
Esto corretas somente:
a) I e I.

67

13.

14.

15.

b) II e V.
18.
c) II e III.
d) I e II.
e) III e V.
(FUVEST) Hidroxiapatita, mineral presente em ossos e dentes,
constituda por ons fosfato (
) e ons hidrxido. A sua frmula
qumica pode ser representada por
(
) ( ). O valor de x nessa
frmula :
a) 1
19.
b) 2
c) 3
d) 4
e) 5
(UFRGS-RS) No processo de produo do sal refinado, a lavagem do sal
marinho provoca a perda do iodo natural, sendo necessrio, depois,
acrescent-lo na forma de iodeto de potssio. Outra perda significativa
a de ons magnsio, presentes no sal marinho na forma de cloreto de
magnsio e sulfato de magnsio. Durante este processo so tambm
adicionados alvejantes como o carbonato de sdio. As frmulas
representativas
das
substncias
sublinhadas
no
texto
so
respectivamente:
a) KI, MgC , MgSO4 e NaCO3
20.
b) K2I, MgC 2, Mg2SO4 e Na2CO3
c) K2I, Mg2C , MgSO4 e Na(CO3)2
d) KI, MgC 2, MgSO4 e Na2CO3
e) KI2, Mg2C , Mg(SO4 )2e Na3CO3
(MACK-SP/2000)
Fora e solubilidade de bases em gua
bases de metais alcalinos
fortes e solveis
fortes e parcialmente solveis, exceto a
bases de metais alcalino-terrosos
de magnsio, que fraca
demais bases
fracas e praticamente insolveis

(UFAL) Considerando que nos processos abaixo est envolvida a mesma


quantidade de amnia, qual deles o mais endotrmico?
a) Liquefao de
( ).
b) Reao de
( ) com cido.
c) Decomposio de
( ) em seus tomos.
d) Decomposio de
( ) em molculas
( )
( ).
e) Escape de
( ) de uma soluo aquosa de amnia.
(Fuvest-SP) Considere a reao de fotossntese e a reao de combusto
da glicose representadas abaixo:
( )

()
( )

( )
( )

( )

( )
()

Sabendo-seque a energia envolvida na combusto de um mol de glicose


2,8 106 J, ao sintetizar meio mol de glicose, a planta:
a) Libera 1,4 106 J.
b) Libera 2,8 106 J.
c) Absorve 1,4 106 J.
d) Absorve 2,8 106 J.
e) Absorve 5,6 106 J.
(ITA-SP) Esta questo se refere comparao do efeito trmico
verificado ao se misturarem 100 cm3 de soluo aquosa 0,10 mol/L de
cada um dos cidos abaixo com 100 cm3 de soluo aquosa 0,10 mol/L
de cada uma das bases abaixo. A tabela a seguir serve para deixar claro
a notao empregada para designar os calores desprendidos.
cidos/bases
HC
HNO3
H3CCOOH
NaOH
|H11|
|H12|
|H13|
KOH
|H21|
|H22|
|H23|
NH4OH
|H31|
|H32|
|H33|

Lembrando que o processo de dissociao de eletrlitos endotrmico,


correto esperar que:
a) |H33| seja o maior dos |H| citados.
b) |H11| = |H13|
Para desentupir um cano de cozinha e para combater a acidez
c) |H23| = |H33|
estomacal, necessita-se, respectivamente, de uma base forte e solvel e
d) |H31| = |H32|
de uma base fraca parcialmente solvel. Consultando a tabela acima,
e) |H21| = |H23|
conclui-se que as frmulas dessas bases podem ser:
21. (UFRN) Quando uma nave espacial est retornando Terra, ao reentrar
a)
( )
( )
na atmosfera provoca as seguintes reaes qumicas dos componentes
b)
( )
do ar:
c)
( )
I.
d)
( ) e
( )
( )
( )
II.
e)
( )
( )
( )
III.
( )
( )
( )
16. (MACK-SP/98) A seqncia numrica correta obtida na associao das
H = +167,2 kJ
substncias da coluna A s afirmaes da coluna B, de cima para baixo
Quanto ao calor envolvido nessas reaes, podemos afirmar que:
:
a) I, II e III so exotrmicas.
A
B
b) I, II e III so endotrmicas.
1) BaSO4
( ) usada em alvejante domstico
c) Apenas III endotrmica.
2) NaC
( ) um dos componentes do soro fisiolgico
d) Apenas I e II so endotrmicas.
3) CaSO4 2H2O ( ) usada com contraste em radiografias de estmago
e) Apenas I e II so exotrmicas.
4) NaC O
( ) usada como preventivo contra as cries
5) NaF
( ) usada na fabricao de gesso
22. (Med. Catanduva-SP) Dadas as equaes termoqumicas:
a) 1; 2; 3; 4; 5
A:
H = +67,6 kJ
( )
( )
( )
b) 2; 1; 5; 3; 4
B:
H = +9,6 kJ
( )
( )
( )
c) 3; 2; 4; 5; 1
So feitas as seguintes afirmaes:
d) 5; 4; 3; 2; 1
I. A equao (B) representa a formao do
( ).
e) 4; 2; 1; 5; 3
II. A decomposio do
( ) , segundo a equao (B), absorve 9,6
17. (Med. Pouso Alegre-MG) Assinale a alternativa correta. Observe o grfico
kJ por mol.
a seguir:
III. A dimerizao do
( ) libera 58,0 kJ/mol de
( ).
So corretas as afirmaes:
a) I, somente
b) I e II
c) I e III
d) II e III
e) I, II e III
23.
A variao de entalpia da reao:
1 Y2 + 1 X2 2 YX :
a) -A
b) B
c) -2 A
d) B - A
e) B + A

(FGV-SP) Em um conversor cataltico, usado em veculos automotores


em seu cano de escape para reduo da poluio atmosfrica, ocorrem
vrias reaes qumicas, sendo que uma das mais importantes :
( )

( )

( )

Dado que as entalpias das reaes abaixo so:


H = -26,4 kcal.
(
)
( )
( )
H = -94,1 kcal.
(
)
( )
( )
Pode-se afirmar que a reao inicial :
a) Exotrmica e absorve 67,7 kcal/mol.
b) Exotrmica e libera 120,5 kcal/mol.
c) Exotrmica e libera 67,7 kcal/mol.
d) Endotrmica e absorve 120,5 kcal/mol.

68

e)
24.

Endotrmica e absorve 67,7 kcal/mol.

(Cesgranrio-RJ) O elemento qumico tungstnio, de smbolo W, muito


utilizado em filamentos de lmpadas incandescentes comuns.
Quando ligado a elementos como carbono ou boro, forma substncia
quimicamente inertes e muito duras.
O carboneto de tungstnio,
( ) , muito utilizado em ponteiras de 29.
ferramentas, como perfuratrizes, esmeris, lixas para metais etc.
Essa substncia pode ser obtida pela reao :
(

( )

a)
b)
c)
d)
e)

I e IV
II e III
III e IV
I e III
I, III e IV

(Odonto-Diamantina-MG) Considerando o grfico a seguir, esto


corretas, exceto:

( )

A partir das reaes a seguir, calcule o H de formao para o


( ).
Dados:

( )
( )
( )
= -840 kJ/mol

(
)
( )
( )
= -394
kJ/mol

( )
( )
( )
( )
= -1.196
kJ/mol
a)
b)
c)
d)
e)

-19 kJ/mol
+38 kJ/mol
-38 kJ/mol
+2.430 kJ/mol
-2.430 kJ/mol

25.

(Odonto-Diamantina-MG) Tendo em vista a tabela de energias de


ligaes fornecida a seguir, calcule quantos gramas de gua podem ser
evaporados pelo calor liberado na queima de 798 gramas de gasolina,
com base nas informaes fornecidas nos itens a seguir.
I. Considere a gasolina composta por 100% de isoctano:
()
30.
II.
()
( ) (T = cte.)

III.
()
( )
( )
()
IV.
Ligao
C C
C H O O
C O
O H
Energia de ligao/kcal83,1
99,5
117,0
173
109
mol-1
Observao: frmula estrutural do isoctano:

31.
a)
b)
c)
d)
e)
26.

1,1
1,1
1,1
1,1
1,1

103
102
103
104
105

(Unisinos-RS) A combusto completado etanol ocorre pela equao:


()

( )

( )

()

Considerando que em 1 h de reao foram produzidos 2.640 g de gs


carbnico, voc conclui que a velocidade da reao, expressa em
nmero de mol de etanol consumido por minuto, :
a) 0,5
b)1,0
c) 23 d)46
e) 69
27.

(PUC-SP) Em determinada experincia, a reao de formao de gua 32.


est ocorrendo com o consumo de 4 mol de oxignio por minuto.
Conseqentemente, a velocidade de consumo de hidrognio de:
a) 2 mol/min
b) 4 mol/min
c) 8 mol/min
d) 12 mol/min
e) 16 mol/min

28.

(Mack-SP) Uma mistura de vapor de gasolina e ar, temperatura


ambiente, no reage. Entretanto, no motor de carros, em presena de
fasca eltrica, ocorre a combusto da gasolina. Dessa constatao, so
feitas as seguintes afirmaes:
I. A fasca fornece mistura a energia necessria para iniciar a
reao.
33.
II. A fasca a nica responsvel pela combusto da gasolina, uma
vez que ela ocorre mesmo em total ausncia de ar.
III. A reao que ocorre exotrmica.
IV. A fasca faz com que as molculas de oxignio se separem do ar e
reajam com a gasolina.
Das afirmaes feitas, somente so corretas:

a)
b)
c)
d)
e)

A energia de ativao da reao A


D igual a 50 kcal/mol.
O H da reao direta +100 kcal/mol.
A reao D A exotrmica.
A energia de ativao da reao direta trs vezes maior que a
energia de ativao da reao inversa.
A entalpia do complexo ativado 150 kcal.

(Unisinos-RS) Nas reaes qumicas, de um modo geral, conseguimos


aumentar a velocidade da reao por meio da elevao de temperatura.
Isto ocorre porque aumenta:
I. A velocidade mdia das molculas reagentes.
II. A energia cintica mdia das molculas dos reagentes.
III. A freqncia das colises entre as molculas.
Das afirmaes:
a) Somente I correta.
b) Somente II correta.
c) Somente I e II so corretas.
d) Somente I e III so corretas.
e) I, II e III so corretas.
(UFMG) A tabela mostra resultados de experincias em que comprimidos
de anticido efervescentes foram dissolvidos em gua.
Tempo para se completar a
Estado do comprimido
Temperatura da gua/C
dissoluo/min
Inteiro
20
1
Inteiro
30
0,5
Pulverizado
20
0,7
Pulverizado
40
0,2
Considerando-se os resultados da tabela e os fatores que, em geral,
influenciam as velocidades da reao, todas as alternativas esto
corretas, exceto:
a) A pulverizao aumenta a energia cintica das partculas.
b) A pulverizao aumenta a freqncia de colises das partculas do
comprimido com molculas de gua.
c) A velocidade de dissoluo depende de mais de um fator.
d) O aquecimento aumenta a energia mdia das colises.
e) O aumento de superfcie de contato favorece a dissoluo.
(UEL-PR) A decomposio do perxido de hidrognio pode ser
representada pela equao:
H < 0
()
()
( )
Das seguintes condies:
I. 25 C e ausncia de luz.
II. 25 C e presena de catalisador.
III. 25 C e presena de luz.
IV. 35 C e ausncia de luz.
V. 35 C e presena de catalisador.
aquela que favorece a conservao do perxido de hidrognio :
a) I
b) II
c) III d)IV
e) V
(Odonto-Diamantina-MG) Analise as afirmaes:
I. As reaes, sejam exotrmicas sejam endotrmicas, ocorrem mais
rapidamente a 50 C que a 20 C.
II. Catalisadores aumentam a velocidade das reaes porque
aumentam a energia de ativao.
III. A 20 C, a reao:
(

()

69

muito mais rpida que a reao:


( )

( )

( )

Esto corretas:
a) I, II e III.
c) I e III.
e) somente I.

d) IV
10
e) V
100
40. (Uniju-RS) O seguinte sistema de equilbrio estabelecido quando se
misturam os gases
( )
( )

()

B)I e II.
D)II e III.

( )

34.

(ITA-SP) Uma certa reao qumica representada pela equao:

35.

onde A e B so as espcies qumicas que foram colocadas para


reagir.
Verificou-se, experimentalmente, numa certa temperatura, que a
velocidade desta reao quadruplica com a duplicao da concentrao
da espcie A, mas no depende da concentrao da espcie B.
Assinale a opo que contm, respectivamente, a expresso correta da
velocidade e o valor correto da ordem da reao.
41.
a) v = k [A]2 [B]2 e 4
b) v = k [A]2 [B]2 e 3
c) v = k [A]2 [B]2 e 2
d) v = k [A]2 e 4
e) v = k [A]2 e 2
(UFRGS-RS) Para a reao:

( )

( )

( )

( )

( )

( )

verifica-se experimentalmente que a velocidade de formao de C


independe da concentrao de B e quadruplicada quando se dobra a
concentrao de A. A expresso matemtica da lei da velocidade para
essa reao :
a) k [A]2 [B]
42.
b) k [A] [B]
c) k [A]2
d) k [A]4
e) k [A]4 [B]
36.

(UEL-PR) No estudo de uma reao representada por:


( )

( )

(UFPA) A expresso da lei do equilbrio KC =


reao:
a)
b)
c)
d)
e)

38.

39.

( )

( )
( )
()

( )
( )

( )

(UFRGS-RS) Para a dissociao do trixido de enxofre:


( )

( )

( )

o valor de KC 5,0 mol/L, a uma certa temperatura. Num recipiente de


10 litros, a essa mesma temperatura, verifica-se que esto presentes, no
equilbrio, 40 mol de
( ) e 50 mol de
( ) Portanto, a quantidade de
matria de
( ) no-dissociada :
a) 4
b) 5
c) 16
d) 20
e) 40
(PUC-SP) O qumico francs Claude Louis Berthollet (1748-1822),
enquanto servia expedio de Napoleo, no Egito, observou que nas
margens dos lagos salgados havia, como resultado da evaporao da
gua da soluo salina, a presena de carbonato de sdio no sedimento.
Era, perfeitamente conhecido o fato de que em laboratrio ocorria o
processo espontneo:

( )

coletou-se os seguintes dados:


[A] inicial
[B] inicial
Velocidade/mol L-1 s-1
0,10
0,10
2,53 10-6
0,10
0,20
5,06 10-6
0,20
0,10
10,01 10-6
A velocidade da reao pode ser expressa por:
a) v = k 2 [A]
b) v = k [B]2
c) v = k [A] [B]
d) v = k [A]2 [B]
e) v = k [A] [B]2
43.
37.

( )

Verifica-se que, ao se colocar 1,0 mol de


( )
( ) e 1,0 mol de
em recipiente de 10 L, havia se formado 0,20 mol de
( ) no
equilbrio. A constante de equilbrio em funo das concentraes em
mol/L, KC, deve ser aproximadamente:
a) 2,00
b) 0,20
c) 0,25
d) 5,00
e) 3,12

( )
( )
( )
( )

1
[O2 ]

( )
( )
( )
( )
( )

()

pertence seguinte

que tendia a se completar devido precipitao de


(
)
A explicao que sugeriu para a ocorrncia do processo inverso foi a de
que a inverso era causada:
a) pela presena de catalisadores, no conhecidos, no lago.
b) por variaes na presso baromtrica nessa regio.
c) pela pequena solubilidade do
em gua.
d) pelo grande excesso de NaC no lago salgado.
e) pelas diminuies drsticas da temperatura durante a noite.
(Cesesp-PE) Qual a variao na constante de equilbrio K, para a reao
endotrmica:
( )

( )

quando:
I. a temperatura (T) aumentada e
II. o volume (V) do sistema diminudo?
I. T aumenta
II. V diminui
a) K aumenta
K diminui
b) K diminui
K aumenta
c) K no se modifica
K no se modifica
d) K no se modifica
K aumenta
e) K aumenta
K no se modifica
(PUC-SP) O gs hidrognio pode ser obtido a partir da reao entre o
monxido de carbono,
( ) , e a gua,
( ) , a altas temperaturas.
Esse processo em equilbrio representado por:
H = 41 kJ
( )
( )
( )
( )
Para tornar mais eficiente a produo de ( ), deve-se:
a) aumentar a presso do sistema.
b) diminuir a presso do sistema.
c) utilizar um catalisador apropriado.
d) aumentar a temperatura do sistema.
e) retirar do sistema o gs carbnico,
( ) , produzido.

(UEL-PR) Dentre os equilbrios qumicos:


I.
( )
( )
( )
44.
II.
( )
( )
III.
( )
( )
( )
IV.
( )
( )
( )
V.
( )
( )
( )
( )
aquele com Kp = KC :
Observaes: Kp a constante de equilbrio em termos de presses
parciais e KC a constante de equilbrio em termos de concentrao em
quantidade de matria.
a) I
b)II
c) III d)IV
e) V
45. (Unip-SP) O esmalte dos dentes consiste em uma substncia insolvel
chamada hidroxiapatita. Na boca existe equilbrio:
(Cesgranrio-RJ) Um grupo de pesquisadores qumicos apresentou
determinado relatrio contendo resultados de estudos sobre processos
alternativos para a produo de substncia de vital importncia para a
A dissoluo da hidroxiapatita chamada de desmineralizao e a sua
populao.
formao chamada de mineralizao. Quando o acar absorvido no
Considerando as constantes de equilbrio, usadas como critrio de
dente e fermenta, produz-se H+1, que provoca todas as transformaes
escolha e apresentadas a seguir para cada processo, espera-se que o
seguintes, exceto:
responsvel opte pelo processo:
a) Ocorre uma desmineralizao.
Processo
KC
b) A concentrao de ( ) aumenta.
a) I
0,01
c) O on
( ) combina com o on
( ).
b) II
0,1
d)
Pode
se
resultar
na
queda
do
dente.
c) III
1

70

e)
46.

47.

A hidroxiapatita no sofre ataque de cidos.

b)

(Odonto-Diamantina-MG) Seja a reao, em equilbrio qumico, a 300 C:


O equilbrio ser deslocado para a direita se:
I. a ele for adicionado HC .
II. a temperatura for reduzida para 200 C.
III. o volume do recipiente, em que se passa a reao, for reduzido para
a metade.
a) Somente II.
B)II e III.
c) I, II e III.
d)I e II.
53.
e)I e III.
(Fuvest-SP) Em soluo aquosa, ons cromato,
segundo a reao:
(

),

de cor laranja,

()

A colorao laranjal fica mais intensa quando se:


a) adiciona OH1-.
b) aumenta a [H3O1+].
c) aumenta a presso.
d) acrescenta mais gua.
e) acrescenta um catalisador.
48.

(PUCCAMP-SP) A equao qumica, no-balanceada:


( )

49.

50.

51.

52.

( )

()

( )

representa a reao que ocorre em solos pouco aerados e envolve a


reduo de nitrato a nitrognio, em meio cido, na presena de
carboidrato. O fenmeno, conhecido como desnitrificao, prejudica a
fixao de
( ) pelas plantas. possvel aumentar a concentrao do 54.
( ) no equilbrio adicionando-se, no solo, espcie qumica capaz de:
a) liberar ons on
( ).
b) liberar ons
( ).
c) absorver
( ).
d) reagir com ( ).
e) consumir
( ).
(FEI-SP) A chuva cida ocorre em regies de alta concentrao de
poluentes provenientes da queima de combustveis fsseis. Numa chuva
normal o pH est em torno de 5,0, e em Los Angeles j ocorreu chuva
com pH em torno de 2,0. A concentrao de ons H 1+ dessa chuva
ocorrida em Los Angeles em relao chuva normal :
a) 1.000 vezes maior.
b) 1.000 vezes menor.
c) 3 vezes maior.
d) 3 vezes menor.
e) 100 vezes maior.
(UFRGS-RS) As leis de proteo ambiental de certas cidades no
permitem o lanamento em rios, entre outros, de efluentes com pH
inferior a 5,0 ou superior a 9,0. No que se refere acidez, os efluentes
aquosos das indstrias X, Y e Z apresentam concentraes da tabela
abaixo:
Indstria
Concentrao no efluente
X
10-3 M de H1+
Y
10-4 M de H1+
Z
10-6 M de OH1Poderiam ser lanados em rios, sem tratamento prvio, apenas os
efluentes de:
a) X
b) Y
c) Z
d) X e Y
e) Y e Z
(Fuvest-SP) temperatura ambiente, o pH de um certo refrigerante,
saturado com gs carbnico, quando em garrafa fechada, vale 4. Ao
abrir-se a garrafa, ocorre escape de gs carbnico. Qual deve ser o valor
do pH do refrigerante depois de a garrafa ser aberta?
a) pH = 4
b) 0 < pH < 4
c) 4 < pH < 7
d) pH = 7
e) 7 < pH < 14

c)
d)
e)

A oxidao e a hidrlise do SO2 na atmosfera levam formao de


cido sulfrico.
cidos fortes podem ionizar-se na gua das chuvas, abaixando seu
pH.
O abaixamento do pH da gua das chuvas significa uma diminuio
na concentrao dos ons hidrognio.
Esttuas e monumentos de mrmore, CaCO3, expostos a chuvas de
baixo pH, podem ser eventualmente destrudos.

(Cesgranrio-RJ) A tabela abaixo apresenta valores mdios de pH


encontrados no corpo humana e em substncias comuns.
Amostras
pH mdio
Suco gstrico
1,7
Suco de limo
2,3
Vinagre comum
3,1
Urina
6,0
gua pura
7,0
Sangue humano
7,4
Smen
8,3
Com base nas informaes contidas na tabela acima, marque a opo
correta.
a) O vinagre comum daria cor vermelha fenolftalena.
b) O suco gstrico aquele que apresenta menor concentrao cida.
c) A concentrao de ons H3O1+ na urina 10-6 mol/L.
d) O smen aquele que apresenta maior concentrao cida.
e) O sangue humano mais cido do que a gua pura.
(Fuvest-SP) O indicador azul de bromotimol fica amarelo em solues
aquosas de concentrao hidrogeninica maior do que 1,0 10-6 mol/L e
azul em solues de concentrao hidrogeninica menor do que 2,5 10 8
mol/L. Considere as trs solues seguintes, cujos valores do pH so
dados entre parnteses: suco de tomate (4,8), gua da chuva (5,6),
gua do mar (8,2). Se necessrio, use log 2,5 = 0,4. As cores
apresentadas por essas solues contendo o indicador so:
Suco de tomate gua da chuva gua do mar
a) amarelo
amarelo
amarelo
b) amarelo
amarelo
azul
c) amarelo
azul
azul
d) azul
azul
amarelo
e) azul
azul
azul

1) B

2) D

3) A

4) B

5) A

6) B

7) E

8) E

9) C

10) C

11) A

12) C

13) E

14) D

15) E

16) E

17) A

18) C

19) C

20) D

21) B

22) C

23) C

24) C

25) D

26) A

27) C

28) D

29) A

30) E

31) A

32) A

33) C

34) E

35) C

36) D

37) E

38) D

39) E

40) E

41) E

42) D

43) E

44) E

45) E

46) C

47) B

48) A

49) A

50) C

51) C

52) D

53) C

54) B

(UFMG) Tem-se notado um abaixamento do pH da gua das chuvas em


muitas regies do mundo. Uma das causas a emisso de dixido de
enxofre, SO2, feita por certas indstrias, o qual oxidado e hidrolisado
na atmosfera. Em relao ao exposto, qual das seguintes alternativas
errada?
a) Uma chuva com pH = 4,6 dez vezes mais cida do que uma chuva
com pH = 5,6.

71

1. (UFC-CE) O glio, utilizado na composio dos chips dos computadores, 6. (UFPA) O silcio ([Ne] 3s2 3p2), elemento fundamental na produo de
apresenta-se como um elemento qumico de notveis propriedades.
microprocessadores, pode ser classificado como:
Dentre elas, destacam-se a de fundir a 30C e somente experimentar
a) metal de transio.
ebulio a 240, presso atmosfrica. (Ga Z = 31)
b) metal pertencente ao quarto perodo da tabela peridica atual.
Com relao a esse elemento, correto afirmar que:
c) no-metal que contm dois eltrons na camada de valncia.
a) sua configurao eletrnica, no estado fundamental, [Ne]
d) no-metal pertencente ao grupo 14 da tabela peridica atual.
3d104s24p1, tendo a formar nions.
e) no-metal pertencente famlia dos halognios.
b) apresenta, no estado fundamental, trs eltrons desemparelhados,
encontrando-se sempre no estado lquido, independentemente da 7. (UFViosa-MG) Eletronegatividade uma propriedade Peridica
temperatura.
importante. Em relao a essa propriedade, assinale a afirmativa
c) seu on mais estvel representado por Ga13+, resultado na mesma
CORRETA:
configurao eletrnica do elemento nenio.
a) O potssio (K) mais eletronegativo que o clcio (Ca).
d) apresenta-se no estado slido, em temperaturas acima de 30C, e
b) O carbono (C) mais eletronegativo que o silcio (Si).
no estado lquido, em temperaturas abaixo de 2403C.
c) O sdio (Na) o mais eletronegativo de todos os elementos.
e) experimenta processo de fuso ao ser mantido por um longo
d) O flor (F) o menos eletronegativo de todos os elementos.
perodo de tempo em contato com a mo de um ser humano
e) O frncio (Fr) o mais eletronegativo de todos os elementos.
normal.
8. (UFMG) Um dos fatores que favorecem a solubilidade de um metal em
2. (PUC-SP) Resolva a questo com base na anlise das alternativas a seguir:
outro a semelhana de suas redes cristalinas. No entanto preciso,
I. Em um mesmo perodo, os elementos apresentam m esmo
tambm, que os seus tomos no sejam muito diferentes quanto ao raio
nmero de nveis.
atmico, a eletronegatividade e a valncia.
II. Os elementos do grupo 2 apresentam, na ltima camada, a
Os metais alcalinos e o ferro, que apresentam redes cristalinas
configurao geral ns2.
semelhantes, no formam ligas por causa das grandes diferenas quanto
III. Quando o subnvel mais energtico tipo s ou p, o elemento de
a essas propriedades.
transio.
Considerando-se as propriedades peridicas do ferro e dos metais
IV. Em um mesmo grupo, os elementos apresentam o mesmo
alcalinos, INCORRETO afirmar que:
nmero de camadas.
a) a eletronegatividade do tomo de ferro maior do que a do tomo
Conclui-se que, com relao estrutura da classificao peridica dos
de sdio.
elementos, estrutura da classificao peridica dos elementos, esto
b) o nmero de oxidao mais comum dos metais alcalinos +1.
corretas as afirmativas:
c) o raio atmico do ferro maior que o do potssio.
a) I e II.
d) o raio atmico de ferro menor que o do rubdio.
b) I e III.
c) II e III.
9. (UFRG-RS) O quadro a seguir apresenta valores comparativos de DUAS
d) II e IV.
propriedades peridicas, medidos em unidades convenientes
e) III e IV.
propriedade.
Elemento
Propriedade 1
Propriedade 2
3. (FUVEST) Os elementos I, II e III tm as seguintes configuraes
Be
1,12
215
4
eletrnicas em suas camadas de valncia:
I. 3s2 3p3
Ca
1,97
141
20
II. 4s2 4p5
Se
1,40
225
34
III. 3s2
As
propriedades
1
e
2
podem
ser,
respectivamente:
Com base nessas afirmaes, indique a afirmao errada.
a) potencial de ionizao e eletropositividade.
a) O elemento I um no-metal.
b) raio atmico e potencial de ionizao.
b) O elemento II um halognio.
c) afinidade eletrnica e raio atmico.
c) O elemento III um metal alcalino-terroso.
d) eletronegatividade e potencial de ionizao.
d) Os elementos I e III pertencem ao terceiro perodo da tabela
e) eletronegatividade e eletropositividade.
peridica.
e)

Os trs elementos pertencem ao mesmo grupo da tabela peridica.

4. (FUVEST) Em seu livro de contos, O Sistema Peridico, o escritor italiano


Primo Levi descreve caractersticas de elementos qumicos e as relaciona
a fatos de sua vida. Dois trechos desse livro so destacados a seguir:
I. [Este metal] mole como a cera; reage com a gua onde flutua
(um metal que flutua!), danando freneticamente e produzindo
hidrognio.
II. [Este outro] um elemento singular: o nico capaz de ligar-se
a si mesmo em longas cadeias estveis, sem grande desperdcio
de energia, e, para a vida sobre a Terra (a nica que conhecemos
at o momento), so necessrias exatamente as longas cadeias.
Por isso,... o elemento-chave da substncia viva.
O metal e o elemento referidos nos trechos I e II so, respectivamente:
a) mercrio e oxignio.
b) cobre e carbono.
c) alumnio e silcio.
d) sdio e carbono.
e) potssio e oxignio.
5. (UEL-PR) Considere o texto a seguir.
Os tomos de cloro, bromo e iodo tm o mesmo nmero de ...X... na
camada de valncia e por isso possuem propriedades ...Y... Todavia no
apresentam mesma aparncia. temperatura ambiente e sob presso
de 1 atm, cloro um gs verde-amarelo, bromo um ...Z... vermelho
escuro e iodo, um slido violeta.
Completa-se corretamente o texto, substituindo-se X, Y e Z,
respectivamente, por:
a) prtons, diferentes e gs.
b) eltrons, diferentes e lquido.
c) eltrons, semelhantes e lquido.
d) prtons, semelhantes e gs.
e) eltrons, semelhantes e gs.

10. (MACK-SP) Dos tomos de 13A 27, INCORRETO afirmar que:


a) formam ctions trivalentes.
b) ao se ligarem a tomos do elemento Y da famlia dos calcognios,
foram o composto A 2Y3.
c) apresentam trs eltrons na camada de valncia.
d) apresentam trs nveis de energia.
e) pertencem famlia dos halognios.
11. (UFU-MG) As propriedades das substncias podem ser relacionadas com
o tipo de ligao que existe entre seus tomos. Por exemplo: compostos
que possuam ligaes inicas tm alto ponto de fuso (geralmente
acima de 350C); todos so slidos temperatura ambiente, grande
parte deles so solveis em gua e suas solues aquosas conduzem
corrente eltrica.
Assinale a alternativa em que aparecem SOMENTE substncias
predominantemente inicas.
a) Perclorato de sdio (NaC O4); etanoato de sdio (CH3CO2Na) e
cloreto de etila (CH3CH2C ).
b) Iodeto de sdio (NaI); hexano (CH3(CH2)4CH3) e carbonato de clcio
(CaCO3).
c) Fluoreto de sdio (NaF); hidrxido de potssio (KOH) e nitrato de
prata (CaCO3).
d) Acetileno (etino) (C2H2); cido perclrico (HC O4) e cloreto de sdio
(Nac ).
e) cido sulfrico (H2SO4); hidrxido de sdio (NaOH) e metanol
(CH3OH).
12. (PUCCamp-SP) Os tomos de certo elemento metlico possuem, cada
um, 3 prtons, 4 nutrons e 3 eltrons. A energia de ionizao desse
elemento est entre as mais baixas dos elementos da tabela peridica.
Ao interagir com halognio, esses tomos tm alterado o seu nmero
de:
a) prtons, transformando-se em ctions.
b) eltrons, transformando-se em nions.
c) nutrons, mantendo-se eletricamente neutros.

72

d)
e)

prtons, transformando-se em nions.


eltrons, transformando-se em ctions.

13. (UFRRJ) O elemento X, do terceiro perodo, que tem maior afinidade


eletrnica, combina-se com o elemento Z, do quarto perodo, que
possui menor energia de ionizao. A frmula do composto resultante
ser:
a)ZX
b) ZX2
c) Z3X
d) ZX3 e) Z2X3
14. (UERJ) Apesar da posio contrria de alguns ortodontistas, est sendo
lanada no mercado internacional a chupeta anticrie. Ela contm
flor, um j consagrado agente anticries, o xylitol, um acar que no
provoca crie e estimula a suco pelo beb.
Considerando que o flor utilizado para esse fim aparece na forma de
fluoreto de sdio, a ligao qumica existente entre o sdio e o flor
denominada:
a) inica
b) metlica
c) dipolo-dipolo
d) covalente apolar
15. (FATEC-SP) Slidos cristalinos com pontos de ebulio e fuso altos,
solveis em solventes polares e que conduzem corrente eltrica quando
fundidos ou em soluo, so exemplos de compostos formados por meio
de ligao:
a) covalente polar
b)covalente apolar.
c) covalente dativa.
d)inica.
e)metlica.
16. (ITA-SP) Uma determinada substncia apresenta as seguintes
propriedades fsico-qumicas:
I. O estado fsico mais estvel a 25C e 1 atm o slido.
II. No estado slido apresenta estrutura cristalina.
III. A condutividade eltrica praticamente nula no estado fsico mais
estvel a 25C e 1 atm.
IV. A condutividade eltrica alta no estado lquido.
A alternativa relativa substncia que apresenta todas as propriedades
acima a:
a) poliacetileno.
b)brometo de sdio.
c) iodo.
d)silcio.
e)grafita.
17. (PUC-SP) Em 1916, G. N. Lewis publicou o primeiro artigo propondo que
tomos podem se ligar compartilhando eltrons. Esse compartilhamento
de eltrons chamado, hoje, de ligao covalente. De modo geral,
podemos classificar as ligaes entre tomos em trs tipos genricos:
ligao inica, ligao metlica e ligao covalente.
Assinale a alternativa que apresenta substncias que contm apenas
ligaes covalentes.
a) H2O, C (diamante), Ag e LiH.
b) O2, NaC , NH3 e H2O
c) CO2, SO2, H2O e Na2O
d) C (diamante), C 2, NH3 e CO2
e) C (diamante), O2, Ag e KC .
18. (UECE) Considere as substncias com seus usos mdicos:
NOME
USO MDICO
Cloreto de amnio
diurtico
Sulfato de brio
facilita o exame de raio-X de rgos internos
Sulfato de magnsio
laxante
Nitrato de prata
anti-sptico
Brometo de sdio
sedativo
Podemos afirmar corretamente que:
a) no cloreto de amnio e no sulfato de magnsio s existem ligaes
covalentes.
b) o brometo de sdio um composto metlico.
c) no sulfato de brio existem quatro ligaes covalentes e uma
ligao inica.
d) no nitrato de prata s existem ligaes inicas.
19. (VUNESP) Duas substncias slidas, X e Y, apresentam as propriedades
listadas na tabela:
Propriedades

Substncia
X
Y
Solubilidade em H2O
solvel
insolvel
Solubilidade em CC 4
insolvel
solvel
Ponto de fuso (C)
880 114
Condutividade eltrica no estado slidono conduz
no conduz
Condutividade da soluo em solvente adequado
conduz
no conduz
Baseando-se nessas informaes, pode-se afirmar que:

a)
b)
c)
d)
e)

X
X
X
X
X

e
e

substncia molecular e Y substncia inica.


substncia inica e Y substncia molecular.
substncia metlica e Y substncia inica.
Y so substncias moleculares.
Y so substncias inicas.

20. (UFRG-RS) Associe as frmulas das substncias abaixo com as


geometrias moleculares correspondentes.
( ) SO3
1 linear
( ) CO2
2 angular
( ) H2S
3 piramidal
4 trigonal plana
A seqncia correta do preenchimento da coluna da esquerda, de cima
para baixo, :
a) 1 2 3
b) 3 1 2
c) 3 2 1
d) 4 2 1
e) 4 1 2
21. (UFSM-RS) Assinale a alternativa que apresenta APENAS molculas
contendo geometria piramidal.
a) BF3 SO3 CH4
b) SO3 PH3 CHC 3
c) NC 3 CF2C 2 BF3
d) POC 2 NH3 CH4
e) PH3 NC 3 PHC 2
22. (UFRGS-RS) As substncias SO2 e CO2 apresentam molculas que
possuem ligaes polarizadas. Sobre as molculas dessas substncias
correto afirmar que:
a) ambas so polares, pois apresentam ligaes polarizadas.
b) ambas so apolares, pois apresentam geometria linear.
c) apenas o CO2 apolar, pois apresenta geometria linear.
d) ambas so polares, pois apresentam geometria angular.
e) apenas o SO2 apolar, pois apresenta geometria linear.
23. (PUC-MG) Relacione a frmula, a forma geomtrica e a polaridade a
seguir, assinalando a opo CORRETA:
a) Frmula CO2; Forma Geomtrica linear; Polaridade polar.
b) Frmula CC 4; Forma Geomtrica tetradrica; Polaridade polar
c) Frmula NH3; Forma Geomtrica piramidal; Polaridade apolar
d) Frmula BeH2; Forma Geomtrica linear; Polaridade apolar
24. (UFRG-RS) O momento dipolar a medida quantitativa da polaridade de
uma ligao. Em molculas apolares, a resultante dos momentos
dipolares referentes a todas as ligaes apresenta valor igual a zero.
Entre as substncias covalentes a seguir
I. CH4
II. CS2
III. HBr
IV. N2
Quais as que apresentam a resultante do momento dipolar igual a zero?
a) Apenas I e II
b) Apenas II e III
c) Apenas I, II e III
d) Apenas I, II e IV
e) I, II, III e IV
25. (PUCCamp-SP) Considere o texto abaixo.
Nos icebergs, as molculas polares da gua associam-se por... (I)...; no
gelo seco, as molculas apolares do dixido de carbono unem-se por...
(II)... . Conseqentemente, a 1,0 atmosfera de presso, possvel
prever que a mudana de estado de agregao do gelo ocorra a uma
temperatura... (III)... do que a do gelo seco.
Para complet-lo corretamente, I, II e III devem ser substitudos,
respectivamente, por:
a) I foras de London; II pontes de hidrognio; III menor
b) I pontes de hidrognio; II foras de Van der Waals; III maior
c) I foras de Van der Waals; II pontes de hidrognio; III maior
d) I foras de Van der Waals; II foras de London; III menor
e) I pontes de hidrognio; II pontes de hidrognio; III - maior
26. (UFF-RJ) A capacidade que um tomo tem de atrair eltrons de outro
tomo, quando os dois formam uma ligao qumica, denominada
eletronegatividade. Essa uma das propriedades qumicas consideradas
no estudo da polaridade das ligaes.
Assinale a opo que apresenta, corretamente, os compostos H 2O, H2S e
H2Se em ordem crescente de polaridade.
a) H2Se < H2O < H2S
b) H2S < H2Se < H2O
c) H2S < H2O < H2Se

73

d)
e)

H2O < H2Se < H2S


H2Se < H2S < H2O

27. (VUNESP) O grfico a seguir foi construdo com dados dos hidretos dos
elementos do grupo 16. Com base neste grfico, so feitas as afirmaes
seguintes.

I.
II.
III.
Das
a)
b)
c)
d)
e)

Os pontos P, Q, R e S no grfico correspondem aos compostos


H2Te, H2S, H2Se e H2O, respectivamente.
Todos esses hidretos so gases a temperatura ambiente, exceto a
gua, que lquida.
Quando a gua ferve, as ligaes covalentes se rompem antes
das intermoleculares.

trs afirmaes apresentadas,


apenas I verdadeira.
apenas I e II so verdadeiras.
apenas II verdadeira.
apenas I e III so verdadeiras.
apenas III verdadeira.

28. (Med. Catanduva-SP) Aps a evaporao de toda a gua de 25 g de uma


soluo saturada (sem corpo de fundo) da substncia X, pesou-se o
resduo slido, obtendo-se 5 g. Se, na mesma temperatura do
experimento anterior, adicionarmos 80 g da substncia X em 300 g de
gua, teremos uma soluo:
a) insaturada.
b) saturada sem corpo fundo.
c) saturada com 5 g de corpo fundo.
d) saturada com 20 g de corpo fundo.
e) supersaturada.
29. (EEM-SP) Evapora-se completamente a gua de 40 g de soluo saturada
de nitrato de prata, sem corpo de fundo, e obtm-se 15 g de resduo
slido. O coeficiente de solubilidade do nitrato de prata para 100 g de
gua na temperatura da soluo inicial :
a) 25 g
b) 30 g
c) 60 g
d) 15 g
e) 45 g
30. (Fatec-SP) No rtulo de uma garrafa de gua mineral l-se, entre outras
informaes:
Contedo 1,5 litro; nitrato de sdio 6,0 ppm.
Considere que 1 ppm equivale a 1 mg de soluto por litro de soluo
aquosa. A massa de nitrato de sdio ingerida por uma pessoa que bebe
um copo de 300 mL dessa gua :
a) 0,003 g
b) 0,0018 g
c) 9,0 g
d) 6,0 mg
e) 1,2 mg
31. (Fatec-SP) Fenol, C6H5OH, conhecido como cido fnico, usado como
desinfetante e na manufatura de plsticos. Dissolvendo-se 0,752 g desse
composto em gua suficiente para 500 mL, obtm-se soluo cuja
concentrao expressa em mol/L :
a) 0,0008
b) 1,504
c) 0,016
d) 1,6
e) 3,2
32. (Fuvest-SP) A concentrao de ons fluoreto em uma gua de uso
domstico de 5,0 10-5 mol/L.
Se uma pessoa toma 3,0 L dessa gua por dia, ao fim de um dia, a
massa de fluoreto, em miligramas, que essa pessoa ingere igual a:
a) 0,9

b) 1,3
c) 2,8
d) 5,7
e) 15
33. (Fuvest-SP) Abaixo representada a concentrao (em MG/kg) de alguns
ons na gua do mar:
on
Mg2+
Na1+
C 1Concentrao
1.350 2.700 10.500 19.000
Dentre esses ons, os que esto em menor e maior concentrao em
quantidade de matria so respectivamente:
a) C 1- e Mg2+
b)
e Na1+
c) Mg2+ e Na1+
d) Mg2+ e C 1e)
e C 134. (FEI-SP) Vamos dar um pau na clera (campanha publicitria
anticlera). Isso possvel com o uso de uma soluo aquosa de
hipoclorito de sdio, NaC O, a uma concentrao mnima de 1,5 10 -5
mol/L.
Partindo-se de uma soluo 0,1 mol/L de NaC O e considerando o
volume de uma gora igual a 0,05mL, indique a alternativa que apresenta
o nmero de gotas desta soluo, por litro de gua, necessrio para
atingir-se aquela concentrao mnima.
a) 1
b) 2
c) 3
d) 4
e) 5
35. (UFOP) O bicarbonato de sdio freqentemente usado como anticido
estomacal.
Considerando que o suco gstrico contenha cerca de 250,0 mL de
soluo de HC 0,1 mol/L, conclui-se que a massa, em gramas, de
NaHCO3 necessria para neutralizar o cido clordrico existente no suco
gstrico :
a) 1,2
b) 1,4
c) 1,8
d) 2,1
e) 2,6
36. (UFRGS-RS) Relacione, quando possvel, a situao de equilbrio cob
presso de 1 atm (coluna esquerda) com sua temperatura (coluna da
direita).
Equilbrio
Temperatura da soluo
1. Gelo em gua
( ) t < 0 C
2. Gelo em soluo aquosa de NaC
( ) t > 100 C
3. gua em ebulio
( ) t = 0 C
4. Soluo aquosa de NaC diluda em
ebulio
A seqncia correta, de cima para baixo, na coluna da direita :
a) 1 3 2
b) 1 3 4
c) 2 4 1
d) 3 1 2
e) 4 2 3
37. (ITA-SP) Sobre a temperatura de ebulio de um lquido so feitas
afirmaes:
I. Aumenta com o aumento da fora da ligao qumica
intramolecular.
II. Aumenta com o aumento da fora da ligao qumica
intermolecular.
III. Aumenta com o aumento da presso externa sobre o lquido.
IV. Aumenta com o aumento da quantidade de slido dissolvido.
Esto corretas:
a) Apenas I e II.
b) Apenas I e IV.
c) Apenas III e IV.
d) Apenas II, III e IV.
e) Todas.
38. (UFMG) Num congelador, h cinco frmas que contm lquidos diferentes
para fazer gelo e picols de limo. Se as frmas forem colocadas, ao
mesmo tempo, no congelador e estiverem, inicialmente, com a mesma
temperatura, vai congelar-se primeiro a frma que contm 500 mL de:
a) gua pura.
b) soluo aquosa contendo 50 mL de suco de limo.
c) soluo aquosa contendo 100 mL de suco de limo.
d) soluo, em gua, contendo 50 mL de suco de limo e 50 g de
acar.

74

e)

soluo, em gua, contendo 100 mL de suco de limo e 50 g de


acar.

quente de bicarbonato de sdio. Neste processo, a prata em contato


com o Ag2S atua como ctodo e o alumnio, como nodo de uma pilha. A
semi-reao que ocorre no ctodo pode ser representada por:
2 ( )
a)
Ag2S(s)
+ ( )
2 Ag(s)
b)
Ag2S(s)
+2
+( )
2 Ag(s)
c)
Ag2S(s)
+ ( )
+2
2 Ag
d)
Ag2S(s)
+2
+S(s)
2 Ag(s)
e)
Ag2S(s)
+ S(s)

39. (Unitau-SP) Duas solues de cloreto de sdio, NaC , e glicose, C 6H12O6,


tm a mesma concentrao em quantidade de matria. A soluo que
congela em temperatura mais baixa :
a) de glicose.
b) de NaC .
c) ambas mesma temperatura.
d) no se congelam.
46. (ITA-SP) Considere a seguinte seqncia ordenada de pares de xidoe) s a glicose se congela.
reduo:
Zn(s)
2
+ ( )
40. (UFRGS-RS) Os pontos normais de ebulio da gua, do etanol e do ter
Fe(s)
2
+ ( )
etlico so, respectivamente, 100 C, 78 C e 34 C. Observe as curvas
H2(g)
2
+2 ( )
no grfico de variao de presso de vapor do lquido (P V) em funo da
temperatura (T).
Cu(s)
2
+ ( )
2 ( )
2
+2 ( )
NO2(g)
+H2O( )
1
+
+2 ( )
( )
Em relao a esta seqncia, so feitas as afirmaes seguintes,
supondo sempre reagentes no seu estado-padro:
I. O on ferroso oxidante frente ao zinco metlico, mas no o
frente ao cobre metlico.
II. Cobre metlico pode ser dissolvido por uma soluo de sal frrico.
III. Cobre metlico pode ser atacado por uma soluo de cido
ntrico.
IV. Zinco metlico menos nobre do que ferro metlico.
As curvas I, II e III correspondem, respectivamente, aos compostos:
V. Colocando ferro metlico, em excesso, dentro de uma soluo de
a) ter etlico, etanol e gua.
sal frrico, acabaremos tendo uma soluo de sal ferroso.
b) etanol, ter etlico e gua.
Em relao a essas afirmaes, podemos dizer que:
c) gua, etanol e ter etlico.
a) todas esto corretas.
d) ter etlico, gua e etanol.
b) todas esto erradas.
e) gua, ter etlico e etanol.
c) s as de nmero par so certas.
d) apenas IV errada.
41. (FEI-SP) Uma salada de alface foi temperada com soluo de vinagre e
e) apenas II e III so erradas.
sal. Aps certo tempo, as folhas de alface murcharam. Esse fenmeno
denominado:
47. (PUC-SP) Na clula eletroqumica A 0/A 3+//Fe2+/Fe0 podemos afirmar
a) disperso.
que:
b) tonometria.
a) o alumnio sofre reduo.
c) ebuliometria.
b) o ferro nodo.
d) crioscopia.
c) os eltrons fluem, pelo circuito externo, do alumnio para o ferro.
e) osmose.
d) h desgaste do eletrodo de ferro.

e) a soluo de A 3+ ir se diluindo.
42. (PUCCAMP-SP) Adicionando em gua pura clulas animais, como os
glbulos vermelhos, observa-se que elas incham at arrebentarem.
48. (FEI-SP) A corroso eletroqumica opera como uma pilha. Ocorre uma
Esse fenmeno explicado pela:
transferncia de eltrons quando dois metais de diferentes potenciais
a) migrao de ons de dentro da clula para a gua pura a fim de
so colocados em contato. Considere uma lata de ao revestida com
igualar as temperaturas de ebulio da soluo celular e da gua
estanho: se a camada de estanho for riscada ou perfurada, o ferro
pura.
funciona como nodo, e o estanho, como ctodo, o que acelera a
b) passagem de molculas da gua para dentro da clula a fim de
corroso. Isso acontece porque:
aumentar a presso da soluo celular.
a) o Fe tem maior capacidade de ganhar eltrons.
c) diminuio da temperatura de congelamento da gua pura devido
b) o Fe tem maior potencial de reduo que o Sn.
adio de um soluto voltil.
c) o Sn um agente redutor.
d) migrao de ons da soluo celular para a gua pura, pois a
d) o Fe tem menor potencial de reduo que o Sn.
temperatura de ebulio da soluo menor que a da gua pura.
e) o Sn tem maior capacidade de doar eltrons.
e) passagem de molculas de gua pura dentro da clula devido
diferena de presso osmtica no interior e no exterior da 49. (UFU-MG) Pequemos pedaos de Zn(s), Fe(s) e Cu(s) so mergulhados,
membrana celular.
separadamente, em soluo aquosa de HC 1,0 mol L -1. correto
afirmar que haver dissoluo do metal no cido para:
43. (Cesgranrio-RJ) A reao espontnea que ocorre numa clula
Dados: potenciais-padro de reduo, a 25 C.
eletroqumica, nas condies-padro :
Semi-reao
E0 (volt)
CuSO4(aq) + Fe(s) FeSO4(aq) + Cu(s)

Zn2+
+ 2
Zn(s)
-0,76
Esta reao indica que:

Fe2+
+ 2
Fe(s)
-0,44
1+
a) o eletrodo Fe(s)/ ( ) o ctodo da clula.

2H
+ 2
H2(g)
0,00
2+
b) o eletrodo ( ) /Cu(s) o nodo da clula.

Cu
+ 2
Cu(s)
+0,34
a) apenas o Cu(s).
c) o metal ferro oxidado.
b) Zn(s) e Fe(s).
d) o CuSO4 o agente redutor.
c) Zn(s) e Cu(s).
e) o metal cobre reduzido.
d) Fe(s) e Cu(s).
e) todos os metais.
44. (Fuvest-SP) Para recuperar prata de solues aquosas contendo ons
1+
Ag , costuma-se adicionar zinco metlico s solues, pois a
50. (UFRJ) Considere os metais com seus respectivos potenciais-padro de
transformao:
reduo:
2 Ag1+ + Zn0 2 Ag0 + Zn2+

Mg2+ + 2
Mg
E0 = -2,37 V
espontnea. Pode-se concluir ento que:

Zn2+
+ 2
Zn
E0 = -0,76 V
a) o potencial de reduo do Ag1+/Ag0 maior do que o do Zn2+/Zn0.

Pb2+
+ 2
Pb
E0 = -0,13 V
b) ocorre transferncia de eltrons do Ag1+ para o Zn0.

c) o Zn0 atua como oxidante e o Ag1+ como redutor.


Cu2+
+ 2
Cu
E0 = +0,34 V

d) Zn0 menos redutor que Ag0.


Ag1+ 2
Ag
E0 = +0,80 V
e) ocorre a eletrlise do Ag1+ e do Zn0.
A alternativa que expressa corretamente a transferncia de eltrons dos
metais acima :
45. (Fuvest-SP) Objetos de prata escurecidos (devido principalmente
a) O melhor agente redutor o Ag.
formao de Ag2S) podem ser limpos eletroquimicamente, sem perda de
b) O Pb cede eltrons mais facilmente que o Zn.
c) A reao Mg2+ + Zn Mg + Zn2+ no espontnea.
prata, mergulhando-os em um recipiente de alumnio contendo soluo

75

O ction magnsio, Mg2+, recebe eltrons mais facilmente do que o 57. (Fatec-SP) Dois eletrodos conectados a uma lmpada so imersos em
ction cobre II, Cu2+.
soluo de Ca(OH)2(aq) (gua de cal).
e) Pode-se estocar, por longo prazo, uma soluo de sulfato de cobre
A lmpada se acende com luz intensa. Com um canudo de plstico
II num recipiente base de zinco.
assopra-se o ar expirado nesta soluo. A luz da lmpada subitamente
torna-se bem fraca. A que se deve tal comportamento?
51. (USJT-SP) Certo tipo de pilha, usada em aparelhos de surdez, funciona
a) dissoluo do gs CO(g) do ar expirado.
em decorrncia da reao qumica:
b) evaporao dos ons
( ) e
( ) provenientes da gua.
Zn + 2 KOH + HgO K2ZnO2 + H2O + Hg
c) precipitao do slido CaCO3(ppt), que reduz a quantidade de ons
Conhecidos os seguintes potenciais de xido-reduo:
na soluo.

Zn
Zn2+ + 2
E0 = +0,76 V
d) dissoluo do gs O2(g) do ar expirado.
1+
0

K
K
+ 1
E = +2,92 V
e) ao aumento da concentrao de ctions
( ) e nions
( ) da

4 OH1O2
+ 2 H2O + 4
E0 = -0,41 V
gua.

Hg
Hg2+ + 2
E0 = -0,80 V
podemos concluir que a diferena de potencial fornecida por essa pilha 58. (PUCCAMP-SP) O cobre com elevado grau de pureza obtido pelo
de:
mtodo eletroltico que consiste na eletrlise de soluo de sulfato
a) 0,39 volt
b)1,17 volt
cprico e cido sulfrico. Utiliza-se cobre impuro como nodo e cobre
c) 1,56 volt
d)2,16 volt e)3,33 volt
puro como ctodo e regula-se convenientemente a voltagem de forma
d)

52. (Vunesp-SP) O funcionamento de uma pilha de combustvel baseada


nas semi-reaes abaixo, cada uma delas
representada com o
respectivo potencial-padro de reduo, E0:
0
2 H2O( ) + 2
1 H2(g) + 2
( ) E = -0,828 V
0
O2(g) + 1 H2O( ) + 2
2
( ) E = +0,401 V
Levando-se em conta essas informaes, afirma-se:
I. A reao global da pilha de combustvel :
1 H2(g) + O2(g) 1 H2O( )
II. O hidrognio sofre oxidao no processo.
III. A diferena de potencial desta pilha de combustvel, em condiopadro, igual a 1,229 V.
Esto corretas as afirmaes:
a) I, apenas.
b) II, apenas.
c) I e II apenas.
d) II e III apenas.
e) I, II e III.
53. (Fuvest-SP) A eletrlise de cloreto de sdio fundido produz sdio
metlico e gs cloro.
Nesse processo, cada on:
a) sdio recebe dois eltrons.
b) cloreto recebe um eltron.
c) sdio recebe um eltron.
d) cloreto perde dois eltrons.
e) sdio perde um eltron.
54. (Mack-SP) Considere as tabelas em ordem decrescente de prioridade de
descarga, respectivamente do nodo e do ctodo.
nions no oxigenados > OH1- > nions oxigenados e F1-

que, no ctodo, ocorra apenas reduo:


(

60. (Cesgranrio-RJ) Um sal de um metal de peso atmico 196,99 foi


eletrolisado durante 5 minutos por uma corrente de 3,86 A, fornecendo
um depsito de 0,788 g do metal no ctodo. Pelos dados, podemos
afirmar que o nmero de oxidao do metal do sal :
a) 1
b) 2
c) 3
d) 4
e) 6
61. (FAAP-SP) Sabe-se que numa pilha eletroltica, a reao global :
( )

56. (Odonto-Diamantina-MG) Com relao eletrlise do KC , em soluo


aquosa, quantas das seguintes afirmaes esto corretas?
I. Durante a eletrlise, o pH vai aumentando.
II. No ctodo (plo negativo), h formao de K(s).
III. No nodo (plo positivo), h desprendimento de cloro gasoso,
C 2(g).
IV. Se for adicionada fenolftalena na soluo inicial, haver
mudana, durante a eletrlise, de incolor para vermelho.
a) 4
b) 3
c) 2
d) 1
e) 0

Portanto, o nmero de coulomb movimentado, por mol de ferro oxidado


:
Dado: 1 faraday = 96.500 coulombs.
a) 9,65 104
b) 19,30 104
c) 12,60 105
d) 11,60 103
e) 12,60 10-5

Ag1+ > Cu2+ > Fe2+ > H1+ > Mg2+ > Na1+ > Ca2+ > K1+
Por eletrlise da gua, feita com eletrodos inertes adequados, obtm-se
os gases hidrognio e oxignio, se for adicionado um eletrlito que pode
ser:
a) KBr
b) NaC
c) Fe(NO3)2
d) MgSO4
e) AgF
55. (Fatec-SP) Cloro gasoso pode ser obtido industrialmente a partir da
eletrlise de uma soluo aquosa de:
a) cido perclrico.
b) cloreto de sdio.
c) hexaclorobenzeno.
d) percloroetileno.
e) tetracloreto de carbono.

( )

A quantidade de eltrons, em mol, necessria para a obteno de 254 g


de cobre puro :
a) 8,5
b) 8,0
c) 5,5
d) 4,0
e) 2,0
59. (UFAL) A carga eltrica necessria para que, na eletrlise de uma soluo
de sulfato de cobre (II), se depositem 2 mol de tomos de cobre :
Dado: 1 faraday corresponde quantidade de carga eltrica de 1 mol de
eltrons.
a) 5 faradays
b) 4 faradays
c) 3 faradays
d) 2 faradays
e) 1 faradays

1. E

2. A

3. E

4. D

5. C

6. D

7. B

8. C

9. B

10. E

11. C

12. E

13. A

14. A

15. D

16. B

17. D

18. C

19. B

20. E

21. C

22. C

23. D

24. D

25. B

26. E

27. C

28. C

29. C

30. B

31. C

32. C

33. E

34. C

35. D

36. C

37. D

38. A

39. B

40. A

41. E

42. E

43. C

44. A

45. B

46. A

47. C

48. D

49. B

50. C

51. C

52. E

53. C

54. D

55. B

76

56. B

57. C

58. B

59. B

60. C

61. B

62. (Odonto. Diamantina-MG) Conte o nmero de carbonos tetradricos,


trigonais e digonais, presentes na frmula:

e assinale a alternativa correspondente.


Teradricos
Trigonais
a)
2
1
b)
5
1
c)
1
6
d)
2
6
e)
0
8

Digonais
5
2
1
0
0

a) metil, isopropil e etil.


b) metil, etil e metil.
c) propil, etil e metil.
d) butil, etil e propil.
e) isopropil, etil e metil.
68. (Unisinos-RS) Para o composto orgnico a seguir formulado, aplicando a
nomenclatura IUPAC, o seu nome correto :

63. A estrutura a seguir de um composto saturado derivado do colesterol.


O nmero de carbonos secundrios presentes nessa estrutura :

a)
b)
c)
d)
e)

5-etil-3,3,4-trimetil-5-hepteno.
3,5-dietil-4,5-dimetil-2-hexeno.
2,4-dietil-2,3-dimetil-4-hexeno.
3-etil-4,5,5-propil-2-hepteno.
3-etil-4,5,5-trimetil-2-hepteno.

69. (Unisinos-RS) Considere as seguintes afirmativas:


I. O dimetilpropano apresenta cinco carbonos em sua estrutura
molecular.
II. O propino possui trs tomos de carbono e quatro tomos de
64. (UECE) Das afirmaes:
hidrognio em sua molcula.
I. O carbono apresenta quatro valncias, ou seja, tetravalente.
III. O eteno o alqueno mais simples.
II. A molcula do tetraclorometano tem geometria tetradrica.
Dessas afirmativas:
III. A molcula do metano tem geometria plana.
a) somente I est correta.
IV. Em condies ambientais, os compostos CH4, C2H6, C3H8 e C4H10
b) I, II e III esto corretas.
so gasosos.
c) somente I e III so corretas.
V. Uma cadeia de composto orgnico classificada como aromtica
d) somente I e II so corretas.
quando as ligaes aparecem em nmero par.
e) somente II e III so corretas.
Esto corretas:
70. (Mack-SP) Antigamente, nas minas, era costumeiro o uso de lamparinas
a) I, II e III
que utilizavam como combustvel um hidrocarboneto produzido pela
b) II, III e IV
reao do carbureto, CaC2(s), com a gua. O hidrocarboneto produzido
c) I, IV e V
por esta reao o:
d) I, II e IV
a) butano.
b)propano.
e) nenhuma das anteriores.
c) eteno.
d)propeno.
e) acetileno.
65. (UFRGS-RS) O citral, composto de frmula:
a) 8
d) 14 e)16

b)10

c)12

71. (UECE) Das afirmaes:


I. O carbono apresenta quatro valncias.
II. A molcula do tetraclorometano tem geometria tetradrica.
III. A molcula do metano tem geometria plana.
IV. Em condies ambientais, os alcanos do CH4, ao C4H10 so
gasosos.
tem forte sabor de limo e empregado em alimentos para dar sabor e
V. Uma cadeia de compostos orgnicos classificada como
aroma ctricos. Sua cadeia carbnica classificada como:
aromtica quando as ligaes pi aparecem em nmero par.
a) homognea, insaturada e ramificada.
Esto corretas:
b) homognea, saturada e normal.
a) I, II e III.
c) homognea, insaturada e aromtica.
b) II, III e IV.
d) heterognea, insaturada e ramificada.
c) I, IV e V.
e) heterognea, saturada e aromtica.
d) I, II e IV.
66. (Mack SP) A cadeia carbnica a seguir classifica-se como:

a)
b)
c)
d)
e)

cclica, saturada, heterognea, ramificada.


aberta, saturada, heterognea, normal.
aberta, saturada, heterognea, ramificada.
acclica, insaturada, homognea, ramificada.
aberta, insaturada, homognea, normal.

67. (FUPE) Ao analisar o composto abaixo, verificamos que os radicais


ligados aos carbonos 3, 4 e 5 da cadeia principal so respectivamente:

72. (UFRGS-RS) O biogs, que uma mistura gasosa formada pela atividade
de bactrias sobre matria orgnica em decomposio, tem como
principal componente o:
a) acetileno.
b)eteno.
c) metano.
d)naftaleno.
e) oxignio.
73. (PUCCAMP-SP) A gasolina uma mistura de vrios hidrocarbonetos: o
lcool etlico absoluto uma substncia pura. Quais afirmaes acerca
dos dois lquidos so corretas?
a) Gasolina: tem ponto de ebulio caracterstico. lcool etlico
absoluto: tem ponto de ebulio caracterstico.
b) Gasolina: h uma relao constante entre as massas dos seus
componentes. lcool etlico absoluto: a relao entre as massas dos
seus elementos qumicos varivel.
c) Gasolina: apresenta densidade caracterstica. lcool etlico absoluto:
apresenta densidade caracterstica.
d) Gasolina: no pode ser representada por frmula molar. lcool
etlico absoluto: pode ser representada por frmula molecular.

77

e)

Gasolina: a massa molar maior do que 18. lcool etlico absoluto: 79. (Fuvest-SP) Na vitamina K3 (frmula abaixo), reconhece-se o grupo
a massa molar menor que 18.

funcional:

74. (PUCCAMP-SP) Considere que na coleta seletiva de lixo so separados


objetos e resduos de:
I. papis;
II. plsticos;
III. madeiras;
IV. metais;
V. vidros.
Qual desses materiais, pode ser de difceis reciclagem e decomposio, causa
maior prejuzo ambiental?
a) I
a) cido carboxlico.
b) II
b) aldedo.
c) III
c) ter.
d) IV
d) fenol.
e) V
e) cetona.
75. (UFF-RJ) Umectantes so aditivos que entram na fabricao de bolos,
panetones, rocamboles etc.
80. (Med. Pouso Alegre-MG) Cetonas macrocclicas so usadas em perfumes
A finalidade evitar que a massa resseque. Para tanto, um umectante
porque possuem intenso cheiro de almscar e retardam a evaporao de
precisa possuir alta afinidade pela gua.
constituintes mais volteis:
Indique qual das substncias abaixo usada como umectante.
a) ter etlico
I.
II.
b) benzeno
c) clorofrmio
d) gliceral
e) ciclo-hexano
76. (UFRJ) A vanilina (essncia de baunilha) possui a frmula estrutural
plana abaixo. Com relao a essa molcula, podemos afirmar que os
grupos funcionais ligados ao ncleo aromtico correspondem s funes:

a)
b)
c)
d)
e)

fenol, ter, aldedo.


fenol, ster, cetona.
lcool, ter, cetona.
lcool, ter, aldedo.
fenol, ster, aldedo.

III.

IV.

V.

77. (Fuvest-SP) O bactericida Fomecin A, cuja frmula estrutural :

apresenta as funes:
a) cido carboxlico e fenol.
b) lcool, fenol e ter.
c) ter, fenol e aldedo.
d) ter, lcool e aldedo.
e) cetona, fenol e aldedo.
78. (PUC-SP) A exposio excessiva ao sol pode trazer srios danos pele
humana. Para atenuar efeitos nocivos, costuma-se utilizar agentes
protetores solares como o 2-hidrxi-4-metoxibenzofenona, cuja frmula
est representada a seguir:

A identificao correta da(s) estrutura(s) acima que possui(em) cheiro de


almscar :
a) IV
b) V
c) IV e V
d) I, IV e V
e) I, II, III, IV e V
81. (USJT-SP) Um xampu, de um modo geral, contm, no mnimo, as
seguintes substncias: detergente, espessante, essncia, corante,
bactericida, fungicida, gua desmineralizada, sal de cozinha e cido
ctrico para abaixar o seu pH. A frmula estrutural abaixo corresponde
ao cido mencionado:

Quais as funes orgnicas presentes no composto acima?


Sobre essa substncia correto afirmar que:
a) lcool e cido carboxlico.
a) apresenta frmula molecular C10H4O3 e um hidrocarboneto
b) lcool e aldedo.
aromtico.
c) Enol e aldedo.
b) apresenta frmula molecular C10H4O5 e funo mista: lcool, ter e
d) cido carboxlico e fenol.
cetona.
e) cido carboxlico.
c) apresenta frmula molecular C14H12O5 e carter bsico pronunciado
pela presena do grupo OH.
82. (USJT-SP) Algumas substncias so adicionadas a certos alimentos que
d) apresenta frmula molecular C14H12O3 e um composto aromtico
contm leos e gorduras (como batatas fritas, toucinhos, margarinas,
de funo mista: cetona, fenol e ter.
manteigas etc.) com a finalidade de impedirem as suas oxidaes no ar,
e) apresenta frmula molecular C14H16O3, totalmente apolar e
produzindo compostos com sabores ranosos. Essas substncias so
insolvel em gua.
denominadas antioxidantes.

78

Por exemplo:

nasal base da substncia efedrina (considerada como doping), cuja


frmula estrutural representada por:

Quais as funes presentes no composto acima?


a) Enol e lcool.
b) Enol e ter.
c) Fenol e ster.
d) lcool e ster.
e) ter e lcool.

Com relao a essa molcula, podemos afirmar respectivamente que sua


frmula molecular e os grupos funcionais presentes correspondem s
funes orgnicas:
a) C10H21NO, fenol e amina.
b) C10H20NO, fenol e amida.
c) C10H15NO, lcool e amina.
d) C10H10NO, lcool e amida.
e) C9H10NO, lcool e nitrila.

83. (Odonto. Diamantina-MG) Em que alternativa os nomes das funes


esto corretos em relao s frmulas dadas em seqncia?

1.

a)
b)
c)
d)
e)

2.

3.

a)
b)
c)
d)
e)

1.
1.
1.
1.
1.

ter
ster
ster
ter
ster

87. (Cesesp-PE) Considerando-se as seguintes aminas:


I. metilamina
II. dimetilamina
III. fenilamina
escolha a alternativa que indica a ordem decrescente de basicidade:

2. amina terciria 3. lcool tercirio


2. amina terciria 3. lcool tercirio
2. amida
3. fenol
2. amina terciria 3. fenol
2. amida
3. lcool tercirio

II
III
I
III
II

>
>
>
>
>

I
II
II
I
II

>
>
>
>
>

III
I
III
II
I

88. (UFMG) A alternativa que apresenta uma substncia orgnica de


comportamento bsico em gua :
a) HCOOH
b) CH4
c) CH3OH
d) CH3NH2

e)
89. (Mack-SP) O nome da molcula esquematizada abaixo :

84. (UEMA) O nome do composto abaixo, que pode ser usado para dar sabor
de morango a balas e refrescos, :

a)
b)
c)
d)
e)

butanoato de metila.
cido etil-n-propilmetanico.
propanoato de metila.
butanoato de etila.
etanoato de butila.

85. (USJT-SP) Alguns compostos so muito utilizados para intensificar o


sabor de carnes enlatadas, frangos, carnes congeladas e alimentos ricos
em protenas.
Por exemplo:

Esse composto no contribui, por si s, para o sabor. Sua funo


explicada por duas teorias:

estimula a atividade das papilas do gosto;

aumenta a secreo celular.


Quais as funes orgnicas existentes no composto acima?
a) Amida, amina e cido.
b) Anidrido de cido e sal orgnico.
c) Amina, cido carboxlico e sal orgnico.
d) Amida, cido carboxlico e sal orgnico.
e) Amido, cido orgnico e ster de cido.
86. (UERJ) Durante a Copa do Mundo de 1994, o craque argentino Armando
Diego Maradona foi punido pela FIFA por utilizar um descongestionante

a)
b)
c)
d)
e)

metil-3-aminopropano
metil-n-propilamina
isobutilamina
t-butilamina
s-butilamina

90. (Vunesp-SP) A asparagina, cuja frmula estrutural encontra-se


esquematizada a seguir, apresenta o(s) grupo(s) funcional(is):

a)
b)
c)
d)
e)

lcool.
ster.
ter e ster.
amida, amina e cido carboxlico.
ter, amida e cido carboxlico.

91. (Med. Catanduva-SP) Dentre os pares de compostos orgnicos


relacionados abaixo, assinale o que exemplifica um caso de isomeria
plana de cadeia:
a) propanal e propanona.
b) n-propilamina e metil-etilamina.
c) 1-butanol e 2-butanol.
d) metoxipropano e etoxietano.
e) propanico e metanoato de etila.
92. (UFRGS-RS) A creolina, usada como desinfetante, contm cresis, que
so hidroximetilbenzenos de frmula molecular C 7H8O. Esse cresis
podem apresentar isomeria:
a) de funo e cadeia.
b) de cadeia e tautomeria.
c) de cadeia e posio.

79

d)
e)

de posio e funo.
de funo e metameria.

93. (UFMG) O cido butrico, cido butanico, um cido carboxlico


encontrado na manteiga ranosa, em queijo velho e na transpirao
humana. Um dos seus ismeros funcionais utilizado na indstria de
alimentos, em mistura ou sozinho, para dar sabor menta ou ma. Qual
dos compostos a seguir, quando colocado na gelatina, deve ter sabor de
ma?
a) CH3(CH2)2COOH
b) CH3COCOCH3
c) CH3COOC2H5
d) CH3CH2CH2COOC2H5
e) (ciclo)C4H7 COOH
94. (Mack-SP) Em relao ao composto a seguir, fazem-se as seguintes
afirmaes:

I. um composto que apresenta isomeria cis-trans.


II. No apresenta isomeria de funo.
III. Seu nome 3,5-dimetil-3-n-propil-5-hepteno.
IV. Apresenta somente ligaes ss-sp3.
So corretas as afirmaes:
a) somente IV.
b) somente I e II.
c) somente III.
d) somente II e III.
e) somente I e III.
95. (UCSal-BA) Um dos componentes da cera de abelha a substncia
abaixo que pode ser obtida a partir de:

a)
b)
c)
d)
e)

dois cidos carboxlicos.


um aldedo e um lcool.
uma cetona e um lcool.
um aldedo e um cido carboxlico.
um lcool e um cido carboxlico.

96. (UFRGS-RS) Um composto alimentar rico em triglicerdeos saturados :


a) a gordura animal.
b) o leo vegetal.
c) a gelatina.
d) a protena da soja.
e) o doce de abbora em calda.
97. (ITA-SP) As gorduras e os leos de origem animal e vegetal de uso mais
comum (banha, sebo, leo de caroo de algodo, leo de amendoim
etc.) so constitudos essencialmente de:
a) cidos carboxlicos alifticos.
b) hidrocarbonetos no-saturados.
c) misturas de parafinas e glicerinas.
d) steres de cidos carboxlicos de nmero de carbonos varivel e
glicerina.
e) teres derivados de alcois com um nmero de carbonos varivel.

1) D

2) D

3) D

4) A

5) B

6) A

7) E

8) B

9) E

10) D

11) C

12) D

13) B

14) D

15) A

16) C

17) D

18) E

19) B

20) A

21) C

22) E

23) D

24) C

25) C

26) A

27) D

28) C

29) D

30) B

31) D

32) C

33) B

34) E

35) A

36) D

37) C

38) D

39) E

80

1) (PUC

SP) Considere a seguinte situao: um nibus movendo-se em


uma estrada e duas pessoas: uma, A, sentada no nibus e outra, B, parada
na estrada, ambas observando uma lmpada fixa no teto do nibus. A diz: a
lmpada no se move em relao a mim, uma vez que a distncia que nos
separa permanece constante. B diz: A lmpada est se movimentando, uma
vez que est se afastando de mim.
a) A est errada e B est certa.
b) A est certa e B est errada.
c) Ambas esto erradas.
d) Cada uma, dentro do seu ponto de vista, est certa.
e) n.d.a.

9) (Vunesp

adapt.) Um rato, em sua ronda procura de alimento, est


parado em um ponto P, quando v uma coruja espreitando-o.
Instintivamente, ele corre em direo sua toca T, localizada a 42 m dali, em
movimento retilneo uniforme e com velocidade v = 7 m/s. Ao ver o rato, a
coruja d incio sua caada, em um mergulho tpico, como mostrado na
figura.

2) (Fatec

SP) Um carro faz uma viagem de So Paulo ao Rio. Os primeiros


250 km so percorridos com uma velocidade mdia de 100 km/h. Aps uma
parada de 30 minutos para um lanche, a viagem retomada, e os 150 km
restantes so percorridos com velocidade mdia de 75 km/h.
A velocidade mdia da viagem completa foi, em km/h:
a) 60
b) 70
c) 80
d) 90
e) 100

3) (UEL

PR) Um carro percorreu a metade de uma estrada viajando a 30


km/h e, a outra metade da estrada, a 60 km/h. Sua velocidade mdia no
percurso total foi de:
a) 60 km/h b) 54 km/h c) 48 km/h
d) 40 km/h e) 30 km/h

4) (Fuvest

SP) Dirigindo-se a uma cidade prxima, por uma autoestrada


plana, um motorista estima seu tempo de viagem, considerando que consiga
manter uma velocidade mdia de 90 km/h. Ao ser surpreendido pela chuva,
decide reduzir sua velocidade mdia para 60 km/h, permanecendo assim at
a chuva parar, quinze minutos mais tarde, quando retoma sua velocidade
mdia inicial.
Essa reduo temporria aumenta seu tempo de viagem, com relao
estimativa inicial, em
a) 5 minutos.
b) 7,5 minutos.
c) 10 minutos.
d) 15 minutos.
e) 30 minutos.

5) (PUC PR) Dois veculos A e B percorrem a mesma pista reta a velocidades

constantes de VA = 35 m/s e VB = 25 m/s, respectivamente. No momento em


que o veculo A est a 30 m na frente do veculo B, o veculo B acelera com
uma taxa constante de 5 m/s2.
Qual o intervalo de tempo para que o veculo B alcance o veculo A?
a) 0,3 minutos
b) 30 segundos
c) 60 segundos
d) 0,1 minutos
e) 1,2 minutos

6) (Mackenzie

SP) Da carroceria de um caminho carregado com areia,


pinga gua razo constante de 90 gotas por minuto. Observando que a
distncia entre as marcas dessas gotas na superfcie plana da rua constante
e igual a 10 m, podemos afirmar que a velocidade escalar do caminho de:
a) 5 m/s
b) 10 m/s c) 15 m/s
d) 20 m/s e) 25 m/s

7) (UFSCar

SP) Um trem carregado de combustvel, de 120 m de


comprimento, faz o percurso de Campinas at Marlia, com velocidade
constante de 50 km/h. Este trem gasta 15 s para atravessar completamente
a ponte sobre o rio Tiet. O comprimento da ponte :
a) 100,0 m.
b) 88,5 m. c) 80,0 m.
d) 75,5 m. e) 70,0 m.

8) (Mackenzie SP) A figura mostra, em determinado instante, dois carros A e

B em movimento retilneo uniforme. O carro A, com velocidade escalar 20


m/s, colide com o B no cruzamento C. Desprezando as dimenses dos
automveis, a velocidade escalar de B :

a) 12 m/s
b) 10 m/s
c) 8 m/s
d) 6 m/s
e) 4 m/s

Ela passa pelo ponto P, 4 s aps a partida do rato e a uma velocidade de 20


m/s.
Qual deve ser a acelerao mdia da coruja, a partir do ponto P, para que ela
consiga capturar o rato no momento em que ele atinge a entrada de sua
toca?
a) 1 m/s2 b) 2 m/s2 c) 3 m/s2
d) 4 m/s2 e) 5 m/s2

10) (UFLA

MG) A figura abaixo apresenta o percurso que um motorista


deve fazer, saindo de um local A para chegar em C, passando por B. O local
A dista 45 km de B, e de C, 120 km. O motorista deve deixar A s 7 horas e
chegar em C obrigatoriamente s 9h30. O motorista, ao deixar A, s 7 horas,
encontra muita neblina no trecho entre A e B, e por segurana, percorre o
trecho com velocidade mdia de 30 km/h. A partir de B, sem os problemas
climticos, e para chegar em C no horrio previsto, 9h30, deve desenvolver
uma velocidade mdia de

a) 120 km/h
b) 90 km/h c) 60 km/h
d) 75 km/h e) 80 km/h

11) (Unifesp SP) A funo da velocidade em relao ao tempo de um ponto

material em trajetria retilnea, no SI, v = 5,0 2,0t. Por meio dela podese afirmar que, no instante t = 4,0 s, a velocidade desse ponto material tem
mdulo
a) 13 m/s e o mesmo sentido da velocidade inicial.
b) 3,0 m/s e o mesmo sentido da velocidade inicial.
c) zero, pois o ponto material j parou e no se movimenta mais.
d) 3,0 m/s e sentido oposto ao da velocidade inicial.
e) 13 m/s e sentido oposto ao da velocidade inicial.

12) (Cefet

MA) Dois mveis, A e B, realizam movimentos que so

descritos

pelas

funes

SB t 10 t 20

horrias

S A 2t 2 4t 10

, com unidades do Sistema Internacional. Esses dois

mveis tm a mesma velocidade no instante:


a) 4,5 segundos
b) 3,5 segundos
c) 3,8 segundos
d) 4,0 segundos
e) 3,0 segundos

13) (UEPB PB) Um automvel move-se com velocidade constante de 20 m/s


por uma avenida e aproxima-se de um semforo com fiscalizao eletrnica,
situado em frente a uma escola. Quando o automvel se encontra a 60
metros do semforo, o sinal muda de verde para amarelo, permanecendo
amarelo por um tempo de 2,0 segundos. Portanto, a menor acelerao
constante que o carro deve ter para passar pelo semforo e no ser multado
em m/s2, vale:
a) 10,0
b) 6,0
c) 8,0
d) 7,0
e) 12,0

14) (Fatec

SP) Um objeto se desloca em uma trajetria retilnea. O grfico


abaixo descreve as posies do objeto em funo do tempo.

81

Analise as seguintes afirmaes a respeito desse movimento:


I. Entre t = 0 e t = 4s o objeto executou um movimento retilneo
uniformemente acelerado.
II. Entre t = 4s e t = 6s o objeto se deslocou 50m.
III. Entre t = 4s e t = 9s o objeto se deslocou com uma velocidade mdia de
2m/s.

a)

S 6 2,5.t

c)

S 6 2,5.t 2.t 2

e)

Deve-se afirmar que apenas


a) I correta.
b) II correta.
c) III correta.
d) I e II so corretas.
e) II e III so corretas.

S 6 7,5.t 2.t

b)

S 6 t2

d)

S 6 5.t t 2

18) (UFES)

15) (Unifor

CE) Entre duas estaes, um trem metropolitano realiza o


movimento cujo grfico da velocidade em funo do tempo representado
abaixo.

Um projtil disparado do solo, verticalmente para cima, com


velocidade inicial de 200 m/s. Desprezando-se a resistncia do ar, a altura
mxima alcanada pelo projtil e o tempo necessrio para alcan-la so,
respectivamente:
a) 4000 m; 40 s
b) 4000 m; 20 s
c) 2000 m; 40 s
d) 2000 m; 20 s
e) 2000 m; 10 s

19) (UFPI)

Uma pequena bola lanada da janela de um edifcio com


velocidade de 10m/s, dirigida para cima. Simultaneamente, uma outra bola
abandonada, em repouso, da mesma posio. Suponha que ambas as bolas
esto sujeitas somente ao da gravidade. Um segundo tempo, aps o
lanamento, a separao entre as bolas de, aproximadamente: (Dados: g =
10 ms2).
a) 5 m
b) 10 m
c) 15 m
d) 20 m
e) 25 m

20) (UFAM) O diagrama abaixo representa uma seqncia de fotografias,


A acelerao do trem no primeiro trecho do movimento, em m/s 2, e a com intervalo de 1s, de uma bola lanada verticalmente para cima num local
onde a acelerao da gravidade tem valor g1. Sabe-se que a bola lanada
distncia entre as estaes, em m, valem, respectivamente,
do ponto A, com velocidade inicial vA, e atinge sua altura mxima no ponto B.
a) 0,50 e 1,2 . 103
b) 0,50 e 9,0 . 102
Com base neste diagrama, podemos afirmar que vA e g1 valem,
c) 1,0 e 6,0 . 102
d) 2,0 e 9,0 . 102
respectivamente:
e) 2,0 e 6,0 . 102
16) (Cefet PR) O

grfico a seguir mostra como varia a velocidade de um


mvel em funo do tempo. Sabe-se que, no instante t0 = 0, o mvel se
encontrava 5 m direita da origem das posies.

A respeito dessa situao, so feitas as afirmativas a seguir:


I A funo horria x = f(t) desse movimento

dada

por:

x 5 20 t 2t .
II O mvel sofre inverso no sentido do movimento, no instante t = 5s.
III A velocidade do mvel no instante t = 8s igual a 12 m/s.
Assinale a alternativa correta.
a) apenas a I correta.
b) apenas a I e a II so corretas.
c) apenas a I e a III so corretas.
d) apenas a II e a III so corretas.
e) I, II e III so corretas.

17) (UFES)

Uma partcula que realiza movimento retilneo uniformemente


variado tem seu grfico S x t representado abaixo. A equao horria que
descreve o movimento dessa partcula dado por:

a) 20 m/s e 7 m/s2
c) 20 m/s e 8 m/s2
e) 40 m/s e 7 m/s2

b) 40 m/s e 10 m/s2
d) 40 m/s e 8 m/s2

21) (UFPE

PE) Um automvel se movimentando numa estrada retilnea


percorre trs trechos consecutivamente. O primeiro trecho percorrido para
a direita em duas horas, com velocidade mdia de mdulo 30 km/h. O
segundo trecho percorrido tambm para a direita em duas horas, com
velocidade mdia de mdulo 90 km/h.
O terceiro trecho percorrido para a esquerda em uma hora, com velocidade
mdia de mdulo 60 km/h. O mdulo da velocidade mdia do automvel no
percurso total vale, em km/h,
a) 30
b) 36
c) 60
d) 72
e) 81

22) (UESPI)

A figura abaixo mostra trs vetores,


X, Y e Z .

Para essa

situao, assinale a alternativa que mostra a relao vetorial correta.

82

26) (UFMG MG) Um pequeno bote, que navegava a uma velocidade de 2,0

m/s em relao margem de um rio, alcanado por um navio, de 50,0m de


comprimento, que se move paralelamente a ele, no mesmo sentido, como
mostrado nesta figura:

a)
b)
c)
d)
e)


YZ X

XYZ

YZX 0

Y Z X

XZ Y

23) (CESJF MG) Sendo o vetor x perpendicular ao vetor y




vetorial ( x + y ) e a diferena ( x - y ) sero sempre vetores:

, a soma

a) iguais
b) de mdulos iguais
c) de sentidos opostos
d) perpendiculares
e) de direes iguais

Esse navio demora 20 segundos para ultrapassar o bote. Ambos movem-se


com velocidades constantes.
Nessas condies, a velocidade do navio em relao margem do rio de,
aproximadamente:
a) 0,50 m/s
b) 2,0 m/s c) 2,5 m/s
d) 4,5 m/s

27) (UFPE) Um submarino em combate lana um torpedo na direo de um

navio ancorado. No instante do lanamento o submarino se movia com


velocidade v = 14 m/s. O torpedo lanado com velocidade v ts, em relao
ao submarino. O intervalo de tempo do lanamento at a coliso do torpedo
com o navio foi de 2,0 min. Supondo que o torpedo se moveu com
velocidade constante, ao calcularmos vts em m/s, encontramos:

24) (Mackenzie

SP) Para se dirigir do ponto A ao ponto B da estrada


abaixo, o veculo teve que passar pelo ponto C e gastou 15,0 minutos. Com
relao ap plano da estrada, o mdulo do vetor velocidade mdia entre A e B
foi:

a) 10 m/s b) 15 m/s c) 17 m/s


d) 21 m/s e) 25 m/s

28) (UCDB

MS) O movimento de um corpo pode ser o resultado da


composio de vrios movimentos realizados simultaneamente. O movimento
de um barco em um rio uma composio de movimentos. Se o barco sobre
o rio com velocidade constante de 10 m/s, em relao s margens, e desce
com velocidade constante de 30 m/s, pode-se concluir que a velocidade da
correnteza, em m/s, em m/s, igual a:
a) 8
b) 10
c) 12
d) 15
e) 20

a) zero;
b) 72 km/h;
c) 120 km/h;
d) 144 km/h;
e) 168 km/h.

29) (UFSM)

25) (UFLA MG) Num espetculo circense, o artista posiciona-se no alto de

uma plataforma, quando seu cavalo adentra o picadeiro num movimento


retilneo uniforme. O profissional do circo deixa-se cair verticalmente da
plataforma e atinge exatamente a sela do animal, o que provoca uma
exploso de aplausos.

Um barco se movimenta com velocidade constante em relao


margem de um rio. Uma pedra arremessada verticalmente, para cima, de
dentro do convs do barco.
Para um observador fixo na margem:
I no instante inicial do lanamento, a velocidade horizontal da pedra igual
velocidade do barco, e a velocidade vertical zero.
II no ponto mais alto da trajetria da pedra, o vetor velocidade tem mdulo
zero.
III a trajetria da pedra uma parbola.
Est(ao) correta(s):
a) apenas I
b) apenas II
c) apenas II e III
d) apenas III
e) I, II e III

30) (ITA)

Uma bola lanada horizontalmente do alto de um edifcio,


tocando o solo decorridos aproximadamente 2 s. Sendo de 2,5 m a altura de
cada andar, o nmero de andares do edifcio : (Use g = 9,8 m/s 2)
a) 5
Considerando que g = 10 m/s2; a altura vertical Hh plataforma-sela 3 m, e a
b) 6
velocidade do cavalo 5 m/s, pode-se afirmar que a distncia horizontal DH c) 8
entre a sela e a plataforma no momento do salto, em metros, :
d) 9
e) indeterminado, pois a velocidade horizontal de arremesso da bola no foi
a) 60
b) 30
c) 20
fornecida.
d)

10

e)

15

31) (UERJ)

Um projtil lanado segundo um ngulo de 30 com a


horizontal e com uma velocidade de 200 m/s. Supondo a acelerao da

83

gravidade igual a 10 m/s2 e desprezando a resistncia do ar, conclumos que


o menor tempo gasto por ele para atingir a altura de 480 m acima do ponto
de lanamento ser de:
a) 8 s
b) 10 s
c) 9 s
d) 14 s
e) 12 s

32) (UFPB)

Dois carros de corrida percorrem, lado a lado, uma curva


circular. O primeiro percorre um semicrculo de raio R1, com velocidade
angular 1 e linear v1 e o segundo, um semicrculo de raio R2, com velocidade
angular 2 e linear v2. Como os dois carros esto sempre lado a lado, os dois
semicrculos so percorridos ao mesmo tempo. Sabendo-se que R1 < R2,
podemos concluir que:
a)

1 2 e v 1 v 2

b)

1 2 e v 1 v 2

c)

1 2 e v 1 v 2

d)

1 2 e v 1 v 2

e)

1 2 e v 1 v 2

(entre os ps e o solo) grande e para frente, enquanto a que ocorre no


carro (entre os pneus e solo) pequena e para trs.
IV. O carro move-se porque a fora que a pessoa exerce sobre o carro e a
fora que o carro exerce sobre a pessoa so iguais, de sentidos contrrios,
mas aplicados em corpos diferentes e, portanto, cada um exerce o seu efeito
independentemente.
A partir da anlise feita, assinale a alternativa correta:
a) Apenas a proposio IV verdadeira.
b) Apenas as proposies III e IV so verdadeiras.
c) Apenas as proposies I e III so verdadeiras.
d) Apenas as proposies II e III so verdadeiras.
e) Apenas as proposies II e IV so verdadeiras.

36) (Unicemp PR) Nos quadrinhos acima, Garfield enunciou parte da 1 Lei
de Newton (Lei da Inrcia).

33) (Mackenzie)

Duas partculas A e B descrevem movimentos circulares


uniformes com velocidades escalares respectivamente iguais a v e 2v. O raio
da trajetria descrita por A o dobro do raio daquela descrita por B. A
relao entre os mdulos de suas aceleraes centrpetas :

1
a cB
8
1
a cB
4
1
a cB
2
a cB

a)

a cA

b)

a cA

c)

a cA

d)

a cA

e)

a cA 2a cB

34) (PUC SP) Em uma bicicleta o ciclista pedala na coroa e o movimento

transmitido catraca pela corrente. A freqncia de giro da catraca igual


da roda. Supondo os dimetros da coroa, catraca e roda iguais,
respectivamente, a 15 cm, 5,0 cm e 60 cm, a velocidade dessa bicicleta, em
m/s, quando o ciclista gira a coroa a 80 rpm, trem mdulo mais prximo de:

Complementando tal lei, outro caso de equilbrio o _______________ no


qual a fora resultante que atua sobre o corpo tambm ______________.
Escolha a alternativa que melhor completa os espaos deixados no texto
acima:
a) movimento circular uniforme; centrpeta;
b) movimento uniformemente variado; varivel;
c) movimento retilneo uniforme; nula;
d) movimento circular uniforme; nula;
e) movimento retilneo uniforme; varivel.

37) (UEFS - BA)


Velocidade (m/s)
Tempo (s)

a) 5
d) 11

b) 7
e) 14

c) 9

35) (UEPB

PB) Um automvel movendo-se em uma BR, guiado por um


aluno de fsica, falta combustvel ao se aproximar de um posto de gasolina.
Lembrando-se de uma aula sobre o princpio de ao e reao, ele
raciocinou: se eu descer do carro e tentar empurr-lo com uma fora F, ele
vai reagir com uma fora - F e ambas vo se anular e eu no conseguirei
mover o carro. Mas uma pessoa que vinha com ele, no concordando com
este raciocnio, desceu do carro e o empurrou, conseguindo mov-lo. Como
voc justificaria o carro mover-se?
Com base na compreenso desta lei, analise as proposies a seguir.

5
0

11
3

17
6

23
9

A velocidade de um corpo, de massa 5 kg, em movimento retilneo


uniformemente variado, se comporta, em relao ao tempo, de acordo com a
tabela.
Nessas condies, atua, sobre o corpo em movimento, uma fora resultante
de mdulo igual, em newtons a:
a) 2
b) 4
c) 5
d) 7
e) 10

38) (UEL)

A figura mostra um sistema em que se considera desprezvel o


atrito entre o bloco B e a superfcie de apoio, bem como as massas das polias
e dos fios.

I. O carro move-se porque a pessoa d um rpido


empurro no carro e, momentaneamente, essa fora
maior do que a fora que o carro exerceu sobre
ela.
II. O carro move-se porque a pessoa empurra o
carro para frente com uma fora maior do que a
fora com que o carro exerce sobre ela.
III. O carro move-se porque a fora que a pessoa
exerce sobre o carro to intensa quanto a que o
carro exerce sobre ela, no entanto, a fora de atrito que a pessoa exerce

84

42) (Fafi

BH) Um peso, P = 100 N, est preso a um fio. Considere as


seguintes afirmativas (g = 10 m/s2):
I Se T = 150 N, o fio com o peso se eleva com uma acelerao de 5 m/s 2.
II Quando o fio como peso desce com uma acelerao de 5 m/s 2, a tenso
no fio vale 50 N.
III Se T = 100 N, o peso poder subir com velocidade constante.

(Dados: mA = 2,0 kg; mB = 5,0 kg; mC = 3,0 kg; g = 10 m/s2).


A acelerao adquirida pelos corpos, quando o sistema abandonado, vale,
no Sistema Internacional de Unidades:
a) 1,0
b) 2,0
c) 3,0
d) 5,0
e) 10

39) (PUC

SP) Dois corpos esto presos s extremidades de um barbante


que passa por uma polia, como mostra o esquema da figura. Desprezando os
atritos, massa do barbante e massa da polia, e considerando a acelerao da
gravidade igual a 10 m/s2, o mdulo da acelerao do corpo P igual a:

Assinale:
a) se todas as afirmativas so falsas.
b) se todas as afirmativas esto corretas.
c) se apenas as afirmativas I e II esto corretas.
d) se apenas as afirmativas I e III esto corretas.
e) se apenas as afirmativas II e III esto corretas.

43) (MACK SP) Um elevador cujo peso de 1200 N desce com acelerao

constante de 1 m/s2. Admitindo g = 10 m/s2, podemos afirmar que a trao


no cabo de:
a) 980 N
b) 890 N
c) 1100 N
d) 1080 N
e) No sei

44) (UFLA

MG) Um corpo de massa 2 kg se desloca numa trajetria


horizontal. O diagrama abaixo, fora versus distncia, mostra as foras do
operador F e do atrito cintico fatc que atuam no corpo na direo do
movimento, o qual ocorre na horizontal. Considerando g = 10 m/s2 e a massa
do corpo 2 kg, pode-se afirmar que

a) 1,3 m/s2 b) 4,0 m/s2 c) 5,0 m/s2


d) 10 m/s2 e) 13 m/s2

40) (Fatec - SP) Um fio, que tem suas extremidades presas aos corpos A e
B, passa por uma roldana sem atrito e de massa desprezvel. O corpo A, de
massa 1,0 kg, est apoiado num plano inclinado de 37 com a horizontal,
suposto sem atrito.

a) a acelerao do corpo de 10 m/s2.


b) o coeficiente de atrito cintico de 0,5.
c) o movimento do corpo uniforme.
d) o trabalho realizado pela fora resultante no percurso de 2 m de 60 J.

45) (UFMS MS) Um pequeno bloco colocado em repouso

sobre um plano
AB inicialmente horizontal ( = 0). O plano pode girar em torno de um eixo
perpendicular ao plano da pgina que contm o ponto A (Figura ao lado).
Girando lentamente o plano, quando o ngulo alcana 30, o bloco inicia
seu movimento percorrendo 1,0m em 1,0s. Adotando a acelerao da
gravidade g=10m/s2, correto afirmar que:

(Adote g = 10 m/s2, sen37=0,60 e cos37=0,80).


Para o corpo B descer com acelerao de 2,0 m/s 2, o seu peso deve ser, em
newtons:
a) 2,0
b) 6,0
c) 8,0
d) 10
e) 20

41) (Mackenzie

SP) No sistema a seguir, o fio e a polia so considerados


ideais e o atrito entre as superfcies em contato desprezvel. Abandonandose o corpo B a partir do repouso, no ponto M, verifica-se que, aps 2 s, ele
passa pelo ponto N com velocidade de 8 m/s. Sabendo-se que a massa do
corpo A de 5 kg, a massa do corpo B
a) 1 kg
b) 2 kg
c) 3 kg
d) 4 kg
e) 5 kg

a) a energia mecnica do bloco permanece constante durante o seu


movimento
b) o coeficiente de atrito esttico entre o bloco e o plano

3/2

c) a acelerao do bloco durante o seu movimento 0,2 m/s2


d) o coeficiente de atrito cintico entre o bloco e o plano

3 /5

e) a energia potencial que o bloco perde, durante o seu movimento,


totalmente transformada em energia cintica

46) (UFAL)

Uma fora F horizontal e de intensidade 30 N aplicada num


corpo A de massa 4 kg, preso a um corpo B de massa 2 kg que, por sua vez,
se prende a um corpo C.

85

velocidade constante. Admite-se g = 10 m/s2, despreza-se o atrito e so


dados:
Sen30=cos60=0,5 e cos120= - 0,5.

O coeficiente de atrito entre cada corpo e a superfcie horizontal de apoio


0,10 e verifica-se que a acelerao do sistema , nessas condies, 2,0 m/s 2.
Adote Adote g = 10 m/s2 e analise as afirmaes.
01) A massa do corpo C 5,0 kg.
02) A trao no fio que une A e B tem mdulo 18 N
04) A fora de atrito que age no corpo A tem mdulo 4,0 N.
O trabalho realizado pelo peso do bloco, nesse percurso, em J, vale:
08) A trao no fio que une B a C tem mdulo 8,0 N.
a) 0,8 J
b) 0,8 J c) 1,2 J
16) A fora resultante no corpo B tem mdulo 4,0 N.
d) 1,6 J
e) 1,6 J

51) (UEPB)

Uma empresa de gerao de energia construiu uma usina


hidreltrica em que a queda dgua, com vazo de 360.000 m 3/h, encontrase 60 m acima do ponto onde se localiza a turbina, conforme observa-se na
a seguir. Considerando-se a acelerao da gravidade de 10 m/s 2,
47) (FMTM) Segurando as mos de seu filho com as suas, uma me pe-se figura
densidade da gua de 103 kg/m3 e desprezando-se as perdas de energia,
a gir-lo uniformemente a uma razo de meia volta por segundo, fazendo
correto afirmar que a potncia mdia, em kW (quilowatts), a ser gerada por
com que o menino fique temporariamente suspenso no ar. Se durante a
esta usina, vale:
rotao a criana de 15 kg tem seu centro de massa mantido a 1 m do corpo
da me, a fora centrpeta que o centro de massa da criana experimenta,
em N, :
D como resposta a soma dos nmeros que precedem as afirmativas
corretas.

(Adote:
a) 45
d) 110

3)

b) 60
e) 135

c) 95

48) (PUC SP0 Um automvel de 1720 kg entra em uma curva de raio r =


200 m, a 108 km/h. Sabendo que o coeficiente de atrito entre os pneus do
automvel e a rodovia igual a 0,3, considere as afirmaes:
I O automvel est a uma velocidade segura para fazer a curva.
II O automvel ir derrapar radialmente para fora da curva.
III A fora centrpeta do automvel excede a fora de atrito.
IV A fora de atrito o produto da fora normal do automvel e o
coeficiente de atrito.
Baseado nas afirmaes acima, responda:
a) apenas I est correta.
b) as afirmativas I e IV esto corretas.
c) apenas II e III esto corretas.
d) esto corretas I, III e IV
e) esto corretas II, IIII e IV.

49) (FGV SP) Mantendo uma inclinao de 60 com o plano da lixa, uma

pessoa arrasta sobre esta a cabea de um palito de fsforos, deslocando-o


com velocidade constante por uma distncia de 5 cm, e ao final desse
deslocamento a plvora se pe em chamas.

a) 0,6.103 b) 6.103
d) 6000.103

e) 60.103

c) 600.103

52) (Unifesp

- SP) Uma criana de massa 40 kg viaja no carro dos pais,


sentada no banco de trs, presa pelo cinto de segurana. Num determinado
momento, o carro atinge a velocidade de 72 km/h. Nesse instante, a energia
cintica dessa criana :
a) igual energia cintica do conjunto carro mais passageiro.
b) zero, pois fisicamente a criana no tem velocidade, logo, no tem energia
cintica.
c) 8000 J em relao ao carro e zero em relao estrada.
d) 8000 J em relao estrada e zero em relao ao carro.
e) 8000 J, independentemente do referencial considerado, pois a energia
um conceito absoluto.

53) (UFRGS)

Um carrinho de 5 kg de massa move-se horizontalmente em


linha reta, com velocidade de 6 m/s. O trabalho da resultante necessrio para
alterar a velocidade para 10 m/s deve ser, em joules:
a) 40
b) 90
c) 160
d) 400
e) 550

54) (FMIT

MG) Numa montanha-russa, que tem um de seus trechos


representados abaixo, com qual velocidade o carrinho passa pelo ponto 2 se
no ponto sua velocidade de 8 m/s?
Considere g = 10 m/s2 e despreze todas as foras dissipativas.

Se a intensidade da fora, constante, aplicada sobre o palito 2 N, a energia


empregada no acendimento deste, desconsiderando-se eventuais perdas, :

Dados: sen60
a)

5 3.10 2
2

b)

5.10

c)

2 3.10 2

d)

2.10 2

e)

3.10

J
J

50) (Unesp

3
1
; cos 60
2
2

SP) Na figura, sob a ao da fora de intensidade F = 2 N,


constante, paralela ao plano, o bloco percorre 0,8 m ao longo do plano com

a) 8 m/s
b) 18 m/s
c) 5 m/s
d) 22 m/s
e) nenhuma das respostas anteriores
55) (UFSC) A figura mostra um bloco, de massa m = 500 g, mantido
encostado em uma mola comprimida de x = 20 cm. A constante elstica da
mola k = 400 N/m. A mola solta e empurra o bloco que, partindo do
repouso no ponto A, atinge o ponto B, onde pra. No percurso entre os
pontos A e B, a fora de atrito da superfcie sobre o bloco dissipa 20% da
energia mecnica inicial no ponto A. (Faa g = 10 m/s2).

86

60) (UA

AM) Um corpo A de massa igual a 5,0 kg move-se com


velocidade constante de 4 m/s sobre um plano horizontal at colidir
frontalmente com outro corpo B de massa igual a 10 kg, inicialmente em
repouso. Aps a coliso, o corpo A fica parado. Pode-se afirmar que a
energia cintica final do corpo B vale:
a) 40 J
b) 10 J
c) 20 J
d) 30 J
e) 50 J
Assinale a(s) proposio(es) correta(s):
(01) Na situao descrita, no h conservao da energia mecnica.
(02) A energia mecnica do bloco no ponto B igual a 6,4 J.
(04) O trabalho realizado pela fora de atrito sobre o bloco, durante o seu
movimento, foi 1,6 J.
(08) A energia mecnica total do bloco, no ponto A, igual a 8,0 J.
(16) A fora peso no realizou trabalho no deslocamento do bloco entre os
pontos A e B, por isso no houve conservao da energia mecnica do bloco.
(32) O ponto B situa-se a 80 cm de altura, em relao ao ponto A.
(64) A energia potencial elstica do bloco, no ponto A, totalmente
transformada na energia potencial gravitacional do bloco, no ponto B.

61) (PUC Campinas) O grfico abaixo representa, em um certo sistema de


referncia, os valores das quantidades de movimento de duas esferas iguais,
de massa 2,0 kg cada, que se movem sobre uma mesma reta e realizam um
choque central.

56) (UESPI

adapt.) Analise as seguintes afirmaes sobre gravitao


universal:
I A primeira lei da gravitao universal afirma que todos os planetas
descrevem rbitas elpticas em torno do Sol, e este ocupa um dos focos desta
elipse.
II De acordo com a segundo lei da gravitao universal, os planetas
percorrem distncias iguais em intervalos de tempo iguais em sua trajetria
em torno do Sol.
III Dados os planetas, A e B, girando em torno de uma estrela, se o raio
mdio de A for o dobro do de B, a razo entre seus perodos

TA
TB

ser

8.

De acordo com o grfico, correto afirmar que:


a) a energia cintica de cada esfera se conservou no choque.
b) a quantidade de movimento de cada esfera se conservou no choque.
c) o choque foi totalmente inelstico
d) o choque foi parcialmente elstico, com coeficiente de restituio 0,5
e) o choque foi perfeitamente elstico.

62) (Mackenzie

SP) Utilizando-se de cordas ideais, dois garotos, exercendo


foras de mesmo mdulo, mantm em equilbrio um bloco A, como mostra a
figura. Se a fora de trao em cada corda tem intensidade de 20 N, a massa
do bloco suspenso :

IV Partindo do pressuposto de que a fora de atrao gravitacional entre


dois corpos vale F, se as massas dos corpos e a distncia entre eles forem
duplicadas, a fora de atrao entre eles permanecer inalterada.
As nicas afirmaes verdadeiras so:
a) I e III
b) I e IV
c) II e IV
d) I, II e III
e) I, III de IV

57) (UFSM)

Dois corpos esfricos e homogneos de mesma massa tm


seus centros separados por uma certa distncia, maior que o seu dimetro.
Se a massa de um deles for reduzida metade e a distncia entre seus
centros, duplicada, o mdulo da fora de atrao gravitacional que existe
entre eles ficar multiplicado por:
a) 8
b) 4
c) 1
d)

1
4

e)

Adote: g = 10 m/s2
a) 1,0 kg b) 2,0 kg
d) 4,0 kg e) 5,0 kg

1
8

58) (FGV

SP) Uma ema pesa aproximadamente 360 N e consegue


desenvolver uma velocidade de 60 km/h, o que lhe confere uma quantidade
de movimento linear, em kg.m/s, de: (Dado: acelerao da gravidade = 10
m/s2).
a) 36
b) 360
c) 600
d) 2160
e) 3600

c) 3,0 kg

63) (FMTM

MG) Com o intuito de expor a rplica de um biplano em


posio de mergulho, dois cabos de ao, um preso na parede e outro preso
no teto da sala de um museu, sustentam o modelo da aeronave, que tem
massa 6 kg.

59) (Cefet PR) Uma pequena bola de borracha, de massa 200 g, desloca-

se para direita com velocidade constante de 5 m/s. Ao bater em uma parede,


ele retorna com velocidade de 3 m/s, em mdulo, como indica a figura a
seguir. Supondo que o choque entre a parede e a bola tenha durado um
centsimo de segundo, a intensidade da fora aplicada pela parede bola foi
de:

a) 40 N
d) 210 N

b) 80 N
e) 600 N

c) 160 N

Admita que os cabos so inextensveis, de massa desprezvel e que suas


aes ocorrem sobre o centro de massa do biplano; que o avio
suficientemente pequeno para ser considerado um ponto material; que o
avio est em repouso em relao ao interior da sala.
Dados: g = 10 m/s2; sen30=0,50; cos30=0,87.
As tenses, medidas em newtons, sobre o cabo horizontal e o cabo
transversal so, respectivamente:

87

e) 1,2 105

a) 12,0 e 14,4
b) 30,0 e 52,2
c) 60,0 e 60,0
d) 90,0 e 552,0
e) 104,4 e 120,0

69) (UFPR PR) Uma tarefa de rotina em depsitos de combustveis consiste

64) (MACK)

Uma barra AB homognea, de seco transversal uniforme e


peso 400 N est apoiada sobre um cavalete e mantida em equilbrio
horizontal pelo corpo Q, colocado na extremidade A. A barra tem
comprimento de 5 m. O peso do corpo Q :

em retirar uma amostra de lquido dos tanques e colocar em provetas para


anlise. Ao inspecionar o contedo de um dos tanques de um certo depsito,
observou-se na parte inferior da proveta uma coluna de 20 cm de altura de
gua e, flutuando sobre ela, uma coluna com 80 cm de altura de leo.
Considerando a densidade da gua igual a 1,00 g/cm 3, a do leo igual a 0,80
g/cm3, a acelerao da gravidade igual a 10 m/s2 e a presso atmosfrica
igual a 1,01 105 Pa, a presso no fundo desse tubo :
a) 1,094 x 105 Pa.
b) 9,41 x 105 Pa.
c) 1,03 x 105 Pa.
d) 1,66 x 105 Pa.
e) 0,941 x 105 Pa.

70) (Unioeste
a) 100 N
d) 250 N

b) 150 N
e) 300 N

c) 200 N

65) (FGV SP)

Usado no antigo Egito para retirar gua do rio Nilo, o shaduf


pode ser visto como um ancestral do guindaste. Consistia de uma haste de
madeira onde em uma das extremidades era amarrado um balde, enquanto
que na outra, uma grande pedra fazia o papel de contra-peso. A haste
horizontal apoiava-se em outra verticalmente disposta e o operador, com
suas mos entre o extremo contendo o balde e o apoio (ponto P), exercia
uma pequena fora adicional para dar ao mecanismo sua mobilidade.

Dados:
Peso do balde e sua corda ........ 200 N
Peso da pedra e sua corda ........ 350 N
Para o esquema apresentado, a fora vertical que uma pessoa deve exercer
sobre o ponto P, para que o shaduf fique horizontalmente em equilbrio, tem
sentido
a) para baixo e intensidade de 100 N.
b) para baixo e intensidade de 50 N.
c) para cima e intensidade de 150 N.
d) para cima e intensidade de 100 N.
e) para cima e intensidade de 50 N.

PR) Um tubo em U est aberto nas duas extremidades e


parcialmente cheio dgua (densidade = 1 x 10 3kg/m3). Num dos braos,
derrama-se querosene (densidade = 0,82 x 10 3 kg/m3), formando-se uma
coluna de 6,0cm de altura.
Qual a diferena (h) da altura
dos dois nveis de lquido em
cada ramo do tubo?
a) 6,00 cm.
b) 4,92 cm.
c) 1,08 cm.
d) 10,92 cm.
e) 7,08 cm.

71) (UEPB

PB) O fsico e
matemtico francs Blaise
Pascal (1623-1662), um dos
precursores no estudo da
hidrosttica, props um princpio (denominado de Pascal), que tem uma
importante aplicao em mquinas hidrulicas, capazes de multiplicar foras.
O princpio desta mquina tambm empregado nos elevadores de
automveis, nas cadeiras de dentistas e barbeiros e nos freios hidrulicos,
etc. Observe a seguir, uma aplicao deste princpio.
Num posto de gasolina, um elevador deve erguer um automvel de 1000 kg
de massa. Se o pisto sobre o qual est o carro tem rea de 600 cm 2,
adotando a acelerao da gravidade 10 m/s2, o mdulo da fora que deve ser
aplicada outra extremidade do pisto, cuja rea de 30 cm2, vale:
a) 410,0 N b) 500,0 N c) 420,0 N
d) 430,0 N e) 580,0 N

72) (FMTM)

A prensa hidrulica representada na figura consta de dois


recipientes cilndricos que se intercomunicam e esto preenchidos com um
lquido homogneo e incompressvel. Quando se aplica ao mbolo E1 uma
fora

F1 ,

transmite-se ao mbolo E2 uma fora

F2 .

A relao entre os

dimetros dos mbolos d2 = 10.d1. Pode-se afirmar que:

66) (Unifor

CE) Um ourives fundiu 20 g de um material de densidade 20


g/cm3, juntamente com 40 g de outro material de densidade 10 g/cm 3. O
valor da densidade, em g/cm3, do composto formado :
a) 18
b) 16
c) 13
d) 12
e) 11

67) (UECE) Tripulantes de um submarino danificado que se encontra a 50m

abaixo do nvel do mar precisam sair dele pela porta de um alapo. A porta
pode ser aberta por um sistema eltrico que exerce uma fora perpendicular
a ela. Supondo-se que a densidade da gua do mar igual a 1025kg/m 3, o
peso da porta desprezvel e que suas dimenses so 0,40m e 0,60m, a
menor fora (em Newtons) necessria para o sistema abrir a porta ,
aproximadamente:
a) 0,6 x 105
b) 1,2 x 105
c) 2,4 x 105
d) 3,6 x 105

68) (Fatec SP) Uma piscina possui 10 m de comprimento, 5,0 m de largura


e 2,0 m de profundidade e est completamente cheia de gua.
Dados:
densidade da gua = 1,0103 kg/m3
presso atmosfrica local = 1,0105 N/m2
acelerao da gravidade local = 10 m/s 2
A presso no fundo da piscina, em N/m 2, vale
a) 2,0 105
b) 1,8 105
c) 1,6 105
d) 1,4 105

a)

F2
1

F1 100

b)

F2
1

F1
10

d)

F2
10
F1

e)

F2
100
F1

c)

F2
5
F1

73) (UEL

PR) Segundo o Princpio de Arquimedes: um corpo total ou


parcialmente imerso num fluido recebe do fluido um empuxo igual e contrrio
ao peso da poro de fluido deslocada e aplicado no centro de gravidade da
mesma.
Baseando-se neste princpio, considere as afirmativas a seguir.
I. Uma pessoa, no limite de suas foras, consegue flutuar na gua salgada e,
nas mesmas condies fsicas, no consegue flutuar em gua doce, porque a
gua doce no exerce empuxo sobre a pessoa.
II. Se uma pessoa, ao flutuar, desloca a mesma poro de gua estando em
gua doce ou salgada, a fora de empuxo exercida pela gua salgada
maior que a exercida pela gua doce, porque a densidade da gua doce
menor que a densidade da gua salgada.
III. Um corpo flutuante est em equilbrio quando a resultante entre a fora
de empuxo e a fora peso diferente de zero e dirigida para cima.
IV. A fora de empuxo uma fora vertical, dirigida para cima e
proporcional densidade e ao volume do lquido em que o corpo est imerso.

88

d) 0,8
Esto corretas apenas as afirmativas:
a) I e II.
b) I e III.
c) II e IV.
d) I, III e IV.
e) II, III e IV.

74) (UFSC) Dados: g = 10 m/s2

e) 0,6

76) Um pote de plstico fechado, cujo volume 1.000 cm3, flutua na gua

com 60% do seu volume imerso. Adotando a densidade da gua igual a 1,0
g/cm3 e a acelerao da gravidade igual a 10 m/s 2, pode-se determinar o
peso e a densidade desse pote. O peso do pote, em newtons, e a densidade,
em g/cm3, so, respectivamente:
a) 4,0 e 0,40
b) 4,0 e 0,60
c) 5,0 e 0,50
d) 6,0 e 1,0
e) 6,0 e 0,60

A figura representa um navio flutuando em equilbrio, submetido ao


apenas de seu prprio peso e do empuxo exercido pela gua.

Considerando a situao descrita, indique a(s) proposio(es) correta(s):


(01) Mesmo sendo construdo com chapas de ao, a densidade mdia do
navio menor do que a densidade da gua.
(02) O empuxo exercido sobre o navio igual ao seu peso.
(04) Um volume de gua igual ao volume submerso do navio tem o mesmo
peso do navio.
(08) O empuxo exercido sobre o navio maior do que seu peso. Caso
contrrio, um pequeno acrscimo de carga provocaria o seu afundamento.
(16) Se um dano no navio permitir que a gua penetre no seu interior,
enchendo-o, ele afundar totalmente, porque, cheio de gua, sua densidade
mdia ser maior do que a densidade da gua.
(32) Sendo o empuxo exercido sobre o navio igual ao seu peso, a densidade
mdia do navio igual densidade da gua.

75) (UFU MG) Prximo aos plos da Terra, podemos observar a presena

de icebergs nos oceanos, dificultando a navegao, uma vez que grande


parte deles est submersa e no pode ser vista pelo navegador. (Dados:
densidade aproximada da gua do mar = 1,0 g/cm 3; densidade aproximada
do gelo = 0,9 g/cm3).
A frao do iceberg que pode ser observada pelo navegador :
a) 0,9
b) 0,1
c) 0,2

1) D
6) C
11) D
16) E
21) B
26) D
31) A
36) C
41) C
46) 22
51) E
56) E
61) E
66) D
71) B
76) E

2) C
7) B
12) E
17) D
22) A
27) D
32) A
37) E
42) B
47) E
52) D
57) E
62) B
67) B
72) E

3) D
8) A
13) A
18) D
23) B
28) B
33) A
38) A
43) D
48) E
53) C
58) C
63) E
68) E
73) C

4) A
9) A
14) C
19) B
24) C
29) D
34) C
39) B
44) B
49) B
54) E
59) C
64) A
69) A
74) 23

5) D
10) D
15) D
20) D
25) E
30) C
35) A
40) D
45) D
50) E
55) 15
60) C
65) D
70) C
75) B

89

1) (Unifesp

SP) O texto a seguir foi extrado de uma matria sobre


congelamento de cadveres para sua preservao por muitos anos, publicada
no jornal O Estado de S. Paulo de 21.07.2002.
Aps a morte clnica, o corpo resfriado com gelo. Uma injeo de
anticoagulantes aplicada e um fluido especial bombeado para o corao,
espalhando-se pelo corpo e empurrando para fora os fluidos naturais. O
corpo colocado numa cmara com gs nitrognio, onde os fluidos
endurecem em vez de congelar. Assim que atinge a temperatura de 321, o
corpo levado para um tanque de nitrognio lquido, onde fica de cabea
para baixo.
Na matria, no consta a unidade de temperatura usada.
Considerando que o valor indicado de 321 esteja correto e que pertena a
uma das escalas, Kelvin, Celsius ou Fahrenheit, pode-se concluir que foi
usada a escala:
a) Kelvin, pois trata-se de um trabalho cientfico e esta a unidade adotada
pelo Sistema Internacional.
b) Fahrenheit, por ser um valor inferior ao zero absoluto e, portanto, s pode
ser medido nessa escala.
c) Fahrenheit, pois as escalas Celsius e Kelvin no admitem esse valor
numrico de temperatura.
d) Celsius, pois s ela tem valores numricos negativos para a indicao de
temperaturas.
e) Celsius, por tratar-se de uma matria publicada em lngua portuguesa e
essa ser a unidade adotada oficialmente no Brasil.

2) (PUC SP) O grfico representa a relao entre a temperatura medida em

c) 3,2 cm
d) 4,0 cm
e) 6,0 cm

7) (UFG

GO) Uma bala perdida disparada com velocidade de 200,0 m/s


penetrou na parede ficando nela incrustada. Considere que 50% da energia
cintica da bala foi transformada em calor, ficando nela retida. A variao de
temperatura da bala, em C, imediatamente ao parar,
Considere: Calor especfico da bala: 250 J/kgC
a) 10
b) 20
c) 40
d) 80
e) 160

8) (Mackenzie

SP) Ao nvel do mar, certa pessoa necessitou aquecer 2,0


litros dgua, utilizando um aquecedor eltrico de imerso, cuja potncia til
constante e igual a 1,0 kW. O termmetro disponibilizado estava calibrado
na escala Fahrenheit e, no incio do aquecimento, a temperatura indicada era
122 F. O tempo mnimo necessrio para que a gua atingisse a temperatura
de ebulio foi
Dados:

gua 1,0

uma escala de temperatura hipottica W e a temperatura medida na escala


Celsius, sob presso normal.

A temperatura de fuso do gelo e a


de ebulio da gua so, em graus
W, respectivamente iguais a
a) 40 e 40
b) 40 e 110
c) 20 e 110
d) 40 e 100
e) 20 e 100

cm3
c gua 1cal / g.C

1cal 4 ,2 J
a) 1 min 40 s
d) 7 min

b) 2 min
e) 10 min

c) 4 min 20 s

9) (UFAM AM) O

grfico representa a variao da temperatura de 100 g de


um corpo slido, em funo da quantidade de calor recebida por uma fonte
que libera energia a uma taxa de 60 Cal/min.

3) (UFAM

AM)
Uma
escala
termomtrica X construda de modo
que a temperatura de 0 X
corresponde a 4 F, e a
temperatura de 100 X corresponde a 68 F. Nesta escala X, a temperatura
de fuso do gelo vale:
a) 30 X
b) 20 X c) 50 X
d) 40 X e) 10 X

4) (Mackenzie

SP) Um mdico criou para uso prprio uma escala


termomtrica linear, adotando, respectivamente, 10,0 M e 190 M para os
pontos de fuso do gelo e de ebulio da gua sobre presso normal. Usando
um termmetro graduado nessa escala, ele mediu a temperatura de um
paciente e encontrou o valor 68 M. A temperatura dessa pessoa na escala
Celsius era:
a) 39 C
b) 38 C
c) 37,5 C
d) 37 C
e) 36,5 C

5) (Mackenzie

SP) Um estudante observa que, em certo instante, a


temperatura de um corpo, na escala Kelvin, 280 K. Aps 2 horas, esse
estudante verifica que a temperatura desse corpo, na escala Fahrenheit, 86
F. Nessas 2 horas, a variao da temperatura do corpo, na escala Celsius,
foi de:
a) 23 C
b) 25 C
c) 28 C
d) 30 C
e) 33 C

Em 4 minutos , a variao de temperatura sofrida pelo corpo de :


a) 60C
d) 50C

b) 80C
e) 50C

c) 90C

10) (PUC

PR) Uma fonte de energia (trmica), de potncia constante e


igual a 20 cal/s, fornece calor a uma massa slida de 100 g.
O grfico a seguir mostra a variao de temperatura em funo do tempo:

6) (Mackenzie

SP) Um profissional, necessitando efetuar uma medida de


temperatura, utilizou um termmetro cujas escalas termomtricas
inicialmente impressas ao lado da coluna de mercrio estavam ilegveis. Para
atingir seu objetivo, colocou o termmetro inicialmente numa vasilha com
gelo fundente, sob presso normal, e verificou que no equilbrio trmico a
coluna de mercrio atingiu 8,0 cm. Ao colocar o termmetro em contato com
gua fervente, tambm sob presso normal, o equilbrio trmico se deu com
a coluna de mercrio atingindo 20,0 cm de altura. Se nesse termmetro
utilizarmos as escalas Celsius e Fahrenheit e a temperatura a ser medida for
expressa pelo mesmo valor nas duas escalas, a coluna de mercrio ter
altura de:
a) 0,33 cm
b) 0,80 cm

Marque a alternativa correta:


a) O calor latente de fuso da substncia de 200 cal/g.
b) A temperatura de fuso de 150C.
c) O calor especfico no estado slido de 0,1 cal/gC.
d) O calor latente de fuso de 20 cal/g.
e) O calor especfico no estado lquido de 0,4 cal/gC.

90

11) (Mackenzie

SP) Durante um trabalho em laboratrio, dois estudantes


resolveram comparar seus resultados. O primeiro, A, aqueceu uma massa de
gelo (gua no estado slido) a partir da temperatura de 10 C e levou-a
ao estado lquido, at a temperatura de 20 C. O segundo, B, resfriou uma
massa de gua, igual do primeiro, a partir da temperatura 10 C e levoua ao estado slido, at a temperatura de 20 C. A relao entre o valor
absoluto da quantidade de calor recebida pela primeira massa dgua (QA) e
o valor absoluto da quantidade de calor perdida pela segunda (QB) :
Dados:
calor especfico do gelo

cal
g. C

(gua no estado slido)................

Cg 0,500

calor especfico da gua lquida....

cal
C a 1,000
g. C

calor latente de fuso do gelo......

L f 80,0

a)

QA
0,05
QB

b)

QA
0,55
QB

d)

QA
1,05
QB

e)

QA
1,5
QB

c)

cal
g
QA
0,95
QB

12) (UFAM AM) O grfico representa a temperatura dois corpos slidos

Ae
B de massas iguais, em funo da quantidade de calor Q recebida. Colocando
A a 20 C em contato com B a 100 C e admitindo que a troca de calor s
ocorra entre eles, a temperatura final de equilbrio em C :

Considere um sistema constitudo por uma quartinha cheia d gua. Parte da


gua que chega superfcie externa da quartinha, atravs de seus poros,
evapora, retirando calor do barro e da gua que o permeia. Isso implica que
tambm a temperatura da gua que est em seu interior diminui nesse
processo.
Tal processo se explica porque, na gua que evapora, so as molculas de
gua:
a) com menor energia cintica mdia que escapam do lquido, aumentando,
assim, a energia cintica mdia desse sistema.
b) que, ao escaparem do lquido, aumentam a presso atmosfrica,
diminuindo, assim, a presso no interior da quartinha.
c) com maior energia cintica mdia que escapam do lquido, diminuindo,
assim, a energia cintica mdia desse sistema.
d) que, ao escaparem do lquido, diminuem a presso atmosfrica,
aumentando, assim, a presso no interior da quartinha.

16) (Enem)

Ainda hoje, muito comum as pessoas utilizarem vasilhames


de barro (moringas ou potes de cermica no esmaltada) para conservar
gua a uma temperatura menor do que a do ambiente. Isso ocorre porque:
a) o barro isola a gua do ambiente, mantendo-a sempre a uma temperatura
menor que a dele, como se fosse isopor.
b) o barro tem poder de gelar a gua pela sua composio qumica. Na
reao, a gua perde calor.
c) o barro poroso, permitindo que a gua passe atravs dele. Parte dessa
gua evapora, tomando calor da moringa e o restante da gua, que so
assim resfriadas.
d) o barro poroso, permitindo que a gua se deposite na parte de fora da
moringa. A gua de fora sempre est a uma temperatura maior que a de
dentro.
e) a moringa uma espcie de geladeira natural, liberando substncia
higroscpicas que diminuem naturalmente a temperatura da gua.

17) (Unesp

SP) Considere seus conhecimentos sobre mudanas de fase e


analise as afirmaes I, II e III, referentes substncia gua, um recurso
natural de alto valor.
I. Durante a transio de slido para lquido, a temperatura no muda,
embora uma quantidade de calor tenha sido fornecida gua.
II. O calor latente de condensao da gua tem um valor diferente do calor
latente de vaporizao.
III. Em determinadas condies, a gua pode coexistir na fase slida, lquida
e gasosa.
Pode-se afirmar que
a) apenas a afirmao I correta.
b) apenas as afirmaes I e II so corretas.
c) apenas as afirmaes I e III so corretas.
d) apenas as afirmaes II e III so corretas.
e) as afirmaes I, II e III so corretas.

a) 50
d) 70

b) 80
e) 90

c) 60

13) (UFMA) Maria e Joo estavam acampados numa praia de So Lus onde

a temperatura ambiente era de 35C ao meio dia. Nesse momento,


verificaram que dois litros de gua mineral estavam na temperatura
ambiente. Resolveram ento baixar a temperatura da gua, colocando-a num
recipiente de isopor juntamente com 200g de gelo a 4C. Aps a fuso de
todo o gelo, e estabelecido o equilbrio trmico da mistura, a temperatura da
gua era aproximadamente:
Dados:
Lgelo = 80cal/g;
cgelo = 0,5cal/gC;
cgua = 1,0 cal/gC
a) 26,9C
b) 22,3C
c) 24,4C
d) 20,3C
e) 29,4C

18) (Uni-Rio

RJ) Admita que voc est com muita fome e deseja cozinhar
batatas em uma panela comum de alumnio num ambiente termicamente
isolado. Considerando que voc s se alimentar quando as batatas
estiverem completamente cozidas, em que local voc poderia saciar sua fome
mais rapidamente?
1. Despreze as perdas de calor para o meio ambiente.
2. Considere a mesma temperatura inicial do conjunto em todos os
ambientes.
a) No Po de Acar Rio de Janeiro.
b) Na Pedra do Sino - Petrpolis .
c) No Pico das Agulhas Negras Itatiaia.
d) No Pico da Bandeira - ES.
e) No Pico da Neblina Serra do Imeri - RR .

19) (Fuvest)

O diagrama esboado mostra os estados fsicos do CO 2 em


diferentes presses e temperaturas. As curvas so formadas por pontos em
que coexistem dois ou mais estados fsicos.

14) (FFFCMPA RS) Uma garrafa trmica, cuja capacidade trmica no pode

ser desprezada, contm 280g de gua. A temperatura do sistema garrafagua de 30 C. Uma massa de 70g de gelo dgua a 0 C colocada na
garrafa. Sabe-se que o calor especfico da gua no estado lquido de 1
cal/g.C e que o calor latente de fuso da gua de 80 cal/g. Considerando
que o sistema garrafa-gua-gelo adiabtico, que a presso de 1 atm e
que a temperatura de equilbrio de 10 C, qual a capacidade trmica da
garrafa?
a) 5 cal/C b) 10 cal/C
c) 20 cal/C
d) 25 cal/C
e) 35 cal/C

15) (UFRN

RN) Cotidianamente so usados recipientes de barro (potes,


quartinhas, filtros etc.) para esfriar um pouco a gua neles contida.

91

I. As trocas de calor por irradiao so resultantes da fragmentao de


ncleos de tomos instveis num processo tambm conhecido por
radioatividade.
II. A conduo trmica o processo de transferncia de calor de um meio ao
outro atravs de ondas eletromagnticas.
III. No pode haver propagao de calor nem por conduo, nem por
conveco, onde no h meio material.
IV. O fenmeno da inverso trmica ocorre mais freqentemente no inverno
e acentua a poluio, j que no ocorre conveco.
correto o contido em apenas
a) I e II.
b) I e III. c) II e III.
d) II e IV. e) III e IV.

23) (FFFCMPA RS) A sensao distinta de quente e frio quando colocamos a

Um mtodo de produo de gelo-seco (CO2 slido) envolve:


I compresso isotrmica do CO2 gasoso, inicialmente a 25C e 1 atm, at
passar para o estado lquido.
II rpida descompresso de 1 atm, processo no qual ocorre rpido
abaixamento de temperatura e aparecimento de CO2 slido.
Em I, a presso mnima a que o CO 2 gasoso deve ser submetido para
comear a liquefao, a 25C, y e, em II, a temperatura deve atingir x. Os
valores de y e x so, respectivamente:
a) 67 atm e 0C
b) 73 atm e 78C
c) 5 atm e 57C
d) 67 atm e 78C
e) 73 atm e 57C
20) Na patinao sobre o gelo, o deslizamento e facilitado porque, quando
o patinador passa, parte do gelo se transforma em gua, reduzindo o atrito.
Estando o gelo a uma temperatura inferior a 0C, isso ocorre porque a
presso da lamina do patim sobre o gelo faz com que ele derreta.
De acordo com seus conhecimentos e com as informaes do texto, correto
afirmar que a fuso do gelo acontece porque:
a) a presso no influencia no ponto de fuso.
b) o aumento da presso aumenta o ponto de fuso.
c) a diminuio da presso diminui o ponto de fuso.
d) a presso e o ponto de fuso no se alteram.
e) o aumento da presso diminui o ponto de fuso.

21) (Unifor

CE) Considere o diagrama de estado de uma substncia e as


afirmaes que seguem.

I. No ponto T, representado no diagrama, a substncia pode


simultaneamente apresentar-se nas fases slida, lquida e gasosa.
II. Se a substncia for aquecida, sob presso constante de 60 cm Hg, desde
a temperatura de 40C at chegar a 90C, ela sofrer duas mudanas de
fase.
III. Somente pode ocorrer a sublimao dessa substncia se t < 40 C e p <
40 cmHg.
Est correto o que se afirma em
a) II, somente.
b) I e II, somente.
c) I e III, somente.
d) II e III, somente.
e) I, II e III.

mo em uma barra de alumnio ou em uma barra de madeira, que esto em


equilbrio trmico com um ambiente a 27C, se deve ao fato de que
a) a quantidade de calor trocada entre a pele e a barra de alumnio menor
do que a trocada entre a pele e a barra de madeira.
b) a quantidade de calor trocada entre a pele e a barra de alumnio maior
do que a trocada entre a pele e a barra de madeira.
c) o alumnio se encontra em uma temperatura superior da madeira.
d) o alumnio se encontra em uma temperatura inferior da madeira.
e) a quantidade de calor trocada entre a pele e as duas barras serem iguais.

24) (Urca)

Iglos (ou iglus) so abrigos feito de neve, ou gelo, construdo


para proteger pessoas do frio das regies muito geladas. Que opo explica
corretamente o fato dessas construes servirem como abrigos trmicos:
a) a gua, mesmo no seu estado slido, possui calor especfico elevado,
funcionando como isolante trmico por no permitir a conduo de calor;
b) a gua, mesmo no seu estado slido, possui calor especfico elevado,
funcionando como isolante trmico por no permitir que o calor produzido
pelo corpo humano seja conduzido com facilidade para fora do ambiente
fechado do iglo;
c) a gua em seu estado slido (gelo) funciona, em certas ocasies, como
aquecedor natural devido o fato do calor comprimido por suas molculas ser
expulso de sua estrutura molecular da gua;
d) a gua em seu estado slido capta com facilidade o calor do ambiente
externo direcionando para o interior do iglo aquecendo as pessoas que ficam
em seu interior;
e) a gua no estado slido possui um calor especfico muito baixo,
funcionando nesse caso como isolante trmico por no permitir que o calor
produzido pelo corpo humano seja conduzido com facilidade para fora do
ambiente fechado do iglo.

25) (UEPB PB) At o incio do sculo XIX,

acreditava-se que a temperatura


de um corpo estava associada a uma substncia fluida, invisvel e de peso
desprezvel, denominada calrico, contida no interior do corpo. No decorrer
do mesmo sculo essas idias foram contestadas e, atravs de algumas
experincias, a exemplo de uma realizada pelo fsico ingls James Prescott
Joule (1818 - 1889), identificou-se definitivamente o calor como energia.
Com base nas informaes contidas no texto acima e em suas experincias
dirias, analise as seguintes proposies:
I. Quando colocamos a mo na maaneta e na madeira de uma porta, a
sensao distinta de quente e frio est associada diferena de temperatura
entre ambas.
II. Ao colocar a mo embaixo de uma panela retirada do fogo a uma certa
distncia, tem-se a sensao de quente, uma vez que a troca de calor neste
processo d-se por conveco.
III. Retirando-se da geladeira uma lata e uma garrafa (de vidro) de
refrigerante em equilbrio trmico, tem-se a impresso de que a lata est
mais fria que a garrafa. Esta sensao diferenciada explicada por a lata,
que geralmente de alumnio, apresentar maior coeficiente de condutividade
trmica do que a garrafa de vidro.
IV. As garrafas trmicas so constitudas de um recipiente de vidro de
paredes duplas, espelhadas interna e externamente. A quase inexistncia de
ar entre as paredes dificulta a propagao do calor, quer por conduo, quer
por conveco.
A partir da anlise feita, assinale a alternativa correta:
a) Todas as proposies so verdadeiras
b) Apenas as proposies I e III so verdadeiras
c) Apenas as proposies II e III so verdadeiras
d) Apenas as proposies II e IV so verdadeiras
e) Apenas as proposies III e IV so verdadeiras

26) (Mackenzie

SP) Numa noite de inverno, o dormitrio de Serginho


apresentava uma temperatura ambiente de 10 C. Para no sentir frio
durante a madrugada, ele esticou sobre a cama trs cobertores de l bem
espessos e aguardou alguns minutos. Em seguida, deitou-se e percebeu que
a cama continuava muito fria. Aps um certo tempo na cama, bem coberto,
22) (UFTM) A transmisso de calor entre os corpos pode ocorrer por trs sentiu que o frio passou e que a cama estava quente.
processos diferentes. Sobre estes processos, considere:

92

Tal fato explica-se, pois:


a) o frio no existe e a sensao de Serginho era apenas psicolgica.
b) os cobertores no so aquecedores, mas isolantes trmicos. Depois de
Serginho deitar-se, seu corpo aqueceu a cama.
c) a cama provavelmente no tinha lenis de l e, ento, o calor produzido
pelos cobertores foi perdido para o ambiente. Quando Serginho se deitou,
interrompeu esse processo.
d) os cobertores de l provavelmente eram de cor clara e, por isso,
demoraram muito para aquecer a cama. Aps Serginho ter-se deitado, foi
necessrio mais algum tempo para que a cama ficasse quente.
e) a l utilizada para a confeco dos cobertores um aquecedor natural
muito lento e a temperatura de Serginho, de aproximadamente 37 C, no
era suficiente para aquecer a cama.

27) (Unesp

SP) Um corpo I colocado dentro de uma campnula de vidro


transparente evacuada. Do lado externo, em ambiente presso atmosfrica,
um corpo II colocado prximo campnula, mas no em contato com ela,
como mostra a figura.

Dado: ao= 1,27 105 C1


a) 3,18 104 e o relgio atrasar.
b) 3,18 104 e o relgio adiantar.
c) 5,53 104 e o relgio atrasar.
d) 6,35 104 e o relgio adiantar.
e) 6,35 104 e o relgio atrasar.

31) (UFS SE) Uma chapa retangular de zinco, de dimenses 80 cm x 25cm,

sofre elevao uniforme de 45F em sua temperatura. Sabese que, sob


presso de 1,0 atmosfera, aos pontos de fuso do gelo e de ebulio da gua
correspondem os valores 32 e 212 graus Fahrenheit, respectivamente, e que

As temperaturas dos corpos so diferentes e os pinos que os sustentam so


isolantes trmicos. Considere as formas de transferncia de calor entre esses
corpos e aponte a alternativa correta.
a) No h troca de calor entre os corpos I e II porque no esto em contato
entre si.
b) No h troca de calor entre os corpos I e II porque o ambiente no interior
da campnula est evacuado.
c) No h troca de calor entre os corpos I e II porque suas temperaturas so
diferentes.
d) H troca de calor entre os corpos I e II e a transferncia se d por
conveco.
e) H troca de calor entre os corpos I e II e a transferncia se d por meio
de radiao eletromagntica.

28) (UDESC)

A figura (a) mostra um dispositivo que pode ser usado para


ligar ou desligar um forno, dependendo da temperatura do local onde se
encontra o sensor (barra AB). Essa barra constituda de dois metais
diferentes e, ao ser aquecida, fecha o circuito, como indicado na figura (b).

o coeficiente de dilatao linear do zinco de 25 10


aquecimento, o aumento da rea da chapa , em cm 2,
a) 1,0
b) 2,5
c) 4,0
d) 5,5
e) 8,0

C 1 .

Nesse

32) (UFG GO) Num dia quente em Goinia, 32 C, uma dona de casa coloca

lcool em um recipiente de vidro graduado e lacra-o bem para evitar


evaporao. De madrugada, com o termmetro acusando 12 C, ela nota
surpresa que, apesar do vidro estar bem fechado, o volume de lcool
reduziu. Sabe-se que o seu espanto no se justifica, pois trata-se do
fenmeno da dilatao trmica. A diminuio do volume foi de Considere o
coeficiente de dilatao trmica volumtrica do lcool:
lcool = 1,1 x.103 C1 >> vidro
a) 1,1%
b) 2,2%
c) 3,3%
d) 4,4%
e) 6,6%

33) (Unifor

CE) Um cilindro de metal possui, a 20C, volume de 600 cm 3.


Aquecido at 120C ele sofre uma dilatao equivalente a 0,03% do seu
volume inicial. Nessas condies, o coeficiente de dilatao linear do metal,
em C1, vale
a) 3 . 108 b) 1 . 107 c) 3 . 107
d) 1 . 106 e) 3 . 106

34) (UFC

CE) Numa
experincia de
laboratrio,
sobre dilatao
superficial, foram feitas vrias medidas das dimenses de uma superfcie
S de uma lmina circular de vidro em funo da temperatura T. Os
resultados das medidas esto representados no grfico abaixo.

O funcionamento do dispositivo acima indicado ocorre devido:


a) a metais diferentes possurem calores especficos diferentes.
b) a metais diferentes possurem condutividades trmicas diferentes.
c) ao calor fluir sempre de um corpo a uma temperatura maior para um
corpo a uma temperatura menor, e nunca ocorrer o fluxo contrrio.
d) a metais diferentes possurem calores latentes diferentes.
e) a metais diferentes possurem coeficientes de dilatao trmica diferentes.

29) (Urca)

Considerando o coeficiente
5

ao 11x10 C

de dilatao linear do ao
5
1
e do alumnio Al 22x10 C , qual a

relao entre os comprimentos iniciais de uma barra de ao e uma barra de


alumnio para suas dilataes serem sempre as mesmas?
a) L0ao = L0Al
b) 2L0ao = L0Al
c) L0ao = 2L0Al
d) L0ao = 3L0Al
e) 3L0ao = L0Al

30) (FMTM

MG) Um relgio tipo carrilho foi calibrado a 20C. Nessa


temperatura o comprimento da haste de seu pndulo 1,25 m. Se a
temperatura ambiente sofrer um aumento de 20C, o acrscimo aproximado
do comprimento da haste ser, em metros, igual a:

Com base nos dados experimentais fornecidos no grfico, pode-se


afirmar, corretamente, que o valor numrico do coeficiente de dilatao
linear do vidro :
a) 24x106 C1.
b) 18x106 C1.
c) 12x106 C1.
d) 9x106 C1.
e) 6x106 C1.

93

35) Um

recipiente de vidro, cujas paredes so finas, contm glicerina. O


conjunto se encontra a 20C. O coeficiente de dilatao linear do vidro

40) (Mackenzie

SP) Uma massa de gs supostamente ideal, inicialmente a


47 C, sofre uma variao de temperatura de 80 C durante uma
transformao isobrica. O volume dessa massa gasosa, aps esse
5,0.10 4 C 1 . Se a temperatura do conjunto se elevar para 60C, pode- aquecimento, sofreu um aumento, em relao ao seu volume inicial, de
a) 2,5 %
b) 4,0 %
c) 25 %
se afirmar que o nvel de glicerina no recipiente:
d) 40 %
e) 80 %
a) baixa, porque a glicerina sofre um aumento de volume menor do que a
capacidade do recipiente.
b) se eleva, porque a glicerina aumenta de volume e a capacidade do 41) (UFRR RR) Um balo de volume V que se encontra sob uma
atmosfera de presso e a uma temperatura negativa de 33C inflado com
recipiente diminui de volume.
gs Hlio que se encontra num cilindro de 1m3 de volume a uma
c) se eleva, porque apenas a glicerina aumenta de volume.
temperatura de 27C sob uma presso de 20atm. O volume final do balo :
d) se eleva, apesar da capacidade do recipiente aumentar.
3
b) 10 m3
c) 160 ml
e) permanece inalterado, pois a capacidade do recipiente aumenta tanto a) 8 m 3
d) 1,6 m e) 16 m3
quanto o volume da glicerina.

8.10 6 C 1

e o coeficiente de dilatao volumtrica da glicerina

36) (UFU - MG) Um frasco de capacidade para 10 litros est completamente 42) (UFSM)

O diagrama presso x volume representa um processo


cheio de glicerina e encontrase temperatura de 10C. Aquecendose o isotrmico em que um gs ideal levado do estado A ao estado D.
frasco com a glicerina at atingir 90C, observase que 352 ml de glicerina
transborda do frasco. Sabendose que o coeficiente de dilatao volumtrica
da glicerina 5,0 x 104 C1, o coeficiente de dilatao linear do frasco ,
em C1.
a) 6,0 x 105
b) 2,0 x 105
c) 4,4 x 104
d) 1,5 x 104

37) (Uniderp MS) Um recipiente de vidro, com a capacidade de 3000 cm3,

est completamente cheio de lquido, a 0C. O conjunto est aquecido at


100C e observa-se que 15 cm3 desse lquido extravasam do recipiente.
Considerando-se o coeficiente de dilatao linear do vidro como sendo
constante no referido intervalo trmico e igual a 4.10 -6 C-1, o coeficiente de
dilatao real desse lquido, em 10-5 C-1, igual a:
A presso PB do gs, no estado B, vale, em atm:
a) 1,5
b) 4,0
a) 4
b) 7
c) 8
c) 5,0
d) 12
e) 24
d) 5,4
43) (UERN) Um gs perfeito, sob presso constante de 200 N/m 2, recebe
e) 6,2
60 J de calor, sofrendo uma variao de volume de 0,2 m3. A variao de
energia interna sofrida pelo gs, nesse processo, foi de:
38) (Unama PA) A dilatao anmala da gua fundamental para a a) 60 J
b) 50 J
c) 40 J
preservao da vida nas regies de clima frio, em que rios e lagos tm as d) 30 J
e) 20 J
guas superficiais congeladas durante o inverno. A camada superficial de
gelo, isolante trmico, impede o congelamento das guas mais profundas,
permitindo a sobrevivncia da fauna e da flora. Caso a gua no tivesse 44) (PUC RS) Considere as afirmativas a seguir e o grfico Presso (P) x
comportamento anmalo, o congelamento ocorreria por inteiro, inviabilizando Volume (V), que apresenta quatro transformaes de um gs, cujo
a continuidade de vida nas regies. A formao dessa camada se deve ao comportamento o de um gs ideal. Duas das transformaes so
isotrmicas, e o gs est contido em um cilindro com mbolo.
resfriamento da camada superficial que, no intervalo de 4C a 0C:
a) mantm constante o volume e permanece na parte superior.
b) diminui o volume e permanece na parte superior.
c) diminui de densidade e permanece na parte superior.
d) aumenta de densidade e permanece na parte superior.

39) (PUC PR) No grfico a seguir, temos a densidade da gua como funo

Densidade
da gua (g/cm)

da temperatura no intervalo de temperaturas de 0C a 100C, elaborado com


dados experimentais.

1.00000
0.99987

I. A transformao do estado 1 para o estado 2 isomtrica com


aquecimento.
II. Na passagem do estado 2 para o estado 3, no ocorre variao de
temperatura, e o gs realiza trabalho positivo.
III. Na passagem do estado 3 para ao estado 4, h resfriamento do gs e
no h realizao de trabalho, pois a transformao isomtrica.
IV. Na transformao do estado 4 para o estado 1, no h variao da
energia interna do gs, e um agente externo realiza um trabalho sobre ele.

0.95838

100
Temperatura (C)

Das seguintes afirmativas, a INCORRETA :


a) a densidade da gua aumenta com a temperatura no intervalo de 0C
4,0C.
b) no intervalo de temperatura dado, a densidade da gua mxima
temperatura de 4,0C.
c) o volume especfico (volume por unidade de massa) da gua mnimo
temperatura de 4,0C.
d) o volume especfico da gua mximo temperatura de 4,0C.
e) o volume especfico da gua diminui no intervalo de 0C a 4,0C
aumenta no intervalo de 4,0C a 100C.

Esto corretas apenas


a) I e II.
b) II e III.
c) I e IV.
d) III e IV.
e) II, III e IV.

45) (UFES)

A figura mostra a variao de volume de um gs ideal,


presso constante de 4 N/m2, em funo da temperatura. Sabe-se que,
durante a transformao de estado de A a B, o gs recebeu uma quantidade
de calor igual a 20 joules. A variao da energia interna do gs entre os
estados A e B foi de:

94

comprimento do fio A 4 vezes maior que o comprimento do fio B. A relao


entre as freqncias :

a) 4 J
b) 16 J
c) 24 J
d) 380 J
e) 420 J

46) (Unisinos)

O ciclo de Carnot consiste em quatro transformaes: duas


isotrmicas alternadas com duas adiabticas, conforme mostrado no grfico
Presso versus Volume, na figura.
A respeito do ciclo de Carnot, afirma-se que:
I na transformao AB, a energia interna
no varia.
II na transformao BC, h absoro de
calor.
III na transformao CD, a presso e o
volume so inversamente proporcionais.
Das afirmativas acima:
a) apenas I correta.
b) apenas II correta.
c) apenas I e II correta.
d) apenas I e III so corretas.
e) I, II e III so corretas.

47) (Ufam)

Um inventor diz ter desenvolvido uma mquina trmica que,


operando entre duas fontes trmicas, quente e fria, com temperaturas de
500 K e 250 K, respectivamente, consegue, em cada ciclo, realizar uma
quantidade de trabalho equivalente a 75% do calor desenvolvido da fonte
quente, rejeitando 25% da energia gerada por essa fonte. De acordo com as
leis da Termodinmica, possvel que o inventor tenha realmente
desenvolvido tal mquina?
a) No possvel, uma vez que esta mquina teria um rendimento maior do
que uma mquina de Carnot, operando entre as mesmas fontes.
b) No possvel, uma vez que o rendimento da mquina 100%.
c) possvel, uma vez que no violaria a primeira lei da Termodinmica.
d) No possvel, uma vez que violaria a primeira lei da termodinmica.
e) possvel, uma vez que esta mquina teria um rendimento de uma
mquina de Carnot, operando entre as mesmas fontes.

48) (Mackenzie)

Um corpo de 250 g de massa encontra-se em equilbrio,


preso a uma mola helicoidal de massa desprezvel e constante elstica k igual
a 100 N/m, como mostra a figura abaixo.

a)

fA
4
fB

d)

fA
1

fB
2

b)

fA
2
fB

c)

fA
1

fB
4

50) (Cefet PR) Com relao ao estudo das ondas, so feitas as seguintes

afirmaes:
I Quando uma mola passa de um meio material para outro, ocorre o
fenmeno chamado refrao, com alterao da freqncia da onda.
II As ondas sonoras podem ser polarizadas, pois so ondas longitudinais.
III O fenmeno da decomposio da luz branca em seu espectro
denominado disperso.
(so) correta(s) a(s) afirmao(es):
a) I
b) II
c) III
d) I e II
e) I e III

51) (Unicruz)

Observando o mar, de um navio ancorado, um marinheiro


avaliou em 12 m a distncia entre as cristas das ondas, que se sucediam
numa freqncia de 9 Hz. Qual a velocidade de propagao das ondas?
a) 1.3 m/s b) 108 m/s c) 10,8.106 m/s
d) 0,75 m/s
e) 75 m/s

52) (UFAL)

Uma onda peridica se propaga numa corda fina com


velocidade de 8 m/s e comprimento de onda igual a40 cm. Essa onda se
transmite para outra corda grossa onde a velocidade de propagao 6,0
m/s.

Na corda grossa, essa onda peridica tem freqncia em hertz e


comprimento de onda em centmetros, respectivamente, iguais a:
a) 20 e 60
b) 20 e 30
c) 15 e 60
d) 15 e 30
e) 15 e 20
Um trem de ondas planas de comprimento de onda , que
se propaga para direita em uma cuba com gua, incide em um obstculo que
apresenta uma fenda de largura F. Ao passar pela fenda, o trem de ondas
muda sua forma, como se v na figura abaixo.

53) (UFRGS)

O atrito entre as superfcies em contato desprezvel. Estica-se a mola, com


o corpo, at o ponto A, e abandona-se o conjunto, com velocidade zero. Em
um intervalo de 1,0 s, medido a partir desse instante, o corpo retornar ao
ponto A:
a) uma vez.
b) duas vezes
c) trs vezes
d) quatro vezes
e) seis vezes.

49) (Unieube

- MG) A figura mostra dois pndulos simples presos em um


suporte. A massa do corpo A duas vezes maior que a do corpo B, o

95

a) I e III, apenas.
b) II e IV, apenas.
c) I, III e IV, apenas. d) II, III e IV, apenas.
e) I, II, III e IV.

59) (UFMS

- MS) Um grande espelho plano serve como pano de fundo em


um palco de teatro, durante a apresentao de uma dana. A bailarina se
coloca entre o espelho e o pblico, que assiste dana. Um observador do
pblico est em uma posio da qual, num dado momento, v a imagem
refletida da bailarina no espelho e v tambm a bailarina na mesma linha de
seus olhos (veja a figura). Nesse momento, a bailarina se aproxima do
espelho com velocidade V com relao ao palco. Se a bailarina v sua prpria
imagem e tambm a do observador refletida no espelho, correto afirmar
que

Qual o fenmeno fsico que ocorre com a onda quando ela passa pela fenda?
a) Difrao b) Disperso
d) Reflexo e) Refrao

c) Interferncia

54) (FGV

SP) Na tabela a seguir, qual dos itens expressa, corretamente,


caractersticas de uma onda sonora?
Natureza da oscilao Meio de propagao
a)transversal
b)longitudinal
c)transversal
d)longitudinal
e)mista

qualquer, incluindo o vcuo300


qualquer meio material
340
lquidos
340
vcuo
300
lquidos
300

Velocidade no ar
(aproximada)
000 km/s
m/s
m/s
000 km/s
000 km/s

55) (UFSM)

O ouvido humano capaz de perceber vibraes mecnicas


com freqncias que variam entre 20 Hz e 20000 Hz. Sabendo-se que a
velocidade de propagao do som no ar de 340 m/s, os comprimentos de
onda correspondentes s freqncias acima, ou seja, aqueles que limitam as
ondas sonoras percebidas pelos humanos, so, respectivamente:
a) 17 m e 0,017 m
b) 0.017 m e 17 m
c) 6,8.103 m e 6,8.106 m
d) 6,8.106 m e 6,8.101 m
e) 5,9.10-2 m e 5,9.101 m

56) (UFRGS) So exemplos de ondas os raios X, os raios gama, as ondas de

rdio, as ondas sonoras e as ondas de luz. Cada um desses cinco tipos de


onda difere, de algum modo, dos demais.
Qual das alternativas apresenta uma afirmao que diferencia corretamente o
tipo de onda referido das demais ondas acima citadas?
a) Raios X so as nicas ondas que no so visveis
b) Raios gama so as nicas ondas transversais.
c) Ondas de rdio so as nicas ondas que transportam energia
d) Ondas sonoras so as nicas ondas longitudinais
e) Ondas de luz so as nicas ondas que se propagam no vcuo com
velocidade de 300.000 km/s.

57) (UEPB PB) luz do dia, a ilustrao ao lado refere-se a um jarro preto,
contendo flores brancas. Com relao cor desta figura pode-se afirmar que:
a) As flores parecero vermelhas e o jarro preto se, dentro de uma sala
escura, forem iluminados com luz vermelha.
b) As flores e o jarro tero sempre a mesma cor, pois a cor uma
propriedade do corpo, independente da luz que o ilumina.
c) As flores e o jarro parecero pretos se, dentro de uma sala escura, forem
iluminados com luz azul.
d) Num ambiente escuro, as flores e o jarro parecero totalmente brancos, se
iluminados com luz branca.
e) Num ambiente escuro, as flores parecero vermelhas e o jarro parecer
preto, se forem iluminados simultaneamente com luz vermelha e verde.

58) (FGV

SP) O professor pede aos grupos de estudo que apresentem


classe suas principais concluses sobre os fundamentos para o
desenvolvimento do estudo da ptica Geomtrica.
GRUPO I Os feixes de luz podem apresentar-se em raios paralelos,
convergentes ou divergentes.
GRUPO II Os fenmenos de reflexo, refrao e absoro ocorrem
isoladamente e nunca simultaneamente.
GRUPO III Enquanto num corpo pintado de preto fosco predomina a
absoro, em um corpo pintado de branco predomina a difuso.
GRUPO IV Os raios luminosos se propagam em linha reta nos meios
homogneos e transparentes.
So corretas as concluses dos grupos:

a) o observador percebe que a imagem da bailarina, refletida no


aproxima-se dele com velocidade 2V.
b) a bailarina percebe que a imagem do observador, refletida no
aproxima-se dela com velocidade 2V.
c) a bailarina percebe que sua prpria imagem, refletida no
aproxima-se dela com velocidade 2V.
d) a imagem refletida da bailarina no espelho uma imagem real.
e) a distncia da bailarina at o espelho o dobro da distncia da
at sua imagem refletida.

espelho,
espelho,
espelho,
bailarina

60) (UFPel)

Os espelhos planos podem ser associados, isto , colocados


lado a lado em ngulo ou dispostos paralelamente entre si. H a possibilidade
de essas associaes deslocarem ou multiplicarem o nmero de imagens de
um objeto.
Baseado em seus conhecimentos sobre ptica Geomtrica, em relao s
imagens produzidas entre dois espelhos planos em ngulo, correto afirmar
que
a) existe a formao de uma nica imagem, para um ngulo de 180, o que,
na prtica, significa um nico espelho.
b) no haver formao de imagens, quando o ngulo for de 0, j que os
espelhos ficam dispostos paralelamente.
c) a expresso

360
1 no apresenta limitaes, fornecendo o

nmero de imagens para qualquer ngulo entre 0 e 360.


d) haver a formao de 6 imagens, se os espelhos estiverem dispostos
perpendicularmente.
e) podem ser produzidas teoricamente infinitas imagens, desde que os
espelhos fiquem dispostos paralelamente, ou seja,

180 .

61) (Unipar PR) Analise as proposies abaixo:


I. O campo visual de um espelho plano sempre o mesmo, qualquer que
seja a posio do observador.
II. O campo visual de um espelho tanto maior quanto mais prximo de seu
centro estiver o observador.
III. O campo visual de um espelho plano cresce conforme aumenta a
distncia do observador em relao ao espelho.
Podemos afirmar que:
a) somente a proposio I est correta.
b) somente a proposio II est correta.
c) somente a proposio III est correta.
d) as proposies I, II e III esto corretas.
e) as proposies I, II e III esto erradas.

62) (UFAC

AC) Um espelho plano fornece uma imagem de um objeto


situado a uma distncia de 20cm do espelho. Afastando-se o espelho 30cm
numa direo normal ao seu plano, a distncia que separar a antiga imagem
da nova ser:
a) 50cm
b) 30cm
c) 70cm
d) 60cm
e) n.d.a

63) (UESPI PI) Um pequeno objeto real de altura h posicionado na frente


de um espelho plano, a uma distncia d do mesmo (veja figura). Assinale a
alternativa correta com relao imagem fornecida por tal espelho.

96

As imagens fornecidas pelos espelhos convexos


a) so sempre reais, menores e invertidas.
b) so sempre virtuais, maiores e invertidas.
c) so sempre virtuais, menores e direitas.
d) so sempre reais, maiores e direitas.

69) (UFMG MG) Um professor pediu a seus alunos que explicassem por que

um lpis, dentro de um copo com gua, parece estar quebrado, como


mostrado nesta figura:
a) A imagem virtual, tem altura h e est localizada a uma distncia d do
espelho.
b) A imagem real, tem altura h e est localizada a uma distncia d do
espelho.
c) A imagem virtual, tem altura menor que h e est localizada a uma
distncia d/2 do espelho.
d) A imagem real, tem altura maior que h e est localizada a uma distncia
2d do espelho.
e) Independente de sua natureza (real ou virtual), a imagem ter altura h e
estar localizada no foco do espelho.

64) (Unifenas

MG) O armrio do banheiro de minha casa, possui duas


portas espelhadas as quais tm as dobradias no mesmo lugar. Como uma
gira em sentido horrio e a outra, em sentido anti-horrio, abri as duas e
coloquei a cabea entre elas, para tentar observar a minha orelha. Percebi,
ento, que 5 imagens de meu rosto foram formadas. Nesta situao, o
ngulo entre as duas portas igual a
a) 300.
b) 450.
d) 600.
d) 720.
e) 900.

65) (Mackenzie SP) Dispe-se de dois espelhos esfricos, um convexo e um

cncavo, com raios de curvatura 20,0 cm cada um, e que obedecem s


condies de Gauss. Quando um objeto real colocado perpendicularmente
ao eixo principal do espelho convexo, a 6,0 cm de seu vrtice, obtm- se
uma imagem conjugada de 1,5 cm de altura. Para que seja obtida uma
imagem conjugada, tambm de 1,5 cm de altura, colocando esse objeto
perpendicularmente ao eixo principal do espelho cncavo, sua distncia at o
vrtice desse espelho dever ser:
a) 11,0 cm b) 15,0 cm c) 26,0 cm
d) 30,0 cm e) 52,0 cm

Bruno respondeu: Isso ocorre, porque a velocidade da luz na gua menor


que a velocidade da luz no ar.
Toms explicou: Esse fenmeno est relacionado com a alterao da
freqncia da luz quando esta muda de meio.
Considerando-se essas duas respostas, correto afirmar que:
a) apenas a de Bruno est certa.
b) apenas a de Toms est certa.
c) as duas esto certas.
d) nenhuma das duas est certa.

70) (UEPB

PB) Em 1621, o cientista holands Willebrord van Roijen SNELL


(1591 - 1626) investigou o fenmeno fsico da propagao da luz em
diversos meios, e estabeleceu, baseado na evidncia experimental, a lei que
levou o seu nome Lei de Snell ou Lei da Refrao. Considere esta lei
aplicada seguinte situao: O ndice de refrao absoluto (n) de um meio
material (conforme a figura) definido como sendo a razo entre a
velocidade da luz no meio 1 e a velocidade da luz no meio 2.

66) (Unifor

CE) Um espelho esfrico projeta sobre uma tela a imagem de


uma pequena vela acesa, ampliada 5 vezes. A distncia da vela at a tela
de 6,0 m. Nestas condies, o raio de curvatura do espelho, em metros, vale
a) 3,0
b) 2,5
c) 2,0
d) 1,5
e) 1,0

A tabela a seguir relaciona o ndice de refrao para sete meios materiais


diferentes. Se necessrio, adote c = 3.108 m/s.

Meio material ndice de refrao


Vcuo
1,0000

67) (PUC SP) Um objeto colocado a 30cm de um espelho esfrico cncavo

perpendicularmente ao eixo ptico deste espelho. A imagem que se obtm


classificada como real e se localiza a 60cm do espelho. Se o objeto for
colocado a 10cm do espelho, sua nova imagem
a) ser classificada como virtual e sua distncia do espelho ser 10cm.
b) ser classificada como real e sua distncia do espelho ser 20cm.
c) ser classificada como virtual e sua distncia do espelho ser 20cm.
d) aumenta de tamanho em relao ao objeto e pode ser projetada em um
anteparo.
e) diminui de tamanho em relao ao objeto e no pode ser projetada em
um anteparo.

Ar
gua

1,0003
1,3300

lcool etlico
leo
Vidro (crown)
Vidro (flint)

1,3600
1,4800
1,5000
1,6600

68) (UEG

GO) Por possuir a propriedade de ampliar o campo visual do


observador, os espelhos esfricos apresentam vrias aplicaes.

Com base nessa tabela, correto afirmar que:


a) a velocidade da luz no se altera quando muda de meio.
b) a velocidade da luz no vidro (crown) a mesma que no vidro (flint).
c) o ar o meio onde a luz apresenta maior velocidade.
d) o vidro (flint) o meio onde a luz viaja mais rpido do que no leo.
e) na gua a luz viaja mais rpido do que no lcool etlico.

71) (Unesp

SP) O ndice de refrao absoluto de um determinado material


encontrado fazendo uma relao entre a velocidade da luz no vcuo e no
material. Considerando o ndice de refrao da gua como sendo,
aproximadamente, 1,3 e a velocidade da luz no vcuo como sendo 3,0x10 8
m/s, a melhor estimativa para a velocidade da luz na gua
a) 0,4x108 m/s.
b) 0,9x108 m/s.

97

c) 2,3x108 m/s.
e) 3,9x108 m/s

d) 3,0x108 m/s.

75) (UFU

72) (Mackenzie SP) Quando um raio de luz monocromtica, proveniente de

MG) Um pescador, ao observar um peixe dentro da gua, sabe


que deve atirar com o arpo alguns centmetros abaixo da posio do peixe
observada por ele, para acertlo.

um meio homogneo, transparente e istropo, identificado por meio A, incide


sobre a superfcie de separao com um meio B, tambm homogneo,
transparente e istropo, passa a se propagar nesse segundo meio, conforme
mostra a ilustrao ao lado. Sabendo-se que o ngulo menor que o
ngulo , podemos afirmar que:

a) no meio A a velocidade de propagao da luz menor que no meio B.


b) no meio A a velocidade de propagao da luz sempre igual velocidade
no meio B.
c) no meio A a velocidade de propagao da luz maior que no meio B.
d) no meio A a velocidade de propagao da luz maior que no meio B,
somente se o ngulo limite de incidncia.
e) no meio A a velocidade de propagao da luz maior que no meio B,
somente se o ngulo limite de refrao.

73) (FGV

SP) Um feixe de luz monocromtica, proveniente de um meio


ptico A, incide sobre a superfcie de separao desse meio com um meio
ptico B. Aps a incidncia, o raio segue por entre os dois meios, no
refletindo nem penetrando o novo meio.

Isso ocorre porque:


a) a luz proveniente do peixe que atinge o olho do pescador sofre uma
refrao ao sair da gua e entrar no ar
b) a luz, ao entrar na gua, sofre uma disperso, separando os diferentes
comprimentos de onda (diferentes cores) de forma a enganar o pescador
sobre a posio real do peixe.
c) a gua funciona com uma lente e, portanto, a imagem do objeto nem
sempre real
d) a gua funciona como um espelho cncavo, devido ao movimento
ondulatrio de sua superfcie, fazendo com que a imagem seja virtual e no
real

76) (ITA

SP) Um pescador deixa cair uma lanterna acesa em um lago a


10,0 m de profundidade. No fundo do lago, a lanterna emite um feixe
luminoso formando um pequeno ngulo com a vertical (veja figura).

Considere: tg
sen
e o ndice de refrao da gua n = 1,33.
Ento, a profundidade aparente h vista pelo pescador igual a:
a) 2,5 m b) 5,0 m c) 7,5 m
d) 8,0 m e) 9,0 m

Com relao a esse acontecimento, analise:


I. O meio ptico A tem um ndice de refrao maior que o meio ptico B.
II. Em A, a velocidade de propagao do feixe maior que em B.
III. Se o ngulo de incidncia (medido relativamente normal superfcie de
separao) for aumentado, o raio de luz reflete, permanecendo no meio A.
IV. Se o raio de luz penetrasse o meio B, a freqncia da luz monocromtica
diminuiria.
Est correto o contido apenas em:
a) I e III. b) II e III. c) II e IV.
d) I, II e IV.
e) I, III e IV.

77) (Unifor)

Um prisma de vidro tem ngulo de refringncia A=60 e seu


ndice de refrao, em relao ao ar, para a luz monocromtica amarela vale

2 . Um raio luminoso amarelo, no ar, incide em uma das faces do prisma


sob ngulo de incidncia de 45. O desvio sofrido pelo raio ao emergir do
prisma vale
Dados:
sen 30 = cos 60 = 1/2
sen 45 0 cos 45 0
0

74) (Fatec

SP) O esquema abaixo representa a direo de um feixe


luminoso monocromtico incidente e as direes dos respectivos feixes
refletido e refratado.

2 /2

sen 60 cos 30 3 / 2
a) 0
d) 60

b) 30
e) 90

c) 45

78) (Mackenzie

Sabendo-se que o ngulo de reflexo vale 60, que o ndice de refrao do


meio A vale 1 e que o do meio B vale

correto afirmar que o ngulo de

refrao vale
Dados:

sen 30 o cos 60 o

1
2

sen 60 o cos 30 o

3
2

a) 15
d) 60

b) 30
e) 90

c) 45

SP) Uma lente delgada convergente tem distncia focal de


20 cm. Para se obter uma imagem conjugada de um objeto real, maior que o
prprio objeto e no invertida, esse dever ser colocado sobre o eixo
principal da lente,
a) a 40 cm do centro ptico.
b) a 20 cm do centro ptico.
c) a mais de 40 cm do centro ptico.
d) entre 20 cm e 40 cm do centro ptico.
e) a menos de 20 cm do centro ptico.

79) (Mackenzie

SP) Uma lente esfrica delgada de convergncia 10 di


utilizada para obter a imagem de um objeto de 15cm de altura. A distncia, a
que o objeto deve estar do centro ptico da lente, para se obter uma imagem
invertida de 3cm de altura, de:
a) 60cm
b) 50cm
c) 42cm
d) 24cm
e) 12cm

98

80) (UFRGS)

A figura abaixo representa um objeto real 0 colocado diante


de uma lente delgada de vidro, com pontos focais F 1 e F2. O sistema todo
est imerso no ar.

I. No olho mope, a imagem ntida se forma atrs da retina, e esse defeito da


viso corrigido usando uma lente divergente.
II. No olho com hipermetropia, a imagem ntida se forma atrs da retina, e
esse defeito da viso corrigido usando uma lente convergente.
III. No olho com astigmatismo, que consiste na perda da focalizao em
determinadas direes, a sua correo feita com lentes cilndricas.
IV. No olho com presbiopia, ocorre uma dificuldade de acomodao do
cristalino, e esse defeito da viso corrigido mediante o uso de uma lente
divergente.
Est(o) correta(s) apenas a(s) afirmativa(s)
a) I e II.
b) III.
c) II e IV.
d) II e III. e) I e IV.

Nessas condies, a imagem do objeto fornecida pela lente


a) real, invertida e menor que o objeto
b) real, invertida e maior que o objeto
c) real, direta e maior que o objeto
d) virtual, direta e menor que o objeto
e) virtual, direta e maior que o objeto

83) (UEM

PR) O diagrama abaixo representa as linhas de um campo


magntico uniforme.

81) (UFSM)

A figura que melhor representa o comportamento refrativo de


um olho sadio

Assinale a alternativa que melhor representa a posio da agulha de uma


bssola colocada em um ponto P, no mesmo plano do campo magntico.

a)

b)

84) (UDESC) Considere as seguintes afirmativas:


c)

d)

I. A experincia de Hans Christian Oersted comprovou que um eltron


desviado, ao se deslocar em um campo magntico, na mesma direo do
campo.
II. Ao partirmos um m ao meio, separamos o plo Norte magntico do plo
Sul magntico, dando origem a dois novos ms monopolares.
III. Quando uma partcula carregada desloca-se paralelamente ao vetor
campo magntico, a fora magntica sobre ela nula.
Assinale a alternativa correta.
a) Somente as afirmativas I e II so verdadeiras.
b) Somente as afirmativas I e III so verdadeiras.
c) Somente a afirmativa III verdadeira.
d) Somente as afirmativas II e III so verdadeiras.
e) Todas as afirmativas so verdadeiras.
85) (UEPB PB) Na segunda dcada do Sculo XIX, o fsico dinamarqus
Hans Christian Oersted (1777-1851), constatou que ao aproximar uma
bssola de um fio percorrido por uma corrente eltrica, sua agulha sofria um
desvio. Da concluiu: toda corrente eltrica gera no espao que a envolve
um campo magntico. Considere a permeabilidade magntica para o vcuo,

0 4.107

e)

82) (UFPel

INV 2007) O olho humano um sofisticado sistema ptico


que pode sofrer pequenas variaes na sua estrutura, ocasionando os
defeitos da viso.
Com base em seus conhecimentos, analise as afirmativas abaixo.

T.m /A.

Sobre o eletromagnetismo correto afirmar que:


a)
a intensidade do campo magntico no interior de uma espira
circular de raio 2,5 cm, quando percorrida por uma corrente de 4,0 A, de
2.105 T;
b)
a intensidade do campo magntico produzido por uma corrente
eltrica de 3,0 A que percorre um fio metlico reto e extenso, distante de
0,25 m de 1,2.106 T;
c)
a direo do campo magntico no centro de uma espira circular
perpendicular ao plano da espira;
d)
um condutor percorrido por uma corrente i, tem num ponto P um
vetor induo magntico B com o sentido mostrado na figura abaixo:

99

86) (UFAM AM) Duas espiras concntricas e coplanares de raios R e 2R so


percorridas por uma corrente i, como mostra a figura abaixo.

O vetor induo magntica resultante no centro O das espiras


perpendicular ao plano da figura e de intensidade:
a)
b)
c)
d)

e)

0i
, orientado para fora
2R
0i
, orientado para fora
4R
30i
, orientado para fora
4R
0i
, orientado para fora
4R
0i
, orientado para dentro
2R

87) (FMTM

MG) O texto discursa sobre os efeitos causados no campo


magntico resultante devido imantao de substncias inseridas em um
campo magntico uniforme.
As substncias __________ so conhecidas por provocarem um pequeno
aumento no valor do campo resultante.
Quanto s substncias __________ ocorre o contrrio, visto que seus ms
elementares opem-se ao campo original.
J com relao s substncias _____________, devido fortssima
imantao, o campo gerado supera em muitas vezes aquele na qual a
substncia foi inserida.
(Alberto Gaspar, Fsica vol. 2, adaptado.)
As substncias que preenchem corretamente as lacunas so,
respectivamente,
a) diamagnticas .... paramagnticas .... ferromagnticas.
b) paramagnticas .... ferromagnticas .... diamagnticas.
c) paramagnticas .... diamagnticas .... ferromagnticas.
d) ferromagnticas .... diamagnticas .... paramagnticas.
e) ferromagnticas .... paramagnticas .... diamagnticas.
88) (ITA SP) Uma espira retangular colocada em um campo magntico
com o plano da espira perpendicular direo do campo, conforme mostra a
figura. Se a corrente eltrica flui no sentido mostrado, pode-se afirmar em
relao resultante das foras, e ao torque total em relao ao centro da
espira, que

a) A resultante das foras no zero, mas o torque total zero.


b) A resultante das foras e o torque total so nulos.
c) O torque total no zero, mas a resultante das foras zero.
d) A resultante das foras e o torque total no so nulos.
e) O enunciado no permite estabelecer correlaes entre as grandezas
consideradas.

Considerando a ao de foras magnticas sobre cargas eltricas em


movimento uniforme, e as trajetrias de cada partcula ilustradas na figura,
pode-se concluir com certeza que
a) as partculas 1 e 2, independentemente de suas massas e velocidades,
possuem necessariamente cargas com sinais contrrios e a partcula 3
eletricamente neutra (carga zero).
b) as partculas 1 e 2, independentemente de suas massas e velocidades,
possuem necessariamente cargas com sinais contrrios e a partcula 3 tem
massa zero.
c) as partculas 1 e 2, independentemente de suas massas e velocidades,
possuem necessariamente cargas de mesmo sinal e a partcula 3 tem carga e
massa zero.
d) as partculas 1 e 2 saram do recipiente com a mesma velocidade.
e) as partculas 1 e 2 possuem massas iguais, e a partcula 3 no possui
massa.

90) (UFPE PE) Uma partcula com carga q = 3,2 x 10-19C e massa m = 3,2 x

10-30 kg desloca-se em uma regio de campo magntico uniforme com B =


0,05 T, descrevendo uma trajetria circular de raio r = 5,0 mm. Determine a
velocidade da partcula em m/s.
a) 1,2 x 107
b) 1,5 x 107
c) 2,3 x 107
d) 2,5 x 107
e) 3,2 x 107

91) (UEL

PR) Uma maneira de produzir corrente eltrica a partir de um


campo magntico , por exemplo, movimentando um im dentro de uma
bobina ou selenide, princpio dos geradores eltricos e dnamos. Considere a
figura, que mostra um m sendo empurrado perpendicularmente ao centro
de uma espira de cobre fechada. Sobre essa situao, indique a afirmativa
correta:

a) Ser gerada na espira uma corrente eltrica no sentido anti-horrio,


somente se o movimento do m for acelerado.
b) O m sofrer uma fora de resistncia ao seu movimento devido a uma
corrente eltrica induzida na espira no sentido anti-horrio.
c) Se o m for empurrado com os plos invertidos em relao figura, ele
no sofrer uma fora de resistncia ao seu movimento e surgir na espira
uma corrente eltrica induzida no sentido horrio.
d) Se deslocarmos a espira em torno do m, agora parado, uma corrente
eltrica induzida circular sobre a espira de cobre no sentido horrio e far
surgir uma fora de resistncia ao movimento da espira.
e) Independentemente do sentido, a corrente induzida ser sempre horria
enquanto o m estiver em movimento atravs da espira.

92) (FMTM

MG) Um m em forma de barra aproxima-se de uma espira,


conforme indica a figura.

89) (Unesp

SP) Uma mistura de substncias radiativas encontra-se


confinada em um recipiente de chumbo, com uma pequena abertura por
onde pode sair um feixe paralelo de partculas emitidas. Ao sarem, trs tipos
de partcula, 1, 2 e 3, adentram uma regio de campo magntico uniforme B
com velocidades perpendiculares s linhas de campo magntico e descrevem
trajetrias conforme ilustradas na figura.

100

d) O sentido da corrente induzida o indicado pela seta (2) e o plo norte do


m repelido pela extremidade A da bobina.
e) O sentido da corrente induzida o indicado pela seta (2) e sobre o plo
norte do m no atua nenhuma fora.

94) (UDESC)

O transformador um dispositivo usado para transformar a


tenso. Por exemplo, um aparelho que funciona com tenso de 110 V
ligado a um transformador que, por sua vez, ligado em uma tomada em
que a tenso 220 V. Diz-se que esse transformador possui tenso de
entrada 220 V e tenso de sada 110 V.

Nessas condies, a espira se comporta como um m posicionado


espacialmente conforme o indicado em:
a)

b)

A figura mostra, esquematicamente, um transformador: o circuito primrio


constitudo de N1 espiras enroladas em torno de um ncleo de ferro, e o
circuito secundrio com N2 espiras enroladas em torno do mesmo ncleo; a
tenso de entrada U1 e a tenso de sada U2.
Considere as afirmativas abaixo, com relao ao transformador acima.
c)

d)

e)

93) (UFAM AM) A lei de Lenz estabelece que o sentido da corrente induzida

num circuito aquele que tende a se opor variao do fluxo do campo


magntico que a produz. Considere a situao indicada na figura, que mostra
o plo norte de um m aproximando-se da extremidade A de uma bobina
AB. Usando a lei de Lenz, correto afirmar que:

a) O sentido da corrente induzida o indicado pela seta (2) e o plo norte do


m atrado pela extremidade A da bobina.
b) O sentido da corrente induzida o indicado pela seta (1) e o plo norte do
m repelido pela extremidade A da bobina.
c) O sentido da corrente induzida o indicado pela seta (1) e o plo norte do
m atrado pela extremidade A da bobina.

I. Quando uma corrente eltrica varivel percorre o circuito primrio, um


fluxo magntico produzido no ncleo de ferro.
II. A tenso induzida no circuito secundrio obedece relao U1 . N1 = U2 .
N2.
III. De acordo com o princpio de conservao de energia, a potncia no
circuito primrio igual potncia no circuito secundrio.
IV. O sentido da corrente induzida no circuito secundrio tal que o campo
magntico criado por ela tende a contrariar a variao do fluxo magntico
que a originou.
Assinale a alternativa correta.
a) Somente as afirmativas I e IV so verdadeiras.
b) Somente as afirmativas I, II e IV so verdadeiras.
c) Somente as afirmativas I, III e IV so verdadeiras.
d) Somente as afirmativas I e II so verdadeiras.
e) Somente as afirmativas III e IV so verdadeiras.

1) C
6) C
11) D
16) C
21) E
26) B
31) B
36) B
41) E
46) D
51) B
56) D
61) B
66) B
71) C
76) C
81) A
86) D
91) B

2) B
7) C
12) B
17) E
22) E
27) E
32) B
37) E
42) C
47) A
52) B
57) A
62) D
67) C
72) C
77) B
82) D
87) C
92) B

3) C
8) D
13) C
18) A
23) B
28) E
33) D
38) C
43) E
48) C
53) A
58) C
63) A
68) C
73) A
78) E
83) B
88) B
93) B

4) A
9) B
14) E
19) D
24) B
29) C
34) D
39) D
44) E
49) D
54) B
59) C
64) C
69) A
74) B
79) A
84) C
89) A
94) C

5) A
10) D
15) C
20) E
25) E
30) A
35) D
40) C
45) B
50) C
55) A
60) A
65) C
70) E
75) A
80) D
85) C
90) D

101

1) (UEPB

PB) As primeiras descobertas das quais se tem notcia,


relacionadas com fenmenos eltricos, foram feitas pelos gregos, na
Antiguidade. O filsofo e matemtico Thales, que vivia na cidade de Mileto do
sculo VI a.C., observou que um pedao de mbar adquiria a propriedade de
atrair corpos leves
(Alvarenga B. e Mximo A. Curso de fsica, vol. 2. So Paulo: Scipione, 2000).

e) I, II e III.

Modernamente sabe-se que todas as substncias podem ser eletrizadas ao


serem atritadas com outra substncia. Abaixo tmse algumas afirmaes
relacionadas ao processo de eletrizao.

de 30,00cm. Ao se dobrar a distncia entre as esferas, a fora de interao


eletrosttica tem intensidade 3,6 N.
Cada uma dessas esferas est eletrizada com carga de

I. Estando inicialmente neutros, atrita-se um basto de plstico com l,


conseqentemente esses dois corpos adquirem cargas eltricas de mesmo
valor e sinais opostos.
II. Um corpo, ao ser eletrizado, ganha ou perde eltrons.
III. Num dia seco, ao pentear os cabelos, o pente utilizado atrai pedacinhos
de papel, por ser o pente um condutor eltrico.

a)

Das afirmativas acima


a) esto corretas I e III.
b) esto corretas I e II.
c) esto corretas II e III.
d) apenas I correta.
e) apenas III correta.

2) (UFRGS)

Duas pequenas esferas metlicas idnticas e eletricamente


isoladas, X e Y, esto carregadas com cargas eltricas +4 C e -8 C,
respectivamente. As esferas X e Y esto separadas por uma distncia que
grande em comparao com seus dimetros. Uma terceira esfera Z, idntica
s duas primeiras, isolada e inicialmente descarregada, posta em contato,
primeiro, com a esfera X e, depois, com a esfera Y.
As cargas eltricas finais nas esferas X, Y e Z so, respectivamente,
a) +2 C, -3 C e -3 C
b) +2 C, +4 C e -4 C
c) +4 C, 0
e -8 C
d) 0,
-2 C e -2 C
e) 0,
0
e -4 C

3) (UFPR PR) Dois corpos metlicos so aproximados entre si. Um deles (A)
tem forma de paraleleppedo e neutro. O outro (B) esfrico e est
carregado eletricamente com carga de mdulo Q. Aps atingido o equilbrio,
os corpos ficam posicionados conforme ilustrado na figura abaixo. Despreze
as interaes eltricas com os demais componentes do sistema.

5) (Mackenzie SP)
( Q1 Q 2 Q )

Duas pequenas esferas eletrizadas com cargas idnticas


interagem

(k 0 9 10 N m / C )
9

mutuamente

no

ar

quando esto separadas, uma da outra, cerca

6 , 0 C

b) 12C
c) 18C
d)

24C

e)

36C

6) (Unimontes MG) A figura abaixo ilustra um sistema em equilbrio no qual

duas esferas de massa M, carregadas com carga positiva Q, esto em


vrtices de um tringulo issceles em que o ngulo entre os dois lados iguais

2 90 . A alternativa em que a relao entre D e as grandezas K


(constante eltrica), Q, M e g (mdulo da acelerao da gravidade) est
CORRETA

a)

D 2Q K / Mg

b)

D 2K Q / Mg

c)

D Q K / Mg

d)

KQ
Mg

7) (UEL PR) Trs partculas carregadas positivamente, cada uma com carga

q, ocupam os vrtices de um tringulo retngulo cujos catetos so iguais e


medem d. Sabendo-se que as cargas esto num meio cuja constante
dieltrica k, a fora eltrica resultante sobre a carga do ngulo reto dada
pela expresso:
a)

d)

b) O ngulo independe das massas de A e de B.


c) possvel identificar o sinal da carga eltrica em excesso na esfera.
d) O ngulo independe da carga em excesso contida na esfera.
e) A nica fora que atua na esfera a fora eltrica.

4) (Unifor

CE) Uma pequena esfera condutora est suspensa por um fio


isolante. Um basto de vidro aproximado da esfera e verifica-se que ela
atrada.
So feitas as seguintes afirmaes:
I. O basto e a esfera esto eletrizados com cargas de sinais opostos.
II. O basto est eletrizado, mas a esfera est neutra.
III. O basto est neutro, mas a esfera est eletrizada.
Pode estar correto o que se afirma em:
a) I, somente.
b) I e II, somente.
c) I e III, somente.
d) II e III, somente.

2d 2
2 k q2
d2

b)

e)

2 k q2
2d2

c)

k q2
d2

2 k q2
d2

8) (UFF

RJ) Uma barra de 30,0 cm de comprimento isolante e de peso


desprezvel, est sustentada por um apoio em torno do qual pode girar. Na
extremidade direita da barra est presa uma carga positiva q 1, de mdulo
3,00 x 10-6 C. Acima de q1, a uma distncia vertical de 10,0 cm, fixada uma
carga positiva q2, de mdulo 6,00 x 10-6 C.
Dado: Constante eletrosttica k = 9,00 x 109 Nm2/C.
q
+

F
q
+

x 10cm x

Assinale a alternativa correta.


a) As faces a e b do corpo A adquirem concentraes de cargas de sinais
contrrios.

k q2

/////////////////////////////////////////////////////////
x
x
x
10cm
20cm
A intensidade da fora vertical F necessria para manter a barra em equilbrio
:
a) 4,86 x 102 N
b) 3,24 x 101 N
c) 1,62 x 103 N
d) 1,62 x 102 N
e) 3,24 x 102 N

9) (Fatec

SP) Considere uma carga positiva Q de 4,0 C, no ar, e um


ponto M a 20 cm de distncia desta carga, Dentre as alternativas seguintes, a
que contm as informaes corretas sobre a intensidade, direo e sentido
do campo eltrico em M, devido a Q, :

102

(Dado: constante eletrosttica = 9.109 N.m2/C2)


I. Tomando-se a mesma carga eltrica, isolada de outra qualquer, entre os
Intensidade
Direo
Sentido
mdulos do campo eltrico e do potencial eltrico em um mesmo ponto do
(N/C)
9,0 101
a)
Linha reta que une Q e M
De Q para Mespao, o primeiro sofre uma diminuio mais rpida que o segundo,
se
aumenta
a
distncia
at
a
carga.
9,0 105
b)
Linha reta que une Q e M
De Q para Mconforme
II. Comparativamente, a estrutura matemtica do clculo da fora eltrica e
Tangente linha circular de centro em Q
5
9,0 10
c)
Horrio da fora gravitacional so idnticas. Assim como as cargas eltricas esto
e de raio QM
1,8 105
d)
Linha reta que une Q e M
De M para Qpara as massas, o campo eltrico est para a acelerao da gravidade.
III. Uma diferena entre os conceitos de campo eltrico resultante e potencial
Tangente linha circular de centro em Q
1,8 105
e)
Anti-horrioeltrico resultante que o primeiro obtm-se vetorialmente, enquanto o
e de raio QM
segundo

obtido
por
uma
soma aritmtica de
escalares.
10) (PUC SP) Seis cargas eltricas puntiformes se encontram no vcuo fixas correto o contido em:
nos vrtices de um hexgono regular de lado . As cargas tm mesmo a) I, apenas
b) II, apenas
mdulo, |Q|, e seus sinais esto indicados na figura.
c) I e III, apenas
d) II e III, apenas
e) I, II e III
Dados:

14) (UFAC)

Deseja-se estudar o potencial eltrico de uma carga negativa


nos pontos A e B, no vcuo, obtendo-se o potencial VA no ponto A
distncia dA
da carga, e o potencial VB no ponto B distncia dB da carga. Sabe-se que dB
o triplo de dA.
Ento, pode-se dizer que, em valores absolutos, o potencial no ponto A :

k0 = 9,0109 Nm2/C2
= 3,0101 cm; |Q| = 5,0105 C

No centro do hexgono, o mdulo e o sentido do vetor campo eltrico


resultante so, respectivamente,
a) 5,0106 N/C; de E para B.
b) 5,0106 N/C; de B para E.
c) 5,0106 N/C; de A para D.
d) 1,0107 N/C; de B para E.
e) 1,0107 N/C; de E para B.

11) (Mackenzie

SP) Nos vrtices A e C do quadrado abaixo colocam-se


cargas eltricas de valor +q. Para que no vrtice D do quadrado o campo
eltrico tenha intensidade nula, a carga eltrica que deve ser colocado no
vrtice B deve ter o valor:

a)
d)

2q

b)

2q

c)

3 2
q
2

2 2q e) 2 2q

12) (Mackenzie

SP) Um pequeno corpo, de massa m gramas e eletrizado


com carga q coulombs, est sujeito ao de uma fora eltrica de
intensidade igual de seu prprio peso. Essa fora se deve existncia de
um campo eltrico uniforme, paralelo ao campo gravitacional, tambm
suposto uniforme na regio onde as observaes foram feitas. Considerando
que tal corpo esteja em equilbrio, devido exclusivamente s aes do campo
eltrico

a) 3 vezes maior do que o potencial no ponto B.


b) 3 vezes menor do que o potencial no ponto B
c) 9 vezes maior do que o potencial no ponto B
d) 9 vezes menor do que o potencial no ponto B
e) n.d.a.

15) (UFPE) O grfico mostra o potencial eltrico em funo da distncia ao

centro de uma esfera condutora carregada de 1,0 cm de raio, no vcuo. O


potencial eltrico a 3,0 cm do centro da esfera, em volts, vale:

a) 50 V
b) 54 V
c) 60 V
d) 62 V
e) 76 V
16) (PUC MG) A figura mostra um campo eltrico uniforme e trs
superfcies equipotenciais, representadas por A, B e C. Considerando-se o
mdulo do campo eltrico como
necessrio para se levar uma carga

4,0 x 10 2 V / m ,
-6

q 1,0 x 10 C

ento o trabalho
do ponto 2 at o

ponto 6 pela trajetria retilnea 2 5 6 ser de:

e do campo gravitacional (g = 10 m/s2), podemos afirmar que a

intensidade do vetor campo eltrico :


a)

E 1,0.10 2

c)

E 1,0.104

e)

m
N/C
q

m
N/C
q
q
E 1,0.101 N / C
m

b)

d)

E 1,0.101
E 1,0.102

m
N/C
q
q
N/C
m

13) (FGV-SP) Com respeito eletrodinmica, analise:


a) W = 4,0 x 104 J

103

b) W = 1,0 x 104 J
c) W = 6,0 x 105 J
d) W = 8,0 x 105 J

17) (UFU

MG)

equipotenciais,

a) 33 dias. b) 38 dias. c) 46 dias.


d) 54 dias. e) 61 dias.

Na

A- E

figura

abaixo,

so

apresentadas

cinco

linhas

, com os respectivos valores do potencial eltrico.

Inicialmente, um aglomerado de partculas com carga total igual a 2,0 C est


sobre a equipotencial A. Esse aglomerado deslocado para a equipotencial B.
Em B o aglomerado sofre uma mudana estrutural e sua carga passa de 2,0
C para 1,5 C. Esse novo aglomerado de 1,5 C deslocado para a
equipotencial C e, em seguida, para D, conservando-se a carga de 1,5 C. Em
D ocorre uma nova mudana estrutural e sua carga passa para 1,0 C. Por
ltimo, esse aglomerado de 1,0 C deslocado para a equipotencial E.
Considerando as afirmaes apresentadas no enunciado acima, assinale a
alternativa que corresponde ao trabalho realizado sobre o aglomerado para
desloc-lo de A para E.
a) 12 J
b) 16 J
c) 8 J
d) 10 J

18) (Unifor)

Um condutor esfrico isolado, eletrizado positivamente com


carga Q, est em equilbrio eletrosttico. Considere, para essa situao, as
afirmativas abaixo.
I A carga Q, em excesso no condutor, se concentra no seu centro.
II O mdulo do vetor campo eltrico no interior do condutor constante e
no-nulo.
III O mdulo do potencial eltrico no interior do condutor constante e
no-nilo.
Dessas afirmativas, apenas:
a) I correta.
b) II correta.
c) III correta.
d) I e II so corretas.
e) I e III so corretas.

21) (UFJF MG) Sabe-se que diferenas de potencial relativamente pequenas

podem causar graves danos ao corpo humano, dependendo da resistncia


oferecida por este passagem da corrente eltrica. O valor dessa resistncia
pode variar entre, aproximadamente, 100 k, para a pele seca, e cerca de
1,00 k, para a pele molhada. Freqentemente, pessoas levam choques
eltricos ao tocarem em chuveiros mal instalados. O choque, que pode ser
inofensivo, se a pessoa estiver com o corpo seco, pode tambm ter graves
conseqncias, e at levar morte, quando o corpo estiver molhado.
As correntes que passariam pelo corpo da pessoa que fosse submetida a uma
diferena de potencial de 120 V com o corpo seco ou com o corpo molhado
seriam, respectivamente:
a) 120 A e 1,20 105 A
b) 120 mA e 1,20 mA
c) 120 A e 1,20 A
d) 1,20 mA e 1,20 103 A
e) 1,20 mA e 120 mA

22) OBSERVAO

Na questo, adote para g, acelerao da gravidade na


superfcie da Terra, o valor de 10 m/s2; para a massa especfica (densidade)
da gua, o valor de 1000 kg/m3 = 1 g/cm3; para o calor especfico da gua, o
valor de 1,0 cal /(g C); para uma caloria, o valor de 4 joules.
(UFF RJ) Um aquecedor eltrico, cujo elemento fundamental um resistor,
foi projetado para funcionar ligado a uma diferena de potencial de 220V e
aquece uma certa quantidade de gua de 20 C a 80 C em 4 minutos.
Assinale a temperatura final da gua, caso este aquecedor seja ligado a uma
diferena de potencial de 110V e usado para aquecer a mesma quantidade
de gua, inicialmente a 20 C, durante os mesmos 4 minutos.
a) 35 C
b) 40 C
c) 50 C
d) 65 C
e) 80 C

23) (Unifesp

SP) Um consumidor troca a sua televiso de 29 polegadas e


70 W de potncia por uma de plasma de 42 polegadas e 220 W de potncia.
Se em sua casa se assiste televiso durante 6,0 horas por dia, em mdia,
pode-se afirmar que o aumento de consumo mensal de energia eltrica que
essa troca vai acarretar , aproximadamente, de
a) 13 kWh. b) 27 kWh. c) 40 kWh.
d) 70 kWh. e) 220 kWh.

24) (PUC PR) Observe o grfico:

19) (UFSCar

SP) O capacitor um elemento de circuito muito utilizado em


aparelhos eletrnicos de regimes alternados ou contnuos. Quando seus dois
terminais so ligados a uma fonte, ele capaz de armazenar cargas eltricas.
Ligando-o a um elemento passivo como um resistor, por exemplo, ele se
descarrega. O grfico representa uma aproximao linear da descarga de um
capacitor.

O comportamento de R1 e R2 no se altera para valores de ddp at 100 V. Ao


analisar este grfico, um aluno concluiu que, para valores abaixo de 100 V:
I. A resistncia de cada um dos condutores constante, isto , eles so
hmicos.
II. O condutor R1 tem resistncia eltrica maior que o condutor R2.
III. Ao ser aplicada uma ddp de 80 V aos extremos de R 2, nele passar uma
corrente de 0,8 A.
Quais as concluses corretas?
a) Apenas I e III.
c) Apenas II e III.
e) Todas.

b) Apenas II.
d) Apenas I.

25) (Unifesp SP) Voc constri trs resistncias eltricas, RA, RB e RC, com
o fios de mesmo comprimento e com as seguintes caractersticas:
nmero de portadores de carga que fluram durante essa descarga est mais I. O fio de RA tem resistividade 1,0x106 .m e dimetro de 0,50 mm.
prximo de
II. O fio de RB tem resistividade 1,2x106 .m e dimetro de 0,50 mm.
a) 1017.
b) 1014.
c) 1011.
8
5
III. O fio de RC tem resistividade 1,5x106 .m e dimetro de 0,40 mm.
d) 10 .
e) 10 .
Pode-se afirmar que:
b) RB > RA > RC.
20) (FMTM MG) Aps um ms de incansveis ... apaga a luz!..., ... desliga o a) RA > RB > RC.
d) RC > RA > RB.
chuveiro!... a esposa comunica ao marido a reduo de 130 kWh no consumo c) RB > RC > RA.
mensal de energia. No dando o brao a torcer, o marido atribui ao sucesso e) RC > RB > RA.
da economia o fato de no mais se ter deixado acesa durante a noite aquela 26) (Mack) Entre os pontos A e B do trecho de circuito eltrico abaixo, a
lmpada de 100 W do corredor, que sua esposa achava indispensvel ficar ddp 80 V. A potncia dissipada pelo resistor de resistncia 4 :
acesa. Apesar de o no uso dessa lmpada ter contribudo para a economia
obtida, ela jamais poderia ter sido a nica responsvel, uma vez que, com a
energia economizada, essa lmpada poderia permanecer ininterruptamente
acesa por, aproximadamente,
Sabendo que a carga eltrica fundamental tem valor

q 1,6.10 19 C ,

104

a) 4 W
d) 27 W

b) 12 W
e) 36 W

c) 18 W

27) (UFPE)

No circuito eltrico abaixo, qual o menor valor da resistncia R


que devemos colocar em paralelo com a lmpada de 6,0 W, para evitar a
queima do fusvel de 3,0 A?

a) 89 V
d) 126 V

b) 95 V
e) 130 V

c) 110 V

31) (UEPB)

a) 8,8
d) 5,8

b) 7,8
e) 4,8

No laboratrio de eletricidade, uma equipe de alunos recebe a


orientao do professor para montar o circuito apresentado na figura a
seguir. Neste circuito existe um cilindro condutor com comprimento de 1 m,
rea de seo transversal de 10-6 m2 e resistividade do material de 2.10-5
.m. Desprezando-se a resistncia dos fios, a corrente i indicada no circuito
vale:

c) 6,8

28) (Fatec

SP) No circuito abaixo os aparelhos de medida so ideais. O


voltmetro V1 indica 24V.

10

20

20

V2

40

a) 0,20 A b) 0,30 A c) 0,12 A


d) 0,24 A e) 0,15 A

32) (Unifor) Uma pilha de fora eletromotriz 6,0 V e resistncia interna 0,20

V1
As indicaes do ampermetro A e do voltmetro V2 so, respectivamente:
a) 1,0 A e 24 V
b) 1,2 A e 36 V
c) 1,2 A e 24 V
d) 2,4 A e 36 V
e) 1,0 A e 36 V

29) (Unicap PE) No circuito abaixo, a corrente I1 igual a 5 A. O gerador


e os fios de ligao so idias.

fornece uma corrente de 2,0 A ao circuito externo. Nessas condies,


correto afirmar que:
a) a ddp nos terminais da pilha vale 6,0 V.
b) a potncia eltrica fornecida pela pilha ao circuito externo de 12 W.
c) o rendimento eltrico da pilha de 80%.
d) a pilha fornece ao circuito extremo energia eltrica na razo de 11,2 J por
segundo.
e) o circuito externo constitudo por um resistor de resistncia eltrica 4,8
.

33) (UFPR

adapt.) Um gerador com fora eletromotriz E = 15 V e


resistncia interna r = 1,0 fornece a energia de duas lmpadas idnticas,
L1 e L2, com resistncia 4 cada uma, conforme o circuito da figura.
A potncia dissipada pela lmpada L1 quando a chave est aberta e quando a
chave est fechada vale, respectivamente:
a) 36 W e 25 W
b) 25 W e 36 W
c) 36 W e 24 W
d) 24 W e 12 W
e) 12 W e 12 W

34) (UFSCar)
Podemos dizer corretamente que:
a) o potencial do ponto A menor do que o do ponto B.
b) A corrente I2 menor do que I3.
c) A resistncia equivalente do circuito 10
d) A potncia total dissipada de 500 W
e) Em 5 s, passa atravs do gerador uma carga total de 1 C.
30) (UFSM adapt.) No circuito representado na figura, a corrente eltrica
no resistor R1 tem intensidade de 4 A. A fem do gerador vale, em volts:

Considere o
figura, onde esto associadas
resistncias (R1, R2 e R3) e trs
E2 e E3) de resistncias
desprezveis.

circuito

da
trs
baterias (E1,
internas

Um voltmetro ideal colocado entre P e Q indicar:


a) 11 V

105

b) 5 V
c) 15 V
d) 1 V
e) zero.

35) (UEL) Pelo circuito eltrico esquematizado flui uma corrente eltrica:

1,5V

1F

10

9V
A diferena de potencial, em volts, nos terminais do resistor de 2,0 e a
potencia dissipada nele, em watts, so, respectivamente:

1,5
a) 10,5 C b) 9,0 C
d) 4,5 C

a) 1,0 e 0,50
b) 1,0 e 2,0
c) 2,0 e 2,0
d) 2,0 e 4,0
e) 4,0 e 8,0

c) 1,5 C
e) Zero

40) (Unimar

SP) No circuito eltrico abaixo, os valores da corrente do


circuito, ddp entre A e B e a carga eltrica armazenada no capacitor
respectivamente, so:

36) (UFSM) A diferena de potencial no resistor R2 do circuito mostrado na


figura vale, em V:

U E r i (gerador)

Dados: U R I

Q CU

(capacitor)

A
8
+

E = 32V

C = 2F
6

a) 48
d) 8

b) 32
e) 4

c) 16

Capacitor

37) (Osec SP)


B
a) 3,2 A ;
b) 2 A ;
c) 1,6 A ;
d) 0,8 A ;
e) 1,6 A ;

19,2 V ; 38,4C
12 V ;
24C
9,6 V ;
19,2C
4,8 V ;
19,2C
4,8 V ;
19,2C

41) (UFLA

MG) O circuito eltrico abaixo representa um banco de


capacitores. Pode-se afirmar que a tenso no capacitor de 3 F de

No circuito acima, as intensidades das correntes i1, i2 e i3, em ampres, valem


respectivamente:
a) 1,0; 2,5; 3,0
b) 1,0; 2,0; 3,0
c) 1,0; 1,5; 2,0
d) 2,0; 3,0; 1,0
e) 1,0; 2,0; 2,5

38) (Mack)

A carga de um capacitor plano 20 C. Duplicando-se a


distncia entre as armaduras desse capacitor, a energia armazenada por ele:
a) reduz-se a um quarto.
b) quadruplica
c) reduz-se metade
d) duplica
e) no se altera

39) (UFLA

MG) Dado o circuito abaixo e supondo o capacitor carregado,


qual ser a sua carga?

a) 30V
b) 60V
c) 5V
d) 6V

42) (UEL-PR) A Teoria da Relatividade Restrita, proposta por Albert Einstein

(1879-1955) em 1905, revolucionria porque mudou as idias sobre o


espao e o tempo, mas em perfeito acordo com os resultados experimentais.
Ela aplicada, entretanto, somente a referenciais inerciais. Em 1915, Einstein
props a Teoria Geral da Relatividade, vlida no s para referenciais
inerciais, mas tambm para referenciais no-inerciais. Sobre os referenciais
inerciais, considere as seguintes afirmativas:
I. So referenciais que se movem, uns em relao aos outros, com
velocidade constante.

106

II. So referenciais que se movem, uns em relao aos outros, com


velocidade varivel.
III. Observadores em referenciais inerciais diferentes medem a mesma
acelerao
para
o
movimento
de
uma
partcula.
Qual das seguintes alternativas est correta?
a) Apenas a afirmativa I verdadeira.
b) Apenas a afirmativa II verdadeira.
e) As afirmativas I e II so verdadeiras.
d) As afirmativas II e III so verdadeiras.
e) As afirmativas I e III so verdadeiras.

43) (Uepa) As afirmativas abaixo referem-se ao efeito fotoeltrico:

I Quando se aumenta apenas a intensidade da luz na superfcie


fotoeltrica, o nmero de eltrons emitidos por unidade de tempo aumenta.
II necessria uma energia mnima dos ftons da luz incidente, para
arrancar os eltrons do metal que constitui uma fotoclula.
III O efeito fotoeltrico parte do pressuposto de que a energia da luz
quantizada.
IV Quanto maior o comprimento de onda de luz, tanto menor a energia do
fton.
Pode-se afirmar que:
a) apenas I e IV so verdadeiras.
b) todas so verdadeiras.
c) apenas I e III so verdadeiras.
d) apenas III e IV so verdadeiras.
e) todas so falsas.

44) (UFG)

Transies eletrnicas, em que ftons so absorvidos ou


emitidos, so responsveis por muitas das cores que percebemos. Na figura
abaixo, v-se parte do diagrama de energia do tomo de hidrognio.

Na transio indicada, (E3 E2), um fton de energia:


a) 1,9 eV emitido. b) 1,9 eV absorvido
c) 4,9 eV emitido
d) 4,9 eV absorvido
e) 3,4 eV emitido.

45) (UFRGS)

Assinale a alternativa que preenche corretamente as lacunas


do texto abaixo, na ordem em que aparecem.
De acordo com a Fsica Quntica, a energia interna de um tomo est
quantizada em nveis discretos. Pelo modelo atmico de Bohr, os valores de
energia dos nveis discretos do tomo de hidrognio livre so dados por:

En

2,18.10 18
, n 1,2,3,...
n2

Em que n o nmero quntico que identifica cada nvel de energia. Sendo h


= 6,6.10-34 Js o valor aproximado da constante de Planck, para sofrer uma
transio atmica do nvel inicial n = 3 para o nvel fundamental n = 1, um
tomo de hidrognio dever ____________ radiao eletromagntica de
freqncia aproximadamente igual a __________ hertz.
a) absorver 1,6.1014
b) emitir 2,5.1014
c) absorver 3,6.1014
d) emitir 2,9.1015
e) absorver 3,3.1015

46) (FMTM

MG) Existem radiofarmacos utilizados no tratamento de


molculas da tireide que contm I-131, meia vida de 8 dias. Esse
radioistopo obtido a partir do bombardeamento do Te-130 com nutrons.
A reao emitida na reao nuclear para obteno do I-131 e a porcentagem
da atividade inicial restante numa amostra desse radioistopo aps 24 dias
so:
a) beta e 25%
b) beta e 12,5%
c) gama e 25%
d) alfa e 12,5%
e) prton e 12,5%

47) (UFRGS)

Num reator, ncleos de urnio-235 capturam nutrons e,


ento, sofrem um processo de fragmentao em ncleos mais leves,
liberando energia e emitindo nutrons. Esse processo conhecido como:
a) fuso
b) fisso
c) espalhamento
d) reao termoinuclear
e) aniquilao

1) B

2) A

3) A

4) E

5) B

6) C

7) D

8) B

9) B

10) E

11) E

12) A

13) E

14) A

15) D

16) B

17) A

18) C

19) A

20) D

21) E

22) A

23) B

24) A

25) E

26) E

27) E

28) B

29) D

30) D

31) D

32) D

33) A

34) A

35) A

36) B

37) B

38) D

39) D

40) B

41) A

42) E

43) B

44) A

45) D

46) B

47) B

107

01 - (UFMG MG) Um carro bicombustvel percorre 8 km com um litro de


lcool e 11 km com um litro do combustvel constitudo de 75% de gasolina e
de 25% de lcool, composio adotada, atualmente, no Brasil.
Recentemente, o Governo brasileiro acenou para uma possvel reduo,
nessa mistura, da porcentagem de lcool, que passaria a ser de 20%.
Suponha que o nmero de quilmetros que esse carro percorre com um litro
dessa mistura varia linearmente de acordo com a proporo de lcool
utilizada.
Ento, CORRETO afirmar que, se for utilizado um litro da nova mistura
proposta pelo Governo, esse carro percorrer um total de
a) 11,20 km.
b) 11,35 km.
c) 11,50 km.
d) 11,60 km.

d)

1
2

e) 1
05 - (UCS RS) O custo total para uma certa empresa fabricar q unidades de
um determinado produto dado pela funo definida por
C(q) 22 000 850q . O valor total recebido pela venda de q unidades
desse produto, a um preo de k reais por unidade, dado pela funo
definida por R(q) kq .

Se, vendendo 200 unidades do produto, a empresa gerou R$ 187 000,00 de


receita, o nmero mnimo de unidades que ela precisa comercializar para
obter lucro
02 - (UCS RS) Em uma experincia realizada na aula de Biologia, um grupo a) igual a 200.
de alunos mede o crescimento de uma planta, em centmetros, todos os dias. b) igual a 236.
Plotando os pontos (t,a), em que t corresponde ao tempo em dias, e a c) igual a 259.
corresponde altura da planta em centmetros, os alunos obtiveram a figura d) maior do que 260.
a seguir.
e) menor do que 200.
06 - (UEM PR) Com respeito funo

f : IR IR

definida por

f ( x ) 4 x 2 , assinale o que for correto.


a) a funo inversa de f

f 1 : IR IR

definida por

f 1 ( x )

1
4x 2

.
b) a funo composta

f o f (x) definida por (4 x 2) 2 .

c) Para todo x pertencente ao domnio de f, tem-se que f(x) um nmero


par.
d) se um ponto (a, b) pertence ao grfico de f, ento
e) f no uma funo decrescente.
Se essa relao entre tempo e altura da planta for mantida, estima-se que,
no 34 dia, a planta tenha, aproximadamente,
a) 10 cm.
b) 6 cm.
c) 8 cm.
d) 5 cm.
e) 7 cm.
03 - (UFG GO) Em um experimento de confinamento, dois bois com mesmo
peso inicial p0 receberam diferentes quantidades de rao. Aps vrias
pesagens, observou-se uma evoluo de seus pesos, conforme o grfico
abaixo:

a b .

07 - (UEL PR) Os produtos farmacuticos devem especificar as dosagens


recomendadas para uso de adultos e de crianas. As frmulas a seguir so
utilizadas para modificar a dosagem de uso dos adultos para a dosagem de
uso por crianas (y).
Frmula A:
Frmula B:

1
( t 1) a
24
1
y
t a
21
y

Onde a denota a dosagem de adulto em miligramas e t a idade da criana em


anos.
Assinale a alternativa que apresenta a idade da criana na qual as duas
frmulas especificam a mesma dosagem.
a) 2 anos.
b) 6 anos.
c) 7 anos.
d) 8 anos.
e) 10 anos.
08 - (UDESC SC) A soma dos coeficientes a e b da funo
para que as afirmaes

Sabendo que, aps 60 dias, a diferena de peso (p 2 p1) entre eles era de
15 kg, calcule a diferena de peso aps 100 dias.
04 - (Mackenzie SP) O grfico de

Se
a)

f (5) f (3)

3
5
1
8

b) 1
c) 2

, ento

f ( 4)
4

y f ( x ) est esboado na figura.

a) 4
c) 2
e) 4

f ( x ) ax b ,

f (0) 3 e f (1) 4 sejam verdadeiras, :

b) 3
d) 5

09 - (UFPA PA) Uma locadora de veculos apresenta, para aluguel de certo


tipo de carro, a seguinte tabela:

Em uma diria, com percurso no superior a 100km, para que a 2 opo


seja menor em reais, necessrio que o nmero de quilmetros percorridos
pelo locatrio pertena ao intervalo
a) [60,100]
b) ]60,100[
c) ]60,100]
d) [0,60]
e) [0,60[
10 - (Unimes SP) Uma empresa, para construir uma estrada, cobra uma
taxa fixa mais uma taxa que varia de acordo com o nmero de quilmetros
de estrada construda.

108

O grfico abaixo descreve o custo da obra, em milhes de reais, em funo


do nmero de quilmetros construdos.
Assinale a alternativa que indique o custo total da obra, sabendo que a
estrada ter 50 km de extenso.

02.
Nesse caso, o preo p cresceria a uma taxa constante de 34%.
03.
Para que a segunda opo seja mais vantajosa que a primeira
para o dono do barco, o nmero mnimo de horas inteiras que o passeio
deveria durar 5.
04.
A segunda opo ser sempre mais vantajosa para o dono do
barco.
13 - (Mackenzie SP) Se, na figura ao lado, temos o esboo do grfico da
funo y f(x) , o grfico que melhor representa y f(x 1) 1

a) R$ 2.000.000,00
b) R$ 4.000.000,00
c) R$ 7.000.000,00
d) R$ 8.000.000,00
e) R$ 9.000.000,00
11 - (UFG GO) A funo, definida para todo nmero real x, cujo grfico :

tem a seguinte lei de formao:

a)

c)

e)

5
x 4, x 5

f (x) 2
5 x 9, x 5

4
2
x 4, x 5

5
f (x)
4 x 9, x 5

b)

d)

2
5 x 4, x 5
f (x)
4 x 9, x 5
5
2
5 x 4, x 5
f (x)
4 x 9, x 5
5

5
x 4, x 5
f (x) 2
5 x 9, x 5
4

12 - (UFAL AL) O grfico abaixo estabelece a relao entre o preo total p,


em reais, cobrado pelo aluguel de um barco de turismo em um passeio pelo
litoral norte de Alagoas e o nmero de horas x gasto no passeio.

14 - (PUC RJ) Quantos nmeros inteiros satisfazem simultaneamente as


desigualdades
a) 4
d) 2

2x 3 x 7 3x 1 :
b) 1
e) 5

c) 3

15 - (UFAM AM) O conjunto das solues, no conjunto R dos nmeros

x
x
x 1
{x R ; x 1}

reais, da inequao
a)

c) vazio
e) {x R ;

:
b)

{x R ; x 1}
d) R

x 0}

16 - (Unioeste PR) Considere a inequao

2
3,
x 1

x 1. Indique

qual(is) dos conjuntos dados esto contidos no conjunto-soluo dessa


inequao.
01. A = {x R | x < 1}
02. B = {x R | x > 5/3}
04. C = {x R | x 5/3}
08. D = {x R | x 1}
16. E = {x R | x < 1 ou x 5/3}
32. F = {x R | x < 1 ou x > 5/3}
64. G = {x R | x < 1 e x > 5/3}

17 - (UERJ RJ) O conjunto soluo da inequao

2x - 3
1
3x - 2

seguinte intervalo:

Analisando o grfico conclui-se que:


00.
A cada passeio, cobrada uma quantia fixa de R$ 28,00.
01.
O preo p cresce a uma taxa constante de 30%.
Ateno: Para responder as alternativas seguintes considere uma segunda
opo: o aluguel ser cobrado somente pelo nmero de horas do passeio, a
R$ 18,00 por hora.

a)

(- , -1]

c)

[-1 ,

2
3

2
3

b)

(- ,

d)

[-1 , )

109

e)

2
3

, 1]

1
1

x (1 x ) x

18 - (UFMG MG) O conjunto soluo da inequao

IR | 0 < x < 1 }
IR | x < 1}
IR | x < 1 e x 0 }
IR | x > 0}
IR | x > 1}

a) {x
b) {x
c) {x
d) {x
e) {x

o grfico da funo

reta.
01.

[ 1, 2) [3, )

b)

( 1, 2] (3, )

c)

[1, 3]

d)

[3, 2)

e)

[3, 2] (2, )

24 - (UEPB PB) Um jogador chuta uma bola que descreve no espao uma
2
parbola dada pela equao: y 3t 150t 288 . Dizemos que a bola
atinge o ponto mais alto de sua trajetria quando t for igual a:
a) 35
b) 20
c) 30
d) 25
e) 40

19 - (UnB DF) Julgue os itens abaixo.


00.

a)

f(x)

x 4
x2
2

, para x > -2, uma semi-

25 - (UDESC SC) Os valores reais de n, para os quais a equao


2x 2 4x n 0 , tm razes reais distintas, que so:
a) somente
c)

Se o grfico de f(x) = ax + b

e)

n 2
n 2

n 2

b)
d)

n 2
n 2

26 - (UDESC SC) Uma fbrica de determinado componente eletrnico tem a


receita financeira dada pela funo R(x) 2x 20x 30 e o custo de
produo dada pela funo

02. Se

b
0
a

27 - (Unifor CE) Seja f uma funo quadrtica cujas razes so 4 e 3. Se o


grfico de f intercepta o eixo das ordenadas no ponto (0; 2), ento

e ab > 0.

f(x) x , e g(x) = x2 1, ento o domnio de gof o conjunto

dos nmeros reais.


03. o grfico de

1 x
x

1
x 2 1

, para x 0, o simtrico do grfico de

a)

2 1 2x - 2 1 4x - 8

igual a:

b) 1

c)

d) 2

e)

d)

b) f crescente para todo x > -2.


c) f decrescente para todo x < 0.
d) f positiva para todo x > 0.

7
8

28 - (Unifesp SP) De um carto retangular de base 14 cm e altura 12 cm,


deseja-se recortar um quadrado de lado x e um trapzio issceles, conforme
a figura, onde a parte hachurada ser retirada.

1 2

2, b) ,2
2,0
e) 0,1

c)

1,2

x 1 3x 3
2
x 4 0

O valor de x em centmetros, para que a rea total removida seja mnima,


a) 3.
b) 2.
c) 1,5.
d) 1.
e) 0,5.
29 - (Furg RS) Determine os nmeros reais a e b para que a funo

22 - (PUC PR) O grfico a seguir de um trinmio do 2 grau:

quadrtica

Assinale a alternativa que melhor representa o trinmio:


a) y = x2 + 2x + 5
b) y = x2 + 2x + 2
c) y = x2 + 2x + 6
d) y = x2 + 3x + 2
e) y = x2 + 2x + 3
23 - (UFJF MG) Os valores de x que satisfazem
x 2 2x 3
0 pertencem a:

x2

49

24 , .

21 - (Mackenzie SP) Em R, a soluo do sistema


a)

a) o conjunto imagem de f o intervalo

e) o valor mximo de f

, para 0 < x 1, em relao reta y = x.

20 - (ESPM SP) A diferena entre a maior e a menor raiz real da equao

2x -

em que a varivel x

representa o nmero de componentes fabricados e vendidos. Se o lucro


dado pela receita financeira menos o custo de produo, o nmero de
componentes que deve ser fabricado e vendido para que o lucro seja mximo
:
a) 32
b) 96
c) 230
d) 16
e) 30

ento f(0) < 0,

C(x) 3x 2 12x 30 ,

f (x)

b
x2
x 5 2
a
a

tenha valor mximo no ponto x = 3 e que

esse valor mximo seja 5 :


a) a=6, b=12,5.
b) a=3, b=12,5.
c) a=3 , b=10.
d) a=6, b=10.
e) a=6, b=15.
30 - (UFC CE) Para cada nmero real
inequao

x 1,

define-se f (x) por

x
f (x)
. Ento, f (f (x)) sempre igual a:
x 1
a) x
c) f (x)
e) f (x2)

b) x
d) f (x)2

110

31 - (UFRR RR) Sabe-se que as funes reais f(x) e (fog) (x) tem as
seguintes leis de formao respectivamente: f(x)=4x+2 e (fog)
(x)=4x2+8x+10. Ento a lei de formao de g(x) igual a:
a) 4X + 2
b) 2X + 1
c) X2 + 1
d) X2 + 2X + 2
e) 4X2 + 2X
32 - (FGV) Sejam f e g duas funes de R em R, tais que f(x) = 2x e g(x) =
2 x.
Ento, o grfico cartesiano da funo f (g (x)) + g (f (x))
a) passa pela origem.
b) corta o eixo x no ponto (4,0).
c) corta o eixo y no ponto (6,0).
d) tem declividade positiva.
e) passa pelo ponto (1,2).

Considerando a funo quadrtica correspondente a cada time da tabela e o


critrio de Joo, pode-se afirmar que:
a) PRAIANO obteve o melhor desempenho;
b) SERRANO obteve o melhor desempenho;
c) CAMPRESTE obteve o melhor desempenho;
d) SERRANO e PRAIANO ficam com o segundo e terceiro lugares,
respectivamente, em termos de seus desempenhos;
e) PRAIANO e CAMPESTRE ficam com o segundo e terceiro lugares,
respectivamente, em termos de seus desempenhos.
37 - (UEPB PB) Sendo x igual ao menor ngulo formado pelos ponteiros das
horas e dos minutos quando so 7 horas e 55 minutos, o valor da expresso
x + 240 igual a:
a) 12010 b) 9510 c) 120
d) 95
e) 11050

33 - (FGV) Considere as funes reais dadas por

38 - (Unimontes MG) Quando os ponteiros de um relgio marcam


1h50min, qual a medida do ngulo central formado por eles?
a) 120
b) 115
c) 110
d) 95

f(x) = 2 x 1, g(x) = f(x) x e h(x) = g(f(x)).

39

(UDESC

cos x senx tgx


cos x

As retas que representam as funes f e h


a) so perpendiculares no ponto (2,1).
b) so perpendiculares, no ponto (0,0).
c) no so perpendiculares, mas se encontram no ponto (1,2).
d) passam pelos pontos (1,1) e (0,1).
e) no se encontram, isto , so paralelas.

SC)

expresso

trigonomtrica

dada

por

uma identidade trigonomtrica com o termo:

a) cotg2 x
b) cotg x
c) cosec2 x
d) sec2 x
e) tg2 x

34 - (UEPG PR) Sobre funes, assinale o que for correto.


01.
Se f(x) = 3x 5 e f[g (x)] = x2 +2, ento g(1) = 4.

40 - (PUC PR) Os valores reais de z que satisfazem a equao


02.
Se considerarmos f : A B , ento a relao f-1 uma funo sen x z 2 6z 9 pertencem ao intervalo:
de B em A se, e somente se, f bijetora.
a) 0 z 3
04.
O
domnio
da
funo
f(x)
=
logx
(3x2)

b) 1 z 1
2

c) 1 z 3
D x R / x e x 1 .
3
d) 2 z 4

08.
Um retngulo de base x tem permetro de 40 cm. A sua rea, em e) 3 z 3
funo da base, expressa por A (x) = 20x x2.
16.
Dadas as funes f(x) = x2 x 2, e g(x) = 1 ax, se a = 2,
3

41 - (UDESC SC) Se senx


e 0x
, o valor numrico da
ento f [g (x)] = 4x2 2x 2.

35 - (UFViosa MG) Seja:

A, B, C, D, E, F, G, H, I, J, K, L,..., X, Y, Z,

conjunto

das

um

letras do alfabeto brasileiro (incluindo K, W,Y). Considere


subconjunto de

R e f : 1

a funo definida por f(A) =3, f(B)

=27, f(C) =243, f(D) = 2187 e assim por diante. Suponha, ainda, que f
bijetora e que f1 sua inversa.
Calculando f1 (3) f1 (323) f1 (39) f1 (325) e mantendo esta ordem, obtmse a palavra:
a) A N E L
b) A L G O
c) A L E M
d) A M E I
e) A N I L
36 - (UFF RJ) A tabela a seguir mostra as estatsticas de trs times num
torneio de futebol.

GOLS
FINALIZAES GOLS A
SOFRIDOS
EM GOL
FAVOR
GS
FG
GF
CAMPESTRE
2
48
12
TIME

PRAIANO
SERRANO

6
3

50
35

13
9

No satisfeito com o resultado do torneio, Joo criou, para cada time, a


funo quadrtica:

P( x )

1
x - GS 2FG x GF2 R
2

substituindo GS, FG e GF pelos valores correspondentes na tabela.


Segundo o critrio de Joo, o desempenho de cada time representado pelo
valor mnimo de P(x), de modo que, quanto maior o valor mnimo de P(x),
melhor ser o desempenho do time correspondente.

expresso
a)
b)
c)
d)
e)

y (cos2 x)(tgx )

25
12
3
5
12
25
5
3
4
3

42 - (UFAM AM) Se

A cos2 x sen2 y (senx cos y) 2 ,

onde

x y , ento A :
2
a) 2cos2 x
b) 2sen2 x
c) sen x
d) cos x
e) sen 2x
43 - (FGV) A soma das razes da equao sen2x sen (x) = 0, no intervalo
[0,2 ] :
a)
c)

7
2
5
2

b)

9
2

d)

111

e)

44

3
2
-

(Furg

RS)

Para

todo


x 0, ,
2

1
1

tg(x)
tg(x) sen2 (x)
cos(
x
)
cos(
x
)

expresso
c)

igual a:

a) sen(x) + cos(x)
b) 1 + sen2(x)
c) cos(x) sen(x)
d) cos2(x)
e) sen2(x)
45 - (UFAM AM) A expresso tgx cotg(-x) +
, em que
a)
b)

0x

sen x cos( x)
2

d)

, equivalente a:

x
tgx
2
sen 2 x

c) cos 2x
d)

cot gx
x

e)

e) x secx
46 - (IME RJ) Resolva a equao:


log (sen x cos x) (1 sen 2x) 2, x - ,
2 2
47 - (UFF RJ) Nas comunicaes, um sinal transmitido por meio de ondas
senoidais, denominadas ondas portadoras.
Considere a forma da onda portadora modelada pela funo trigonomtrica

f ( t ) 2 sen (3t - ), t R
3

48 - (Mackenzie SP) A figura mostra os esboos dos grficos das funes

x
f (x) sen
k

g ( x ) cos(m x) . Ento,

Pode-se afirmar que o grfico que melhor representa f(t) :

a)

a)

| m | k

c)

| m |

d)

m k
1
m k
2

e)

b)

m 2k

b)

1
k
3

49 - (UEM PR) Uma esteira rolante de um supermercado com dois andares


faz um ngulo de 30 com o plano determinado pelo piso inferior. Assinale o
que for correto, considerando o comprimento da esteira 12 metros.
a) Uma pessoa que sai do piso inferior e vai ao piso superior se eleva 6 (seis)
metros.
b) Faltam dados para se calcular a altura total que uma pessoa se eleva ao ir
do piso inferior ao piso superior utilizando a esteira.
c) Se uma pessoa caminha 2 metros na esteira durante o percurso entre o
piso inferior e o piso superior, ento a pessoa se eleva, no total, 5 (cinco)
metros.
d) Uma pessoa que sai do piso inferior e vai ao piso superior se eleva

6 3

metros.
e) Se uma pessoa caminha 2 metros na esteira durante o percurso entre o
piso inferior e o piso superior, ento a pessoa se eleva, no total,

5 3

metros.

112

50 - (Mackenzie SP) Na figura, se


, ento

3n m

A ( m;0) , B ( n ;0) e C ( 4;0)

as respectivas medidas dos ngulos

igual a

2
3
sen
2
2

, concluise que

AED e ADE ,
0 sen

ento sabendo que

2 6
4

53 - (UFPR PR) Calcule o seno do maior ngulo de um tringulo cujos lados


medem 4, 6 e 8 metros.

15
4

a)
b)

15
2

a)

c)

b) 8
c)

5 3
25
3

e)

51 - (Mackenzie SP) Uma estao E, de produo de energia eltrica, e


uma fbrica F esto situadas nas margens opostas de um rio de largura

10
4
3
2

d)

d) 9
e)

1
4
1
2

km. Para fornecer energia a F, dois fios eltricos a ligam a E, um por

3
terra e outro por gua, conforme a figura. Supondo-se que o preo do metro
do fio de ligao por terra R$ 12,00 e que o metro do fio de ligao pela
gua R$ 30,00, o custo total, em reais, dos fios utilizados :

54 - (UEPG PR) Na figura abaixo, em que o ponto B localiza-se a leste de A,


a distncia AB = 5 km. Neste momento, um barco passa pelo ponto C, a
norte de B, e leva meia hora para atingir o ponto D. A partir destes dados,
assinale o que for correto.

30

D
a) 28 000
b) 24 000
c) 15 800
d) 18 600
e) 25 000

.
.

A
01.

52 - (UFAL AL) Uma pessoa encontra-se no aeroporto (ponto A), vinda de


uma viagem. Ela pretende ir para sua casa (ponto C), distante 18 km do
aeroporto. Alm do percurso direto, ela tem outras opes para chegar em
casa, percorrendo trechos retilneos de estrada, conforme mostra o esquema
abaixo e no qual esto indicados alguns dados que permitiro a anlise da
veracidade das afirmaes que seguem.

02.
04.

AC = 10 km
AD = 2,5 km
BC = 5 3 km

08. O ngulo BD mede 60


16. A velocidade mdia do barco de 15 km/h
55 - (UFAM AM) O menor valor no negativo cngruo ao arco de

21
5

rad igual:
a)
b)

00. Optando pelo caminho

AD DC ,

que se tivesse escolhido o trecho


01. O percurso

ela percorreria x metros a mais do

AC , 2.400 < x < 2.800.

AB BC tem 6 3 km

AD DC

c)

rad

d)

9
rad
5

e)

2 rad

02. Considere que, para ir para casa, essa pessoa poderia pegar um txi no
aeroporto e o valor da corrida, em reais, fosse calculado pela expresso V(x)
= 12 + 0,5x, em que x o nmero de quilmetros percorridos. Nesse caso,
para percorrer o trecho

rad
5
7
rad
5

ela gastaria R$ 22,20. (Use:

3 1,7 )

03. Supondo que, percorrendo o trecho AC , ela gastou 12 minutos para


chegar em casa, ento a velocidade mdia do carro que a transportou era de
90 km/h.

56 - (UEPB PB) Sendo

cossec 2460 sec1110


cot g 2205

, ento o valor de A

igual a:
a)

4
3

1
3
04. Considere que, percorrendo o trecho AC , o automvel em que essa
4
e)
pessoa estava parou em um posto para abastecer (ponto E). Se e so
3
c)

b)

d)

8
3

8
3

113

57 - (UFAL AL) Determine o valor do 458o termo da seqncia (cos 30o, cos 63 - (UEG GO) Entre os 486 funcionrios de uma agroindstria, h seis
60o, cos 90o, cos 120o, ...).
agrnomos e oito tcnicos agrcolas. Deseja-se constituir uma comisso
formada com cinco destes 14 profissionais, sendo que a comisso deve
58 - (UEPB PB) O valor de cos 1 200 igual ao valor de:
conter dois agrnomos e trs tcnicos agrcolas. A quantidade de comisses
a) cos 30
diferentes que podem ser formadas
b) sen 30
a) 10.080.
c) sen 60
b) 2.002.
d) cos 60
c) 840.
e) cos 45
d) 71.
59 - (Unimontes MG) Observe atentamente a simetria da figura abaixo.

64 - (UFSC SC) Assinale a(s) proposio(es) CORRETA(S).


01. Considerando-se um hexgono regular e tomando-se ao acaso uma das
retas determinadas pelos seus vrtices, a probabilidade de que a reta passe
pelo centro do hexgono

Sabendo-se que

11
sen
6
1
1
a)
e
2
2
1
1
c)
e
2
2

sen

2
1

b)
d)

0x

e
e

tg (x) 4 3

2
1
2

, assinale a(s) alternativa(s) correta(s).

, ento o valor de sec(x) 7.

cos(2x) sen (x) cos2 (x)


2

08. Se

tg (x) 4 3 , ento

cos sec 2 ( x ) tg ( x )
2

sec ( x )
16. Se
32.

3
12

sec(x ) a , ento a 1 .

1
1
1
1
1

2 1 sen 2 ( x ) 1 cos2 ( x ) 1 sec2 ( x ) 1 cossec2 ( x )


sen 2 ( x ) cos2 ( x ).

61 - (UFPI PI) Sabendo-se que

senx cos x

3
4

, podemos afirmar

que sen(2x) :
a)

c)
e)

13
16

13
16
3
4

b)

3
16

d)

02. Se cinco atletas disputam uma prova de corrida de 800 metros, ento o
nmero de resultados possveis para os dois primeiros lugares, sem que haja
empates, 10.
04. Antnio, Cludio, Carlos e Ivan montaram uma empresa de prestao de
servios e decidiram que o nome da empresa ser a sigla formada pelas
iniciais dos seus nomes, por exemplo, CACI. O nmero de siglas possveis
12.
08. Numa lanchonete h cinco tipos de sucos: laranja, abacaxi, acerola, limo
e morango. Eles so servidos em copos de trs tamanhos: pequeno, mdio e
grande. No permitido misturar sabores. O nmero de maneiras possveis
e de se pedir um suco 15.
16. Quando sete pessoas se encontram e todas se cumprimentam, o nmero
de apertos de mo possvel, sem que os cumprimentos se repitam, 42.

(sec(x) tg (x))(sec(x) tg (x)) sen2 (x) cos2 ( x) .

02. Se
04.

19
sen
6

, ento os valores de

so, respectivamente,

60 - (UEM PR) Para


01.

6 2

1
8

13
16

62 - (Mackenzie SP) Em uma sala de aula h 25 alunos, quatro deles


considerados gnios. O nmero de grupos, com trs alunos, que pode ser
formado, incluindo pelo menos um dos gnios,
a) 580
b) 1200
c) 970
d) 1050
e) 780

65 - (UEL PR) Antnio e Bruno so membros atuantes do Grmio Estudantil


e esto se formando numa turma de 28 alunos. Uma comisso de formatura,
com 5 membros, deve ser formada para a organizao dos festejos. Quantas
comisses podem ser formadas de modo que Antnio e Bruno sejam
membros?
a) 2600
b) 9828
c) 9288
d) 3276
e) 28
66 - (UFAM AM) O campeonato brasileiro de futebol da srie A tem 20
times que jogam todos entre si, duas vezes. Ento o nmero total de jogos
de:
a) 368
b) 388
c) 376
d) 386
e) 380
67 - (Unipar PR) No restaurante onde voc almoa todos os dias so
oferecidos quatro tipos de saladas, cinco tipos de pratos quentes e dois tipos
de sobremesas. De quantas maneiras voc pode combinar uma refeio com
uma salada, um prato quente e uma sobremesa:
a) 20
b) 25
c) 30
d) 40
e) 45
68 - (UEPG PR) Em relao aos anagramas da palavra "cidade", assinale o
que for correto.
01. Em 72 anagramas as vogais aparecem juntas.
02. Podem ser formados 360 anagramas.
04. Em 72 anagramas as consoantes aparecem juntas.
08. 60 anagramas comeam com "c".
16. 180 o nmero de anagramas que comeam por vogal.
69 - (UFRN RN) Arranjam-se os dgitos 1, 2, 3 e 4 de todos os modos
possveis, formando-se 24 nmeros de 4 dgitos distintos. Listam-se, em
ordem crescente, os 24 nmeros formados.
Nessa lista, o nmero 3.241 ocupa a
a) 14a posio.
b) 13a posio.
c) 16a posio.
d) 15a posio.
70 - (UFAL AL) TRAIPU um municpio alagoano situado prximo s
margens do rio So Francisco com populao aproximada de 24 000
habitantes. Considerando as letras da palavra TRAIPU, o nmero de
anagramas em que as vogais nunca aparecem juntas
a) 696
b) 684
c) 600

114

d) 576
e) 144
71 - (UFPI PI) Sob as retas paralelas no-coincidentes r e s, marcam-se 5 e
9 pontos distintos, respectivamente. O nmero de quadrilteros convexos
com vrtices nesses pontos :
a) 720
b) 360
c) 260
d) 148
e) 46
72 - (UFPel RS) Maurcio de Sousa, criador de uma famosa revista com
histrias em quadrinhos, baseou a criao de seus personagens em amigos
de infncia e nos filhos, conferindo a cada um deles caractersticas distintivas
e personalidades marcantes. A turma da Mnica e todos os demais
personagens criados pelo escritor esto a, com um tipo de mensagem
carinhosa, alegre, descontrada e at matemtica, dirigida s crianas e aos
adultos de todo o mundo.

76 - (UEM PR) Uma senha bancria composta de 3 (trs) dgitos que


podem variar de 0 a 9 (zero a nove). Assinale o que for incorreto.
a) Se uma possvel senha testada a cada segundo, ento todas as
possveis senhas sero verificadas em menos de 17 minutos.
b) H mais de mil possveis senhas distintas.
c) Existem apenas 10 senhas com todos os dgitos idnticos.
d) H 720 senhas com todos os dgitos distintos.
e) H 100 senhas identificadas com nmeros menores que o nmero 100
(cem).
77 - (UEPG PR) Se
01.
02.
04.
08.
16.

A n 1,8 A n,7 16A n, 6

, assinale o que for correto.

Cn 3, 5 = Cn, 8
Cn, 2 = 28
Pn 5 um nmero mpar.
n um nmero par.
Na + 2, 2 = 90

78 - (Mackenzie SP) Um trem de passageiros constitudo de uma


locomotiva e 6 vages distintos, sendo um deles restaurante. Sabendo que a
locomotiva deve ir frente e que o vago restaurante no pode ser colocado
imediatamente aps a locomotiva, o nmero de modos diferentes de montar
a composio :
79 - (UFSC SC) Quantos nmeros diferentes obteremos, permutando os
algarismos do nmero 336 223?
80 - (UERJ RJ) Observe o quadrinho abaixo.

Se os personagens da histria em quadrinhos acima continuassem


permutando as letras, com o objetivo de formar todos os anagramas
possveis, eles obteriam mais
a) 718 anagramas.
b) 360 anagramas.
c) 720 anagramas.
d) 362 anagramas.
e) 358 anagramas.
73 - (UESPI PI) Quantos nmeros com trs dgitos distintos podem ser
formados usando os algarismos {1, 2, 3, 4, 5}?
a) 60
b) 120
c) 140
d) 180
e) 200

As quatro pessoas que conversavam no banco da praa poderiam estar


sentadas em outra ordem.
Considerando que o fumante ficou sempre numa das extremidades, o
nmero de ordenaes possveis :
a) 4
b) 6
c) 12
d) 24
e) 48
81 - (FGV) Sendo k um nmero real positivo, o terceiro termo do
desenvolvimento de (2x + k)12, ordenado segundo expoentes decrescentes
de x, 66x10. Assim, correto afirmar que k igual a:

1
66
1
74 - (UFJF MG) Cinco amigos vo viajar utilizando um carro com cinco
d)
lugares. Sabendo-se que apenas dois deles podem dirigir, o nmero de
33
maneiras que os cinco amigos podem se acomodar para viagem :
a) 12
b) 24
c) 48
d) 120

75 - (UFAC AC) Emanuel investigou os seguintes nmeros: A quantidade


mxima de maneiras de preencher, ao acaso, a folha de respostas de uma
prova de matemtica que contm 7 questes do tipo mltipla escolha, tendo
cada questo 5 alternativas e a quantidade mxima de tringulos que podem
ser construdos com vrtices tomados sobre 30 pontos distintos de uma
circunferncia de raio r > 0.
Se seus clculos foram feitos corretamente, neles podemos ver que:
a) O maior nmero de tringulos que podem ser construdos maior que o
maior nmero possvel de folhas de respostas preenchidas ao acaso.
b) Os nmeros investigados so iguais.
c) Os nmeros investigados so maiores que 4.070.
d) Os nmeros investigados so menores que 70.000.
e) O maior nmero de tringulos que podem ser construdos menor que o
maior nmero possvel de folhas de respostas preenchidas ao acaso.

a)

1
64
1
e)
32
b)

82 - (UEPG PR) Considerando o binmio

c)

1
58

2 1
x 3
x

, assinale o que for

correto.
01.
Se o desenvolvimento desse binmio possui cinco termos, a soma
de seus coeficientes 32.
02.
Se n = 4, o coeficiente do termo mdio desse binmio 12.
04.
Se n um nmero mpar, o desenvolvimento desse binmio tem
um nmero par de termos.
08.
Se a soma dos coeficientes do desenvolvimento desse binmio
64, ento n = 6.
16.
O produto do primeiro termo do desenvolvimento desse binmio
pelo seu ltimo termo xn, para qualquer valor de

n N .

115

2 y
x
2

83 - (UEPB PB) O 6 termo no desenvolvimento do binmio

ser:
a)
c)
e)

63 8 5
x y
16
5
4
4
3

b)
d)

84 6 6
x y
16
6
5

90 - (UFViosa MG) No jogo abaixo, o jogador precisa descobrir em quais


dos oitenta e um quadradinhos esto colocadas 10 bombas. No quadradinho
onde aparece um nmero certeza que no h uma bomba. Por sua vez, o
nmero que aparece dentro do quadradinho indica quantas bombas h nos
oito quadradinhos que o cercam.
Por exemplo, o nmero 2 indica que h duas bombas espalhadas nos oito
quadradinhos que cercam o nmero 2. Considere Q a regio delimitada pelo
quadrado que contm o nmero 2, formada por nove quadradinhos; e R a
regio delimitada pelo retngulo que contm os nmeros 1 e 3, formada por
dezoito quadradinhos.

10

84 - (Unimontes MG) No desenvolvimento de


independente de x
a) 90.
c) 45.

4 1
x
x

, o termo

b) 54.
d) 80.

85 - (UEPB PB) O termo independente de x no desenvolvimento de


6

x
x

a) 6
c) 15
e) 30

igual a:
b) 20
d) 21

10

86 - (UECE CE) O termo mdio no desenvolvimento de


a) 126
c) 252

x
x

b) 126x5
d) 252x5

87 - (FGV) Os resultados de 1 800 lanamentos de um dado esto descritos


na tabela abaixo:

n da face 1
2
3
4
5
6
frequncia 150 300 450 300 350 250

Baseado nestas informaes, assinale a afirmativa INCORRETA:


a) As bombas podem estar distribudas na regio Q de 28 maneiras distintas.
b) A probabilidade de o jogador escolher um quadradinho que no contenha
bomba maior na regio R do que na regio Q.
c)A probabilidade de o jogador escolher um quadradinho na regio Q que
contenha uma bomba igual a 0,25.
d) A probabilidade de o jogador escolher um quadradinho que no contenha
uma bomba na regio R igual a 0,75.
e) As bombas podem estar distribudas na regio R de 448 maneiras
distintas.
91 - (UEPB PB) No lanamento simultneo de dois dados honestos, um
amarelo e outro branco, a probabilidade de no sair soma 5, igual a:
a)

Se lanarmos esse mesmo dado duas vezes, podemos afirmar que:


a) a probabilidade de sair pelo menos uma face
b) a probabilidade de sair pelo menos uma face
c) a probabilidade de sarem duas faces
d) a probabilidade de sarem as faces

1
6
11
4
36

c)

e)

1
3

3 e 4

b)
d)

4
9
5
6

92 - (EFOA MG) Na tabela abaixo esto apresentados dados referentes a


um grupo de estudantes matriculados em quatro cursos de uma
universidade, distribudos segundo o sexo, sendo que cada estudante est
matriculado em apenas um curso.

1
18

e) a probabilidade de sarem duas faces maiores que

8
9
1
9
5
9

35
36

88 - (Unimontes MG) Uma urna contm 40 cartes, numerados de 1 a 40.


Se retirarmos ao acaso um carto dessa urna, qual a probabilidade de o
nmero escrito no carto ser um mltiplo de 4 ou mltiplo de 3?
a)
b)
c)
d)

23
40
7
40
1
4
1
2

89 - (Unipar PR) Um casal pretende ter trs filhos. A probabilidade de


nascerem dois meninos e uma menina, independentemente da ordem, de:
a) 3/5
b) 3/8
c) 3/10
d) 3/14
e) 3/16

Uma pessoa desse grupo de estudantes escolhida ao acaso. Sejam p 1, p2,


p3 e p4, respectivamente, as probabilidades de ser homem, mulher, aluno de
Matemtica e aluno de Cincias da Computao. Sabendo-se que
e que

p 4 2 p 3 , ento a b

p1 3 p 2

vale:

a) 165
b) 145
c) 155
d) 135
e) 175
93 - (ESPM SP) Dado no plano cartesiano o tringulo de vrtices A (4, 0), B
(0, 2) e C (8, 8), a medida da altura relativa ao vrtice A igual a:
a) 4
b) 5
c)

4 2

d)

3 2

116

e)

2 3

94 - (UECE CE) Na linha poligonal PQRSTU, plana e aberta como mostra a


figura, dois segmentos consecutivos so sempre perpendiculares, a medida
de PQ 1m e, a partir de QR, inclusive, os demais comprimentos dos
segmentos so obtidos, dobrando o valor do segmento anterior.

102 - (FEI SP) Calcular a distncia da origem ao vrtice da parbola: y = x 2


6x + 10.
103 - Achar a rea do quadriltero ABCD, dados A(2 ; 5), B(7 ; 1), C(3 ; 4)
e D(2 ; 3).

104 - A rea do tringulo ABC da figura :

205

b)

215

c)

15

d)

235

95 - (Vunesp SP) O tringulo PQR, no plano cartesiano, de vrtices P =


(0,0), Q = (6,0) e R = (3,5), :
a) equiltero.
b) issceles, mas no equiltero.
c) escaleno.
d) retngulo.
e) obtusngulo.
96 - (Unifor CE) Sejam os pontos
segmento de reta
a) 2
b) 6
c)

A(3,2)

B(5,4).

A medida do

AB

10

4 2

d)

2 7

e)

2 6

.
B

-1

A distncia do ponto P ao ponto U, em metros, :


a)

y
A

97 - (Vunesp SP) Os vrtices da base de um tringulo issceles so os


pontos (1, 1) e (3, 4) de um sistema de coordenadas cartesianas
retangulares. Qual a ordenada do terceiro vrtice, se ele pertence ao eixo das
ordenadas?
98 - (UFOP MG) Sabe-se que a reta 2x y + 4 = 0 passa pelo ponto mdio
do segmento que une os pontos A(2k, 1) e B(1, k). O valor de k :
a) 3
b) 3
c) 2
d) 2
e) 0

-2

a) 18
b) 9
c) 9
d) 15
e) 18
105 - (Cescem) Os grficos x2 + y = 10 e x + y = 10 interceptamse em
dois pontos; a distncia entre esses pontos :
a) menor que 1
b) 2
c) 1
d) 2
e) maior que 2
106 - (Cescem SP) As retas x 1 = 0, y = x e x + y 4 = 0 determinam
um tringulo. As frases corretas so:
a) o tringulo equiltero
b) o tringulo retngulo
c) o tringulo obtusngulo
d) o tringulo issceles
e) o tringulo est inscrito numa circunferncia de centro na origem
107 - (FGV) A rea do trapzio determinado pelas retas de equao x = 3, y
= 5; y = x +1 e pelo eixo y :
a) 7,5
b) 7
c) 6,5
d) 6
e) 5,5
108 - (UFOP MG) Num sistema de coordenadas cartesianas, localizam-se o
ponto P (3,4) e a reta r de equao x+ y 3 = 0. Seja Q o ponto de r cuja
abscissa o dobro da ordenada.

99 - (UFC CE) A distncia entre o ponto de encontro (interseo) das retas


x + y - 2 = 0 e x - y - 4 = 0 e a origem do sistema de coordenadas, (0 ,
0), :
a) 3
b) 7
c) 4
d)

11

e)

10

100 - (PUC Campinas) Os pontos (0; 0), (1; 3) e (10; 0) so vrtices de


um retngulo. O quarto vrtice do retngulo o ponto:
a) (9; 3)
b) (9; 2)
c) (9; 1)
d) (8; 2)
e) (8; 1)
101) Os pontos A(3; 8), B(11; 3) e C(8; 2) so:
a) alinhados
b) vrtices de um tringulo issceles
c) vrtices de um tringulo escaleno
d) vrtices de um tringulo eqiltero
e) vrtices de um tringulo retngulo

A distncia de P at Q :
a) 10
b)

10

c) 4
d)

2 2

109 - (UEPB PB) A distncia entre as retas paralelas

r:yx

s : y x 7 igual a:

117

2
7

a)
c) 7

b)
d)

7 2
7

e)

b)

c) 1

d) 2

e)

2
7
2

1
2

a) 2

3
2

115 - (UFAM AM) No grfico abaixo, a reta s que passa pelo ponto P
paralela a reta r e tem como equao:

110 - (UEPB PB) A distncia entre o ponto P(3, 5) e a reta r, de equao x


+ 2y 8 = 0, igual a:
a) 5
c)

b)

d)

e) 3
111 - (UFPel RS) Engenheiros do Instituto Militar de Engenharia (IME)
desenvolveram uma argila calcinada, material que poder baratear a
construo de estradas. Essa argila no existe em nenhum outro pas.
A pesquisa comeou em 1997, com um objetivo: encontrar um material que
pudesse ser utilizado na Amaznia. A regio carente de rochas, e as
dificuldades no transporte encarecem a brita, comercializada por mais de R$
100,00 o metro cbico. Segundo o IME, o custo da argila calcinada fica em
torno de R$40,00.
Foram estudadas vrias famlias de solos da Amaznia, chegando-se a
concluses animadoras nos ltimos anos. O agregado artificial poder ser
usado em pavimentao rodoviria, pois resiste a desgaste, compresso e
abraso, e tambm em obras de concreto. Segundo o coordenador da
pesquisa, o material pode ser utilizado em qualquer regio do pas.
http://www1.folha.uol..com.br/folha/cincia/ult306u13159.shtml - acessado
em 06/05/2005. [adapt.].
Tambm com o objetivo de baratear custos, na execuo do projeto de
novas estradas, deve ser considerada sempre a menor distncia entre os
pontos a serem alcanados.
As cidades A e B, localizadas no mapa, com coordenadas A (8, 5) e B (12, 8),
so ligadas por uma rodovia em linha reta. A construo de um novo trecho
de menor dimenso que ligue a rodovia existente cidade C (10, 5), medir

a)

3x y 1 0

b)

3x y 3 0

c)

x 3y 1 0

d)

3x y 1 0

e)

3x y 1 0

116 - (UPE PE) Considere a reta (r) de equao


00. a reta (s) de equao

3x 4 y 10 0 . Ento

4 x 3y 5 0 perpendicular reta (r).

01. a reta (r) secante circunferncia de equao

x 2 y2 4 .

02. o tringulo, cujos vrtices so a origem e os pontos de interseo da reta


(r) com os eixos coordenados, tem rea igual a

25
6

03. a tangente do ngulo que d a direo de (r)

3
4

04. a equao da reta paralela reta (r) e que passa por (1,2)
3x 4 y 11 0 .
117 - (UFAM AM) Uma circunferncia passa pelos pontos A = (0, 2), B =
(0, 8) e C = (8, 8). Ento a equao da circunferncia :
a) (x 4)2 + (y +5)2 = 25
b) (x + 4)2 + (y 5)2 =25
c) (x 5)2 + (y 4)2 = 25
d) (x + 5)2 + (y + 4)2 = 25
e) (x 4)2 + (y - 5)2 = 25
118 - (Unimontes MG) A equao 2 x2 + 2y2 12x + 8y 6 = 0 de uma
circunferncia
a) de centro (3, 2) e raio 4.
b) de centro (3, 2) e raio 16.
c) de centro (3,
d) de centro (3, 2) e raio 2.
a) 720 km. b) 300 km. c) 648 km.
d) 1200 km
e) 126 km.
112 - (Cefet PR) Considere as retas ( r ) 4x 3y + 17 = 0
3y 8 = 0. A distncia entre ( r ) e ( s ) :
a) 17/9.
b) 25/3.
c) 50.
d) 25.
e) 5.

119 - (FGV) A circunferncia da figura seguinte tangente aos eixos x e y


e tem equao x2 + y2 6x 6y + 9 = 0. A rea da superfcie sombreada
e

( s ) 4x

113 - (UFSCar SP) Considere P um ponto pertencente reta (r) de


equao 3x + 5y 10 = 0 e equidistante dos eixos coordenados. A equao
da reta que passa por P e perpendicular a (r)
a) 10x 6y 5 = 0.
b) 6x 10y + 5 = 0.
c) 15x 9y 16 = 0.
d) 5x + 3y 10 = 0.
e) 15x 3y 4 = 0.
114 - (FGV) As retas de equaes
perpendiculares.
O valor de a :

a 1
x 12
a 2

y x 1 e y

so a) 9( 1)
c)

9( 4 )
4

b) 81 9
d)

9(9 4)
4

118

e)

16. O volume do prisma de base triangular ABC e altura 5 de 125 unidades


de volume.
32. A altura do cilindro reto que tem como base o crculo delimitado pela

6 ( 6 )
4

circunferncia que passa pelos pontos A, B e C, e que possui 150


unidades de volume, de 15 unidades de comprimento.
64. O cone de base igual ao crculo delimitado pela circunferncia que passa

120 - (UFPel RS)

pelos pontos A, B e C, e altura 6 unidades de comprimento, possui


unidades de volume.

50

124 - (FGV) No plano cartesiano, a circunferncia que passa pelo ponto


2
2
P(1,3) e concntrica com a circunferncia x y 6x 8y 1 0
tem a seguinte equao:
a) x2 + y2 + 6x + 8y 40 = 0
b) x2 + y2 3x 4y + 5 = 0
c) x2 + y2 6x 8y + 20 = 0
d) x2 + y2 + 3x + 4y 25 = 0
e) x2 + y2 3x + 4y 19 = 0
Os laboratrios de fsica nuclear, at 1930, dispunham de aceleradores de
partculas apenas na forma linear. O inconveniente desses aceleradores que
necessitam uma extenso muito grande para as partculas atingirem altas
velocidades. A partir daquele ano, Ernest Lawrence inventou o cclotron, no
qual as partculas so aceleradas em trajetrias circulares.
Com base no texto e em seus conhecimentos, correto afirmar que uma
partcula que descreve uma trajetria circular sobre uma circunferncia de
equao x2 + y2 16x 12y = 0 percorre, nessa trajetria, uma distncia
igual a
a)

b)

c)

d)

20 u.c
100 u.c
e) 56 u.c

10 u.c
28 u.c

121 - (UEG GO) A equao da circunferncia que aparece no grfico abaixo


pode ser escrita na forma

a)

x 2 10x y2 8y 32 0

b)

x 2 10x y2 8y 38 0

c)

x 2 10x y2 8y 32 0

d)

x 2 10x y2 8y 38 0

122 - (PUC PR) O dimetro de uma circunferncia o segmento da reta


4x 3y 12 0 , situado entre os eixos de coordenadas.
A equao dessa circunferncia :
a) x2 + y2 + 4x + 2y = 0
b) x2 + y2 + 4x 2y = 0
c) x2 + y2 + 3x 4y = 0
d) x2 + y2 4x + 3y = 0
e) x2 + y2 + 8x 6y = 0

125 - (UEL PR) Considere a reta r de equao y2x2=0. Com relao


representao geomtrica da reta r no plano cartesiano, pode-se afirmar:
I. A rea do tringulo formado pela reta r e pelos eixos coordenados tem o
valor de 1 unidade quadrada.
II. A circunferncia de equao x2 + y2 = 2 contm todo o tringulo formado
pela reta r e pelos eixos coordenados.
III. A circunferncia de equao x2 + y2 + 2x 4y=0 tangencia a reta r .
IV. A reta r perpendicular reta 2y+x+10=0 .
A alternativa que contm todas as afirmativas corretas :
a) I e II
b) I e III
c) I e IV
d) II e III
e) II, III e IV

1) A
4) B
7) C
10) E
13) A
16) 99
19) CEEC
22) E
25) E
28) D
31) D
34) 30
37) B
40) D
43) B


46) S ,0 0,
4 2
49) A
52) VFVVF
55) A
123 - (Unioeste PR) Tome trs pontos distintos A ( 6,0) , B (1,5) e
58) B
C ( 4,0) no plano cartesiano. Assim, verdade:
61) A
01. A reta que passa pelos pontos A e B intercepta o eixo y em 8.
02. A circunferncia que passa por esses trs pontos tem equao igual a 64) 12
67) D
(x 1) 2 y 2 25 .
04. A reta s : y x 6 tangencia a circunferncia determinada pelos 70) D
73) A
pontos A, B e C.
08. Os pontos A, B e C determinam uma circunferncia de rea igual a 30 76) B

2) E
5) C
8) A
11) C
14) D
17) C
20) A
23) A
26) D
29) C
32) E
35) C
38) B
41) C
44) D

3) 25kg
6) D
9) C
12) VFFVF
15) B
18) C
21) A
24) D
27) A
30) A
33) E
36) A
39) D
42) A
45) B

47) A

48) B

50) B
53) A
56) E
59) D
62) C
65) A
68) 31
71) B
74) C
77) 27

51) A
54) 31
57) 60) 59
63) C
66) E
69) C
72) E
75) E
78) 600

unidades de rea.

119

79) 60
82) 12
85) B
88) D
91) A
94) A
97) 23/10
100) A
103) 42,5

80) C
83) A
86) C
89) B
92) A
95) B
98) B
101) B
104) C

81) E
84) C
87) B
90) B
93) A
96) A
99) E
102)
105) B

106) B e D
109) D
112) E
115) E
118) C
121) C
124) C

A x N / 3 x 8

04.
16.

AB
AB A

B x R / 3 x 8

02. A = B
08.

AB B

06 - (UDESC SC) Considere os conjuntos:

(UEPG

PR)

A {3, 7, x, 5, 9}

Considere dois conjuntos,


B {1, 5, x, 8, y , 4} .
e

e B, tais
Sabendo-se

que
que

A B {5, 9, 6} , assinale o que for correto.


01.

A B {1, 2, 3, 4, 5, 6, 7, 9}

02.

A B {3, 7}

04.
08.

AB
8 A

16.

x y 15

A ] ,5[

B [3,[ , assinale o que for correto.

01. AB = [3,5[
04. 5 A
16. AB=],3]

O conjunto C A B , :
a) {2, 3, 4, 5}
b) {6, 7}
c) {... -8, -7, -6}
d) {0, 1, 2, 3, 4, 5}
e) {0, 1}
07 - (UEG GO) Na escola do professor Golias, so praticadas duas
modalidades de esportes: o futebol e a natao. Exatamente 80% dos alunos
praticam futebol e 60%, natao.
Se a escola tem 300 alunos e todo aluno pratica pelo menos um esporte,
ento o nmero de alunos que praticam os dois esportes :
a) 240
b) 204
c) 180
d) 139
e) 120
08 - (Cefet PR) Dados os conjuntos A = { 1, 2, 3, 4, 5 }; B = { 4, 5, 6, 7 };

04 - (UEPG PR) Em relao aos intervalos de nmeros reais


e

A {x N/ | x 1 | 4} e

B {x Z / | x 2 | 3} .

02 - (EFOA MG) Em uma cidade com 40.000 habitantes h trs clubes


recreativos: Colina, Silvestre e Campestre. Feita uma pesquisa, foram obtidos
os seguintes resultados: 20% da populao freqenta o Colina; 16% o
Silvestre; 14% o Campestre; 8% o Colina e o Silvestre; 5% o Colina e o
Campestre; e 4% o Silvestre e o Campestre. Somente 2% freqentam os trs
clubes. O nmero de habitantes que no freqentam nenhum destes trs
clubes :
a) 26000
b) 30000
c) 28000
d) 32000
e) 34000
03

108) B
111) A
114) E
117) E
120) A
123) 82

a) 95
b) 100
c) 105
d) 108
e) 120

01 - (UEPG PR) Sejam os conjuntos:


Assinale o que for correto.
01. A B

107) A
110) D
113) A
116) VFVVV
119) C
122) C
125) C

02. {3,6}A
08. 3B

05 - (UEG GO) Considere os dados abaixo.


Uma enquete com os 450 alunos de uma escola para saber os tipos de
calados mais usados apresentou o seguinte resultado:
48% dos alunos usavam sandlia;
22% dos alunos usavam tnis;
30% dos alunos usavam sapato.
Esse resultado foi representado em um grfico de setores:

Tnis
(ngulo )
Sandlia
(ngulo )

C A = { 7, 8, 9 }; C B = { 3, 8, 9 } e A
de elementos do conjunto C :
a) 6.
b) 7.
c) 3.
d) 4.
e) 5.

C = { 4 }, o nmero

09 - (UFMG MG) Em uma pesquisa de opinio, foram obtidos estes dados:


40% dos entrevistados lem o jornal A.
55% dos entrevistados lem o jornal B.
35% dos entrevistados lem o jornal C.
12% dos entrevistados lem os jornais A e B.
15% dos entrevistados lem os jornais A e C.
19% dos entrevistados lem os jornais B e C.
7% dos entrevistados lem os trs jornais.
135 pessoas entrevistadas no lem nenhum dos trs jornais.
Considerando-se esses dados, correto afirmar que o nmero total de
entrevistados foi:
a) 1 200
b) 1 500
c) 1 250
d) 1 350
10 - (UFAL AL) Considere os conjuntos
A ] - , 2], B [ 3, 4] e C ] - 1, 3[ para analisar as afirmativas
abaixo.

Sapato
(ngulo )

00.

A B [ 3, 2]

01.

BC C

02.

A C ] - , - 1

O ngulo no grfico acima mede:

120

03.

[ 2, 5] A

Cursos
Alunos
Apenas A
9
Apenas B
20
Apenas C
10
A e B
13
A e C
8
B e C
18
A, B e C
3

04. O complementar de C em relao a B [ 3, - 1] [3,4]


11 - (UFSCar SP) Nas eleies do dia 1 de outubro passado, dos eleitores
que compareceram s urnas em uma determinada cidade, 29 % deles
votaram, para prefeito, no candidato U, 36 % no candidato V, 25 % no
candidato W e os 20 000 eleitores restantes votaram em branco ou
anularam seus votos. Com base nesses dados, pode-se afirmar que o nmero
de eleitores que votou no candidato V foi:
a) 50 000.
b) 58 000.
c) 72 000.
d) 180 000.
e) 200 000.

Analise as afirmativas seguintes com base nos dados apresentados na tabela.

12 - (Cefet PR)

/ x < 6}.

O conjunto D,

C, :
a) {3, 2, 1, 0, 7, 9}.

I.
II.
III.
IV.

33 pessoas se inscreveram em pelo menos dois cursos.


52 pessoas no se inscreveram no curso A.
48 pessoas se inscreveram no curso B.
O total de inscritos nos cursos foi de 88 pessoas.

A alternativa que contm todas as afirmativas corretas :


a) I e II.
b) I e III.
c) III e IV.
d) I, II e III.
e) II , III e IV.

c) {2, 4, 5}.
d) {3, 1}.
e) {1, 3}.

13 - (Cefet PR) A figura seguinte mostra os conjuntos A, B e C. Nela,


a regio hachurada corresponde a:

16 - (Unimontes MG) Uma cidade com 15000 habitantes tem dois clubes
de futebol: Campos e So Bento. Numa pesquisa feita com seus habitantes,
constatou-se que 1500 pessoas no apreciam nenhum dos dois clubes, 1800
apreciam os dois e 4900 apreciam o Clube Campos. Quantas pessoas
apreciam o Clube So Bento?
a) 10400
b) 8600
c) 13500
d) 3100
17 - (UF Campina Grande PB) Uma escola de Campina Grande abriu
inscries para aulas de reforo nas disciplinas Matemtica, Fsica e Qumica
do 2 ano do Ensino Mdio, sem que houvesse coincidncia de horrios, de
modo que permitisse a inscrio simultnea em mais de uma dessas trs
disciplinas. Analisando o resultado final das inscries, o coordenador
pedaggico constatou:
Dos 62 alunos inscritos para as aulas de Fsica, 22 inscreveram-se
exclusivamente para essas aulas;
38 alunos se inscreveram para as aulas de Matemtica;
26 alunos se inscreveram para as aulas de Qumica;
Nenhum aluno se inscreveu simultaneamente para as aulas de Matemtica
e de Qumica;
O nmero de alunos inscritos exclusivamente para as aulas de Matemtica
o dobro do nmero de alunos inscritos exclusivamente para as aulas de
Qumica.

a) (A B) C
b) (A B) - C
c) (A B) - C
d) (A - C.) B
e) (A B) C
14 - (UFRRJ RJ) Considerando a figura plana no desenho abaixo, correto
afirmar que a regio negrita pode ser representada por:

B
a) (B C) (C A).
c) (C B) (A C).
e) (C B) (C A).

O nmero de alunos inscritos simultaneamente para as aulas de Matemtica


e de Fsica
a) 26
b) 20
c) 18
d) 24
e) 22
18 - (FMTM MG) Em uma amostra de indivduos, 40% foram afetados pela
doena A, 20% foram afetados pela doena B e 5% foram afetados por
ambas as doenas. Dos indivduos da amostra que no foram afetados nem
por A nem por B, 2% morreram. A porcentagem de indivduos da amostra
que morreram sem terem sido afetados por quaisquer das duas doenas
analisadas de
a) 0,7%.
b) 0,8%.
c) 0,9%.
d) 1,0%.
e) 1,1%.

b) (A C) (B C).
d) (C A) (B A).

15 - (UEL PR) Uma Universidade est oferecendo trs cursos de extenso


para a comunidade externa com a finalidade de melhorar o condicionamento
fsico de pessoas adultas, sendo eles:
Curso A: Natao.
Curso B: Alongamento.
Curso C: Voleibol.
As inscries nos cursos se deram de acordo com a tabela seguinte:

1) 29
5) D
9) B
13) E
17) E

2) A
6) A
10) VFVFV
14) E
18) C

3) 26
7) E
11) C
15) B

4) 13
8) E
12) B
16) A

121

01 - (UFMG MG) Um barril cheio, contendo uma mistura com 70% de vinho
puro e 30% de suco, custa CR$ 24.000,00. O preo do litro de vinho puro
CR$ 600,00 e o preo do litro de suco CR$ 200,00. A capacidade do barril,
em litros, :
a) 30
b) 40
c) 50
d) 75
e) 120

e assim por diante.


a) Qual a estimativa do aumento percentual do PIB de 1999 em relao ao
de 1998?
b) Em que ano o PIB ser aproximadamente o dobro do que era em 1996?
Use aproximao por valores superiores e adote os seguintes dados:
log 2 = 0,3010
log 13 = 1,1139

02 - (UERJ RJ) O Cear atravessa a maior seca do sculo. H mais de cinco


meses, Fortaleza vem sofrendo racionamento de gua e estava ameaada
por um colapso no fornecimento, em setembro. Para combater este
problema, o Governo do Estado construiu a maior obra da histria do Cear:
o CANAL DO TRABALHADOR, ligando o rio Jaguaribe ao Aude Pacajus, com
115 quilmetros de extenso.
Para se ter uma idia da dimenso desta obra, basta dizer que ela 18
quilmetros maior que o canal do Panam em extenso, e que representa
um grau da curvatura da Terra.
(Revista VEJA, 22/09/93)
Considere a Terra esfrica e o canal construdo como parte de um crculo
mximo. Com essas informaes e usando o valor 3 para , o raio da Terra,
em km, seria:
a) 20.700
b) 13.800
c) 10.350
d) 6.900
e) 6.300

08 - (PUC SP) Um capital C, aplicado a juros compostos a uma taxa unitria


i por perodo, produz, ao final de n perodos, o montante M, dado por M = C
(1 + i)n. Nessas condies, utilizando-se log 2 = 0,30 e log 3 = 0,48, o
capital de R$ 2000,00, aplicado a juros compostos taxa de 20% ao ano,
produzir o montante de 5000,00, ao final de um perodo.
a) 4 anos
b) 4 anos e 2 meses
c) 4 anos e 8 meses.
d) 5 anos
e) 5 anos e 6 meses

03 - (Unificado RJ) 3 profissionais fazem 24 peas em 2 horas, e 4


aprendizes fazem 16 peas em 3 horas. Em quantas horas 2 profissionais e 3
aprendizes faro 48 peas?
a) 2
b) 3
c) 4
d) 5
e) 6
04 - (UERJ RJ) Leia com ateno:

Voc deve concordar que, em casos como este, justo que cada um pague
proporcionalmente ao que consumiu. A conta foi de 28 (vinte e oito) reais.
Considere que Hagar tenha consumido o triplo do que consumiu o seu
acompanhante; assim, proporcionalmente, Hagar deve pagar:
a) R$ 18,00
b) R$ 19,00
c) R$ 20,00
d) R$ 21,00
e) R$ 24,00
05 - (UFMG MG) A quantia de CR$ 15.000.000,00 emprestada a uma taxa
de juros de 20% ao ms .
Aplicando-se juros compostos, o valor que dever ser pago para quitao da
dvida , trs meses depois, :
a) CR$ 24.000.000,00
b) CR$ 25.920.000,00
c) CR$ 40.920.000,00
d) CR$ 42.000.000,00
e) CR$ 48.000.000,00
06 - (UFRJ RJ) Uma loja oferece duas formas de pagamentos para seus
clientes: vista ou em duas parcelas iguais. A loja anuncia, na sua vitrine,
um vestido por um preo total de R$200,00 para pagamento em duas vezes,
sendo R$100,00 no ato da compra e R$100,00 em trinta dias aps essa data.
Para pagamento vista , a loja oferece um desconto de 10% sobre o preo
total de R$200,00 anunciado na vitrine. Considerando o preo vista como o
preo real do vestido, determine a taxa de juros cobrada pela loja no
pagamento em duas vezes.
07 - (FGV) A funo P = 60 (1,04)t representa a estimativa do Produto
Interno Bruto em bilhes de dlares (PIB) de um pas no ano t adotando-se a
seguinte conveno:
t = 0 representa o ano de 1996
t = 1 representa o ano de 1997
t = 2 representa o ano de 1998

09 - (UFG GO) Uma indstria consome mensalmente 150 m3 de um certo


reagente. Uma unidade dessa indstria passou a produzir esse reagente e,
no primeiro ms de produo, produziu 10% do seu consumo mensal. Se a
unidade aumenta a produo do reagente em 3 m3 por ms, quantos meses
sero necessrios, a partir do incio da produo, para que a unidade
produza, em um nico ms, 70% do volume mensal desse reagente
consumido pela indstria?
a) 21
b) 24
c) 28
d) 31
e) 36
10 - (UFSCar SP) Com o reajuste de 10% no preo da mercadoria A, seu
novo preo ultrapassar o da mercadoria B em R$ 9,99. Dando um desconto
de 5% no preo da mercadoria B, o novo preo dessa mercadoria se igualar
ao preo da mercadoria A antes do reajuste de 10%. Assim, o preo da
mercadoria B, sem o desconto de 5%, em R$,
a) 222,00.
b) 233,00.
c) 299,00.
d) 333,00.
e) 466,00.
11 - (Mackenzie SP) Supondo que, neste Processo Seletivo 2007, a relao
candidato/vaga seja 5,5, e que, para 2008, haja um aumento de 18% no
nmero de candidatos e um aumento de 10% no nmero de vagas
oferecidas, a relao candidato/vaga para 2008 ser de
a) 5,9
b) 5,4
c) 5,7
d) 6
e) 6,1
12 - (UFMG MG) Francisco resolveu comprar um pacote de viagem que
custava R$ 4 200,00, j includos R$ 120,00 correspondentes a taxas de
embarque em aeroportos.
Na agncia de viagens, foi informado de que, se fizesse o pagamento vista,
teria um desconto de 10%, exceto no valor referente s taxas de embarque,
sobre o qual no haveria nenhum desconto.
Decidiu, pois, pagar o pacote de viagem vista. Ento, CORRETO afirmar
que Francisco pagou por esse pacote de viagem
a) R$ 3 672,00.
b) R$ 3 780,00.
c) R$ 3 792,00.
d) R$ 3 900,00.
13 - (UFG GO) Na turma de Mrcia, que cursa o 3 ano do 2 grau h 25
alunos. Aps baterias de testes vocacionais, Mrcia e mais 7 de seu colegas
decidiram pela rea de exatas, 1/5 dos alunos pela rea de biolgicas e 36%
pela rea de humanas.
Com base no exposto, correto afirmar que:
01. 28% dos alunos optaram por exatas;
02. 9 alunos decidiram por humanas;
03. todos os alunos fizeram opo;
04. 20% dos alunos decidiram por biolgicas;
05. 3% dos alunos no fizeram opo.
14 - (UFG GO) Para o proprietrio de um posto distribuidor de combustvel
o preo de custo do lcool Cr$ 320,00 o litro. Sabe-se que o preo de
venda deste combustvel Cr$ 356,00 o litro e que a este preo, vende-se
1200 litros de lcool por dia. Com base nestes dados, pode-se dizer que:
01. o lucro obtido no dia de Cr$ 43.200,00;
02. para dar desconto no preo de venda e obter acrscimo no lucro o
proprietrio deve atrair mais consumidores;
04. o lucro obtido por litro inferior a 11%;

122

08. se o proprietrio der um desconto de Cr$ 10,00 por litro, consegue


vender 1620 litros no dia. Neste caso obtm-se acrscimo no lucro do dia;
16. dando um desconto de x cruzeiros por litro de lcool h um aumento de
60x litros na venda; ento, o valor de x que torna mximo o lucro Cr$ 8,00;
32. para manter o lucro dando um desconto de Cr$ 9,00 por litro
necessrio ampliar a venda para 1600 litros por dia.

d)

4
3

e)

2
3

23 - (UFSC SC) Assinale a(s) proposio(es) CORRETA(S).


01.
Uma avenida em linha reta possui 20 placas de sinalizao
igualmente espaadas. A distncia entre a stima e a dcima placa 1.200
metros. A distncia entre a primeira e a ltima placa 7.600 metros.
Se o preo de uma cesta bsica , hoje, R$ 98,00 e esse valor
15 - (UEPG PR) Em relao P. A. (a, b, c), onde a = m 2, b = m + 5 e c 02.
diminui 2% a cada ms que passa em relao ao valor do ms anterior,
= 3m 4, assinale o que for correto.
ento daqui a nove meses o preo da cesta bsica ser de 100.(0,98)10 reais.
01. a + b um nmero mpar.
04.
Uma cliente levar 12 meses para saldar uma dvida de R$
02. a + b + c maior que 40.
6.400,00 com uma loja de mveis, pagando R$ 500,00 no primeiro ms, R$
04. m mltiplo de 3.
550,00 no segundo ms, R$ 600,00 no terceiro ms e assim por diante.
c
08.
um nmero natural.
08.
Se trs nmeros inteiros positivos no-nulos formam uma
a
progresso aritmtica, e a soma deles igual a 36, ento o valor mximo que
16. A razo da P. A. um nmero mpar.
o maior desses nmeros pode ter 24.
16.
No livro O Cdigo da Vinci, de Dan Brown, no local onde o corpo
de Jacques Saunire encontrado, alguns nmeros esto escritos no cho.
Estes nmeros fazem parte da Seqncia de Fibonacci, que uma seqncia
infinita de nmeros em que cada termo, a partir do terceiro, igual soma
dos dois termos que imediatamente o antecedem. Assim, o dcimo primeiro
16 - (Mackenzie SP) Observe a disposio, abaixo, da seqncia dos termo da Seqncia de Fibonacci 1, 1, 2, 3, 5, 8, 13,... o nmero 79.
nmeros naturais mpares.
24 - (Mackenzie SP) Se a soma dos 20 primeiros termos da progresso
1a linha 1
aritmtica (log x, log x3, ...) 200, o valor de x4
a) 2000
b) 10000 c) 100
2 a linha 3,5
d) 1000
e) 3000
a

3 linha 7,9,11

4 a linha 13,15,17,19
5 linha 21,23,25,27,29
............
...............................
a

O quarto termo da vigsima linha


a) 395
b) 371
d) 401
e) 399

25 - (IME RJ) Um quadrado de lado igual a um metro dividido em quatro


quadrados idnticos. Repete-se esta diviso com os quadrados obtidos e
assim sucessivamente por n vezes. A figura abaixo ilustra as quatro
primeiras etapas desse processo. Quando n , a soma em metros dos
permetros dos quadrados hachurados em todas as etapas :

c) 387

17 - (Furg RS) Se k o quinto termo da progresso aritmtica (log104,


log1012, log1036,...) ento 10k dado por:
a) 344.
b) 314.
c) 304.
d) 324.
e) 348.
18 - (FGV) Observe as cinco primeiras figuras de uma seqncia infinita.

O nmero de quadradinhos escuros da figura que ocupa o 59. lugar nessa


seqncia
a) 3 481.
b) 1 741. c) 900.
d) 841.
e) 600.
19 - (UF Campina Grande PB) Num perodo de 10 meses consecutivos,
uma fbrica deseja produzir 60.000 pares de calados, de modo que a
produo a cada ms (a partir do segundo) seja 900 pares a mais, em
relao ao ms anterior. Nessas condies, a produo ao final do primeiro
ms deve ser de
a) 1.980 pares
b) 1.950 pares
c) 1.910 pares
d) 1.890 pares
e) 1.850 pares
20 - (EFOA MG) Para angariar recursos para formatura, uma turma de 3
ano do ensino mdio de um colgio organizou uma rifa, cujos bilhetes foram
numerados de 3 em 3, de 100 a 997. Sabendo-se que os bilhetes foram
vendidos a R$ 8,00 cada um e que foram vendidos 92% do total de bilhetes,
o valor arrecadado com a rifa, em reais, foi:
a) 2304
b) 2128
c) 2248
d) 2136
e) 2208
21 - (UFG GO) A cada dia, um atleta em treinamento deseja correr x metros
a mais que no dia anterior. Considere que, no primeiro dia, ele correu 10 km
e pretende correr 42 km no 21o dia. Nessas condies, o valor de , x em
metros, dever ser de
a) 1.200 b) 1.300
c) 1.400
d) 1.500
e) 1.600
22 - (UFC CE) Um tringulo tem os lados em progresso aritmtica e seu
crculo circunscrito tem raio medindo 25 cm. A tangente do ngulo oposto
ao maior cateto vale:
a)

3
4

b) 1

c)

1
2

a) 4
b) 6
c) 8
d) 10
e) 12
26 - (Unipar PR) Um peregrino caminha sempre 400 metros a mais do que
no dia anterior rumo ao templo sagrado. Ao final de onze dias ele havia
caminhado 35,2 km. A distncia, em metros, que ele caminhou no ltimo dia
foi de:
a) 4.800
b) 4.900
c) 5.200
d) 5.500
e) 5.600
27 - (Unipar PR) Conta a histria da Matemtica que, ainda criana, Gauss
solucionou o seguinte problema em alguns minutos. O problema consistia em
dar o resultado da soma:
1 + 2 + 3 + 4 + .......... + 98 + 99 + 100 = X
Podemos afirmar que o valor de X igual a:
a) 11.000
b) 10.001
c) 9.990

123

d) 9.900
e) 5.050
28 - (EFOA MG) Para arrecadar doaes, uma Entidade Beneficente usou
uma conta telefnica do tipo 0800. O nmero de pessoas que ligaram, por
dia, variou de acordo com uma progresso aritmtica de razo 4. Sabendo-se
que cada doao foi de R$ 0,40 e que no primeiro dia duas pessoas ligaram,
o nmero mnimo de dias a fim de que o total arrecadado atingisse o valor de
R$ 81.920,00 foi:
a) 230
b) 280
c) 250
d) 320
e) 300

d)

29 - (UFBA BA) Considerando que os nmeros reais a, b e c formam, nessa e)


ordem, uma progresso geomtrica e satisfazem a igualdade

log 2 a

1
2 log 4 c 9 , determine o valor de b.
log b 2

30 - (Mackenzie SP) Se

(1 senx, 1 cos x , 1 senx) , 0 x

uma progresso geomtrica,

1
a)
2
b)
c)
d)
e)

3
2
1

2
3
2
2

cos2x

vale

,
2

32 - (UEG GO) Em uma progresso geomtrica, a razo 3, o primeiro


termo 4 e o ltimo termo 8.748. Essa progresso possui:
a) 14 termos
b) 12 termos
c) 10 termos
d) 8 termos
33 - (UEPG PR) Em relao seqncia
termo geral dado por
01.
02.
04.
08.
16.

(a1 , a 2 , a 3 , , a n , ) ,

cujo

a n n 2(n 3) , assinale o que for correto.

uma P.A. de razo 3.


O primeiro termo um nmero negativo.
uma P.G. de razo 3.
O 5 termo um nmero natural quadrado perfeito.
de termos decrescentes.

34 - (PUC MG) Os nmeros reais a e b so tais que a seqncia

a, b, 2a b uma progresso aritmtica e a seqncia

3 , 27, 3
a

uma progresso geomtrica. Ento o valor de a :


a) 1,5
31 - (PUC RS) Uma Progresso Geomtrica tem n termos, ( a1, a2, ... , an ). b) 2,5
Os pontos ( 1, a1 ), ( 2, a2 ), ... , ( n, an ) podem localizar-se sobre o grfico c) 3,5
de:
d) 4,5
35 - (Fepecs DF) Se
a)

b)

c)

1 2 3
4
n

...... n .....
3 9 27 81
3

ento s

igual a:
a) 1/2
b) 2/3
c) 3/4
d) 8/9
e) 1
36 - (UFPel RS) Em 1970, poca em que a populao brasileira crescia, a
cada 40 anos, numa progresso geomtrica de razo q, ramos 90 milhes
de brasileiros.
Esse crescimento foi alterado devido a transformaes ocorridas nas famlias
brasileiras, como a entrada das mulheres no mercado de trabalho e o
planejamento familiar, principalmente com a popularizao dos
anticoncepcionais. Nesse novo contexto, o Instituto Brasileiro de Geografia e
Estatstica (IBGE) projeta, para o ano de 2050, uma populao de 260
milhes de brasileiros, 302,5 milhes de pessoas a menos do que teramos se
a populao tivesse mantido o crescimento na mesma progresso geomtrica
de razo q, a cada 40 anos.
www.ibge.gov.br
Com base nos textos e em seus conhecimentos, correto afirmar que a
razo q igual a
a) 2.
b) 1,4.
c) 3,12.
d) 6,25.
e) 2,5.
37

(UFSCar

SP)

conjunto

soluo

da

equao

8 8 8
sen
... cos x, com x [0,2] ,
9 27 81

124

a)

2 4
,
3 3

b)

5 7
,
6 6

c)

3 5
,
4 4

d)

11
,

6 6

e)

5
,
3 3

38 - (UEPB PB) Se a soma dos termos da P.G.

1 1

1, , 2 ,....
x x

com x > 1, o valor de x igual a:


a)
c)
e)

7
6
5
4
4
3

b)

1
f (x)
2

c)

1
f (x)
3

d)

1
f (x)
4

e) (2)x
igual a 4, 41 - (UFPA PA) Um professor de Matemtica Aplicada enviou a seguinte
mensagem ao seu melhor aluno, um estudante chamado Nicphoro, que
gostava muito de desenhar e traar grficos:

3
2
6
d)
5
b)

Prezado Nicphoro,
Estive analisando cuidadosamente aquele problema de Matemtica e percebi
que ele regido por uma funo pulso-unitrio definida por

1 se x 1
f (x)
0 se x 1

39 - (UFMS MS) Considere a seqncia infinita de circunferncias tangentes


interiores no ponto A, a seguir, obtida de tal forma que, a partir de uma
circunferncia, que tangencia os eixos coordenados nos pontos A e B,
determina-se a prxima circunferncia tal que seu dimetro seja igual ao raio
da anterior e o centro da circunferncia anterior pertena circunferncia
imediatamente posterior, e assim sucessivamente, como ilustrado na figura
seguinte:

Trace, por favor, usando os seus conhecimentos, o grfico desta funo e o


envie para mim.
Um abrao e saudaes matemticas.
Euclides Arquimedes.
Nicphoro traou corretamente o grfico da funo acima e o enviou ao prof.
Euclides Arquimedes.
O grfico enviado foi

b)
a)

Assinale a(s) alternativa(s) correta(s).


01.
As medidas dos raios de todas as circunferncias, em ordem da
maior para a menor, formam uma progresso geomtrica de razo 2.
02.
As medidas das reas de todas as circunferncias, em ordem da
maior para a menor, formam uma progresso geomtrica de razo

1
4

04.
As medidas dos comprimentos de todas as circunferncias, em
ordem da maior para a menor, formam uma progresso aritmtica de razo

1
2

d)
c)

e)

08.
Os valores das abscissas dos centros de todas as circunferncias,
em ordem da maior para a menor, formam uma progresso geomtrica de
razo 2.
16.
A soma das reas de todas as circunferncias da seqncia
infinita

4
3

42 - (ESPM SP) O grfico em destaque representa uma funo real y = f


(x). Entre as alternativas dadas, assinale a que melhor representa a funo

y f(x 1)

da rea da primeira circunferncia.

40 - (FGV) No grfico seguinte esto representados os trs primeiros


trapzios de uma seqncia infinita. Pelos vrtices A, B, C, D desses
trapzios passa o grfico de uma funo exponencial f(x) = a x. Se a rea
total dos infinitos trapzios dessa seqncia

5
6

, ento

a) f(x) = 3x

125

a)

b)

a)

d)
c)

b)

e)
c)

46 - (UDESC SC) A soluo da inequao

(x 1) 2 3

a) x 2 ou x 4
b) x > 4
c) x > 0
d) 2 < x < 4
e) x < 2 ou x > 4

d)

47 - (UEG GO) Dada a funo real

e)

f (x ) sen x

, faa o que se pede:

3
A 0, , , ,2
2
2

a)

Determine a imagem do conjunto

funo f.
b)

Esboce o grfico de f para 0 x 2.

pela

48 - (UNIFOA MG) Seja f a funo definida no intervalo aberto (1, +1)


por f(x) =
43 - (UFG GO) Sejam

f (x) x e g(x) x x - 2x 1

funes definidas

para todo nmero real. O nmero de pontos de interseo, entre os grficos


de f e , g :
a) zero
b) um
c) dois
d) trs
e) quatro
44 - (UFRN RN) Sendo

f ( x ) | x 2 2x | ,

o grfico que melhor

representa f :
a)

b)

x
1 x

1
:
3

. Ento f

a) 2
b)

1
2

c)

d)

1
3

e)

1
2

1
3

49 - (Unifor CE) Se x > 4, quantos nmeros inteiros satisfazem a sentena

20 5x
4x

c)

8x - 136?

a) 10
b) 11
c) 12
d) 13
e) 14

d)

50 - (USP SP) Resolver a inequao |x2 4| < 3x.


51 - (UFPA PA) Um professor de Matemtica props o seguinte problema
aos seus alunos:
Determine o valor preciso da seguinte expresso, em que os algoritmos so
todos calculados na base 10 (logaritmos decimais):

45 - (UFAM AM) O grfico que melhor representa a relao


para todo x,

y R , :

| y | x 1 ,

1
2
3
4
x log log log log
2
3
4
5
5
6
7
8
9
log log log log log
6
7
8
9
10

126

Os alunos que resolveram corretamente esta questo concluram que


a)x = 1/2
b)x = 1
c)x = 2
d)x = 2
e)x = 1
52 - (ITA SP) Sejam x, y e z nmeros reais positivos tais que seus
logaritmos numa dada base k so nmeros primos satisfazendo
logk ( xy ) = 49,
logk ( x / z) = 44.
Ento, logk (xyz) igual a
a)52.
b)61.
c)67.
d)80.
e)97.
53 - (UFRR RR) Sejam a e b nmeros reais positivos tais que

log b (5 ab) 5 . Ento:


a)logb a = 25
b)logb a = 25
c)logb a = 10
d)logb a = 24
e) log b a

log 2 a e log 3 b , ento o valor de x em 8 x 9

2b
3a
2a
b)
3b
b
c)
a
a
d)
b
3b
e)
2a
a)

1
log , sabendo que a e b so razes
ab

x 2 7 x 10 0 ,

concluir que o valor da expresso (log A)2 (log B)2 igual a:


a)21
b)4
c)10
d)40
60

(UFAM

AM)

Considere

as

funes

f(x)

log3

(9x2)

1
g(x ) log 3 , definidas para todo x > 0. Ento, 1 + f(x) + g(x) igual
x

61

(UEM

PR)

Os

valores

2log 3 x log 9 x log 81 3

de

que

satisfazem

equao

so:

c)

3 e

4 3
3

d)

3 e

4 27
3

e) 4

3 e

3
3

62 - (FGV) Considere a matriz

log x
A x
log 3 1

log 3 9

log 9 3

com x R, x > 0 e

(1) 6x + 3 = 0
2

d) 1

(2)

1
2

x 0
2

(3) 9x 3 = 0

56 - (UEPB PB) O valor de

80,666 log 2 0,5

igual a:

a) 4
b) 2
c) 1
d) 3
e) 5
57 - (FGV) Daqui a t anos, o nmero de habitantes de uma cidade ser
N 40 000(1,02)t . O valor de t para que a populao dobre em relao a
de hoje :

log 2
log1,02

b)50
c)(log 2)(log 1,02)
d) 2

A
3 , pode-se
B

x 1 e seja n, o determinante de A.
Considere as equaes:

1
c)
2

a)

log

1
1
a)
e 2
4
1
1
b)
e
2
4

55 - (Mackenzie SP) O valor de

e)

admite em R:

59 - (UFRN RN) Sabendo-se que log AB = 7 e

da equao
a) 2
b) 1

- 9log9 3x 0

a:
a)1 = log3 x
b)3 + log3 x
c)3 log3 x
d)1 log3 x
e)3 log3 x

25

54 - (PUC RS) Se

log3 x2

58 - (UEPB PB) A equao 3


a)uma nica raiz menor que 5
b)duas razes diferentes
c)duas razes positivas
d)uma nica raiz maior que 5
e)duas razes maiores que 4

log 2
log1,02

e)2(log 2)(log 1,02)

1
4
1
2
(5)x =
2
(4)x2 =

Pode-se afirmar que n raiz da equao


a) (1).
b) (2).
c) (3).
d) (4).
e) (5).
63 - (Mackenzie SP) Se (x,y) a soluo do sistema
x

y
3

3 3

log( x 1) log y
log 3

, o valor de

xy

a)5

127

b)6
c)7
d)8
e)9
64 - (UFPel RS) A natureza dotou a espcie humana de uma sensibilidade
auditiva que diminui com o aumento do nvel da presso sonora.
O nvel de presso sonora (NPS) pode ser definido pela expresso:

P
NPS 20 log
P0

em que P o valor da presso medida e P 0 a presso de referncia, isto ,


a menor presso percebida pelo ouvido humano (P 0 = 2 . 105), medidas em
Pascal.
http://portalteses.cict.fiocruz.br acessado em 25/04/2006
Com base no texto e em seus conhecimentos, correto afirmar que,
considerando log 2 = 0,3, a expresso NPS pode ser escrita como
a)20 log P + 1,5
b)20 log P - 1,5
c)20 log P + 94.
d)log P + 9,4
e)5 log P + 20

Seja N o maior nmero dessas caixas de som que podem ser ligadas,
simultaneamente, sem que se atinja o nvel de 115 dB, que o mximo
suportvel pelo ouvido humano.
Ento, CORRETO afirmar que N
a)menor ou igual a 25.
b)maior que 25 e menor ou igual a 50.
c)maior que 50 e menor ou igual a 75.
d)maior que 75 e menor ou igual a 100.
69 - (UFRN RN) A escala decibel de som definida pela seguinte
expresso:

67 - (Mackenzie SP) A figura mostra os esboos dos grficos das funes


f(x) = 22x e g(x) = log2 (x + 1).

Nessa expresso, B o nvel do som, em decibis (dB), de um rudo de


intensidade fsica I, e Io a intensidade de referncia associada ao som mais
fraco percebido pelo ouvido humano.
De acordo com a expresso dada e a tabela abaixo, pode-se concluir que, em
relao intensidade de uma conversao normal, a intensidade do som de
uma orquestra

Som

65 - (Fepecs DF) Se x = log104 + log1025, ento x igual a:


a)1;
b)2;
c)log1029;
d)log1025/4;
e)1,4020.
66 - (Cefet PR) Dados log 2 = 0,301 e log 3 = 0,477, o valor mais
prximo de x real na equao 3 + 6x . 4 = 183 :
a)1,93.
b)2,12.
c)2,57.
d)2,61.
e)2,98.

I
B 10 log
I0

Nvel do som em

Som mnimo

dB
0

Raspagem de folhas

10

Sussurro

20

Conversao normal

60

Banda de rock

80

Orquestra

90

Mximo suportvel

120

a)1000 vezes superior.


c)100 vezes superior.

b)200 vezes superior.


d)2000 vezes superior.

70 - (UFRRJ RJ) A intensidade M de um terremoto medida na escala


Richter um nmero que varia de M = 0 at M = 8,9 para o maior
terremoto conhecido. M dado pela frmula emprica (baseada apenas na
experincia)

E
2
1og 10
3
E0

sendo E a energia liberada no terremoto em

quilowatt-hora (Kwh) e E0 = 7x103 kwh.


Sabe-se que uma cidade utiliza cerca de 2x10 5 kwh de energia eltrica por
dia.
Se a energia liberada por um terremoto pudesse ser, de alguma forma,
transformada em energia eltrica, quantos dias de fornecimento de energia
eltrica seriam produzidos para essa cidade por um terremoto de intensidade
M = 6?
71 - (Fatec SP) Na figura abaixo est representada a funo real f, dada
por f ( x ) log a x , para todo x 0 .

A rea do tringulo ABC

1
4
5
b)
2
3
c)
2
a)

d)3
e)

1
3

68 - (UFMG MG) Em uma danceteria, h um aparelho com vrias caixas de


som iguais. Quando uma dessas caixas ligada no volume mximo, o nvel R
de rudo contnuo de 95 dB.
Sabe-se que
R = 120 + 10.log10Is , em que Is a intensidade sonora, dada em watt/m2;
e
a intensidade sonora Is proporcional ao nmero de caixas ligadas.

De acordo com os dados da figura, correto concluir que a rea do trapzio


ABCO, em unidades de superfcie,
a)4
b)4,5
c)5
d)5,5
e)6
72 - (UFSC SC) Assinale a(s) proposio(es) CORRETA(S).
01.
Dentre todos os retngulos com 40m de permetro, o de maior
rea aquele com lado de 20m e rea de 400m 2.
02.
O grfico abaixo mostra quanto cada brasileiro pagou de impostos
(em reais per capita) nos anos indicados.
04.
Em certa fbrica, durante o horrio de trabalho, o custo de
fabricao de x unidades de C(x) = x2 + x + 500 reais. Num dia normal
de trabalho, durante as t primeiras horas de produo, so fabricadas x(t) =
15t unidades. O gasto na produo, ao final da segunda hora, de R$
1.430,00.

128

08.
Certa substncia radioativa que se desintegra uniformemente ao
longo do tempo tem sua quantidade ainda no desintegrada, aps t anos,
dada por

M(t) M 0

.2 20

onde M0 representa a quantidade inicial dessa

substncia. A porcentagem da quantidade ainda no desintegrada aps 40


anos em relao quantidade inicial M0 de, aproximadamente, 50%.
16.
Uma cidade servida por trs empresas de telefonia. A empresa
X cobra, por ms, uma assinatura de R$ 35,00 mais R$ 0,50 por minuto
utilizado. A empresa Y cobra, por ms, uma assinatura de R$ 20,00 mais R$
0,80 por minuto utilizado. A empresa Z no cobra assinatura mensal para at
50 minutos utilizados e, acima de 50 minutos, o custo de cada minuto
utilizado de R$ 1,20. Portanto, acima de 50 minutos de uso mensal a
empresa X mais vantajosa para o cliente do que as outras duas.

77 - (UFJF MG) Dada a equao


que sua soluo um nmero:
a)natural.
b)maior que 1.
c)de mdulo maior do que 1.
d)par.
e)de mdulo menor do que 1.
78 - (UFAM AM) Seja
x 2 2

5
1

125
25

2 x

. Ento,

23x 2 8 x 1 4 x 1 ,

podemos afirmar

o menor nmero que soluo da equao

um nmero

a)par
b)primo
c)no real
d)divisvel por 5
e)irracional
79 - (UEL PR) Um barco parte de um porto A com 2k passageiros e passa
pelos portos B e C, deixando em cada um metade dos passageiros presentes
k

Com base nos dados fornecidos pelo grfico, pode-se afirmar que no ano
2000 houve um aumento de 20% no gasto com impostos, em relao a
1995.
73 - (UEPG PR) Sendo A o conjunto das razes da equao 3x + 31x = 4,
assinale o que for correto.
01. A N A
02. A

{1,0,1,2}

04.A soma dos elementos do conjunto A 4.


08.A um conjunto unitrio.
16.Se B = {0,1,2,3,4,5}, ento A B {0,1}
x 2 2x
74 - (UEPB PB) O conjunto soluo da inequao (0,04) 2

0,008

igual a:
a) S x

R / x 3
b) S x R / x -1 ou x 3
c) S x R / x 1 x 3
d) S x R / x 1 ou x 3
e) S x R / - 1 x 3
75 - (UEPG PR) A equao
os nmeros a e b, com
01

5 2 x 125 6.5 x 1 ,

no momento de chegada, e recebendo, em cada um, 2 2 novos passageiros.


Se o barco parte do porto C com 28 passageiros e se N representa o nmero
de passageiros que partiram de A, correto afirmar que:
a)N mltiplo de 7
b)N mltiplo de 13
c)N divisor de 50
d)N divisor de 128
e)N primo
80 - (FGV) A posio de um objeto A num eixo numerado descrita pela lei

1 7 0, 5 t
2
8 8

objeto B, de acordo com a lei 2t.


Os objetos A e B se encontraro num certo instante tAB. O valor de tAB, em
segundos, um divisor de:
a)28.
b)26.
c)24.
d)22.
e)20.
81 - (UEG GO) Em uma pesquisa, aps n meses da constatao da
existncia de uma epidemia, o nmero de pessoas por ela atingidas era

f (n )

admite como solues

onde t o tempo em segundos. No mesmo eixo, move-se o

40.000
2 15 4 2n

. Nestas condies, o tempo para que a epidemia

atinja pelo menos 4.000 pessoas de aproximadamente


Dados: log 2 = 0,3 e log 3 = 0,48
a)9 dias.
b) 8 dias.
c)7 dias.
d) 5 dias
82 - (UFSCar SP) Para estimar a rea da figura ABDO (sombreada no
desenho), onde a curva AB parte da representao grfica da funo f(x) =
2x, Joo demarcou o retngulo OCBD e, em seguida, usou um programa de
computador que plota pontos aleatoriamente no interior desse retngulo.

a b . Ento, assinale o que for correto.

b
1
a

02a.b um nmero par.


04a > 0 e b < 0
08a + b < 5
16

a
b

um nmero natural.

76 - (UEPB PB) O valor de x na inequao exponencial


dado por:
a) x 2

d)

x 2
x2
x2

e)

b)
c)

1
2

5
0,16 .
2

Sabendo que dos 1000 pontos plotados, apenas 540 ficaram no interior da
figura ABDO, a rea estimada dessa figura, em unidades de rea, igual a
a)4,32.
b)4,26.
c)3,92.
d)3,84.
e)3,52.

129

83 - (UFBA BA) A temperatura Y(t) de um corpo em funo do tempo t


0, dado em minutos varia de acordo com a expresso Y(t) = Ya + Bekt,
sendo Ya a temperatura do meio em que se encontra o corpo e B e k
constantes.
Suponha que no instante t=0, um corpo, com uma temperatura de 75C,
imerso em gua, que mantida a uma temperatura de 25C .
Sabendo que, depois de 1 minuto, a temperatura do corpo de 50C, calcule
o tempo para que, depois de imerso na gua, a temperatura do corpo seja
igual a 37,5C.

32.A norma de z z = 4.
89 - (Integrado RJ) As imagens dos complexos z = x + yi tais que |z - (2
+ 3i)| = 4 formam, no plano complexo, uma ...
a)reta.
b)parbola.
c)circunferncia com centro no primeiro quadrante e raio 2.
d)circunferncia com centro no primeiro quadrante e raio 4.
e)circunferncia com centro no quarto quadrante e raio 2.

84 - (UEPB PB) Os tomos de um elemento qumico radioativo possuem


uma tendncia natural de se desintegrarem, diminuindo, portanto, sua
quantidade original com o passar do tempo. Suponha que certa quantidade

90 - (UFU MG) O conjunto-soluo da equao 2|z|2 + 3z 4 z = 8 + 7i :


a){1 + 2i, 1 (3/2)i}
b){2 i, 3 + 2i}
de um elemento radioativo, com massa inicial m 0 (gramas), com m 0 0 , c){2 + i, (3/2) + i}
d){3 + i, i}
t

e){4 + i, 3 + i}
decomponha-se conforme o modelo matemtico m( t ) m 0 10 70 , em
91 - (Integrado RJ) Considere u = 2 + 2i e v = 2 2i. Ento, u28 . v27
que m(t) a quantidade de massa radioativa restante no tempo t(anos). igual a:
Usando a aproximao log102=0,3, a quantidade de anos para que esse a)2 2i
b)-2 + 2i
1
elemento se decomponha at atingir
da massa inicial ser:
c)2 + 2i
8
d)2 2i
a)60
e)i
b)62
c)64
92 - (Gama Filho RJ) z um complexo diferente de zero. Os pontos A, O e
d)63
B so as imagens, no plano complexo, de z, 0 e iz, respectivamente. O
e)70
ngulo AB mede:
a)90
85 - (Unesp SP) A temperatura mdia da Terra comeou a ser medida por
b)60
volta de 1870 e em 1880 j apareceu uma diferena: estava (0,01) C (graus c)45
Celsius) acima daquela registrada em 1870 (10 anos antes). A funo t(x) =
d)30
(0,01).2(0,05)x, com t(x) em C e x em anos, fornece uma estimativa para o
e)0
aumento da temperatura mdia da Terra (em relao quela registrada em
1870) no ano (1880 + x), x 0 . Com base na funo, determine em que
93 - (Unimep RJ) O valor de (1 + i)10 onde i a unidade imaginria, :
ano a temperatura mdia da Terra ter aumentado 3 C. (Use as
aproximaes log2(3) = 1,6 e log2(5) = 2,3).

86 - (Unicamp SP) O decaimento radioativo do estrncio 90 descrito pela


funo

P(t ) P0 2 bt , onde t um instante de tempo, medido em anos,

b uma constante real e P0 a concentrao inicial de estrncio 90, ou seja,

a concentrao no instante t = 0.
a)
Se a concentrao de estrncio 90 cai pela metade em 29 anos,
isto , se a meia-vida do estrncio 90 de 29 anos, determine o valor da
constante b.
b)
Dada uma concentrao inicial P0, de estrncio 90, determine o
tempo necessrio para que a concentrao seja reduzida a 20% de P 0.
Considere

log2 10 3,32 .

87 - (UERJ RJ) O valor de

1 2i
1 i

94 - (Unificado RJ) Se Z = 1 + i, o argumento principal do complexo

1
z2

:
a)zero
b)/4
c)/2
d)
e)3/2
95 - (UFSC SC) Determine o valor de x para que o produto (12 2i) [18 +
(x 2) i] seja um nmero real.

3 1
i.
2 2
3 1
i.
b)
2 2
3 1
i.
c)
2 2
3 1
i.
d)
2 2

96 - (FEI SP) Seja o nmero complexo

a)

z 1 3i .

Escreva na forma

algbrica o complexo z-1.


97 - (FEI SP) Seja ao nmero complexo

z 1 3i . Escreva o complexo

z na forma trigonomtrica.
98 - (UFSM RS) O mdulo do complexo cos a i . sen a :
a)1
b)i
c)i
d) i4
e)n.d.a

e)3.

99 - Considere o nmero complexo

88 - (UFSC SC) Dados

z 1 i 3 ,

determine a soma dos nmeros

associados (s) afirmaes verdadeira(s):

z 1 i 3

02.O quadrado de z

z 2(1 3i)

. A forma trigonomtrica ou

a)z = 2(cos + i . sen )

z 1 i 3

08.O produto de z pelo seu conjugado


16.O mdulo de z z = 10.

z 3 i

polar de z :

b)z = 2(cos - i . sen )

01.O conjugado de z

04.O oposto de z

a)64 i
b)128 i
c)32 i
d)-32 i
e)nenhuma das anteriores

z.z 4

c)z = cos - i . sen

3
3

d)z = cos
+ i . sen
6
6
e)n.d.a

130

100 - (Mapofei SP) Determinar as razes quadradas do nmero complexo z


= 5 12i.

1
A
1

101 - (FEI SP) Achar o mdulo e o ngulo polar (argumento) do nmero


3
complexo z 1 i .

1 i

102 - (Santa Casa SP) Se os nmeros complexos z1 e z2 so tais que z1 =


2 (cos 135 + i . sen 135) e z2 = z1 2, ento o mdulo de z2 igual a:
109 - (USP SP) Sejam
a) 2

b) 2

c) 2

3 2

d) 4 2
e) 2

z 2 3 2i

1 2

A
1 4

b)150
d)300

104 - (Cesgranrio RJ) A inversa da matriz

3
1

c) inexistente

2 -1

B
x y

duas matrizes. Se

16 1 10

13 1
8
11 1 7

4 3

:
- 1 - 1

1 3

b .
-1 - 4

111 - (UFPB PB) A inversa da matriz

-1 1
d . 4 3
1 - 1

A 1

105 - (Mackenzie SP) Seja a matriz

k . Se A = A-1, os valores de

cotg cosc

A
cosc cotg

1 0 1
A 0 1 0 a matriz

2 3 4

4 x 1
1
0 2 0 . Ento, o valor de x
2

2 3 1

a)-1
b)0
c)1
d)3
com e)2

so:

112 - (Poli SP) Dadas as matrizes

k
a)
, k inteiro
2

2 1
1
2 1

e B
A
0

1
2

1 0

e sendo At a matriz transposta de A, calcular At . B.

b)2k, k inteiro
c)todos os nmeros reais
d)inexistentes
e)n.d.a

106 - (Fuvest SP) A matriz

Lembrando que A . A-1 = I3, a segunda linha de A :


a)(1 1 1)
b)(3 -2 -2)
c)(2 1 3)
d)(0 -1 -1)
e)(2 -2 3)

- 4 -3

e .
-1 1

113 - (Osec SP) Em

sen cos 0 1

sen cos 0 0
sen
1
0 0

0
0
1
0

inversvel, se e

somente se:

y3

y 2

3x -y

4x 2y

4 0

, x e y valem,
5 - 1

respectivamente:
a)4 e -1
b)4 e 1
c)4 e 0
d)1 e 1
e)1 e 0

2
a)
4
3
b)
4
3
c)
3

c. n /n z
2

d. n/n Z
4
e. R

107 - (Faap SP) Dadas as matrizes

x 2
2
x

114 - (Med-Jundia SP) A matriz transposta da matriz quadrada A = (aij)


de ordem 2 com aij = ij + 2, 1 i 2, 1 j 2, :

a. n / n Z
b. 2n /n Z

calcular A . B + A-1.

6 5

5
3

110 - (Mackenzie SP) A matriz inversa da matriz A

2 2

103 - (USP SP) O argumento do nmero complexo

1
a. 4
1

3 x
3 4

B a inversa de A, ento x + y, vale:


a)3/2
b)1/2
c)1
d)1
e)zero

a)120
c)210
e)330

2 3

3 2
A

7 5

1 1
B

- 1 1

108 - (Mau SP) Determine as condies que x deve satisfazer para que a
matriz A seja invertvel.

3
d )
6
2
e)
4

6
4

4
3

131

115 - (UFRN RN) Dadas as matrizes

1 2
1 3 2

A 3 4 e B
,
2 0 1

5 6

ento a matriz A Bt :

igual

sua transposta, o valor de x + 2y :


a)-20
b)1
c)1
d)13
e)20

0 1
a

1 4
2 0

b 0 4
3 5

119 - (Unifor CE) Assinale a alternativa verdadeira

1 0 0

a) 1 1 0 uma matriz diagonal.

1 1 1
1 1
b)
uma matriz triangular.
0 1

2 0 3
c

0 4 5
2 1
d

1 4

0 0

b 0 4
3 5

116 - (PUC RJ) Se

118 - (UEL PR) Sabendo-se que a matriz

5
x2 2 y

3x
49 y
1 21
0

0 1
uma matriz identidade
1 0
1 0
d)
uma matriz simtrica
1 1
c)

5
25


A 12 , B - 8
3
13

- 1

C 10
-1

ento a matriz X

tal que A + B C X = 0 :

31

a -6
17

0 0

1 1

e)

uma matriz transposta da matriz

1 0
1 0

120 - (UFOP MG)


a) Dadas as matrizes:
A = (aij)3x4, com

17

b -6
31

com

- 31

c -6
- 17

21

d -6
17

1, se,i j 4
a ij
, e, B = (b )
1, se,i j 4

ij 4x3

1, se, i j 4
, calcule o determinante de AB.
bij
1, se, i j 4

b) Observe a matriz

1 2 3
0 x 4
0 0 y

. Chama-se trao de uma matriz quadrada

a soma dos elementos de sua diagonal principal. Determine x e y na matriz


acima de tal forma que seu trao valha 9 e seu determinante 15.
121 - (PUC Campinas) A matriz A = (aij) quadrada de ordem 2

31

e 0
17

com

117 - (Unesp SP) Considere trs lojas, L1, L2 e L3 , e trs tipos de produtos,
P1, P2 e P3. A matriz a seguir descreve a quantidade de cada produto vendido
por cada loja na primeira semana de dezembro. Cada elemento aij da matriz
indica a quantidade do produto Pi vendido pela loja Lj , i, j = 1, 2, 3.

a ij 2i j para i j

a ij 3i 2 j para i j

O determinante de A igual a:
a)1
b)2
c)4
d)5
e)6
122 - (Mackenzie SP) Se

L1 L 2 L 3
P1 30 19 20

P2 15 10 8
P3 12 16 11
Analisando a matriz, podemos afirmar que
a)
a quantidade de produtos do tipo P2 vendidos pela loja L2 11.
b)
a quantidade de produtos do tipo P1 vendidos pela loja L3 30.
c)
a soma das quantidades de produtos do tipo P3 vendidos pelas
trs lojas 40.
d)
a soma das quantidades de produtos do tipo Pi vendidos pelas
lojas Li, i = 1, 2, 3, 52.
e)
a soma das quantidades dos produtos dos tipos P1 e P2 vendidos
pela loja L1 45.

- sen x
0
0
a)0
c)/4
e)/3

0 x 2 , o maior valor de x tal que:

8
-5
- sen x cotg x 0
0
cos x

b)/6
d)/2

123 - (FGV) O determinante associado matriz

6
1 11

2
4

3 nulo

3 7
2

porque:

132

a)tem duas linhas proporcionais


b)tem duas colunas proporcionais
c)tem elementos negativos
d)uma coluna combinao linear das outras
e)n.d.a

03. O grfico da funo

h(x)

sempre voltada para baixo.

f(x)
,
x 1

para x 1, tem concavidade

129 - (UFOP MG) Resolva:


124

(UFU

MG)

Dadas

as

1
2 0

A
3
2

matrizes

1 0
12 3 x
27 9

1 4 0
, o determinante do produto At.B vale:
B
2
1
1

a)zero
c)104
e)224

b)164
d)50

130

125 - (Santa Casa SP) Seja a matriz quadrada A = (aij) de ordem 2, tal
que:

cos 2i j , se i j
a ij

sen
, se i j

i j

(UFOP

0
3 x 10
1

MG)

cos2

sen 2

log1

log2 2

tg 4

sen 3
2

cos

log3 27

determinante

matriz

sen
igual a:

a)1
b)2
c)3
d)4
e)nenhuma das respostas anteriores

o determinante de A igual a:
a) 3/4
b) 1/4
c) 0
d) 1/4
e) 3/4

131 - (UFSC SC) Sendo A uma matriz dada por

0 1 0

8 0
5
A
1 3 7

4
4 2

k
x
(k z) , o determinante
2
132 - (Mackenzie SP) Na funo real definida por

1
sec2 x cossec2 x
sen2 x
1
1
cos2 x

tg 2 x

0
.
0

Calcule det(A).

126 - Qualquer que seja x R, tal que

igual a:

cotg2 x

a)secx . cossecx
b)1
c)1
d)zero
e)n.d.a

x 2 4
f (x) x 3 9
x 4 16

f(0,001) vale:
a)0,02
b)1000-1
c)10-2
d)500-1
e)0,5
133 - (Unifor CE) Quantos nmeros inteiros satisfazem a sentena

127 - (ITA SP) Sabendo-se que a soma das razes da equao:

1 1
x 0
0 b
b x

0
x
x
2

2
0
0
x
b

8/3 e que S o conjunto destas razes, podemos afirmar que:


a)S [-17,-1]
b)S [1,5]
c)S [-1,3]
d)S [-10,0]
e)S [0,3]

128 - (UnB DF) Dada

da

1 0 0
A 3 4 5 ,
2 1 3

x
2
0

0
x1
1

1
2 4 ?
1

a)Dois.
b)Trs.
c)Quatro.
d)Cinco.
e)Seis.

134 - (Unip SP) Se

1 2 3
x y z
6 9 12 12 , ento 2 3 4
x y z
1 2 3

vale:

a)-4
b)4/3
c)4/3
d)4
e)12
considere a funo f(x) = det

(xI-A), em que I a matriz identidade 3 x 3.


Julgue os itens abaixo.
00. Existe um nmero irracional x0, tal que f(x0) = 0.
01. f assume dois mximos e um mnimo em algum intervalo (a,b) da reta
real.
02. A uma matriz inversvel.

135 - (UFU MG) Qual a afirmativa errada?


a)O determinante de uma matriz que tem duas linhas (ou colunas) iguais,
igual a zero.
b)O determinante de uma matriz no se altera quando se trocam na matriz,
duas linhas (ou colunas) entere si.
c)O determinante de uma matriz fica multiplicado por k quando se multiplica
uma linha (ou coluna) da matriz por k.
d)A adio a uma linha , de uma matriz, de uma combinao linear das
demais no altera o valor do seu determinante.
e)O determinante de uma matriz que tem duas linhas (ou colunas)
proporcionais igual a zero.
136 - (Mackenzie SP) A uma matriz quadrada de ordem 4 e det A = -6.
o valor de x tal que det (2A) = x 97 :

133

a)-12
b)zero
c)1
d)97/2
e)194

142 - (UFAL AL) Considere as matrizes

3 - 1
1 3
2 3 1

A
,
B

e
C

0 1
3
0
1
x
0

y 2

para analisar as

afirmaes seguintes.
00. Se A = B . C, ento x = 1 e y = 7.

01. A matriz inversa de A

137 - (UFU MG) Considere as matrizes:

a b c
a

A d e f , B b
g h i
c
2a

D 2d
2g

d g

e h , C
f i
2b 2c

2e 2f
2h 2i

a b c

g h i
d e f

Se o determinante da matriz A K 0, ento det (B) + det (C) + det (D)


igual a:
a)10 K
b)8 K
c)6 K
d)4 K
e)2 K

0 - 3

1 1
3 9

02. Se x = 1 e y = 1, ento o determinante da matriz (C . B) igual a zero.


03. A matriz A2 anti-simtrica.
04. O determinante da matriz (10 . A) igual a 10 vezes o determinante de
A.

1) C

2) D

3) C

4) D

5) B

6) 25%

8) D

9) D

11) A

12) C

13) ECECE

14) VVFVV

15) 17

16) C

17) D

18) B

19) B

20) E

21) E

22) D

23) 03

24) C

a)1
b)1
c)det A + det B
d)det(AB)

25) C

26) C

27) E

28) D

29) 08

30) C

140 - (UEL PR) Se A uma matriz quadrada de ordem trs com det A = 5,
ento o valor de det 2A :
a)6
b)11
c)15
d)30
e)40

31) A

32) D

33) 11

34) A

35) C

36) E

37) B

38) E

39) 018

40) D

41) D

42) C

43) D

44) A

45) B

138 - (UFU MG) Dada as matrizes

a b c

A d e f
g h i

a d g 7) a) 4%

B b e h b) 2014
c f i 10) A

det A = 5, considere as matrizes C = 3A e D = B 1. Ento, det C + det (CD)t


:
a)90
b)18
c)210
d)162
e)54
139 - (UFU MG) Se A e B so matrizes inversveis de mesma ordem, ento

det(A 1BA )
det B

igual a

141 - (UEPG PR) Assinale o que for correto.

01.Se

02.Se

04.Se

08.Se

1
4
A
1
0
a

A 0

1
A
2
1
A
0

0
0
2
0

2
8
2
0

5
3
, ento det(A) = 0
1

b c

d e , ento det(A) = a.d.f

0 f

1
, ento det(A) = det(At)
3
2
, ento [det(A)]n = 1, para n N*
1

sen a cos a
16.Se A
cos a sen a , ento det(A) = cos2a

46) E

47) a) 0,1,0,1,0
fim do gab.

b) No
48) C

49) C

50) {x R | 1 < x < 4}51) E

52) A

53) D

54) A

55) B

56) D

57) A

58) A

59) A

60) B

61) D

62) C

63) A

64) C

65) B

66) B

67) C

68) D

69) D

134

70) 35 DIAS

71) E

72) 20

73) 18

74) E

75) 26

76) A

77) E

78) C

79) D

80) C

81) C

82) A

83) 37,5 aps 2min. 84) D

85) 2044

86) a) b = 1/29 b) 67,28


87) A

88) 45

89) D

90) C

91) A

92) A

93) C

94) E

95) 05

96) 1 3 i

01 - (Mackenzie SP) Dois paraleleppedos retngulos de mesmas


dimenses cortam-se conforme a figura, sendo igual a 1 o volume da regio
assinalada. Se ABCD um quadrado, e o volume total do slido obtido,
incluindo a regio assinalada, 9, a dimenso b igual a

anos

)
97) 2(cos + i . sen98)
3
3 D

99) A

3
100) 3 2i; 3 + 2i 101) |z| = 1; arg(z) =102)
E
2

103) B

104) B

106) A

6
107)
- 5

109) E

110) B

105) E

15

108) x

a) 2
b) 6
c) 5
d) 3
e) 4

1
15

111) D

02 - (UFOP MG) Admita que, ao congelar-se, a gua aumenta em

seu volume. O volume de gua a congelar para obter-se um bloco de gelo de


10 cm5cm6 cm, em mL , de:
a) 280
b) 281,25
c) 300
d) 311,25

2 1

112) 3 2
0
1

113) D

114) C

115) B

116) A

117) E

118) B

119) B

121) E

122) D

123) D

124) A

125) C

126) D

127) D

128) CECE

130) C

131) 70

132) D

133) C

134) D

135) B

136) C

137) B

138) D

139) A

140) E

141) 15

120) a) 64
b) 5 e 3

03 - (UFOP MG) As medidas das arestas de uma caixa retangular so 18


cm, 30 cm e 42 cm. O menor nmero de cubos que enchem completamente
essa caixa :

129) x = 0 ou
x=2

a) 210
b) 190
c) 105
d) 95
04 - (UFAM AM) A maior distncia entre dois pontos de um cubo de aresta

2
142) VFVFF
47) b)

, :

a)

b)

c)

2 3

d) 3 2
e) 2
05 - (UFPel RS)

135

A charge acima ilustra uma campanha de conscientizao da populao sobre


a necessidade de se evitar o desperdcio de gua. Os domiclios so os
campees do desaproveitamento de gua. A mangueira da ilustrao, ligada
a uma torneira com vazo constante, enche em 34 minutos uma caixa dgua
cujas medidas internas so 0,80 m de comprimento, 1 m de largura e x m
de profundidade.
www.saaej.sp.gov.br/ambiente/desperdcio/htm - acessado em 06.04.2005
[adapt].
A partir das leituras acima, considerando

3 1,7

e a profundidade dessa

caixa dgua que igual diagonal de um cubo, correto afirmar que a


aresta do referido cubo de
a) 0,85 m.
b) 0,50 m.
c) 0,37 m.
d) 0,22 m.
e) 0,26 m.
06 - (Unifor CE) A pea de ferro abaixo foi obtida de um paraleleppedo
reto-retngulo de dimenses 20 cm, 30 cm e 40 cm, com a retirada de
quatro cubos iguais de aresta 10 cm.

11 - (UEPG PR) Assinale o que for correto.


01. Quando um cubo tem a sua aresta aumentada de 50%, o seu volume fica
multiplicado por

27
8

02. Uma pirmide quadrangular regular tem faces laterais formadas por
tringulos eqilteros de 2 cm de lado. O volume dessa pirmide

4 2
cm3 .
3
04. Um prisma hexagonal possui 18 arestas.
08. Dois cilindros eqilteros A e B tm os raios da base iguais a r 1 e r2,
respectivamente. Sabendo que

AeB

8
27

r1 2

r2 3

, ento a razo entre os volumes de

16. O crculo mximo de uma esfera mede 12 cm. O volume da esfera vale
288 cm3.
12 - (Unifesp SP) Considere o poliedro cujos vrtices so os pontos mdios
das arestas de um cubo. O nmero de faces triangulares e o nmero de faces
quadradas desse poliedro so, respectivamente:

Se a densidade do ferro 7,8 g/cm3, ento a massa dessa pea, em quilos,


a) 187,2
b) 179,4
c) 171,6
d) 163,8
e) 156
07 - (Mackenzie SP) Se as dimenses de um paraleleppedo reto retngulo
de volume 15 esto em progresso aritmtica e a maior delas 3, a soma
dessas dimenses
a)
c)
e)

25
8
9
2
21
4

b)
d)

19
6
15
2

08 - (UFPel RS) A ourivesaria uma indstria que muito se desenvolveu


nos ltimos anos. Muito se tem investido em design e produo. Em
adolescentes de classe mdia e alta, so usuais colares com pingentes de
ouro de vrias formas.
Sendo a densidade de um corpo o quociente da massa pelo volume,
considerando a densidade do ouro 19,3 g/cm 3 e sendo o grama do ouro R$
31,00, correto afirmar que o custo do material de um pingente de ouro
macio, na forma de um dado (cubo) com 0,64 cm 2 de rea em cada face,
de, aproximadamente,
a) R$ 160,00.
b) R$ 250,00.
c) R$ 306,00.
d) R$ 158,00.
e) R$ 382,00.
09 - (UEG GO) Uma bola de futebol foi confeccionada utilizando-se 32 faces
planas, sendo 20 hexagonais e 12 pentagonais. Considerando-se que a bola
identifica-se com um poliedro assim construdo, esse poliedro possui
exatamente
a) 180 arestas
b) 90 vrtices
c) 60 vrtices
d) 60 arestas
10 - (UPE PE) At 1985, as nicas formas conhecidas de organizao de
cadeias carbnicas puras e estveis eram o diamante e o grafite. Nesse
mesmo ano, trs pesquisadores revelaram ao mundo a terceira forma estvel
de carbono alm do diamante e do grafite. Os fulerenos, substncia cuja
molcula possui tomos de carbono nos vrtices de um poliedro denominado
de icosaedro truncado. Esse poliedro possui 12 faces pentagonais e 20 faces
hexagonais.
Pode-se afirmar que o nmero de vrtices do icosaedro truncado igual a:
a) 80
b) 60
c) 70
d) 90
e) 25

a) 8 e 8.
c) 6 e 8.
e) 6 e 6.

b) 8 e 6.
d) 8 e 4.

13 - (PUC SP) Corta-se uma pirmide de 12cm de altura por um plano


paralelo base e a 4cm desta. Calcule a razo entre a rea da base e a rea
da seco.
a) 12

c) 9
4

b) 7

4
7
d)
9

e) 9

14 - (Osec SP) Um prisma e uma pirmide tm bases com a mesma rea.


Se o volume do prisma o dobro do volume de pirmide, a altura da
pirmide ser:
a) O triplo da do prisma.
b) O dobro da do prisma.
c) O triplo da metade da do prisma.
d) O dobro da tera parte da do prisma.
e) n.d.a
15 - (UEG GO) Calcular o volume de uma pirmide que tem por base uma
das faces de um cubo de 5cm de aresta e por vrtice um ponto do segmento
de reta que liga os centros de duas faces opostas e perpendiculares a essa
base.
16 - (ITA SP) A rea lateral de uma pirmide quadrangular regular de altura
4 m e de rea da base 64 m2 vale:
2
a) 128 m2
b) 64 2m
c) 135 m2
e)

d)

60 5m2

32( 2 1)m2

17 - (UEPG PR) Considere um cubo de aresta a 2 cm , e o poliedro


VABC, como indicado na figura e assinale o que for correto.
3

136

a)
01. O volume do poliedro vale

1
cm3 .
3

02. O poliedro VABC um tetraedro regular.


04. A rea do tringulo ABC vale 1 cm2.
08. O tringulo AVB escaleno.
2

16. A rea total do cubo 6 4 cm .


18 - (Fac. de Med. Jundia) A base de um cone circular reto est inscrita
em uma das faces de um cubo e o vrtice desse cone est no centro da face
oposta a essa base. O volume desse cone de
rea total desse cubo, encontra-se
a) 216 cm2.
b) 108 cm2.
c)

72 3 cm2

d)

18 cm3 .

Calculando-se a

1
2

e) 72 cm2
19 - (UFMG MG) Nesta figura, esto representados o cubo ABCDEFGH e o
slido OPQRST:

1
3

c)

1
4

d)

1
5

e)

1
6

22 - (UFC CE) Um vaso em forma de cilindro circular reto tem medida de


raio da base 5 cm, altura 20 cm e contm gua at a altura de 19 cm
(despreze a espessura das paredes do vaso). Assinale a alternativa na qual
consta o maior nmero de esferas de ao, de 1 cm de raio cada, que
podemos colocar no vaso a fim de que a gua no transborde.
a) 14
b) 15
c) 16
d) 17
e) 18
23 - (Furg RS) O resultado obtido pela diviso do volume de um cubo pela
sua rea total 2.
O valor de

54 4 cm2

b)

a) 84.
c) 36.
e) 96.

1
do volume da esfera inscrita nesse cubo
3
b) 64.
d) 100.

24 - (UEL PR) Um joalheiro resolveu presentear uma amiga com uma jia
exclusiva. Para isto, imaginou um pingente, com o formato de um octaedro
regular, contendo uma prola inscrita, com o formato de uma esfera de raio
r, conforme representado na figura a seguir.

Cada aresta do cubo mede 4 cm e os vrtices do slido OPQRST so os


pontos centrais das faces do cubo.
Ento, CORRETO afirmar que a rea lateral total do slido OPQRST mede
a)

8 2 cm2

b)

8 3 cm2

c)

16 2 cm2

d)

16 3 cm2

Se a aresta do octaedro regular tem 2cm de comprimento, o volume da


prola, em cm3, :

20 - (PUC SP) De um cristal de rocha, com o formato de uma esfera, foi


lapidada uma jia na forma de um octaedro regular, como mostra a figura
seguinte.

Se tal jia tem

9 2 cm3

de volume, quantos centmetros cbicos de rocha

foram retirados do cristal original para lapid-la? (Use:


a)

36 2

b)

32 2

c)

24 2

d)

18 2

e)

12 2

21 - (Mackenzie SP) Num cilindro reto de altura

3)

6 3,

a)
b)

2
3
8
3

c)

8 2
9

d)

4 6
9

e)

8 6
27

25 - (UFOP MG) Um cone circular reto est inscrito em uma esfera. As


geratrizes do cone medem cm 10 e o ngulo , formado entre uma geratriz
qualquer e a altura do cone, tal que cos 0,9 .

completamente

cheio de gua, foi imerso um prisma triangular regular de altura 2 ,


conforme a figura. A razo entre o volume de gua que transbordou e o
volume do cilindro

137

a) Calcule o volume do cone.


b) Calcule o volume da esfera.
26 - (Unifor CE) Duas esferas, de mesmo raio, so tangentes
externamente. Um cilindro circular reto, de volume 108 dm3, circunscrito
s duas esferas, de modo que seu eixo contm um dimetro de cada esfera.
Para cada uma das esferas, o volume, em dm3, e a rea da superfcie, em
dm2, so, respectivamente,
a) 30S e 32S
b) 32S e 30S
c) 32S e 36S
d) 36S e 32S
e) 36S e 36S
27 - (UFMG MG) Um recipiente cbico, sem tampa, cujas arestas medem 4
dm, contm 56 litros de gua. Ao lado desse recipiente, esto os seguintes
slidos, todos de ao macio:
3
uma esfera de raio 2 dm;
um cilindro circular reto com raio da base
um paraleleppedo retangular de dimenses

dm e altura

dm,

dm;

dm e

31 - (Mackenzie SP) Bolas de tnis, normalmente, so vendidas em


embalagens cilndricas contendo trs unidades que tangenciam as paredes
internas da embalagem. Numa dessas embalagens, se o volume no ocupado
pelas bolas 2, o volume da embalagem :
a) 6
b) 8
c) 10
d) 12
e) 4

32) O volume de uma esfera 4000/3m3. Calcule o volume de


um cilindro circular reto, inscrito nessa esfera, sabendo que a
base desse cilindro equivalente superfcie total de um cubo de aresta

6 m .

dm;

33 - (UEPG PR) Considere um cilindro circular reto e um cone circular reto,

ambos eqilteros, que tm bases iguais, de 16 cm2 de rea e assinale o


que for correto.
01. A rea lateral do cilindro o dobro da rea lateral do cone.
Qual desses slidos, quando colocado no recipiente, NO far com que a 02. A rea total do cilindro maior que 250 cm2.
04. O cilindro e o cone tm a mesma altura.
gua transborde?
a) A pirmide
b) O cilindro
08. A razo entre o volume do cilindro e o volume do cone 2 3 .
c) O paraleleppedo
d) A esfera
16. O volume do cone a tera parte do volume do cilindro.
28 - (UFJF MG) Se em um cubo, o raio da esfera inscrita mede 2 cm, o raio
34- Na base de um cone, cujo volume igual a 144m3, est inscrito um
da esfera circunscrita a esse cubo igual a:
uma pirmide reta de altura

dm e de base quadrada com lado

12

dm.

a)

4 2cm

hexgono regular de rea

b)

4 3cm

a)

( 5 1)m2

c)

2 2cm

b)

36 5m2

d)

2 3cm

e)

c)

36( 5 1)m2

3 2cm
d)

36( 5 1)m2

e)

36(1 5 )m2

29 - (UFAL AL) Na figura abaixo tem-se um cone circular reto, cuja geratriz
mede 20 cm, inscrito em um cilindro circular reto de 16 cm de altura.

54 3m 2 . A rea total desse cone :

35 - (Osec SP) O volume de um slido gerado pela rotao de um tringulo


retngulo e issceles, de hipotenusa igual a 1, em torno de um eixo que
contm a hipotenusa igual a:
a)

b)

216 3 cm2 .
01. A razo entre as reas totais do cone e do cilindro, nesta ordem,
02. O volume do cone igual a

2
3

2
7

do volume do cilindro.

3.456 3 cm3

04. Para se obter um outro cone circular reto, cujo volume igual a

1
24

12

36 - (PUC RJ) Um quebra-luz um cone de geratriz 17 cm e altura 15 cm.


Uma lmpada acesa no vrtice do cone projeta no cho um crculo de 2m de
dimetro. A que altura do cho se encontra a lmpada?
a) 1,50m
b) 1,87m
c) 1,90m
d) 1,97m
e) 2,00m

03. O volume do prisma regular de base hexagonal que pode ser inscrito no
cilindro igual a

c)

6
2
e)
3

3
Com base nos dados acima, analise as afirmaes seguintes.

00. A rea do hexgono regular que pode ser inscrito na base do cone d) 24

do

37 - (Integrado RJ) Uma tulipa de chopp tem a forma cnica, como mostra
a figura abaixo. Sabendo-se que sua capacidade de 100 ml, a altura h
igual a:

10cm

volume do cilindro, pode-se interceptar o cone dado por um plano paralelo


sua base e distante 8 cm de seu vrtice.
30 - (UFAL AL) Para analisar as afirmativas abaixo, considere o
paraleleppedo retngulo de dimenses 6 cm, 5 cm e 2 cm.
00. Seu volume maior que 64 cm3.
01. Sua diagonal mede

65 cm

02. Sua rea total igual a 104 cm2.


03. O raio da esfera circunscrita a ele igual a 4 cm.
04. O volume de uma pirmide com vrtice no centro de uma face e cuja
base a face oposta a ela igual a 30 cm 3.

a) 20 cm
b) 16 cm
c) 12 cm
d) 8 cm
e) 4 cm

38 - (PUC RJ) A rea lateral de um cilindro


reto excede de 3cm2 a rea de sua seco
meridiana. Se o raio da base vale 1cm,
ento quanto valer a sua rea lateral?

138

39 - (Mackenzie SP) Aumentando-se de 6 unidades o raio de um cilindro, o


seu volume aumenta Y unidades. Se tivssemos aumentado de 6 unidades a
altura do cilindro inicial, o seu volume teria aumentado igualmente de Y
unidades. Se a altura original 2, o raio original :
a) 2
b) 4
c) 6
d) 6
e) 8
40 - (USP SP) O lquido contido em uma lata cilndrica deve ser distribudo
em potes tambm cilndricos, cuja altura

1 da altura da lata e cujo


4

dimetro de base 1 do dimetro da base da lata. O nmero de potes

necessrios :
a) 6
b) 12
c) 18
d) 24
e) 36
41 - (Faap SP) Um retngulo girando em torno de cada um dos lados gera
dois slidos, cujos volumes medem 360m3 e 600m3. Calcular a medida dos
lados do retngulo.

a) 300
b) 100
c) 400
d) 150
e) 200
50 - (UFRRJ RJ) Carlos um rapaz viciado em beber refrigerante diet. Um
dia, voltando do trabalho, ele passou em frente a uma companhia de gs,
onde viu um enorme reservatrio cilndrico de 3 metros de altura com uma
base de 2 metros de dimetro e pensou... Em quanto tempo eu beberia
aquele reservatrio inteiro, se ele estivesse cheio de refrigerante diet?.
Considerando = 3,14 e sabendo-se que Carlos bebe 3 litros de refrigerante
diet por dia, pose-se afirmar que ele consumir o lquido do reservatrio em
um perodo de:
a) 86 dias.
b) 86 meses.
c) 86 anos.
d) 8,6 anos.
e) 860 meses.
51 - (UnB DF) A figura abaixo representa um coador de caf (em forma de
um tronco de cone) apoiado sobre um vaso cilndrico com permetro da base
igual ao permetro da boca do coador. Calcule r, de acordo com os dados na
figura e sabendo que a capacidade do coador um quarto da capacidade do
vaso.

42 - (PUC SP) Quantos mililitros de tinta podem ser acondicionados no


reservatrio cilndrico de uma caneta esferogrfica, sabendo que seu
dimetro 2mm e seu comprimento 12cm?
a) 0,3768
b) 3,768
c) 0,03768
d) 37,68
e) 0,003768
43 - (EFEI MG) Um cilindro est inscrito em um cubo cuja diagonal mede
20 cm. Calcule a rea lateral do cilindro.
44 - (UFSC SC) Um cilindro reto tem 63 cm de volume. Sabendo que o
raio da base mede 3 cm, determine, em centmetros, a sua altura.
45 - (UFG GO) Tem-se um copo em forma de cilindro, cujo interior possui
raio da base igual a 3cm e altura 6m. Coloca-se guaran neste copo at a
altura de 3 cm e, depois, um cubo de gelo de aresta 0,9cm. Aps colocado
dentro do copo, o cubo de gelo fica parcialmente submerso. Pode-se afirmar
que:
01. o volume do cubo de gelo igual a 0,9 cm 3;
02. a rea lateral interna do copo maior que 108cm 2;
04. a rea da base (interna) do copo menor que 36 cm 2;
08. a capacidade total do copo de 54 cm3;
16. se 0,8 cm da aresta lateral do cubo de gelo fica submersa, a altura do
guaran passa de 3 cm para 3,8 cm;
32. se a densidade do gelo fosse igual densidade do guaran, depois que o
gelo derretesse, o volume de lquido no copo seria de 28 cm3

52 - (Integrado RJ) O volume do slido gerado pela rotao completa da


figura a seguir, em torno do eixo e, , em cm3:
e

46 - (Unimep RJ) Um tambor em forma de cilindro circular reto tem 6 dm


de dimetro e 9 dm de altura e est com gua at a boca. Dentro v-se uma
melancia. Uma pessoa retira a melancia e verifica que o nvel da gua baixou
de 0,25 dm. Podemos dizer que o volume da melancia aproximadamente:
a) 8,510 dm3
b) 7,065 dm3

a) 38
b) 54
c) 92
d) 112
e) 128

c) 85 dm3
d) 5,042 dm3
e) nenhuma das anteriores
47 - (Unificado RJ) Um salame tem a forma de um cilindro reto com 40 cm
de altura e pesa 1 kg. Tentando servir um fregus que queria meio quilo de
salame, Joo cortou um pedao , obliquamente, de modo que a altura do
pedao variava entre 22cm e 26cm. O peso do pedao de :
a) 600g
b) 610g
c) 620g
d) 630g
e) 640g

48 - (UFJF MG) Um reservatrio de formato cilndrico, de altura

5 m,

raio

2
m

2 cm

6 cm
3 cm

3 cm

53 - (UERJ RJ) Uma linha poligonal fechada de trs lados limita um


tringulo de permetro l. Se ela gira em torno de um dos lados, gera uma
superfcie de rea S igual ao produto de l pelo comprimento da circunferncia
descrita pelo baricentro G da poligonal.
A figura abaixo mostra a linha (ABCA) que d uma volta em torno de BC.

est ocupando de gua em 60% de sua capacidade. A

quantidade de gua necessria para completar o reservatrio , em litros:


a) 4
b) 4000
c) 40
d) 400
e) 10000
49 - (UFMA MA) Um recipiente sob a forma de um cilindro reto est repleto
de vinho. Esse vinho deve ser distribudo em copos cilndricos, possuindo,
cada um, altura igual a
a

1
8

da altura do recipiente e dimetro da base igual

do dimetro da base do recipiente. A quantidade de copos necessria

para distribuir todo o vinho :

a) Esboce a figura gerada e indique o clculo da rea de sua superfcie.


b) Calcule a distncia r do baricentro G dessa linha ao eixo de rotao.

139

e) no cair em nenhuma caapa.


03 - (UFRN RN) O relgio ao lado est marcando 2h30min. Passadas duas
horas e quinze minutos, a medida do menor ngulo formado pelos ponteiros
do relgio ser:
a) 127,5
b) 105
c) 112,5
d) 120

1) C

2) B

3) C

4) B

5) B

6) E

7) D

8) C

9) C

10) B

11) 31

12) B

13) C

14) C

15) V 125 cm3

16) B

17) 25

18) A

19) D

20) D

21) C

22) E

23) E

24) E

25) -

26) E

27) C

28) D

29) VFFVV

30) FVVFF

31) A

32) 576m3

33) 11

34) C

35) C

36) B

37) C

38)

39) C

40) E

41) 6m e 10m

S 3 cm2
2

04 - (UniRio RJ) Estrela do


PT nos jardins do
Alvorada.A
primeira-dama
resolveu decorar o
Palcio da Alvorada e a Granja
do Torto com um
paisagismo bastante particular. Slvias especialmente plantadas formam a
estrela vermelha petista nos jardins das duas residncias oficiais.
Correio Braziliense, 2004.
A estrela de cinco pontas foi desenhada como mostra a figura abaixo. A
produo desse paisagismo especial no Palcio do Planalto foi realizada
sabendo que A, B, C, D, E so vrtices de um pentgono regular e que o
ngulo

D
CA

igual a:

a) 72
b) 48
c) 36
d) 24
e) 18
05 - (Furg RS) Na figura abaixo, as retas r e s so paralelas.

42) A

43) S 400 cm2 44) 07


3

45) FVVFFF

46) B

47) A

48) B

49) E

50) D

51) 12

52) E

53) a) S = 36 cm2 b) r = 4/3 cm

A medida do ngulo y, em graus


a) 90.
b) 60.
c) 100.
d) 70.
e) 80.
06 - (UFU MG) Na figura abaixo o ngulo x, em graus, pertence ao intervalo

Exerccios Extras
01 - (ESPM SP) A soma dos ngulos assinalados na figura abaixo igual a:
a) 720
b) 900
c) 1080
d) 1260
e) 1440

a) (0, 15)
b) (15, 20)
c) (20, 25)
d) (25, 30)
07 - (UFMG MG) Observe esta figura:

F
105 A

02 - (UFRRJ
RJ) A figura abaixo
mostra
a
trajetria de uma bola
de bilhar. Sabe-se que, quando ela bate na lateral da mesa (retangular),
forma um ngulo de chegada que sempre igual ao ngulo de sada. A bola
foi lanada da caapa A, formando um ngulo de 45 com o lado AD.

57
E

28

Sabendo-se que o lado AB mede 2 unidades e BC mede 3 unidades, a bola


a) cair na caapa A.
b) cair na caapa B.
c) cair na caapa C.
d) cair na caapa D.

Nessa figura, os pontos F, A e B esto em uma reta e as retas CB e ED so


paralelas.
Assim sendo, o ngulo
a) 39
b) 44

C
AB

mede:

140

c) 47
d) 48
08 - (Unifor CE) Na figura abaixo tm-se as retas r e s, paralelas entre si, e
os ngulos assinalados, em graus.

13 - (UFG GO) A figura abaixo o esboo de uma pista de atletismo, com


cinco raias de 60 cm de largura cada. As raias so delimitadas por retas e
semicircunferncias concntricas, sendo que a raia mais interna circunscreve
um campo de futebol de 70 m por 100 m.

3
0

7
0

Nessas condies, + igual a


a) 50
b) 70
c) 100
d) 110
e) 130

A pista ser revestida com material para amortecimento de impactos que


custa R$ 15,00 o m2.
Qual , aproximadamente, o valor a ser gasto com o material de
revestimento da pista?

09 - (Unifor CE) A medida em graus do ngulo


medida de seu complemento. O ngulo
a) 90
b) 6730'
c) 60
d) 4830'
e) 45

igual ao triplo da

mede

a) 3
b) 6
c) 9
d) 8
e) 4

10 - (Unifor CE) Na figura abaixo, tem-se r//s e t//u

30

14 - (PUC PR) Na circunferncia de centro O e raio 6, os raios OA e OB


formam um ngulo de 60.
Calcule a rea do crculo tangente a OA e OB e circunferncia de centro O.

15 - (UEL PR) Qual a


hachurada na figura a
que o raio da circunferncia maior r?

rea
da
regio
seguir, sabendo-se

70

a)
v

Se os ngulos assinaladostm as medidas indicadas em graus, ento b)


igual a:
a) 100o
b) 80o
c) 70o
d) 50o
c)
e) 30o
11 - (PUC SP) Na figura r//s ento o valor do gulo x :
r//s

12 - (Fepecs DF) Os crculos C1 e C2 da figura a seguir tm raio 1 e so


tangentes no ponto O. A regio sombreada R limitada por C 1 e C2 e por um
arco de crculo de centro O e raio 1.

e)

1
r2

16 - (UFAL AL) Uma praa tinha a forma de um quadrado com 160 m de


permetro. Aps uma reforma, a sua superfcie passou a ter um formato
circular, com dimetro igual a 75% da medida do lado do quadrado original.
Com essa reforma, de quantos metros quadrados foi reduzida a rea da
praa original? (Use 3,14) .
17 - (FMTM MG) A figura mostra uma circunferncia de centro O e raio
igual a 2 e um pentgono regular ABCDO, cujos vrtices A e D pertencem
circunferncia. A regio hachurada tem rea igual a:
a)
b)
c)

A rea de R :
a)

3 3
6

c)

2
3

3
2

e)

2 12

b)

18 2

d)

10


r 2 1
4


r 2 1
2

^
x

1
r2
8 4

d)

10

1
r2
4 2

d)
e)

6
5
8
3
9
4
10
3
12
5

18 - (UEL PR) Na figura, ABCD um quadrado cujo lado mede a. Um dos


arcos est contido na circunferncia de centro C e raio a, e o outro uma
semicircunferncia de centro no ponto mdio de BC e de dimetro a. A rea
da regio hachurada :

141

a)

a 2
6

b)

a
8

c)

a 2 1

3
2

d)

a 2 1

6
3

e)

a 2
1
6

4.(3 3 2)

d)

6.(2 3 1)

e)

12.( 3 1)

24 - (Unesp SP) A figura representa um tringulo retngulo de vrtices A, B


e C, onde o segmento de reta DE paralelo ao lado AB do tringulo.

19 - (UEL PR) Uma lembrana de festa foi confeccionada em cartolina a


partir de um hexgono regular de lado igual a 3 cm. Com centro em cada
vrtice foram construdos semicrculos com raio igual ao lado do hexgono. A
seguir, foi retirada a regio do semicrculo que ficava por baixo do
semicrculo seguinte, resultando na figura abaixo. Use o valor 3,14 para e o
valor 1,73 para

c)

3.

Assinale a alternativa que contm o valor mais aproximado da rea total da


figura.
a) 113,04 cm2
b) 108,14 cm2
c) 103,22 cm2
d) 84,78 cm2
e) 82,52 cm2

Se AB = 15 cm, AC = 20 cm e AD = 8 cm, a rea do trapzio ABED, em cm 2,

a) 84.
b) 96.
c) 120.
d) 150.
e) 192.
25 - (Unicamp SP) A coletnea de textos da prova de redao tambm
destaca o impacto da modernizao da agricultura sobre a produtividade da
terra e sobre as relaes sociais no pas. Aproveitando esse tema,
analisamos, nesta questo, a colheita de uma plantao de cana-de-acar,
cujo formato fornecido na figura ao lado. Para colher a cana, pode-se
recorrer a trabalhadores especializados ou a mquinas. Cada trabalhador
capaz de colher 0,001 km2 por dia, enquanto uma colhedeira mecnica colhe,
por dia, uma rea correspondente a 0,09 km2.

20 - (Fuvest SP) Na figura seguinte, esto representadas um quadrado de


lado 4, uma de suas diagonais e uma semicircunferncia de raio 2. Ento a
rea da regio hachurada :
a)

2
2

b) + 2
c) + 3
d) + 4
e) 2 + 1

21 - (Unesp SP) Um
cavalo se encontra preso
num
cercado
de
pastagem, cuja forma
um quadrado, com lado medindo 50m. Ele est amarrado a uma corda de
40m que est fixada num dos cantos do quadrado. Considerando = 3,14,
calcule a rea, em metros quadrados, da regio do cercado que o cavalo no
conseguir alcanar, porque est amarrado.
a) 1244
b) 1256
c) 1422
d) 1424
e) 1444
22 - (UFF RJ) Calcule a rea da regio hachurada na figura abaixo, sabendo
que as duas circunferncias tm o mesmo raio e esto inscritas no retngulo
ABCD.

a) Se a cana precisa ser colhida em 40 dias, quantos trabalhadores so


necessrios para a colheita, supondo que no haja mquinas?
b) Suponha, agora, que a colheita da parte hachurada do desenho s possa
ser feita manualmente, e que o resto da cana seja colhido por quatro
colhedeiras mecnicas. Neste caso, quantos trabalhadores so necessrios
para que a colheita das duas partes tenha a mesma durao?
Em seus clculos, desconsidere os trabalhadores que operam as mquinas.
26 - (FGV ) A seco transversal de uma caixa de latas de ervilhas um
retngulo que acomoda, exatamente, as latas, como mostra a figura abaixo:

2cm

23 - (Mackenzie SP) Na figura, a circunferncia est inscrita no hexgono


regular de lado 2; adotando
a)

2.(6 3 5)

b)

3.(4 3 3)

3 , a rea da regio sombreada

a) Sabendo que o raio da lata de ervilhas 3,5 cm, determine a rea da


seco transversal. (3)
b) Supondo, ainda, que a altura da lata de ervilhas seja 8,5 cm e que sejam
colocadas 60 latas em cada caixa, calcule o volume da caixa. (4)
27 - (UFRN RN) A figura abaixo de um mosaico quadrado de 1,5 m por
1,5 m, construdo com cermicas quadradas de 0,30 m por 0,30 m, algumas
cortadas em diagonal.

142

A rea, em metros quadrados, da regio cinza que cerca as cermicas postas


em diagonal :
a) 0,08
b) 0,09
c) 0,10
d) 0,11
28 - (Unesp SP) A figura representa um trapzio retngulo em que a
medida de AB k centmetros, o lado AD mede 2k e o ngulo DE mede 30.

Nestas condies, a rea do trapzio, em funo de k, dada por:


a)

k 2 (2 3 )

b)

2 3

k2
2

c)

3k 2 3
2

d)

3k 2 3

e)

k2 3

Em seu testamento, para contemplar igualmente seus dois filhos, o


proprietrio determinou que o terreno fosse dividido em duas partes de
mesma rea por meio de uma cerca paralela ao cateto BC, e que a parte com
a forma de um trapzio fosse do filho mais velho.
Com a morte do dr. Afrnio, seus advogados mandaram medir o
comprimento do lado AB do terreno e receberam a resposta: 156m.
Deveriam, ento, mandar construir a cerca PQ, paralela ao lado BC, de forma
que os dois terrenos tivessem mesma rea, mas, para isso, precisariam
conhecer a medida AP = x.
Sabe-se que o testamento foi cumprido. O valor de x , aproximadamente:
a) 100m.
b) 105m.
c) 110m.
d) 115m.
e) 120m.
32 - (Mackenzie SP) Na figura, se

AB AC , a rea do tringulo ABC

29 - (UFPR PR) Uma pessoa pretende adquirir um terreno de esquina para


construir sua casa, porm ela no sabe a rea do terreno. As nicas
informaes disponveis so que o terreno possui o formato de um trapzio
retngulo com um dos lados medindo 10 m e outro medindo 24 m. Alm
disso, o ngulo entre esses lados de 120 graus, conforme a figura ao lado.
Qual a rea desse terreno?
Considere

3 1,73

a)
c)

a) 332,16 m2
c) 346,54 m2
e) 308,70 m2

e)

b) 314,32 m2
d) 360,58 m2

30 - (UFRJ RJ) No crculo abaixo, a figura formada a partir de semicircunferncias e AC = CD = DE = EB.

S2

D E
S1

1
2
1
4
4
3

d)

3
4
3
2

33 - (Fuvest SP) Na figura abaixo, tem-se AC = 3, AB = 4 e CB = 6. O valor


de CD
a) 17/12
b) 19/12
c) 23/12
d) 25/12
e) 29/12
34 - (UFG
GO) A figura abaixo representa uma pipa simtrica em relao ao segmento
AB, onde AB mede 80 cm. Ento a rea da pipa, em m2, de
a)

0,8 3

b)

0,16 3

c)
31 - (FGV ) No incio do sculo passado, o dr. Afrnio Corra possua um
terreno no centro da cidade com a forma do tringulo retngulo ABC que se d)
v na figura a seguir.
e)

0,32 3

Determine S1/S2, a razo entre as reas hachuradas.

b)

1,6 3
3,2 3

143

35 - (FGV ) Trs nmeros complexos esto representados no plano de


Argand-Gauss por 3 pontos que dividem uma circunferncia de centro na
origem (0, 0) em partes iguais. Um desses nmeros igual a 1. Determine os
outros dois nmeros.
Faa um esboo da circunferncia e calcule a rea do tringulo cujos vrtices
so os trs pontos.

raio 6 cm. Sobre a de raio 3 cm, marca-se um arco de 3 cm de comprimento


e, sobre a de raio 6 cm, marca-se um arco de 6 cm de comprimento.
Em radianos, as medidas dos dois arcos marcados so, respectivamente,
a) 3 e 6
b) 3 e 6
c)
e)

d) 1 e 1

e2

40 - (UFMS MS) Observe a figura a seguir, supondo que todos os arcos so


semicircunferncias.

36 - (Mackenzie SP) No tringulo abaixo, temos AB = BC e CD = AC. Se x


e y so as medidas em graus dos ngulos
x y igual a

B , respectivamente, ento

A partir da observao da figura, correto afirmar que


a) o caminho A o maior de todos.
b) o caminho C maior que o caminho B.
c) o caminho C o maior de todos.
d) o caminho B menor que o caminho A.
e) os caminhos tm o mesmo comprimento.
41 - (UFMG MG) Nesta figura, esto representadas trs circunferncias,
tangentes duas a duas, e uma reta tangente s trs circunferncias:

a) 120
c) 115
e) 105

b) 110
d) 95

Sabe-se que o raio de cada uma das duas circunferncias maiores mede 1
cm.
Ento, CORRETO afirmar que a medida do raio da circunferncia menor

37 - (EFOA MG) Na geometria plana, quando so conhecidos os lados a , b


e c de um tringulo qualquer, possvel calcular a rea S , sem necessidade
da
determinao
de
qualquer
ngulo,
atravs
da
frmula a)

S p(p a )(p b)(p c)

, onde

2 p a b c . Considere um

terreno triangular de lados 2x 1 , x 1 , x, conforme a figura abaixo, cuja


rea e permetro so iguais em valor numrico.

c)

1
cm
3

2
cm
2

b)

d)

1
cm
4

2
cm
4

42 - (UFJF MG) Testes efetuados em um pneu de corrida constataram que,


a partir de 185.600 voltas, ele passa a se deteriorar, podendo causar riscos
segurana do piloto. Sabendo que o dimetro do pneu de 0,5 m, ele
poder percorrer, sem riscos para o piloto, aproximadamente:
a) 93 km.
b) 196 km.
c) 366 km.
d) 592 km.
e) 291 km.

CORRETO afirmar que a rea do terreno igual a:


a) 30
b) 32
c) 34
d) 38
e) 36

43 - (Unesp SP) Paulo fabricou uma bicicleta, tendo rodas de tamanhos


distintos, com o raio da roda maior (dianteira) medindo 3 dm, o raio da roda
menor medindo 2 dm e a distncia entre os centros A e B das rodas sendo 7
dm. As rodas da bicicleta, ao serem apoiadas no solo horizontal, podem ser
representadas no plano (desprezando-se os pneus) como duas
circunferncias, de centros A e B, que tangenciam a reta r nos pontos P e Q,
como indicado na figura.

38 - (UEPB PB) Se em um painel retangular foi afixado um cartaz de


formato triangular, como mostra a figura, a rea S ocupada pelo cartaz
igual a:
a)

5
3m 2
2

b) 10m2
c) 5m2
d)

10 3m 2

e)

5 3m 2

39 - (UCS RS)
Considere duas circunferncias concntricas, uma de raio 3 cm e outra de

a) Determine a distncia entre os pontos de tangncia P e Q e o valor do


seno do ngulo

BP Q .

b) Quando a bicicleta avana, supondo que no haja deslizamento, se os


raios da roda maior descrevem um ngulo de 60, determine a medida, em
graus, do ngulo descrito pelos raios da roda menor. Calcule, tambm,

144

quantas voltas ter dado a roda menor quando a maior tiver rodado 80
voltas.
44 - (Unesp SP) A figura mostra um pequeno crculo de raio r > 0 rodeado
por quatro outros crculos maiores de raio R > r. Os crculos maiores so
tangentes externamente ao menor, e cada um deles tangente a dois outros
maiores.

47 - (UFRJ RJ) Uma roda de 10 cm de dimetro gira em linha reta, sem


escorregar, sobre uma superfcie lisa e horizontal.

Determine o menor nmero de voltas completas para a roda percorrer uma


distncia maior que 10 m.
48 - (Fuvest SP) A figura abaixo representa duas polias circulares C1 e C2
de raios R1 = 4cm e R2 = 1cm, apoiadas em uma superfcie plana em P 1 e P2,
respectivamente. Uma correia envolve as polias, sem folga. Sabendo-se que
a distncia entre os pontos P1 e P2

3 3cm , determinar o comprimento da

correia.

a) Obtenha o valor da razo de R pela distncia do centro do crculo menor a


um dos pontos em que dois dos crculos maiores se tangenciam.
b) Obtenha o valor da razo R/r.
45 - (Furg RS) Dois atletas vo disputar uma corrida em uma pista com a
forma ilustrada na figura a seguir.

49 - (UFG GO) A figura abaixo mostra uma circunferncia de raio r = 3 cm,


inscrita num tringulo retngulo, cuja hipotenusa mede 18 cm.

O percurso tracejado, a ser cumprido pelo atleta que corre na raia 1, inicia no
ponto A e formado pela semicircunferncia de centro O e dimetro AB, pelo
segmento de reta BC, pela semicircunferncia de dimetro CD e centro O e, a) Calcule o comprimento da circunferncia que circunscreve o tringulo ABC.
finalmente, pelo segmento de reta DA. O trajeto a ser percorrido pelo atleta b) Calcule o permetro do tringulo ABC.
que corre na raia 2 tem incio no ponto P e formado pelo arco PQ da
circunferncia com dimetro QT e centro O, pelo segmento de reta QR, pela 50 - (UFOP MG) Num trapzio issceles, os ngulos da base so de 60.
semicircunferncia de dimetro RS e centro O e, finalmente, pelo segmento Este trapzio tem uma rea de 3 3 m 2 e est inscrito num semicrculo,
de reta ST. A chegada para o corredor da raia 1 o ponto A e, para o atleta
conforme mostra a figura.
da raia 2, o ponto T. Sabendo que OA O' D 25metros ,

OT O' S 30metros

BC DA QR ST ,

para que os atletas

percorram a mesma distncia, o comprimento do arco TP deve ser igual a


a) 30 metros
b) 5 metros
c) 25 metros
d) 2 metros
e) 10 metros

Podemos afirmar que o permetro do trapzio e a rea do semicrculo


medem, respectivamente:
a) 10m e 2 m2
b) 10m e 4 m2
c) 8m e 2 m2
46 - (UFLA MG) Se todos os crculos da figura so de raio igual a 5 metros,
2
o comprimento do caminho de A at B que passa pelos centros dos crculos d) 8m e 4 m
51 - (UFSCar SP) Os satlites de comunicao so posicionados em
sincronismo com a Terra, o que significa dizer que cada satlite fica sempre
sobre o mesmo ponto da superfcie da Terra. Considere um satlite cujo raio
da rbita seja igual a 7 vezes o raio da Terra. Na figura, P e Q representam
duas cidades na Terra, separadas pela maior distncia possvel em que um
sinal pode ser enviado e recebido, em linha reta, por esse satlite.

a)

100 2 m

b)
c)

11 2m
110m

d)

10 10 2 m

e) igual metade do permetro do retngulo.

145

a) 108o.
c) 54o.
e) 18o.

b) 72o.
d) 36o.

Se R a medida do raio da Terra, para ir de P at Q, passando pelo satlite,


o sinal percorrer, em linha reta, a distncia de

57 - (UEPB PB) Se as diagonais de um paralelogramo formam entre si um

a)

6 3R

permetro desse paralelogramo em centmetros, igual a:

b)

7 3R

a)

2 1 13

c)

8 3R

c)

4 13

d)

10 2R

e)

2 2 13

e)

11 2 R

ngulo de 30 e seus comprimentos so respectivamente

b)

2 13

d)

1 13

2 3 e 4 cm ,

58 - (FGV ) As bases de um trapzio issceles medem 20 m e 36 m, e a


soma das medidas dos lados no paralelos 20 m. A medida da altura desse
52 - (Unifor CE) Os lados de um octgono regular so prolongados at que trapzio :
se obtenha uma estrela. A soma das medidas dos ngulos internos dos a) 6 m
b) 3 m
vrtices dessa estrela
c) 8 m
d) 4 m
a) 180.
e) 10 m
b) 360.
c) 540.
59 - (UFMG MG) Esta figura representa o quadriltero ABCD:
d) 720.
e) 900.
53 - (UEPB PB) Aumentando-se de 5 unidades o nmero de lados de um
polgono, o nmero de diagonais aumenta de 40. Esse polgono o:
a) heptgono
b) pentgono
c) hexgono
d) octgono
e) enegono
54 - (UEM PR) Dentre as alternativas abaixo, assinale a(s) correta(s).
01. Todo polgono eqiltero eqingulo.
02. Dado um tringulo qualquer, de vrtices A, B e C, sempre possvel
determinar um paralelogramo ABCD tal que a rea desse paralelogramo seja
exatamente o dobro da rea do tringulo ABC.
04. Qualquer que seja o tringulo ABC, possvel determinar uma
circunferncia circunscrita a ele.
08. Sejam AT e AH respectivamente as reas do tringulo eqiltero de lado l
e do hexgono regular tambm de lado l. Ento A H 6A T .
16. Se a diagonal do retngulo de vrtices A, B, C e D (nomeadas no sentido
anti-horrio) mede 10 u.c. e a rea do tringulo de vrtices A, B e C 24
u.a., ento o permetro do retngulo ABCD 28 u.c.
32. Se as faces de um poliedro so todas pentgonos regulares, ento o
nmero mximo de faces que se encontram em um nico vrtice desse
poliedro 5.
64. Todo quadriltero eqingulo , necessariamente, um retngulo.

Sabe-se que

AB 1cm

AD 2cm ;

C
AB

mede 120; e
o ngulo
o segmento CD perpendicular aos segmentos AD e BC.

Ento, CORRETO afirmar que o comprimento do segmento BD


a)

3cm

c)

6
cm
2

b)

5
cm
2
d)

2 cm

60 - (UFJF MG) Seja o tringulo de base igual a 10 m e altura igual a 5 m


com um quadrado inscrito, tendo um lado contido na base do tringulo. O
lado do quadrado , em metros, igual a:

a)
55 - (ITA SP) Considere um prisma regular em que a soma dos ngulos
internos de todas as faces 7200. O nmero de vrtices deste prisma
igual a:
b)
a) 11.
b) 32.
c) 10.
d) 20.
e) 22.
c)
56 - (Unifesp SP) Pentgonos regulares congruentes podem ser
conectados, lado a lado, formando uma estrela de cinco pontas, conforme d)
destacado na figura. Nestas condies, o ngulo mede
e)

10
3
5
2
20
7
15
4
15
2

61 - (FGV ) No teodolito indicado, cada volta completa da manivela aumenta


em 0,5 o ngulo de observao em relao horizontal.

146

a)

21 7
m
2

b)

19 7
m
2

c)

17 7
m
2

d)

8 7
m
2

e) 8m
65 - (Unimontes MG) Uma escada de 6m de comprimento est apoiada
numa parede de

3 3m

verticalmente para baixo,

de altura. Se o topo da escada se deslocar

3m

, ento o deslocamento horizontal do p da

escada , em m, igual a

Se a partir da situao descrita na figura so necessrias mais 45 voltas


completas da manivela para que o teodolito aponte para o topo da parede, a
medida de h, em metros, igual a
a)

0,75 3 1 2

c)


4 2 1

d)

2 6 3

b)

a)

2 6

b)

c)

2 6 3

2 6 3
d)

3 6 2

66 - (UFLA MG) Um aparelho construdo para medir alturas e consiste de


um esquadro com uma rgua de 10 cm e outra rgua deslizante que permite
medir tangentes do ngulo de visada , conforme o esquema a seguir:

2 3 1

Uma pessoa, utilizando o aparelho a 1,5 m do solo, toma duas medidas, com
distncia entre elas de 10 metros, conforme esquema:

3 2 1

e)

62 - (ESPM SP) Na figura abaixo, ABCD um quadrado cujo lado mede

2 3
a)

. O tringulo AEG eqiltero. A medida do segmento DF igual a:

1 3

b) 2
c)

d) 1
e)

Sendo l1

3
2

63 - (PUC MG) Em
um
mapa,
o
parque turstico P e as cidades A, B, C e D esto dispostos conforme a figura
ao lado, sendo AB paralelo a CD. Sabendo-se que, na realidade,

AB 40km , AD 30km e
at o parque P, em quilmetros, :

DC 25km

30 cm

l 2 20 cm , calcule a altura da rvore.

67 - (UFG GO) Para dar sustentao a um poste telefnico, utilizou-se um


outro poste com 8 m de comprimento, fixado ao solo a 4 m de distncia do
poste telefnico, inclinado sob um ngulo de 60, conforme a figura abaixo.

, a distncia da cidade A

a) 65
b) 70
c) 75
d) 80

64 - (UFJF MG)
Uma mesquita possui uma
abboda
semiesfrica de 4 m de raio,
cujo centro dista 7 m
do cho e 5 m das paredes
laterais. A figura ao
lado representa um corte
em perfil, em que
um menino, afastado 6 m
da parede lateral, mirando em A, v o ponto B na abboda. Considerando-se
os olhos do menino a 1 m do cho e desprezando-se a espessura das
paredes para o clculo, a altura do ponto B ao cho :

Considerando-se que foram utilizados 10 m de cabo para ligar os dois postes,


determine a altura do poste telefnico em relao ao solo.
68 - (UFG GO) O desenho abaixo, construdo na escala 1:7000, representa
parte do bairro gua Branca em Goinia. As ruas R. 1, R. 2 e R. 3 so
paralelas Av. Olinda. O comprimento da Av. B, da esquina com a Av. Olinda
at a esquina com a Rua Dores do Indaya, de 350 m.

147

Em relao aos 6 ladrilhos triangulares colocados perfeitamente nos espaos


da figura 1, como indicado na figura 2, correto dizer que
a) 2 so tringulos eqilteros e 4 so tringulos issceles de ngulo da base
medindo 15.
b) 2 so tringulos eqilteros e 4 so tringulos issceles de ngulo da base
medindo 30.
c) 2 so tringulos issceles de ngulo da base medindo 50 e 4 so
tringulos issceles de ngulo da base medindo 30.
d) 2 so tringulos eqilteros e 4 so tringulos retngulos issceles.
Considerando-se que cada rua mede 7 m de largura, calcule quantos metros e) 2 so tringulos eqilteros e 4 so tringulos escalenos.
um pedestre caminhar na Av. B, partindo da esquina com Av. Olinda, at a
73 - (UFMS MS) A escadaria, ilustrada a seguir, de degraus iguais, ser
esquina com a Rua R. 2, sem atravess-las.
reformada e transformada em uma rampa para facilitar o acesso. A nova
69 - (UFPE PE) Na ilustrao a seguir, os segmentos BC e DE so paralelos
rampa iniciar no segmento AB (ao p da escadaria) e terminar no
segmento CD (no topo da escadaria).

Se BC = 12, DG = 7 e GE = 8, quanto mede FC?


a) 6,2
b) 6,3
c) 6,4
d) 6,5
e) 6,6
70 - (UFViosa MG) Sob duas ruas paralelas de uma cidade sero
construdos, a partir das estaes A e B, passando pelas estaes C e D, dois
tneis retilneos, que se encontraro na estao X, conforme ilustra a figura
abaixo.

A distncia entre as estaes A e C de 1 km e entre as estaes B e D, de


1,5 km. Em cada um dos tneis so perfurados 12 m por dia. Sabendo que o
tnel 1 demandar 250 dias para ser construdo e que os tneis devero se
encontrar em X, no mesmo dia, CORRETO afirmar que o nmero de dias
que a construo do tnel 2 dever anteceder do tnel 1 :
a) 135
b) 145
c) 125
d) 105
e) 115
71 - (Furg RS) O valor de x, na figura abaixo,
a) 24.
b) 13.
c) 5.
d) 8.
e) 10.

A norma de segurana determina que a rampa dever ter elevao mxima


de 30%, isto , quando o deslocamento realizado sobre a rampa, a
metragem de subida na vertical dever ser no mximo 30% da metragem do
deslocamento na horizontal.
Sabe-se que cada degrau tem 70 cm de largura e que a altura do perfil de
cada degrau de 18 cm, ento, a respeito da rampa, que iniciar no
segmento AB e terminar no segmento CD, correto afirmar que
(Use se necessrio

2 1,4,

3 1,7 e tg 50 1,2 )

a) ela estar de acordo com a norma de segurana citada


de inclinao inferior a 15.
b) ela estar de acordo com a norma de segurana citada
de inclinao entre 15 e 25.
c) ela estar de acordo com a norma de segurana citada
de inclinao entre 25 e 30.
d) ela no estar de acordo com a norma de segurana
ngulo de inclinao entre 30 e 45.
e) ela no estar de acordo com a norma de segurana
ngulo de inclinao superior a 45.

e ter um ngulo
e ter um ngulo
e ter um ngulo
citada e ter um
citada e ter um

74 - (PUC MG) Dois navios partiram ao mesmo tempo de um mesmo porto,


seguindo em direes perpendiculares; um deles navegando velocidade
constante de

24 km h

e o outro velocidade constante de

32 km h .

Aps 45 minutos, a distncia entre esses dois navios, em quilmetros, era


aproximadamente igual a:
a) 25
b) 30
c) 40
d) 45
75 - (UFLA MG) Uma criana brinca em um balano feito com uma corda de
2 metros de comprimento. Por razes de segurana, sua me a proibiu de
balanar atingindo uma altura superior a 2 metros do solo. O ngulo mximo
que ela pode atingir de

72 - (UFSCar
SP) A figura 1
representa um determinado encaixe no plano de 7 ladrilhos poligonais a) = 60
regulares (1 hexgono, 2 tringulos, 4 quadrados), sem sobreposies e b) = 30
c) = 45
cortes.
d) = 15
e) = 90
76
(Unifesp SP) Imagine uma parede vertical com uma janela retangular, de
lados a e b, conforme a figura, onde a paralelo ao piso plano e horizontal.
Suponhamos que a luz solar incida perpendicularmente ao lado a, com
inclinao de 60 em relao parede.

148

79 - (UEG GO) Deve ser demarcado um terreno na forma de tringulo


retngulo com 600 m2 de rea, cujo maior lado mede 50 m. Quantos metros
lineares de muro sero necessrios para cercar esse terreno?
a) 190
b) 150
c) 130
Se A1 e A2 representam, respectivamente, as reas da janela e de sua d) 120
e) 110
A1
imagem projetada no piso, a razo
vale:
C 30 o . Se
A2
80 - (EFEI MG) Um tringulo ABC tem AB 5 cm e AB
a)

c)
e)

3
3
2

3
2
1
2

b)

d)

3
3

a sua rea mede


a) Escaleno.
c) Issceles.

5 3
cm2 , pode-se afirmar que esse tringulo :
4
b) Eqiltero.
d) Retngulo.

81 - (UFRRJ RJ) Milena, diante da configurao representada abaixo, pede


ajuda aos vestibulandos para calcular o comprimento da sombra x do poste,
mas, para isso, ela informa que o sen = 0,6.

77 - (FMTM MG) A figura 1 indica uma pirmide de base quadrada, cuja


planificao est representada na figura 2.

Calcule o comprimento da sombra x.


82 - (Fuvest SP) Um lateral L faz um lanamento para um atacante A,
situado 32 m sua frente em uma linha paralela lateral do campo de
futebol. A bola, entretanto, segue uma trajetria retilnea, mas no paralela
lateral e quando passa pela linha de meio do campo est a uma distncia de
12m da linha que une o lateral ao atacante. Sabendo-se que a linha de meio
do campo est mesma distncia dos dois jogadores, a distncia mnima que
o atacante ter que percorrer para encontrar a trajetria da bola ser de:
a) 18,8m
b) 19,2m
c) 19,6m
d) 20m
e) 20,4m

O quociente

a) 1.
c)

e)

3 3
2

x
y

igual a:

b)

6
2

d)

78 - (Unimontes MG) Se no tringulo retngulo ABC abaixo

AC 5 , encontre BD .

AB 4

83 - (UEPG
PR)
Na
diviso
do
polinmio
5
p(x) = x
3x3 6 por
s(x) = x 2
obtm-se
quociente q(x) = x4 + bx3 + cx2 + dx + e
assinale o que for correto.
e
01. a = c
02. b = d
04. a + b = 4
08. f = 2
16. b + d = e

e resto r(x) = f. Isto posto,

84 - (UFOP MG) O resto da diviso do polinmio p(x) = x99 2x + 3 pelo


polinmio q(x) = x2 1 :
a) x +3
b) 6
c) 8
d) 3x - 1

149

85 - (UFAC AC) Um polinmio com coeficientes reais (na varivel x) p(x)


divisvel por (x - ) se, e somente se, p() = 0 [J. R. dAlembert (17171783)]. Admitindo este resultado, se temos x 4 2x3 + mx2 64x + n divisvel
por x2 6x + 5, vale que:
a) m = n = 5.
b) m + n = 68.
c) m = 70 e n = 10.
d) m = 5 e n = 70.
e) m = 60 e n = 5.
86 - (UFViosa MG) Sejam f e g funes reais tais que f(g(x)) = x2 3x +

2 e g(x) = 2x 3, para todo x R . A partir dessas informaes,


considere as seguintes afirmativas, atribuindo V para a(s) verdadeira(s) e F
para a(s) falsa(s):
( ) As razes de f so 1 e 1.
( ) O produto de f(3) e g(f(7)) igual a 60.
( ) O resto da diviso de f(g(x)) por g(x) igual a
( ) Para todo

x 3 tem-se que

f(g(x))

2.

1
.
4

95 - (UFF RJ) Na decomposio de um polinmio P(x), um aluno utilizou o


algoritmo conhecido como de Briot-Ruffini, conforme indicado abaixo:

-4

-2

-2

-4

-2 1

-2

Com base nos dados acima, determine o polinmio P(x) e todas as suas
razes.
96 - (UFSC SC) Determine o valor de m, para que o resto da diviso do
polinmio P(x) = x + mx - 2x + 1 por x + 3 seja 43.
97 - (UERJ RJ) A figura abaixo representa o grfico de um polinmio P e
uma reta r que lhe secante nos pontos A (2, -3) e B (4, 15).

A seqncia CORRETA :
a) FFVF.
b) VFVF.
c) FVVF.
d) VVFV.
e) FVFV.

15

r
B

2
87 - (UFRR RR) Um polinmio P(x) dividido por (x-2) d resto 4 e dividido
por (x-4) d resto 12. Ento o resto da diviso de P(x) por (x-2) (x-4) :
4
x
A
-3
a) 4x-4
b) 3x-2
c) 2x-1
d) 4x
e) 0
a) Determine o resto da diviso de P(x) por x - 4.
b) Mostre que a reta r representa graficamente o resto da diviso de P(x) por
88
(UEPB
PB)
O
quociente
da
diviso
de (x - 2) (x - 4).
P(x) 2x 4 3x 3 4x 2 3x 2 por ( x 1) ( x 1) ( x 2) igual 98 - (UFOP MG) Sejam os polinmios P(x) = x4 + ax2 + bx 1 e Q(x) = x3
a:
+ x2 + x + 1. Se P(x) divisvel por Q(x), ento a afirmativa correta :
a) 2x 1
a) a = 0 e b = -1
b) a = 0 e b = 0
b) 1 2x
c) a = 0 e b = 1
d) a = 1 e b = 0
c) 2x + 1
e) a = 1 e b = 1
d) 1 2x
e) 2x
99 - (Unicamp SP) Seja p(x) = x3 12x + 16.
a) Verifique que x = 2 raiz de p(x).
89
(UEM
PR)
Considere
o
polinmio: b) Use fatorao para mostrar que se x > 0 e x 2, ento p(x) > 0.
c) Mostre que, entre todos os prismas retos de bases quadradas que tm
p(x) x 5 x 4 x 3 x 2 x 1 .
volume igual a 8m3, o cubo o que tem menor rea total.
correto afirmar que:
2
100 - (Unimontes MG) O polinmio A(x) representa a rea de um
a) o grau do quociente da diviso de p(x) por d( x ) x x 1 3.
quadrado de lado, a 1 e o polinmio V(x) representa o volume de um
b) o resto da diviso de p(x) por d ( x ) x 2 r ( x ) 63 .
cubo de aresta a 1 . O grau de A(x) V(x)
d(x ) x 1
c) o
quociente
da diviso
de
p(x) por
a) 3.
b) 6.
c) 5.
d) 4.
q(x) x 4 2x 3 3x 2 4x 5 .
101 - (UFMT MT) A diviso de um polinmio de coeficientes reais P(x) por
d) p(x) possui raiz real.
( x 1) apresenta como quociente um polinmio Q(x) de grau 3 com o
e) p 2 7 2 1
coeficiente do termo de maior grau igual a 1 e, como resto, ( x 3) . O

90 - (UFG GO) Determine o valor de

p(x) kx 3 (k 1)x 2 2kx 6

k IR ,

para que o polinmio


2
seja divisvel por x 2 .

grfico de Q(x) mostrado na figura abaixo.

91 - (FEI SP) Determine as razes da equao x3 16x2 + 85x 150 = 0,


sabendo que uma das razes tem multiplicidade 2.
92 - (Mau SP) Dada a equao x3 2x2 + 2x 1 = 0, determine todas as
razes, sabendo-se que x0 = 1 uma raiz.
93 - A raiz x = 1 da equao x4 x3 3x2 + 5x 2 = 0 :
a) simples
b) dupla
c) tripla
d) quadrupla
e) quintupla
94 - (ITA SP) Sabendo-se que 3x 1 fator de 12x3 19x2 + 8x 1 ento
as solues reais da equao 12(33x) 19(33x) + 8(3x) 1 = 0 somam:
a) log312
b) 1
c)

1
log312
3

e) log37

d) -1

A partir dessas informaes, qual a soma dos coeficientes de P(x)?


a) 1
b) 2
c) 0
d) 1
e) 2
102 - (UEM PR) Assinale a(s) alternativa(s) correta(s).
01. Se p(x) e q(x) so polinmios e o grau de p(x) maior do que o grau de
q(x), ento p(x) tem mais razes reais do que o polinmio q(x).

150

p(x) x 3 2x 2 x 1

02. O grfico da funo polinominal

passa pelos

pontos (0,1) e (1,3) e tem uma raiz no intervalo [0,1].


04. O valor mnimo absoluto da funo f dada por

f ( x ) x 2 2x

111 - (UFTM) O polinmio P(x) = x3 + a . x2 + b . x + c tem conhecidos os


valores P(1) = 4 , P(1) = 8 e P(2) = 5. Sabendo-se que uma raiz de
P(x) vale b . a , ento o produto de suas 3 razes igual a:
a) 3.
b) 2.
c) 1.
d) 2.
e) 3.

2
112 - (UFAM AM) Sabe-se que 2 a raiz da equao x 4 4x3 + x2 + 6x =

0. A forma fatorada dessa equao :


a) x (x 2) (x - 1) (x + 3)
b) x (x 2) (x - 1) (x 3)
08. Dividindo-se o polinmio p(x) pelo polinmio q ( x ) x 2 , obteve-se c) x (x + 2) (x + 1) (x 3)
d) x (x 2) (x +1) (x 3)
resultado zero. Pode-se afirmar que 2 uma raiz de p(x).
16. Sabendo-se que o polinmio p(x) tem razes r1, r2, r3, , rn e que o e) x (x + 2) (x - 1) (x 3)
polinmio q(x) tem razes s1, s2, s3, , sn, ento o polinmio
MG) Se a, b, e c so razes reais do polinmio p(x) = 20 x3
h ( x ) p ( x ) q ( x ) tem razes r1 s1 , r2 s 2 , r3 s 3 , , rn s n . 113 - (UFViosa
+ 20 x2 + 9 x + 1, ento log(a2 + b2 + c2), onde log denota logaritmo
32. Uma seqncia de polinmios foi construdas da seguinte forma:
decimal, :
p0(x) = 1
a) 2
b) 1
c) 2
d) 0
e) 1
p1(x) = x
pm + 2(x) = pm(x) + x pm + 1(x), m 0.
114 - (Unioeste PR) O capital livre para investimentos, ou o capital de giro,
Assim, pode-se afirmar que o polinmio p5(x) tem grau 5 e x = 0 uma de de uma empresa (C) em funo dos anos (t) de sua existncia no mercado
suas razes.
3
2
pode ser descrito pela funo C(t) t 14t 56t 64 . Sabe-se que

3
2

103 - (Unimontes MG) Se P(x) um polinmio do segundo grau com


coeficientes reais, tais que P (0) 1 e P ( x 1) P ( x ) 8x 2 , ento
a)

P(x) 2x 2 4x 1

b)

P(x) 4x 2 2x 1

c)

P( x ) 2x 2 4x 1

d)

P(x) 4x 2 2x 1

104 - (UFG GO) Sendo x R ,


para os quais vale a decomposio:

x 1 ,

115 - (FGV) O polinmio

encontre os valores de A, B e C,

x
A
Bx C

2
2
x

1
(x 1)(x 1)
x 1
105 - (FGV ) O polinmio

P(x) ax3 bx 2 cx 2

a) P(1) = 1
c) P(2) = 0
e) P(2) = 12

P(x) x 3 kx 2 6x 5

. ento, a soma das razes da equao


a) 6

b) 7

116

c) 6

d) 9

(UFMS

satisfaz as

divisvel por

e) 3

MS)

Seja

b) P(1) = 0
d) P(2) = 8

conjunto

chamado conjunto dos nmeros

complexos.
Sabe-se que o nmero complexo z = a + bi pode ser representado pelo
ponto (a,b) no plano cartesiano de eixos x e y. Denotaremos por
o conjugado do nmero complexo z.

, qualquer que seja x real. Ento:

x 5

P ( x 1) 0 :

C {z a bi / a, b reais e i - 1}

seguintes condies:

P( 1) 0

3
P ( x ) P ( x ) x

no 4 ano a empresa apresentou um capital de giro igual a zero. Os demais


anos, nos quais a empresa apresentou um capital de giro tambm igual a
zero, somam
a) 8.
b) 5
c) 10.
d) 7.
e) 11.

Considerando as solues, em C, da equao

x2

z a bi

0 , assinale a(s)

proposio(es) correta(s).
01. O mdulo da soma dos quadrados de suas 8 razes 4.
02. As suas razes, representadas no plano cartesiano, formam os vrtices de
um octgono regular.
04. Sendo z uma das razes da equao, ento z tambm o .
08. Existe uma raiz da equao dada que tem multiplicidade 4.

106 - (UEPG PR) Em relao equao x3 7x2 + 14x 8 = 0, assinale o


4
2 2
16. O polinmio p(x) x x
no pode ser decomposto num produto
que for correto.
de fatores do 1 grau, em C.
01. A soma de suas razes 7.
02. Uma das razes nula.
117 - (UEPB PB) Em C, o conjunto soluo da equao x2 6x + 10 = 0,
04. As suas razes constituem uma progresso geomtrica.
igual a:
08. O produto de suas razes um nmero mpar.
a) S = {3i , 3i}
b) S = {3 + i , 3 i}
16. Uma das razes imaginria.
c) S = {i 3 , i + 3}
d) S = {3 + i , 3 i}
107 - (UFSC SC) As dimenses, em metros, de um paraleleppedo retngulo e) S = {3 i , 3 i}
so dadas pelas razes do polinmio x 3 14x2 +56x 64. Determine, em
metros cbicos, o volume desse paraleleppedo.
118 - (UFLA MG) Sabendo-se que x i uma das razes da equao
108 - (PUC SP) Sabe-se que a equao x4 + x3 4x2 + x + 1 = 0 admite
4
3
2
razes inteiras. Se m a maior das razes no inteiras dessa equao, ento polinomial x 2x 3x 2x 2 0 , obtenha as outras razes.
119
(UFRRJ
RJ)
Sabendo
que
a
unidade
imaginria
1
o valor de m
:
i C (C completo e i - 1 raiz do polinmio p(x) = x4 + x3 + kx2 +
m
a) 6
b) 3
x 2, determine:
a) o valor de k ;
c) 0
d) 5
b) o conjunto soluo S da equao p(x) = 0.
e) 2

120 - (UFMS MS) Sabendo-se que os nmeros complexos

109 - (Mackenzie SP) Se a, b e c so as razes da equao x 3 2x2 + 3x

1 1 1
vale

a b c
7
4
b)
c)
3
3

4 = 0, ento
a)

2
3

z2 1 i 5
dada por

d)

3
4

e)

1
4

so razes da equao polinomial com coeficientes reais,

x 4 2x 3 mx2 2x n 0 , calcule o valor de m n .

121 - (UFViosa MG) Considere

110 - (Fepecs DF) A soma das razes da equao x3 6x2 + 11x 6 = 0


igual a:
por
a) 3;
b) 1;
c) 0;
d) 1;
e) 6.

z1 i

f :R R

uma funo real definida

1
cos x 2

f ( x ) det senx 1
2
0 - senx cosx

151

O grfico cartesiano que melhor representa a funo f :

a)

13) C 19.315 reais

14) E

15) B

16) 893,50 m

17) A

18) B

19) C

20) B

21) A

22) A = (4-)cm

23) B

24) B

25) a) 300 trabalhadores / b) 120 trabalhadores


26) a) 343(1+3)cm / b) 8746,5(1++3)cm
27) B
30)
b)

=1

e)

122 - (Furg RS) O polinmio tem x 7x + 16x 12 tem:


a) uma raiz real com multiplicidade 3.
b) uma raiz real com multiplicidade 2.
c) razes reais e distintas.
d) uma raiz complexa.
e) duas razes complexas.

2) B

3) A

4) C

5) E

6) B

7) D

8) C

9) B

10) A

11) 100

12) A

32) A
35) a

36) E

37) E

38) E

39) D

40) E

41) B

43) a)

rea do tringulo

PQ = 43dm; sen (BQ) / b) =90; 120 voltas

b) 2+1
45) E

46) D

47) 32 voltas

48) 63+6cm

49) a) 18

50) A

51) C

52) D

53) A

54) 94

55) E

56) D

57) A

58) A

59) A

60) A

61) C

62) D

63) C

64) B

65) C

66)

67) 12,92m

68) L=168m

69) C

70) C

71) C

72) D

73) B

74) B

75) A

76) D

77) B

78)

79) D

80) C

81) X 13,33m

82) B

83) 27

84) A

85) D

86) B

87) A

88) A

89) B

90) K=2

91) V={5;6}

93) C

94) A

92) {1;
1) B

31) C
34) B

44) a) 1

d)

29) A

33) E

42) E
c)

28) B

95) P(X)=X4+X3-4X2-2x+1
96) 07

97) *

98)

99) **

100) A

101) B

102) 42

103) B

104) A=C= e B=-


105) C

b) 42cm

106) 05

152

107)

108) B

109) D

110) E

111) E

112) D

113) E

114) C

115) D

116) 13

117) B

118)

119) a) k=-1 / s={i, -i, 1,


120) 013
2}

121) A

122) B
*97) a) O resto P(4) = 15.

*99) a)
p(2) = 2 - 12. 2 + 16 = 0, logo 2 raiz
b) p(x) = (x 2) (x + 2x 8) = (x 2) (x 2) (x + 4), logo se x >
0 e x 2, ento (x 2) > 0 e x + 4 > 0. Portanto p(x) > 0
c)

8 = ah

h=

8 (1)
a2

Substituindo 1 em 2 temos:

St = 4ah + 2a (2)

St a2 16 a3

Somando e subtraindo 12a da expresso dos colchetes, temos:

St a2 16 - 12a a3 12a
2
Logo: St 2 (a - 2) (a 4) 24 , como a > 0, S mnimo
a
se, e somente se, (a 2) (a + 4) = 0, da temos que a = 2,
ento de (1) h = 2, e, portanto, o prisma reto o cubo.

b) Se o resto dessa diviso da forma: R(s) = ax + b; e o quociente

Q(x), ento:
P(x) = (x - 2) (x - 4) . Q(x) + ax + b
P(4) = 4a + b = 15
P(2) = 2a + b = -3

Resolvendo o sistema, obtemos a = 9 e b = -21; Assim, R(x) = 9x -21


polinmio do 1 grau cujo grfico uma reta. Essa reta contm os
pontos A e B, pois R(2) = -3 e R(4) = 15, logo a reta r.

1. Tcnicas especiais permitem multiplicar clulas meristemticas da regio


apical do caule em laboratrio. Desta forma, milhares de plntulas podem ser
gerados assexuadamente a partir de um nico vegetal escolhido. Sobre o
exposto, assinale a alternativa correta:
a) os vegetais gerados deste modo perdem a capacidade de realizar
fotossntese.
b) por serem clulas meristemticas, j apresentam alto grau de
diferenciao.
c) se a forma de propagao fosse sexuada, os descendentes no poderiam
se reproduzir assexuadamente.
d) essas tcnicas pouco influem no desenvolvimento de novas prticas
agrcolas.
e) todas as plntulas geradas por esse processo so geneticamente iguais.
2. A indstria madeireira explora principalmente o cerne de uma rvore que
constitudo por:
a) floema mais recente.
b) xilema mais antigo.
c) tecido parenquimtico.
d) tecido esclerenquimtico.
e) tecido meristemtico.
3. Assinale as afirmaes abaixo:
I- Evaporao de gua a partir das clulas dos parnquimas assimiladores.
II- Conduo de gua pelo xilema.
III- Sada de vapor dgua pelos ostolos.
IV- Circulao do vapor dgua pelos espaos intercelulares.
A seqncia de eventos que leva transpirao estomtica, quando da
entrada da dgua na folha :
a) I II III IV.
b) II I IV III.
c) IV - III II I.
d) III I II IV.
e) III IV I II.

As situaes A e B esto relacionadas com os seguintes fenmenos:


Alta (I)
Intensidade de luz
Baixa (II)
Alta (III)
Concentrao de CO2
Baixa (IV)
Alto (V)
Suprimento de gua
Baixo (VI)
Para ocorrer a situao A so necessrios:
a) I, IV e V.
b) I, III e V.
c) I, IV e VI.
d) II, IV e VI.
e) II, III e V.
5. Os esquemas abaixo representam detalhes da epiderme de duas folhas,
designadas por I e II.

4. Considere os seguintes esquemas do estmato:

153

Com relao anlise dos esquemas, pode-se dizer que:


a) em I, as clulas estomticas apresentam menor suprimento hdrico que
em II.
b) em I, as clulas estomticas apresentam maior suprimento hdrico que em
II.
c) em I, as clulas estomticas esto trgidas.
d) em II, as clulas estomticas esto murchas.
e) em I e II, as clulas estomticas devem apresentar o mesmo suprimento
hdrico.
6. Uma haste cortada e florida de roseira foi mergulhada em soluo de azul
de metileno. Aps algumas horas observou-se a alterao na colorao das
flores pelo corante. Explica-se o fenmeno:
a) a presso osmtica das clulas do floema absorvem o corante.
b) a presso atmosfrica promove a ascenso do corante pelo interior do
xilema.
c) a fora resultante da transpirao movimenta a soluo atravs das clulas
do xilema.
d) a coeso das molculas do corante responsvel pela subida da seiva.
e) o corante sobe por difuso atravs dos microcapilares das clulas do lber.
7. Segundo a teoria de suco das folhas de Dixon, pode-se afirmar:
I- Quando maior a transpirao foliar, tanto maior ser a conduo de seiva
bruta pelos vasos xilemticos.
II- O reforo de lignina dos vasos lenhosos impede a colapso desses vasos,
quando a transpirao foliar for muita intensa.
III- A coluna dgua mantm-se contnua no interior dos vasos lenhosos,
gras `fora de coeso das molculas de gua.
Esto corretas:
a) apenas I e II.
b) apenas I e III.
c) apenas II e III.
d) todas.
e) nenhuma.
8. O experimento mostrado na figura abaixo, conhecido como cintamento ou
anel de Malpighi, consiste na retirada de toda a casca do caule e uma
pequena poro da sua parte interna at o cmbio vascular, em torno da
base do mesmo.

Este sistema utilizado para explicar o movimento da seiva orgnica nos


vegetais onde 1, 2 e 3 podem representar, respectivamente:
a) folha raiz floema.
b) raiz folha xilema.
c) folha raiz lenho.
d) flor raiz lber.
e) folha flor xilema.
11.

A respeito da organela representada acima, assinale a alternativa incorreta:


a) Est presente em todos os organismos auttrofos.
b) A estrutura 1 apresenta pigmentos que absorvem energia utilizada na
produo de ATP.
c) Em 2, h enzimas que utilizam CO2 para fabricao de glicose.
d) Essa organela possui capacidade de autoduplicao.
e) O processo realizado por essa organela ocorre em 2 etapas, sendo que
uma delas no depende de luz.
12. Observando-se as reaes de uma planta iluminao, em condies
experimentais, foi possvel construir um grfico, onde a linha pontilhada
representa a respirao e a linha cheia, a fotossntese.

A anlise do grfico permite concluir que a tendncia da planta :


a) desenvolver fototropismo.
b) reproduzir-se.
c) acumular reservas nutritivas.
d) definhar-se por falta de alimento disponvel.
e) crescer.
Sobre esses processos, a alternativa verdadeira :
a) teremos o comprometimento dos vasos liberianos por estarem
posicionados mais externamente no caule.
b) aps alguns dias, observaremos a morte das razes como conseqncia da
interrupo do fornecimento de seiva bruta.
c) este cintamento compromete imediatamente os vasos lenhosos.
d) novos vasos liberianos so formados mais internamente no caule para
compensar a obstruo causada nos vasos mais externos.
e) as folhas nunca sero prejudicadas pelo cintamento.
9. Uma das afirmativas abaixo, sobre a fase luminosa da fotossntese,
falsa. Assinale-a.
a) ocorre quebra de molculas de gua.
b) forma-se NADPH2.
c) h liberao de O2.
d) h produo de ATPs.
e) h consumo de CO2.
10. A figura a seguir representa um conjunto de dois osmmetros, com
concentraes diferentes, ligados por um tubo de vidro e mergulhados em
gua destilada.

13. Considere o ciclo abaixo e analise a alternativa incorreta:

a) em brifitas, 3 a fase haplide e duradoura e 1, a fase diplide e


temporria.
b) em brifitas, 4 pode ser mvel ou imvel.
c) em pteridfitas, 3 fase transitria e 4 haplide.
d) em brifitas e pteridfitas, a fecundao ocorre entre 2 e 3 e a meiose
entre 1 e 2.
e) em samabaias, 2 geralmente forma-se na fase inferior da folha.
14. Qual das figuras abaixo representa a resposta de uma caule que recebeu
luz unilateral vinda da direita?

154

15. O esquema abaixo representa uma experincia com hormnios vegetais:

a) 1, mantendo-se o crescimento para cima.


b) 2, o que explicar o geotropismo negativo dos caules.
c) 3, o que far com que o caule cresa no sentido da fonte luminosa.
d) 3, o que explicar o fototropismo negativo do caule.
e) 4, o que far com que o caule cresa no sentido da fonte luminosa.
19. O esquema abaixo corresponde a uma flor completa de angiosperma.

Uma plntula foi retirada da sua caixa de germinao e colocada


horizontalmente sobre um suporte de madeira durante certo tempo,
suficiente para permitir seu crescimento. O resultado desse experimento est
representado pela seguinte figura:

Assinale a opo que apresenta a relao correta do nmero com a nome da


estrutura indicada.
a) 1-spala; 2-ptala; 3-estilete; 4-antera; 5-ovrio; 6-filete; 7-estigma.
b) 1-spala; 2- ptala; 3- filete; 4-antera; 5-ovrio; 6- estilete; 7-estigma.
c) 1-spala; 2-ptala; 3-filete; 4-antera; 5- ovrio; 6- estigma;7-estilete.
d) 1- spala;2-ptala; 3- filete; 4-antera; 5- estigma; 6- estilete; 7-ovrio.
e) 1-ptala; 2-spala; 3- estilete; 4-antera; 5-ovrio; 6-filete; 7-estigma.

16. Os experimentos abaixo (A e B) forma montados com a finalidade de se


demonstrar a ocorrncia de respectivamente:

a) geotropismo e geotropismo.
b) fototropismo e geotropismo.
c) fototropismo e fototropismo.
d) nastismo e fototropismo.
e) geotropismo e fototropismo.
17. Ao contrrio do que ocorre com a absoro da gua, a absoro dos
nutrientes pelas razes requer que a concentrao interna desses materiais,
nas clulas, seja superada. O fenmeno que torna possvel a penetrao de
nutrientes atravs da raiz, contra o gradiente de concentrao, envolvendo
gastos energticos, conhecido como:
a) difuso.
b) osmose.
c) transporta ativo.
d) transporte passivo.
e) difuso facilitada.

20. O desenho corresponde ao esquema de um vulo de angiosperma. Aps


a dupla fecundao, o zigot e o endosperma triplide sero originados,
respectivamente, a partir de:

a) I e II.
b) II e III.
c) I e IV.
d) I e III.
e) II e IV.

21. O esquema abaixo resume o processo da fotossntese.

18. Abaixo est representado, esquematicamente, o pice de um caule. A


seta indica o ponto onde a luz incide. Pode-se, ento, deduzir, que a
concentrao maior de auxina ser na regio numerada por:

Os nmeros 1, 2, 3 e 4 representam, respectivamente, as seguintes


substncias:
a) gua, oxignio, gs carbnico e glicose.
b) gua, gs carbnico, oxignio e glicose.
c) gs carbnico, oxignio, gua e glicose.
d) gs carbnico, gua, glicose e oxignio.

155

e) oxignio, gs carbnico, gua e glicose.

a) em plena atividade fotossinttica.


b) em estgio de primrdio.
c) em plena maturao.
d) entrando na fase de distenso.
e) atingindo a fase senescente.

22. A maior parte da matria orgnica de uma rvore provm de:


a) gua do solo.
b) gs carbnico do ar.
c) gs oxignio do ar.
d) compostos nitrogenados do solo.
e) sais minerais do solo.
23. Ao compararmos as pteridfitas s angiospermas,
caractersticas a seguir as primeiras no apresentam?
a) independncia de gua.
b) gametfito reduzido.
c) polinizao anemfila.
d) esporfito autotrfico.
e) esporngios.

qual

das

24. Bananeira, cana-de-aucar, coqueiro, laranjeira e pinheiro-do-Paran tm


em comum:
a) o mesmo tipo de caule areo.
b) a mesmo tipo de raiz.
c) a presena de estmatos nas folhas.
d) o fruto com estrutura protetora das sementes.
e) a semente com o mesmo nmero de cotildones.
25. Ao realizar a remoo da extremidade superior do caule de uma planta,
esta geralmente apresenta o desenvolvimento de gemas laterais. O
fenmeno responsvel pela inibio das gemas laterais denominado:
a) fototropismo e controlado hormonalmente.
b) fototropismo e no controlado hormonalmente.
c) dominncia apical e controlado hormonalmente.
d) dominncia apical e no controlado hormonalmente.
e) geotropismo e no controlado hormonalmente.
26. A presena de sementes uma adaptao importante de certos grupos
vegetais ao ambiente terrestre. Caracterizam-se por apresentar sementes:
a) pinheiros e leguminosas.
b) gramneas e avencas.
c) samambaias e avencas.
d) musgos e samambaias.
e) gramneas e musgos.
27. Quando se caminha de grupos mais simples (brifitas) para outros mais
complexos (pteridfitas gimnospermas angiospermas) tem-se:
a) Dependncia de gua para a fecundao.
b) Aparecimento de meiose esprica.
c) Evoluo do gametfito.
d) Involuo do esporfito.
e) Evoluo do esporfito.
28. Quantidades X, 2X e 3X de DNA so encontradas, respectivamente, nas
clulas de:
a) Oosfera, caule e albmen.
b) Oosfera, saco embrionrio e caule.
c) Caule, oosfera e albmen.
d) Albmen, caule e oosfera.
e) Oosfera, albmen e caule.
29. Uma flor hermafrodita possui pelo menos:
a) clice e corola.
b) androceu e receptculo.
c) estames e pistilos.
d) ptalas e estames.
e) nenhuma das anteriores.
30. No caule de uma dicotilednea, aparecem dois meristemas que a fazem
crescer em espessura. Um deles produz lber para fora e lenho para dentro; o
outro, mais perifrico, forma o sber ou cortia. Esses meristemas
secundrios so respectivamente:
a) feloderma e esclernquima.
b) cmbio e felognio.
c) felognio e endoderma.
d) cmbio e esclernquima.
e) felognio e cmbio.
31. Quando o nvel de auxina de uma folha baixa acentuadamente, podemos
pressupor que esta folha esteja:

32. Qual das alternativas a seguir corresponde, respectivamente, ao


comportamento de uma planta de dia curto, com fotoperodo crtico igual a
10horas, e uma planta de dia longo com fotoperodo crtico igual a 14 horas?

a)
b)
c)
d)
e)

Tempo de exposio/dia
15 horas
8 horas
9 horas
11 horas
13 horas

Comportamento
No floresce
Floresce
Floresce
Floresce
No floresce
Floresce
No floresce
Floresce
Floresce
No floresce

33. O pinheiro-do-Paran produz pinhas femininas


que, aps a
fecundao, produzem pinhes. As palavras grifadas nessa
frase
correspondem a, respectivamente:
a) esporfito, estrbilos e frutos.
b) esporfito, estrbilos e sementes.
c) gametfito, cones e frutos.
d) gametfito, cones e sementes.
e) esporfito, inflorescncia e frutos.
34. A presena de semente uma adaptao importante de certos grupos
vegetais ao ambiente terrestre. Caracterizam-se por apresentar sementes:

a) pinheiros e leguminosas.
b) gramneas e avencas.
c) samambaias e pinheiros.
d) musgos e samambaias.
e) gramneas e musgos.
35. Considere os seguintes grupos de plantas: brifitas, pteridfitas,
angiospermas e gimnospermas, e assinale a alternativa correta.

a) brifitas so vasculares.
b) pteridfitas e gimnospermas so vasculares.
c) angiospermas e pteridfitas apresentam gro de plen.
d) brifitas e angiospermas apresentam flores e frutos.
e) gimnospermas foram as primeiras plantas vasculares.
36. Nos vegetais superiores, aps a fecundao, a flor perde suas estruturas
acessrias e os estames. Outras estruturas como ovrio, vulo e oosfera se
desenvolvem, originando respectivamente:

a) semente, fruto e endosperma.


b) fruto, semente e embrio.
c) semente, fruto e cotildones.
d) fruto,semente e endosperma.
e) fruto, endosperma e embrio.
37. Anemofilia, entomofilia e ornitofilia referem-se, respectivamente ao
transporte:
a) da semente pelo vento, por insetos e por aves.
b) do plen por insetos, pelo vento e por aves.
c) do plen pelo vento, por insetos e por aves.

156

d) da semente por aves, por insetos e pelo vento.


e) Nenhuma das anteriores.

e) pteridfitas.
41. comum entre as pteridfitas e gimnospermas a presena de:

38. As adaptaes florais que facilitam a polinizao pelo vento so:

a) frutos.

a) ptalas coloridas e glndulas odorferas.

b) esporfitos duradouros.

b) pequena produo de plen e glndulas odorferas.

c) sementes secas.

c) estigmas pequenos e ptalas coloridas.

d) endosperma triplide.

d) corolas grandes e estiletes longos.

e) fecundao dupla.

e) filetes longos e grande produo de plen.


39. Ao terminarem uma refeio Ronaldo solicitou caju de sobremesa, Bruno
pediu abacaxi e Antnio pediu morango. O que cada um comeu de
sobremesa, respectivamente foi:
a) pednculo floral, receptculo floral e infrutescncia.
b) receptculo floral, infrutescncia e pednculo floral.
c) receptculo floral, pednculo floral e infrutescncia.
d) pednculo floral, infrutescncia e receptculo floral.
e) pseudofruto, pseudofruto mltiplo e pednculo floral.

42. o nadar dos anterozides foi substitudo nas gimnospermas pelo crescer
do tubo polnico.
A frase relaciona-se:
a) aparecimento de fecundao cruzada.
b) polinizao cruzada.
c) independncia da gua para a fecundao.
d) auto-fecundao.
e) vida aqutica.

40. As caractersticas apresentadas no quadro abaixo referem-se a:

sementes

abrigadas no interior do fruto

folhas

com nervuras paralelas e invaginantes

flores

composta por 3 elementos ou mltiplos

frutos

possuem 3 lojas ou mltiplos

raiz

sistema radicular fasciculado

a) brifitas.
b) monocotiledneas.
c) gimnospermas.
d) dicotiledneas.

1)
6)
11)
16)
21)
26)
31)
36)
41)

E
C
A
E
A
A
E
B
B

2)
7)
12)
17)
22)
27)
32)
37)
42)

B
D
D
C
B
E
A
A
C

3)
8)
13)
18)
23)
28)
33)
38)

B
A
D
C
C
A
B
E

4)
9)
14)
19)
24)
29)
34)
39)

A
E
C
B
C
C
A
D

5)
10)
15)
20)
25)
30)
35)
40)

A
A
D
D
C
B
B
B

1. Um pesquisador, ao estudar o desenvolvimento embrionrio de um animal, a) Os adultos tm simetria bilateral e as larvas tm simetria radial.
verificou que o celoma se formava a partir das cavidades de bolsas b) As formas larvais tm simetria bilateral e os adultos tm simetria radial.
evaginadas do arquntero. Com base nessa informao pode-se conclui que c) Tanto adultos quanto larvas tm simetria bilateral.
o animal em questo
d) Tanto adultos quanto larvas tm simetria radial.
a) certamente um aneldeo.
e) Os adultos so assimtricos e as larvas tm simetria bilateral.
b) Pode ser um inseto.
c) Pode ser um mamfero.
5. So exemplos de animais que so esquizocelomados
d) certamente um Cnidrio.
a) Planria lombriga estrela do mar
e) Pode ser um Molusco.
b) Gafanhoto hidra homem
c) Polvo minhoca sapo
2. O arquntero uma cavidade presente no embrio de animais:
d) Marisco sanguessuga pulga
a) Diblsticos e triblsticos.
e) Macaco jacar ourio do mar
b) Pseudocelomados e celomados apenas.
c) Pseudocelomados apenas.
6. Um estudante escreveu o seguinte texto: Como as planrias so animais
d) Celomados apenas.
segmentados, elas so tambm celomadas, pois todos os animais
e) Triblsticos apenas.
segmentados so celomado. Essa afirmao est
a) Correta, pois as planrias so animais segmentados e todos os
3. Um estudante escreveu o seguinte texto: Como os Moluscos so
segmentados so celomados.
animais triblsticos, eles so tambm celomados (do tipo esquizoceloma) pois b) Incorreta, pois as planrias, apesar de serem animais celomados, no
todos os animais triblsticos so celomado. Essa afirmao est:
so segmentadas.
a) Correta, pois os moluscos so animais triblsticos e todos os triblsticos c) Incorreta, pois as planrias no so animais celomados, apesar de serem
so celomados.
segmentados.
b) Incorreta, pois os moluscos, apesar de serem animais triblsticos, no d) Incorreta, pois as planrias no so animais celomados, nem
so celomados.
segmentados.
c) Incorreta, pois os moluscos, apesar de serem animais celomados, no e) Incorreta, pois as planrias so pseudocelomados, apesar de serem
so triblsticos.
segmentados.
d) Incorreta, pois apesar de os moluscos serem triblsticos e celomados,
nem todo animal triblstico celomado.
7. Em relao a grupos de invertebrados, considere as caractersticas citadas
e) Incorreta, pois apesar de os moluscos serem triblsticos o celoma do abaixo.
tipo enteroceloma.
1. Presena de dois pares de antenas.
2. Corpo metamerizado.
4. Nos Equinodermos
3. Hbitat exclusivamente marinho.

157

4.
5.

Presena de exoesqueleto.
Locomoo atravs de sistema ambulacrrio.

Assinale a alternativa que apresenta a correspondncia correta entre o grupo


e suas caractersticas.
a) Aneldeos 2 e 5
b) Moluscos 2 e 4
c) Crustceos 3 e 4
d) Insetos 3 e 5
e) Equinodermos 3 e 5

b)

Os plos (para isolamento trmico) e as glndulas mamrias (para


alimentao dos filhotes) so caractersticas exclusivas do grupo, o que
no verdadeiro para o diafragma (um dos msculos envolvidos na
respirao).
c) No curso de sua evoluo, irradiaram-se ocupando os mais diferentes
nichos, como o alimentar (caso dos mamferos carnvoros, herbvoros e
onvoros), repercutindo em mudanas na dentio, msculos
mastigadores e trato digestrio.
d) So endotrmicos, semelhana das aves, mantm a temperatura
corporal elevada e constante.
e) Apresentam sempre placenta desenvolvida, fecundao interna e
viviparidade, exceto os monotremados e os marsupiais.
13. Os herbvoros desempenham importante papel dentro do ecossistema,
pois so parte integrante das cadeias trficas e possibilitam a circulao de
nutrientes e energia. Considerando essa informao, assinale a afirmativa
incorreta.

8. No ano 2000, pesquisadores da Universidade da Carolina do Norte (EUA)


divulgaram, pela primeira vez, a descoberta de um corao fossilizado, com
quatro cavidades, pertencentes a um dinossauro, Willo, que viveu h 66
milhes de anos. A preservao dessa rara evidncia anatmica veio somarse s feies osteolgicas observadas nos esqueletos fsseis de dinossauros.
Isso refora a hiptese filogentica de que, dos grupos citados nas
alternativas, os mais estreitamente relacionados com os dinossauros so:
a)
a)
b)
c)
d)
e)

Os lagartos e os crocodilos.
Os anfbios e as aves.
Os crocodilos e as aves.
As tartarugas e os mamferos.
Os lagartos e os mamferos.

b)
c)

Os herbvoros so organismos hetertrofos que podem ainda ser


classificados como consumidores primrios, pois so os primeiros a
consumir as plantas na cadeia alimentar.
Muitos herbvoros so mamferos pastadores, como antlopes e cervos,
que apresentam o rmen ou pana, local onde ocorre o mutualismo com
bactrias que digerem a celulose.
A grande variedade de espcies de mamferos pastadores na frica e na
Amrica do Sul existe devido ao fenmeno da simbiose entre estas
espcies pastadoras, e no da competio entre elas, ao longo do
processo evolutivo.
O predatismo ocorre geralmente sobre os animais herbvoros, sendo que
os predadores acumulam menores biomassa e energia que os
herbvoros, na pirmide trfica.
Uma das razes que explica a perda de energia ao longo da cadeia
trfica, como no caso da via capim-herbvoro-carnvoro, a perda de
calor do organismo para o abiente.

9. Considere os quatro txons abaixo relacionados.


1. Bufo dorginyi
2. Lystrophis dorbignyi
d)
3. Didelphys albiventris
4. Didelphys marsupialis
Em relao a eles, correto afirmar que:
e)
a) Todos pertencem mesma espcie.
b) H entre os quatro txons, apenas duas espcies diferentes.
c) Os txons um e dois so de gneros diferentes, mas da mesma espcie.
d) Os txons trs e quatro so de espcies diferentes, mas do mesmo 14. A coevoluo pode ser definida como a evoluo simultnea de duas ou
gnero.
mais espcies que tm um relacionamento ecolgico prximo. Atravs de
e) Os txons um e dois so da mesma subespcie.
presses seletivas, a evoluo de uma espcie torna-se parcialmente
10. Um bilogo, ao caminhar na beira da praia, coletou trs espcimes de dependente da evoluo da outra.
animais marinhos. Ao fazer seu relatrio de campo, descreveu-os da seguinte Com base nessa definio, qual das seguintes interaes ecolgicas NO
forma.
contribuiria potencialmente para a ocorrncia da coevoluo.
Espcime 1 Eumetazorio, diploblstico, com simetria primria radial.
Espcime 2 Eumetazorio, triploblstico, protostmio, celomado, com
simetria primria bilateral.
Espcime 3 Eumetazorio, triploblstico, deuterostmio celomado, com
simetria primria bilateral.
Quais dos animais poderiam corresponder descrio dos espcimes 1, 2 e
3, respectivamente?
a)
b)
c)
d)
e)

A esponja, a lagosta e o ourio-do-mar.


A anmona, o marisco e a lampreia.
O anfioxo, a esponja e a estrela-do-mar.
A estrela-do-mar, o caranguejo e o anfioxo.
O ourio-do-mar, a gua viva e a esponja.

11. Leia as afirmaes abaixo:


1- Existem aranhas em que a fmea muito maior que o macho.
2- Qualquer descuido na hora do sexo e a fmea devora o marido.
3- por isso que nessas aranhas a maioria dos machos vira padre.
I.

A primeira afirmativa refere-se ao dimorfismo sexual, observado nessa


classe de artrpodes.
II. Os aracndeos apresentam fecundao externa, so vivparos e seu
desenvolvimento direto.
III. A seqncia das afirmaes aborda a seleo sexual, a qual se
relaciona com a escolha de um parceiro para a reproduo.
Quais esto corretas?
a)
b)
c)
d)
e)

Apenas I
Apenas II
Apenas I e III
II e III
I, II e III

a)
b)
c)
d)
e)

O comensalismo.
A competio.
A predao.
O parasitismo.
O mutualismo.

15. Uma cachorra que se alimenta de carne bovina est parasitada por
vermes intestinais e pulgas. Esta cachorra amamenta seus filhotes recmnascidos, os quais esto parasitados externamente por carrapatos.
Considerando apenas as relaes trficas descritas, podemos afirmar que:
a)
b)
c)
d)
e)

nenhum dos organismos citados ocupa o mesmo nvel trfico.


os vermes intestinais e os carrapatos ocupam o mesmo nvel trfico.
as pulgas e os carrapatos ocupam o mesmo nvel trfico.
a cachorra e os filhotes ocupam o mesmo nvel trfico.
os filhotes, os vermes intestinais e as pulgas ocupam o mesmo nvel
trfico.

16. Em um ecossistema de savana, em equilbrio, coexistem dois grandes


predadores vertebrados, A e B, com massas corporais equivalentes. Ambos
alimentam-se de grandes herbvoros, como antlopes, zebras e outros.
Ao longo de um ano, foi verificado que, enquanto o predador A consumiu
uma massa de presas 5 vezes superior sua prpria massa corporal, o
predador B
Consumiu uma massa de presas 50 vezes superior sua prpria massa
corporal.
Com base nas informaes acima, considere as seguintes afirmaes.
I. O predador A gasta a maior parte da energia que consome para a
construo de sua biomassa, enquanto o predador B gasta a maior
parte da energia que consome para manter seu metabolismo e
produzir calor.
II. No terceiro nvel trfico existe maior perda de energia na forma de
calor na cadeia em que se encontra o predador A.
III. O predador A um animal ectotrmico, podendo ser um crocodilo, e o
predador B um animal endotrmico, podendo ser um leo.

12. Os mamferos compreendem um grupo muito heterogneo atualmente,


porm h evidncias de uma maior diversificao no passado, durante o
perodo Tercirio. Podem ser divididos em trs grupos atualmente. Sobre os So
Mammalia, incorreto afirmar.
a)
b)
a) Prototheria (os monotremados), os Metatheria (os marsupiais) e os c)
Eutheria (os mamferos com placenta desenvolvida).
d)

corretas:
Apenas I.
Apenas II.
Apenas I e III.
Apenas II e III.

158

e)

I,II e III.

17. Com base nas diferentes adaptaes que os animais endotrmicos


apresentam-nos diferentes biomas em que vivem, foram estabelecidas as
seguintes leis.
1. Lei das propores, ou Lei de Allen: animais de regies quentes
apresentam extremidades e apndices ____________ do que os de
regies frias.
2. Lei da Colorao, ou Lei de Gloger: animais de regies quentes e
midas apresentam colorao mais__________ do que os de regies
frias e secas.
3. Lei do Tamanho, ou Lei de Bergman: animais de regies frias
so___________ do que os de regies quentes.
Assinale a alternativa que preenche corretamente as lacunas dos textos
dessas leis, na ordem em que ocorrem.
a) Menores escura maiores.
b) Maiores clara- menores.
c) Menores escura menores.
d) Menores clara menores.
e) Maiores escura maiores.
f)
18. O mexilho dourado originrio da China e do sudeste da sia. No
Brasil, seu primeiro registro ocorreu em 1998, em frente ao porto de porto
Alegre, provavelmente introduzido por meio de gua de lastro de navios.
Esse molusco capaz de fixar-se em qualquer tipo de substrato submerso,
causando importantes danos ambientais e econmicos. Em 2004, o Ministrio
do Meio Ambiente iniciou campanha nacional, com objetivo de elaborar um
plano de controle de sua expanso.
Considere as afirmaes abaixo, relacionadas introduo dessa espcie
extica.
I. Na ausncia de resistncia ambiental, observa-se um crescimento
exponencial da populao de mexilhes.
II. A curva de crescimento da populao de mexilhes expressa-se na
forma de um S (curva sigmide).
III. Uma das conseqncias da introduo do mexilho ser o aumento da
biodiversidade nos ecossistemas brasileiros.
Quais esto corretas?
a) Apenas I.
b) Apenas II.
c) Apenas I e III.
d) Apenas II e III.
e) I, II e III.
19. Leia as definies abaixo, referentes a tipos de interaes ecolgicas que
ocorrem entre organismos em uma comunidade.
I. Sociedade: associao anatmica entre indivduos da mesma espcie
que passam a formar uma unidade estrutural e funcional.
II. Colnia: interao entre indivduos de uma mesma espcie em que h
diviso de trabalho.
III. Protocooperao: associao facultativa entre indivduos de espcies
diferentes, em que ambos se beneficiam.
Quais esto corretas?
a) Apenas I.
b) Apenas III.
c) Apenas I e II.
d) Apenas II e III.
e) I, II e III.
20. Assinale a afirmao correta sobre os ecossistemas de gua doce.
a)
b)
c)
d)
e)

Lagos e lagoas constituem ambientes lticos, pois no apresentam


correntezas.
Nos lagos e lagoas, no so encontradas algas verdes, cianobactrias e
diatomceas.
Os rios so dependentes de matria orgnica proveniente de outros
ambientes.
Nos rios, a quantidade de sedimento diminui medida que se aproxima
a foz.
Lagos e lagoas eutrficas caracterizam-se por apresentar grande
quantidade de oxignio dissolvido.

21. Quanto caracterizao de grupos vegetais e suas interaes, e correto


afirmar que:
a)
b)
c)
d)
e)

Plantas parasitas se fixam nas plantas hospedeiras atravs de hidatdios.


Liquens so exemplo de mutualismo entre algas e fungos.
Epfitas so plantas parasitas, porque retiram nutrientes da planta
hospedeira.
Orqudeas so angiospermas aclorofiladas saprfitas.
Musgos so vegetais vasculares clorofilados considerados hemiparasitas.

22. Recentemente, o Brasil foi colonizado pela gara boiadeira ( Bubulcus


ibis), conhecida por se alimentar em associao com o gado bovino. A sua
dieta compe-se predominantemente de presas terrestres, como insetos,
aranhas e pequenos vertebrados. O gado, ao se movimentar no pasto, causa
o deslocamento de pequenos animais presentes na vegetao, que so
capturados pelas garas. As vezes, estas se aproximam do gado para
capturar ectoparasitas.
Assinale com V (verdadeiro) ou F (falso) as afirmaes que seguem
referentes s interaes entre os animais citados no texto.
( ) A gara-boiadeira e o gado bovino fazem parte do mesmo nvel trfico.
( ) A gara-boiadeira consumidora secundria quando se alimenta de
ectoparasitas.
( ) As presas terrestres da gara-boiadeira deslocadas pelo movimento do
gado no pasto ocupam um mesmo hbitat.
( ) Ao se alimentar dos ectoparasitas, a gara-boiadeira estabelece com o
gado bovino uma relao de protocooperao.
A seqncia correta de preenchimento dos parnteses, de cima para baixo
a) V-F-V-V
b) F-V-V-V
c) V-V-F-F
d) V-F-F-F
e) F-F-V-V
23. Considere as seguintes afirmaes, a respeito de biocombustveis.
I. O biodiesel um tipo de biocombustvel que pode substituir, com
vantagens ambientais, vrios derivados do petrleo.
II. O babau, palmeira com razes respiratrias, tpica do cerrado, uma
das plantas indicadas para a produo de biodisel.
III. A implantao de uma monocultura, como a cana-de-acar, em reas
de mata atlntica implica queimadas, que contribuem para o aumento
da temperatura e da eroso do solo.
Quais esto corretas?
a) Apenas I.
b) Apenas I e II.
c) Apenas I e III.
d) Apenas II e III.
e) I, II e III.
24. Em um experimento em que foram injetados aminocidos radiativos em
um animal, a observao de uma de suas clulas mostrou os seguintes
resultados: aps 3 minutos, a radioatividade estava localizada na organela X
(demonstrando que a sntese de protenas ocorria naquele local); aps 20
minutos, a radioatividade passou a ser observada na organela Y; 90 minutos
depois, verificou-ser a presena de grnulos de secrees
radioativos, uma evidncia de que as protenas estavam prxima de serem
exportadas.
As organelas X e Y referidas no texto so, respectivamente,
a)
b)
c)
d)
e)

O complexo golgiense e o lisossomo.


O retculo endoplasmtico liso e o retculo endoplasmtico rugoso.
A mitocndria e o ribossomo.
O retculo endoplasmtico rugoso e o complexo golgiense.
O centrolo e o retculo endoplasmtico liso.

25. Assinale a alternativa que preenche corretamente as lacunas do


enunciado abaixo, na ordem em que aparecem.
Em pesquisas que investigam a ancestralidade de uma determinada
populao, seqncias de DNA dos indivduos so comparadas com trechos
de DNA tpicos de diferentes populaes.
Nesse tipo de pesquisa, a anlise.............. permite averiguar a
ancestralidade ................. .
a) Do cromossomo Y masculina de homens e mulheres.
b) Do cromossomo X masculina de homens.
c) Das mitocndrias masculina de homens.
d) Do cromossomo Y feminina de homens.
e) Das mitocndrias - feminina de homens e mulheres.
26. As protenas so sintetizadas pelos ribossomos livres no citoplasma ou no
retculo endoplasmtico rugoso. O ribossomo fornece o local apropriado para
que os aminocidos sejam ligados e assim formem a protena.
Analise as afirmativas.
I. O retculo endoplasmtico rugoso promove a sntese dos ribossomos e
produz RNA.
II. O ribossomo se desloca ao longo da molcula de RNAr e assim vai
traduzindo a informao em protenas.

159

III.

Para cada cdon do DNA mensageiro adicionado um aminocido,


portanto o nmero de cdons correspondente ao nmero de 31. Uma vez no citoplasma a glicose participar do processo de respirao
aminocidos existentes.
celular, resultando, no final, gs carbnico, gua, liberao de energia sob
IV. O retculo endoplasmtico rugoso abriga os ribossomos que sero forma de ATP. Essa transformao ocorre primeiramente no citoplasma e
levados at o RNAm pelo RNAt.
posteriormente no interior de uma organela e a seqncia completa dos
V. As protenas correspondem s informaes que existem no DNA, pois acontecimentos, incluindo o que ocorre no citoplasma, corresponde a:
todos os nucleotdeos do DNA so utilizados no processo de traduo. a) ribossomo, ciclo de Krebs, cadeia respiratria e gliclise.
b) complexo de Golgi, cadeia respiratria, ciclo de Krebs e gliclise.
Quantas afirmativas esto INCORRETAS?
c) mitocndria, ciclo de Krebs, gliclise e cadeia respiratria.
a) Uma.
d) lisossomos, gliclise, ciclo de Krebs e cadeia respiratria.
b) Duas.
e) mitocndria, gliclise, ciclo de Krebs e cadeia respiratria.
c) Trs.
d) Quatro.
e) Cinco.
32. As figuras anteriores so representaes simblicas de nucleotdeos. A
partir de conhecimento da estrutura dos cidos nuclicos visto na aula de
27. Analise as afirmativas abaixo onde so apresentadas estruturas celulares. Biologia do Professor Maurivam, possvel afirmar que inexistem os
I. O retculo endoplasmtico granuloso uma rede de tubos e bolsas nucleotdeos.
achatadas com mitocndrias aderidas sua superfcie, estas atuam R = Ribose
produzindo protenas.
D = Desoxirribose
II. Os clios so estruturas filamentosas mveis compostas de P = cido fosfrico
microtbulos que so constitudos de molculas de protenas
(tubulina).
III. Os cloroplastos so estruturas ovaladas (alongadas), delimitados por
membranas lipoproticas, sendo internamente preenchidos por
pequenas bolsas discoidais (achatadas) empilhadas.
Com base nas afirmaes acima, podemos dizer que apenas,
a) I e II esto corretas.
b) II e III esto corretas.
c) I e III esto corretas.
d) II est correta.
e) I est correta.
28. Considere as seguintes etapas da sntese de protenas.

a) I e II.
b) IV e V.
c) I e III.
I. Transcrio do cdigo gentico do DNA em cdons do RNA
d) III e V.
mensageiro.
e) II e IV.
II. Ligao dos cdons de RNA mensageiro aos anticdons dos RNAs
transportadores.
33. As funes de sntese protica, sntese de lipdios, digesto intracelular,
III. Fixao do RNA mensageiro aos ribossomos.
respirao celular e formao do fuso acromtico so realizadas,
respectivamente, pelas seguintes estruturas:
Qual a seqncia correta dessas etapas durante o processo?
a) I II III
a) ribossomo, retculo endoplasmtico liso, mitocndrias, lisossomos e
b) I III II
centrolos
c) II III I
b) ribossomo, retculo endoplasmtico liso, lisossomos, mitocndrias e
d) III II I
centrolos
e) III I II
c) ribossomo, lisossomos, retculo endoplasmtico liso, mitocndrias e
centrolos
29. As hemcias humanas foram selecionadas ao longo da evoluo de modo
d) retculo endoplasmtico liso, mitocndrias, lisossomos, centrolos e
a que desempenhassem hoje em dia suas funes de maneira eficiente.
ribossomos
Durante este processo evolutivo, as mitocndrias e os ncleos foram perdidos
e) retculo endoplasmtico liso, ribossomos, lisossomos, centrolos e
na fase madura.
mitocndrias
Quais dos processos biolgicos abaixo continuam a ocorrer nas hemcias
maduras apesar desta adaptao?
34. Na espcie humana a determinao sexual feita por um par de
a) Cadeia transportadora de eltrons.
cromossomos, X e Y. O indivduo do sexo feminino apresenta dois
b) Ciclo de Krebs.
cromossomos X enquanto que o indivduo de sexo masculino apresenta um
c) Gliclise.
cromossomo X e outro cromossomo Y. Com base neste texto INCORRETO
d) Replicao.
afirmar:
e) Transcrio.
a)
30. A energia indispensvel para qualquer ser vivo independente do
metabolismo prprio. claro que em nveis de exigncia maior do organismo,
b)
devemos sintetizar maior quantidade de energia, sendo, portanto necessrio
que utilizemos em condies aerbicas os seguintes processos:
c)
d)
I. Gliclise
II. Ciclo de Krebs

e)

O sexo masculino heterogamtico e sempre o homem quem


determina o sexo dos filhos.
O sexo feminino homogamtico e o gameta feminino normal sempre
apresenta um cromossomo X.
Na mulher um cromossomo X est sempre condensado e recebe o nome
de cromatina sexual.
A hipertricose auricular uma caracterstica restrita ao sexo masculino,
pois o gene est localizado no cromossomo Y.
Toda doena hereditria e recessiva ligada ao cromossomo X s afeta as
mulheres.

III.Respirao celular
IV.Cadeia respiratria

a)
b)
c)
d)
e)

I, II e III, sendo que estes em conjunto formam o nmero IV.


I, II e IV, sendo que estes em conjunto formam o nmero III.
I e II, sendo que o IV formado pela unio de I e III.
I e III, sendo que estes formam o IV.
nenhuma das alternativas anteriores.

160

35. O esquema a seguir representa uma clula animal sobre a qual so feitas b)
as
afirmaes
a
seguir:

ocorre, tanto na anfase mittica quanto na anfase I meitica, a


separao das cromtides irms, as quais so puxadas para os plos
opostos da clula.
c) so formadas duas clulas diplides no final da mitose masculina,
enquanto que, no final da meiose, so formadas quatro clulas diplides,
considerando a espcie humana.
d) todas as clulas animais e vegetais iniciam a citocinese (diviso do
citoplasma) da mesma forma, pela invaginao da membrana plasmtica
(movimento centrpeto).
e) ocorre, na fase da telfase mittica, a reorganizao do envoltrio
nuclear e do nuclolo, que foram desorganizados no incio do processo
de diviso.
39. Na novela das 20h, um dos temas tratados o nascimento de uma
criana com sndrome de Down. Trata-se de uma menina gmea de um
menino que no apresenta a sndrome.
Com base nessas informaes, considere as afirmaes abaixo.
I- Esses gmeos so dizigticos
II- A sndrome de Down uma aneuploidia sexual.
III- A sndrome dessa menina pode ser decorrente de um erro durante a
meiose I ou II.

I.

A estrutura 2 representa a parede celular que funciona como uma


barreira seletiva entre o citoplasma e o meio ambiente.
II. A estrutura 3 corresponde ao retculo endoplasmtico rugoso onde
ocorre a sntese protica.
III. A estrutura 4 desempenha funes relacionadas ao processo de
digesto intracelular.
IV. O esquema representa uma clula ingerindo pores lquidas e
formando vesculas pinocitticas.
V. A estrutura 5 est presente apenas nas clulas animais.
VI. Os plastos responsveis pela realizao da fotossntese no esto
presentes nessa clula.
Assinale a alternativa que rene as afirmaes corretas
a) III V e VI.
b) I, II, e IV.
c) II,IV e VI.
d) II, IV, V e VI.
e) I, II, III e V.

Quais so corretas.
a) Apenas I.
b) Apenas II.
c) Apenas III.
d) Apenas I e III.
e) Apenas II e III.
40. O paradigma da biologia molecular descrito no esquema abaixo:

36. Em organismos humanos normais, o nmero de autossomos e o nmero


de cromossomos sexuais que se pode encontrar em um vulo, em uma clula
da pele e no zigoto so, respectivamente,

Os mecanismos responsveis pela manuteno e expresso da informao


gentica de I para III so os seguintes:
a) replicao, duplicao e transcrio.
b) replicao, transcrio e traduo.
c) replicao, transduo e traduo.
a) zero e 2; 46 e zero; 44 e 2.
d) traduo, transcrio e replicao.
b) 22 e 2; 44 e zero; 46 e zero.
e) traduo, replicao e transduo.
c) 22 e 1; 44 e 2; 44 e 2.
41. Analisando-se a composio dos nucleotdeos que compem o cido
d) zero e 1; 44 e 2; 46 e zero.
nuclico de dois diferentes organismos (X e Y), verificaram-se as seguintes
e) 44 e 2; 46 e zero; 22 e 1.
propores: organismo X 21% adenina, 29% citosina, 21% timina e 29%
guanina; organismo Y 28% adenina, 36% uracila, 20% guanina e 16%
37. O grfico a seguir ilustra a variao da quantidade de DNA de uma clula citosina.
durante sua vida:
Com base nestes resultados, escolha a alternativa correta.
a) O organismo X possui RNA de fita simples, e o organismo Y tambm
possui RNA de fita simples.
b) O organismo X possui DNA de fita dupla, e o organismo Y possui RNA de
fita simples.
c) O organismo X possui DNA de fita dupla, e o organismo Y tambm
possui DNA, porm de fita simples.
d) O organismo X possui RNA de fita dupla, e o organismo Y possui DNA de
fita simples.
e) O organismo X possui DNA de fita simples, e o organismo Y possui RNA
de fita simples.
A fase responsvel pela duplicao do DNA :
42. No reino animal h dois tipos de diviso celular: mitose, que o processo
a) A
pelo qual clulas somticas se dividem, e meiose, que o processo de diviso
b) B
que produz gametas haplides.
c) C
Sobre a meiose correto afirmar que.
d) D
e) E
I. Esse processo pode ocorrer a partir de qualquer clula somtica, exceto
dos neurnios e clulas musculares.
38. O ciclo celular envolve a interfase e as divises celulares, que podem ser
II. Cada uma das quatro clulas resultantes da meiose contm apenas um
mitose ou meiose.
representante de cada par de cromossomos.
Durante as fases de divises celulares, ocorrem vrios eventos importantes
III. Durante a prfase I ocorre pareamento entre cromossomos homlogos,
que envolvem o material gentico, as organelas e as estruturas celulares.
O ciclo celular mittico garante a reposio celular, o crescimento dos podendo haver troca de material gentico entre eles.
organismos multicelulares e a reproduo assexuada, enquanto o ciclo celular IV. Ocorrem duas divises por ciclo, e o material gentico (DNA) replicado
meitico um processo importante para a reproduo sexuada.
antes de cada uma dessas divises.
Com base no texto e em seus conhecimentos, correto afirmar que:
Quais afirmativas esto corretas?
a) a interfase um perodo do ciclo celular que apenas antecede a mitose a) Apenas I, II e III.
ou a meiose; nela no ocorrem eventos importantes para a gerao de b) Apenas II e III.
novas clulas.
c) Apenas II e IV.

161

d)
e)

Apenas II, III e IV.


Apenas I, III e IV.

43. Quanto aos processos de diviso celular em humanos, correto afirmar


que:
a) As clulas somticas sofrem meiose, que um processo pelo qual uma
clula me (com 46 cromossomos) origina duas clulas filhas (com 23
cromossomos).
b) As clulas da epiderme sofrem mitose, que um processo pelo qual uma
clula me (com 23 cromossomos) origina duas clulas filhas (com 46
cromossomos).
c) As clulas reprodutivas sofrem meiose, que um processo pelo qual
uma clula me (com 46 cromossomos) origina quatro clulas filhas
(com 23 cromossomos).
d) Os gametas so produzidos atravs de mitose, que um processo pelo
qual uma clula me (com 46 cromossomos) origina quatro clulas filhas
(com 23 cromossomos).
e) As clulas somticas sofrem mitose, que um processo pelo qual uma
clula me (com 23 cromossomos) origina duas clulas filhas (com 46
cromossomos).
44. O Brasil o maior produtor mundial de caf. Alguns pesquisadores
admitem que a espcie Coffea arbica tenha se originado na Etipia, frica, a
partir da hibridizao natural entre duas espcies ancestrais: Coffea
eugenioides ( 2n = 22 cromossomo ) e Coffea canephora (2n = 22
cromossomos). No hbrido resultante, teria havido uma duplicao do nmero
de cromossomos (poliploidizao): em uma das divises celulares, as
cromtides de todos os cromossomos, ao invs de migrarem cada uma delas
para ambas as clulas-filhas, teriam permanecido na mesma clula. Deste
modo, originou-se um novo ramo, ou uma nova planta, tetraplide. Esta nova
planta a Coffea arbica, que hoje cultivamos e que tem importante papel
em nossa economia.
A partir destas informaes, possvel dizer que o nmero de cromossomos
nos gametas das espcies Coffea eugenioides e Coffea canephora e o
nmero de cromossomos nas clulas somticas e nos gametas de Coffea
arbica so, respectivamente:
a) 11, 11, 44 e 22 b)11, 11, 22 e 11
c) 22, 22, 88 e 44
d)22, 22, 44 e 22
e) 22, 22, 22 e 11
45. O fenmeno da osmose est diretamente implicado na manuteno da
vida e se manifesta de diversas formas. Analise a veracidade (V) ou falsidade
(F) das proposies abaixo, sobre as manifestaes do fenmeno da osmose.
( ) Os vegetais adaptados a solos prximos do mar mantm no citoplasma
das clulas das razes grande quantidades de sais para torn-lo hipertnico,
evitando, com isso, que sejam dessecadas pela elevada concentrao de sais
presentes nesses solos.
( ) A turgncia das plantas mantida pela diferena de concentrao de sais,
que maior no interior das clulas, fazendo com que maiores quantidades de
gua passem pela membrana plasmtica, semipermevel, e fiquem retidas no
seu interior.
( ) A maior concentrao de substncias no interior dos protozorios de gua
doce, como o paramcio. Faz com que sua clula, por pinocitose, retenha
gua em excesso, podendo romper a membrana plasmtica. Para evitar que
isso ocorra. Esses protozorios apresentam estruturas celulares conhecidas
como vacolos contrteis. Responsveis pela retirada do excesso de gua da
clula, por osmose.
Assinale a alternativa que preenche corretamente os parnteses, de cima
para baixo.
a) V F V
b) F V V
c) V V V
d) V V F
e) V F F
46. Primeira dcada do sculo passado, o mdico brasileiro Carlos Chagas
iniciou uma srie de estudos que o levaram a descrever o ciclo de vida de um
importante __I__ pertencente
_ II___ Trypanosoma cruzi, agente
etiolgico do mal de Chagas e que tem como transmissor um __III___
pertencente ao __IV__ Triatoma, popularmente conhecido por barbeiro.
No trecho acima, as lacunas I, II, III e IV devem ser substitudas correta e,
respectivamente, por:
a) Protozorio, famlia, inseto e filo.
b) Protozorio, espcie, inseto e gnero.
c) Bacilo, espcie, verme e gnero.
d) Bacilo, famlia, verme e filo.
e) Vrus, ordem, molusco e gnero.
47. Em relao Entamoeba histolytica, correto afirmar que:
a) o agente etiolgico da lcera de Bauru.
b) O homem pode infectar-se por ingesto de gua
c) O trofozoto uma forma de resistncia do parasita.
d) Pode apresentar reproduo sexuada no vetor.
e) Pode ser transmitida ao homem por meio de cistos.

48. Analise as afirmativas abaixo e assinale a alternativa correta.


I A malria (maleita ou impaludismo) uma doena infecciosa do homem e
de outros animais causada por protozorio do gnero Plasmodium.
II O ciclo vital do plasmdio se desenvolve no sangue humano (ciclo
sexuado) e no tubo digestivo do pernilongo Aedes (ciclo assexuado).
III A leishmaniose tegumentar americana causada pelo protozorio
flagelado Leishmania brasiliensis, sendo transmitido pela picada e fezes do
Phlebotomus, um lepidptero noturno.
IV A tricomonase, infeco que na mulher se caracteriza pela eliminao
de um fluxo vaginal purulento (leucorria) e que no homem gera uretrite,
pode ser transmitida pelo uso de roupas mal lavadas e pelo uso comum de
toalhas, piscinas e sanitrios.
a) Todas esto corretas.
b) Apenas I est correta.
c) Apenas I e IV esto corretas.
d) Apenas IV est correta.
e) Apenas II e III esto corretas.
49. Que relao abaixo entre doena, o agente etiolgico e o vetor
incorreta?
a) Doena de Chagas Trypanosoma cruzi Barbeiro (Triatoma sp).
b) Doena do sono Trypanosoma gambiense mosca ts-ts (Glossina
sp).
c) Disenteria Entamoeba histolytica molusco (Biomplalaria sp).
d) Malria Plasmodium sp mosquito (Anopheles sp).
e) lcera de Bauru Leishmania brasiliensis mosquito (Phlebotomus sp).
50. Uma evidente diferena existente entre a anfase da mitose e as
anfases I e II da meiose que os cromossomos em migrao para os plos
celulares so:
a) irmos nas anfases I e II e homlogos na anfase da mitose.
b) homlogos nas anfases I e II e irmos na anfase da mitose.
c) homlogos na anfase I e irmos na anfase II e na anfase da mitose.
d) irmos na anfase I e anfase da mitose e homlogos na anfase II.
e) irmos nas anfases I e II e anfase da mitose.
51. Considerando uma clula com 6 cromossomos (2n=6) que esteja em
diviso, o esquema ao lado representaria uma:
a)
b)
c)
d)
e)

anfase I da meiose.
metfase I da meiose.
metfase II da meiose.
anfase II da meiose.
anfase mittica.

52. Os itens abaixo se referem mitose e todos eles esto corretos, exceto:
a)
b)
c)
d)
e)

um processo de diviso celular importante para o crescimento dos


organismos.
Ocorre nas clulas somticas de animais e vegetais.
Uma clula-me origina duas clulas-filhas com o mesmo nmero de
cromossomos.
A duplicao do DNA ocorre na fase da metfase.
Na fase da telfase, forma-se uma nova membrana nuclear em torno
dos cromossomos e o citoplasma se divide.

53. Relacione as fases meiticas (coluna I) com os respectivos fenmenos


(coluna II):
Coluna 1

Coluna 2

1) zigteno

( ) Migrao
dos cromossomos homlogos para os
plos.
2) paquteno
( ) pareamento dos homlogos.
3) diplteno
( ) migrao dos cromossomos irmos para os plos.
4) anfase I
( ) visualizao dos quiasmas.
5) anfase II
( ) ocorrncia do crossing-over.
A seqncia correta, de cima para baixo, na coluna II
a) 4, 1, 2, 3, 5
b) 4, 1, 5, 2, 3
c) 4, 1, 5, 3, 2
d) 4, 1, 3, 2, 5
e) 4, 2, 5, 1, 3
54. Das afirmativas abaixo:
I- O crossing-over permite a recombinao dos genes localizados em
cromossomos homlogos.

162

II- Meiose um tipo de diviso celular na qual uma clula diplide d origem
a quatro clulas haplides.
III- A interfase um perodo de grande atividade metablica no ncleo.
nessa fase que o DNA se duplica e o RNA sintetizado.
a) apenas a afirmativa I correta.
b) apenas a afirmativa II correta.
c) apenas a afirmativa III correta.
d) apenas duas afirmativas so corretas
e) todas as afirmativas so corretas.

60. As manifestaes relacionadas na coluna 1 so atribudas carncia de


vitaminas na dieta do homem. Assinale na coluna 2, de acordo, com a
seqncia em que so citadas as manifestaes de carncias:

55. Relacione as fases da mitose: anfase, telfase, metfase e prfase, com


os respectivos nmeros das figuras abaixo:

Coluna 1

Coluna 2

1) beribri

( ) vitamina A

2) cegueira noturna

( ) vitamina B1

3) escorbuto

( ) vitamina B2

4) anemia

( )vitamina B12

5) dificuldade de coagulao

( ) vitamina C
( ) vitamina K

a)
b)
c)
d)
e)

1,
1,
2,
2,
2,

2,
2,
1,
1,
1,

4,
5,
5,
3,
4,

5,6
4,6
4,6
4,5
3,5

61.
a) 4-3-2-1
b) 3-4-2-1
c) 1-2-3-4
d) 2-3-4-1
e) 3-1-2-4
56. Na mitose, a duplicao das cromtides e a ligao dos cromossomos ao
fuso ocorrem, respectivamente, na:
a)
b)
c)
d)
e)

metfase e interfase
prfase e anfase
prfase e metfase
interfase e prfase
interfase e metfase

57. A tabela mostra a composio das bases nitrogenadas pricas, adenina e


guanina, nos DNAs do homem e do boi.
Adenina
Guanina
Homem
30,4%
?
Boi
?
21,0%
As porcentagens que esto faltando para o homem e o boi
respectivamente:
a)
b)
c)
d)
e)

19,6
21,0
29,0
19,6
30,4

e
e
e
e
e

29,0.
30,4.
30,4.
21,0.
21,0

58. Um dos filamentos da dupla hlice de uma molcula de DNA tem a


seguinte seqncia: AACGATCGCTCA. O s seus produtos de duplicao e
transcrio so, respectivamente:
a) AACGATCGCTCA; TTGCTAGCGAGT.
b) TTGCTAGCGAGT; UUGCUAGCGAGU.
c) UUCGUACGCACU; TTGCTAGCGAGT.
d) TTGCTAGCGAGT; AACGATCGCTCA.
e) TTCGTACGCACT; AACGATCGCTCA.
59. Ateno na cozinha: no aconselhvel temperar com sal e vinagre, uma
salada de verduras, ou um pedao de carne, muito tempo antes de consumir.
Provavelmente as folhas da verdura ficaro murchas, e a carne vai comear a
liberar muito lquido.
Baseado no texto e em seus conhecimentos correto afirmar que em ambos
os casos ocorrer:
a) difuso do solvente do meio hipertnico para o hipotnico, por isso a
carne e as verduras perdero gua
b) a lise celular e por isso as clulas liberaro gua, pois foram submetidas
a um meio hipotnico
c) a deplasmlise, processo em que h perda de gua para o ambiente e
conseqentemente a diminuio do volume celular
d) um processo de osmose, em que as clulas perdero gua por serem
submetidas a um meio hipertnico
e) um processo de transporte ativo, em que as clulas secretaro gua
para ocorrer a entrada de sal nas prprias clulas

Hemcias humanas foram colocadas em um meio com concentraes


diferentes. Pelo formato das clulas I, II e III, sabe-se que os meios se
classificam, respectivamente, como
a) isotnico - hipotnico - hipertnico.
b) hipertnico - isotnico - hipotnico.
c) hipotnico - hipertnico - isotnico.
d) hipotnico - isotnico - hipertnico.
e) isotnico - hipertnico - hipotnico.
62. Fazendo-se a cariotipagem em trs pacientes portadores de distrbios
citogenticos, verificaram-se os seguintes caritipos: 45 X0; 47 XXY e 47 XY
+ 21.
Podemos afirmar
respectivamente:
a)
b)
c)
d)
e)

sndrome
sndrome
sndrome
sndrome
sndrome

de
de
de
de
do

que

os

portadores

desses

caritipos

apresentam,

Klinefelter, sndrome de Patau e sndrome de Down.


Down, sndrome de Klinefelter e sndrome de Turner.
Patau, sndrome de Edwards e sndrome de Klinefelter.
Turner, sndrome de Klinefelter e sndrome de Down.
duplo Y, sndrome de Down e sndrome de Klinefelter.

63. Mamferos e pssaros que mergulham esto sujeitos a perodos de


hipoxia durante a submerso. Em relao a esse fato, analise as afirmaes
abaixo.
I.
II.

Esses animais utilizam as reservas de oxignio dos tecidos.


Esses animais possuem elevadas concentraes de hemoglobina e
mioglobina.
III. Durante o mergulho, ocorre uma diminuio da freqncia cardaca
desses animais.
Quais esto corretas?
a) Apenas I.
c) Apenas I e III

b)Apenas II.
d)Apenas II e III

e) I, II e III

64. A bile, produzida pelo fgado, tem com funo:


a)
b)
c)
d)
e)

Lubrificar a mucosa intestinal.


Emulsionar as gorduras.
Digerir as protenas.
Provocar contrao da vescula biliar.
Estimular a secreo gstrica.

65. Considere os seguintes fenmenos:


I.
II.
III.

Absoro de alimentos.
Incio da digesto das protenas.
Incio da digesto das gorduras.

163

IV. Adio de cido ao suco digestivo.


Ocorrem no estmago apenas:
a) I e II
c) II e III

b)I e IV
d)II e IV

e) III e IV
66. Nas opes abaixo esto relacionadas cavidades cardacas e vasos
sanguneos. Indique aquela que rene cavidades e vasos nos quais, no
homem adulto, o sangue encontrado ser sempre sangue arterial:
a)
b)
c)
d)
e)

Ventrculo esquerdo Aorta Artria Pulmonar.


trio Esquerdo Veia Pulmonar Aorta.
Ventrculo direito Artria Pulmonar Aorta.
trio direito Veia Cava Veia Pulmonar.
Ventrculo direito Veia Pulmonar Artria Pulmonar.

67. Os esquemas representam processos qumicos que ocorrem nas


hemcias de uma pessoa, envolvendo hemoglobina (Hb), gs oxignio e gs
carbnico.

a)
b)
c)
d)
e)

vulo meiose- haplide.


Ovcito I mitose haplides.
Ovcito II meiose haplides.
Ovcito I meiose diplides.
Corpsculos polares mitose diplides.

71. Nos seres humanos, o desenvolvimento embrionrio depende da correta


implantao do embrio no endomtrio. Quando isso ocorre, inicia-se a
secreo da gonadotrofina corinica humana (HCG), a qual pode causar
nuseas na gestante.
O conjunto de clulas responsveis pela implantao do embrio e a
conseqente produo de HCG denomina-se:
a)
b)
c)
d)
e)

Ectoderma.
Mesoderma
Endoderma.
Mrula.
Trofoplasto.

72. Com relao gametognese animal, qual das


alternativas apresenta apenas clulas diplides?
a)
b)
c)
d)
e)
Os locais onde ocorrem as situaes representadas em I e II so,
respectivamente.
a)
b)
c)
d)
e)

Intestino e msculos.
Pele e msculos.
Rim e intestino.
Rim e pulmo.
Pulmo e pele.

68.

Ovcito I, espermatcitos II.


Espermtides, ovognia.
Espermatognias, primeiro glbulo polar.
Espermatognias, ovcito I.
Espermatcitos II, ovcito II.

73. A enorme variedade de seres vivos tem fascinado a humanidade ao


longo da histria. H cerca de dois sculos, comearam a surgir explicaes
cientficas para a origem e a diversidade da vida.
Analise a veracidade (V) ou falsidade (F) das proposies abaixo,
considerando as evidncias do processo evolutivo dos seres vivos.
( ) Os fsseis constituem-se no principal registro temporal desse processo.
( ) Diferentes espcies podem apresentar estruturas corporais de mesma
origem, mas com funes diferentes.
(
) Durante o desenvolvimento embrionrio, os seres vivos de grupos
semelhantes compartilham caractersticas, o que sugere a existncia de um
ancestral comum.
Assinale a alternativa que preenche corretamente os parnteses, de cima
para baixo.
a)
b)
c)
d)
e)

F-F-V
V-F-F
V-V-V
F-V-F
V-F-V

74. O tubaro tem concentrao osmtica semelhante gua do mar. Em


relao a este fato, analise as afirmaes abaixo:
I. A concentrao osmtica do tubaro mantida pelos altos nveis de
uria.
A respeito do esquema acima, assinale a alternativa correta.
II. O tubaro pode ser considerado hipotnico em relao ao ambiente.
a) Cavalo e zebra pertencem a gneros diferentes.
III. A estrutura mais importante na manuteno da concentrao osmtica
b) As antas apresentam maior parentesco evolutivo com os cavalos do que
do tubaro a glndula retal.
com os rinocerontes.
c) O parentesco evolutivo entre cavalo e zebra maior do que entre Quais esto corretas?
a) Apenas I.
rinocerontes de um chifre e rinocerontes de dois chifres.
b) Apenas I e II.
d) As antas pertencem mesma famlia dos rinocerontes.
c) Apenas I e III.
e) Todos os animais citados pertencem mesma famlia.
d) Apenas II e III.
69. Em um trabalho de pesquisa, foram classificados dois mosquitos como e) I, II e III.
sendo:

'Aedes (Stegomyia) aegypti' e


'Anopheles (Myzomya) gambiae'.
O grau de semelhana entre esses mosquitos permite que sejam colocados
no (a) mesmo (a):
a) Espcie.
b) Subespcie.
c) Gnero.
d) Subgnero.
e) Famlia.
70. Assinale a alternativa que completa corretamente as lacunas do texto
abaixo.
A menina, ao nascer, j realizou as fases de multiplicao e crescimento do
processo denominado gametognese, possuindo um grande nmero de
.............. em processo interrompido de ............. . Essas ltimas clulas
so.........

75. Os animais homeotrmicos fazem trocas de calor com o ambiente. Assim,


a perda de calor ocorre do interior do corpo para a periferia e,
posteriormente, para o ambiente.
Quais dos mecanismos abaixo podem ser utilizados para a manuteno da
temperatura corporal?
I. A circulao para distribuio e disperso de calor.
II. O resfriamento atravs do suor.
III. A respirao para o resfriamento.
a) Apenas II.
b) Apenas I e II.
c) Apenas I e III.
d) Apenas II e III.
e) I, II e III.
76. Qual a alternativa que relaciona os mecanismos de trocas gasosas com
os respectivos animais?
Bagre

Tartaruga

Pingim

Abelha

164

e) V F V V F - F
81. O cultivo de plantas transgnicas, assunto polmico em vrios pases,
inclusive no Brasil, traz uma nova dimenso para os cruzamentos na
agricultura, por possibilitar modificaes genticas direcionadas, envolvendo
espcies totalmente diferentes. Entre os inmeros transgnicos at agora
produzidos, a RR um tipo de soja tolerante a um herbicida ou "mata-mato"
de espectro amplo. Um experimento gentico inseriu no genoma da soja um
77. Dos animais citados, sob o ponto de vista evolutivo, o mais prximo do gene que induz essa tolerncia especfica, facilitando o manejo dessa cultura.
homem :
Uma preocupao de alguns ambientalistas, no caso da soja RR, que esse
a) O tubaro
gene possa "escapar" ou ser transmitido para outras plantas, ervas daninha,
b) A r
por exemplo, as quais poderiam apresentar propriedades inesperadas,
c) O morcego
prejudicando o meio ambiente e os ecossistemas. Outra preocupao ainda
d) O jacar
no serem previsveis os impactos ambientais causados pela quebra de
e) O pingim
barreiras naturais, ao se introduzir plantas transgnicas nas lavouras. Em
relao a esses, fatores, analise a veracidade (V) ou falsidade (F) das
78. Os ecossistemas aquticos caracterizam-se por apresentar ambientes proposies abaixo.
com salinidade e intensidade de luz solar diferentes, fatores determinantes
da diversidade de seres vivos que constituem esses ecossistemas.
( ) Duas populaes de uma mesma espcie, isoladas geograficamente e
sofrendo isoladamente os efeitos dos fatores evolutivos, podero acumular
Em relao a esses fatores, analise a veracidade (V) ou falsidade (F) das tantas diferenas genticas a ponto de no ser mais possvel o cruzamento
proposies abaixo.
entre elas, determinando o surgimento de uma nova espcie.
( ) Em populaes naturais, a transmisso de genes entre indivduos de uma
( ) Os seres nectnicos, capazes de mover-se contracorrente, vivem mesma espcie s acontece atravs da reproduo sexuada.
principalmente na regio euftica.
( ) No processo de especiao, o isolamento reprodutivo precede o
( ) Os seres bentnicos habitam o fundo dos mares lagos, constituindo a isolamento geogrfico.
flora e a fauna de fundo do talassociclo e do epinociclo
( ) O sucesso da reproduo sexuada, principalmente no que se refere a
( ) Os seres planctnicos, que flutuam, vivem principalmente na plataforma descendentes frteis, depende, entre outros fatores, da espcie a que
continental.
pertencem os indivduos envolvidos.
(
)O processo de formao de novas espcies denomina-se
Assinale a alternativa que preenche corretamente os parnteses, de cima espermatognese.
para baixo.
a) V F - F
Assinale a alternativa que preenche corretamente os parnteses, de cima
b) F V - V
para baixo.
c) V V - V
d) F V - F
a) V F F V F.
e) V F - V
b) V F V V F.
c) V V F V F.
79. Os seres vivos representantes dos cinco reinos podem desempenhar d) F F F V F.
papel de produtores ou consumidores
(1)
e) F F F V V.
(2)
nas cadeias alimentares, bem como estabelecer com outras espcies relaes 82. Em audes de pequeno porte, na regio do polgono da seca no Nordeste
ecolgicas ntimas, como, por exemplo, mutualismo e parasitismo.
brasileiro, houve curto perodo de estiagem seguido de longo perodo
(3)
(4)
chuvoso e forte luminosidade. Espera-se que, nessas circunstncias.
Quais desses papis e quais dessas relaes ecolgicas podem ser atribudos
a) ocorra aumento de fitoplncton, deposio de matria orgnica, morte
aos fungos?
de algas, baixo teor de oxignio e mortandade de animais aquticos.
a) Apenas 1 e 3
b) haja proliferao de animais que habitam as regies mais profundas,
b) Apenas 1 e 4
pois eles consumiro mais alimentos.
c) Apenas 2 e 3
c) ocorra aumento da taxa de oxignio para que os microrganismos
d) Apenas 2 e 4
anaerbicos possam atuar consumindo as algas.
e) Apenas 2, 3 e 4
d) ocorra diminuio da quantidade de sais minerais disponveis, o que
permite a proliferao do zooplncton.
80. Voc est montando um aqurio em casa e gostaria que ele
representasse um lago ou um rio de pouca correnteza. Agora deve decidir e) haja acmulo de dejetos animais, que acabam se depositando e
deixando as guas lmpidas para atuao dos decompositores.
sobre os organismos que poder colocar em seu aqurio, em funo de suas
a)
b)
c)
d)
e)

Tegumentar
Tegumentar
Branquial
Filotraqueal
branquial

- branquial traqueal traqueal


-filotraqueal - pulmonar filotraqueal
- pulmonar - pulmonar - traqueal
-branquial - pulmonar - filopulmonar
- pulmonar - branquial traqueal

adaptaes a esse tipo de meio ambiente.


( ) Existem moluscos que vivem em gua doce. Entre eles, os gastrpodes
so animais interessantes para se ter em aqurio, pois limpam as paredes
recobertas de algas devido ao modo de alimentao raspador, para o qual
utilizam a rdula.
( ) Peixes so animais adequados para o aqurio porque possuem brnquias
para respirao e a linha lateral, que lhes permite perceber estmulos
qumicos trazidos pela gua.
( ) Crustceos, de maneira geral, respiram por intermdio de brnquias e,
portanto, podem viver em ambiente dulccola. Os olhos compostos desses
animais tambm esto adaptados ao ambiente aqutico.
( ) Equinodermos no devem ser colocados nesse aqurio porque no
apresentam um rgo responsvel pela osmorregulao, alm de serem
animais exclusivamente marinhos.
( ) Para manter um anfbio no aqurio, pode-se escolher uma salamandra
que mantenha as brnquias na forma adulta, sendo capaz, portanto, de
respirar dentro da gua.
( ) No ser possvel colocar nenhum inseto no aqurio; como as estruturas
respiratrias dos insetos so traquias, todos eles so terrestres e morreriam
afogados dentro da gua.
Em relao a esses fatores, analise a veracidade (V) ou falsidade (F) das
proposies acima e assinale a alternativa que preenche corretamente os
parnteses, de cima para baixo.
a) V V V F F F
b) V F V V V F
c) V V F F V F
d) F V F F F V

1) C

2) A

3) D

4) B

5) D

6) D

7) E

8) C

9) D

10) B

11) C

12) B

13) C

14) A

15) E

16) C

17) E

18) A

19) B

20) C

21) A

22) E

23) C

24) D

25) E

26) E

27) B

28) B

29) C

30) B

31) E

32) D

33) B

34) E

35) C

36) C

37) B

38) E

39) D

40) B

41) B

42) B

43) C

44) A

45) D

46) B

47) E

48) C

49) C

50) C

51) A

52) D

53) C

54) E

55) B

56) E

57) A

58) B

59) D

60) E

165

61) E

62) D

63) E

64) B

65) D

76) C

77) C

66) B

67) D

68) C

69) E

70) D

81) A

82) A

71) E

72) D

73) C

74) C

75) E

1. Analise as proposies abaixo:


I. Segundo a hiptese dos cosmozorios ou panspermia csmica, os
primeiros seres vivos teriam chegado Terra em fragmentos de
corpos celestes, como os meteoritos ou estrelas cadentes.
II. A hiptese da abiognese ou da gerao espontnea, defendida h
mais de 2.000 anos por Aristteles, postulava a existncia do
"princpio vital", capaz de produzir matria viva a partir de matria
bruta.
III. Os cientistas Lazzaro Spallanzani, John Needham e Francesco Redi
contestaram a hiptese da abiognese.
IV. De acordo com a hiptese autotrfica, os seres vivos surgiram de
molculas orgnicas, na forma de organismos muito simples e
incapazes de produzir seu prprio alimento.
Assinale a alternativa correta:
a) Apenas as proposies I e II esto corretas.
b) Apenas as proposies II e III esto corretas.
c) Apenas as proposies III e IV esto corretas.
d) Apenas as proposies II e IV esto corretas.
e) Todas as proposies esto corretas.
2. A Terra tem uma idade estimada de 4,5 bilhes de anos e h cerca de 3,5
bilhes surgiram as primeiras formas de vida que evoluram, originando toda
a biodiversidade existente no planeta. Sobre a origem e evoluo dos seres
vivos correto afirmar:
a) As primeiras formas primitivas de vida surgiram com as algas e bactrias
heterotrficas que por meio da endosimbiose originaram os primeiro
organismos eucariontes. Essa concepo reforada pela presena de
molculas de DNA em mitocndrias e cloroplastos.
b) Os coacervados resultaram de uma combinao de NH3, CH4, H2 e
vapores de gua, presentes numa atmosfera rica em descargas eltricas
e radiaes ultravioleta, o que provocou a origem dos primeiros
aminocidos e associao de molculas mais complexas.
c) De acordo com a teoria evolucionista de Lamarck, os fatores ambientais
alteram as caractersticas fenotpicas dos organismos, porm jamais
sero transmitidas a seus descendentes.
d) O isolamento geogrfico pode gerar espcies novas a partir de uma
espcie ancestral, porm as diversidades genticas surgidas podem ser
revertidas com o fim do isolamento.
e) Estudos sobre mutao e origem da variabilidade gentica permitiram
criar a Teoria do Neodarwinismo, que associa variao e seleo,
anulando teorias como o Fixismo, Lamarkismo e o prprio Darwinismo.
3. Utilize os conceitos enumerados para completar as afirmativas abaixo:
(1) evoluo
(2) teoria sinttica da evoluo
(3) lei da transmisso das caractersticas adquiridas
(4) convergncia adaptativa
(5) seleo natural
I. O mecanismo defendido por Darwin para explicar a adaptao dos
organismos ao ambiente conhecido como ( ).
II. A idia de que o desenvolvimento de um rgo pelo uso intensivo
herdado pela prole conhecida como ( ).
III. A idia de que todos os organismos esto ligados por laos de
ancestralidade e descendncia com modificao conhecida como
( ).
IV. A idia que considera o desenvolvimento de estruturas e formas
corporais semelhantes por adaptao a ambientes parecidos
conhecida como ( ).
V. A mutao, recombinao e seleo natural so os principais fatores
evolutivos considerados pela ( ).
Assinale a alternativa que apresenta a seqncia numrica, lida de cima para
baixo, que completa corretamente as afirmativas:
a) 5, 3, 4, 2 e 1
b) 5, 3, 1, 4 e 2
c) 1, 3, 4, 5 e 2
d) 1, 4, 3, 5 e 2
e) 2, 5, 1, 3 e 4
4. A respeito da formao das espcies, assinale o que for correto.
01. O critrio que melhor distingue duas espcies entre si o das
dessemelhanas fisiolgicas e bioqumicas.
02. Populaes reprodutivamente isoladas de outras podem passar a ter
histria evolutiva prpria e independente.

78) E

79) E

80) B

04. O aparecimento de mecanismos de isolamento evolutivo prejudica a


especiao.
08. Subespcies de uma mesma espcie so separadas por mecanismos de
isolamento reprodutivo.
16. Para que possa ocorrer especiao geogrfica e formao de novas
espcies, as populaes devem estar separadas por barreiras
geogrficas.
5. Um agrupamento de clulas diferenciadas e especializadas na execuo de
uma funo biolgica denomina-se tecido, que so classificados em tecido
epitelial, conjuntivo, muscular e nervoso. So os tecidos que constituem os
rgos e estes constituem os sistemas.
Os sistemas por sua vez, comandam as atividades vitais nos seres vivos.
Nesse contexto, INCORRETO afirmar que o tecido epitelial
a) constitui-se de dois tipos bsicos: o primeiro, de revestimento ou
protetor e o segundo, glandular ou secretor.
b) formado por clulas justapostas, geralmente polidricas, e apresenta
escassez de substncias intercelulares, tendo como principal funo
revestir e proteger as superfcies do organismo.
c) apresenta elevada quantidade de substncia intercelular e suas clulas
possuem formas e funes bastante variadas com diversas
especializaes.
d) pode ser classificado quanto ao nmero de camadas e ao formato das
clulas, tais como: pavimentoso simples e estratificado, cbico simples e
estratificado e prismtico simples.
e) identificado no revestimento da traquia e dos brnquios como
pseudo-estratificado e no revestimento interno da bexiga como
estratificado de transio.
6. Os tecidos de revestimento ou epitlios so formados por clulas
justapostas e apresentam caractersticas peculiares nos diferentes grupos
animais. Todas as funes relacionadas a seguir podem estar associadas s
diferentes funes dos epitlios, EXCETO:
a) Absoro de nutrientes
b) Trocas gasosas com o ambiente
c) Manuteno da temperatura corporal
d) Distribuio de nutrientes para todo o corpo
7. Sobre a estrutura tecidual do Homem podemos afirmar que:
a) A pele formada por um epitlio estratificado pavimentoso, com camada
protetora de queratina; contudo a melanina, produzida pelos
melancitos, que protege a pele da exposio prolongada ao sol.
b) Os tbulos renais formam glndulas revestidas por epitlio cilndrico com
funo de absoro e secreo.
c) O intestino revestido por epitlio cbico estratificado, que forma
volumosas dobras na mucosa, chamadas microvilosidades intestinais,
que compem as reas absortivas deste rgo.
d) As glndulas sebceas e sudorparas so anexos exclusivos da pele.
e) Pncreas, supra-renais e fgado so considerados glndulas mistas por
apresentarem ductos que conduzem secreo e tambm liberam
diretamente hormnios em capilares sangneos.
8. Em algumas situaes de agresso por fatores externos, algumas clulas
podem sofrer alteraes e dar origem a um novo tipo de tecido, processo
denominado de metaplasia. Um exemplo quando o epitlio pseudoestratificado ciliado da traquia em fumantes crnicos, sob a ao irritante da
fumaa de cigarro, transforma-se em epitlio estratificado pavimentoso.
Analise as afirmativas abaixo e assinale a que REPRESENTA ALTERAES
DECORRENTES dessa metaplasia.
a) Reduo do nmero de camadas celulares.
b) Aumento do nmero de clulas ciliadas.
c) Diminuio na quantidade e viscosidade do muco.
d) Formao de um tecido mais resistente, semelhante ao encontrado na
pele.
9. uma espcie de tecido conjuntivo que serve como reserva de energia e
como proteo contra o frio. Alm disso envolve diversos rgos,
protegendo-os contra traumatismos durante os movimentos do corpo. Este
tecido conhecido como: Linhares 118
a) sseo
b) cartilaginoso
c) adiposo
d) muscular
e) nervoso

166

10. Marque a alternativa cuja seqncia


adequadamente as lacunas da seguinte frase:

preenche

respectiva

d) Para preencher o molde, foi utilizado o sangue, que, assim como o tecido
adiposo, exemplo de tecido conjuntivo especializado.
e) A maior parte dos minerais dos ossos se localiza na matriz extracelular.

Na periferia da cartilagem hialina, os _______________ apresentam forma


alongada, com o eixo paralelo superfcie. Mais profundamente, so
arredondados e aparecem em grupos de at oito clulas, chamados grupos 14. Com relao a diferentes tipos celulares observados na espcie humana,
________________, porque suas clulas so originadas de um nico analise as proposies abaixo.
__________________.
00. As plaquetas ou trombcitos se originam de clulas denominadas
a) condrcitos, isgenos, condroblasto
b) condroblastos, isgenos, condrcito
c) condrcitos, pericndrios, condroblasto
d) condroblastos, pericndrios, condrcito
e) pericndrios, condroblastos, condrcito

megacaricitos, que se fragmentam e passam para a circulao.


01. A maturao dos linfcitos pode ocorrer na medula ssea e tambm no
timo e no bao.
02. Os condroblastos tm intensa atividade metablica e de diviso celular,
ao contrrio dos condrcitos.
03. Os osteblastos so as clulas que mantm a parte mineral da matriz
11. Em relao ao tecido conjuntivo cartilaginoso podemos afirmar que:
ssea enquanto os osteoclastos produzem a parte protica dessa matriz.
01. a cartilagem hialina ocorre no nariz, na laringe e na traquia, e no 04. As clulas de Leydig tm funo importante na nutrio dos
esqueleto fetal, antes de ser substituda pelo tecido sseo.
espermatozides em humanos. As clulas de Sertoli (intersticiais) tm
02. os tendes e ligamentos fixam-se aos ossos nas articulaes, em
funo endcrina, pois secretam testosterona.
associao com cartilagens fibrosas.
03. a cartilagem elstica encontrada no pavilho auditivo e rica em fibras 15. Clulas e outras estruturas com funes especializadas formam os
de tecido conjuntivo denso, o que a torna mais resistente tenso.
diferentes tecidos do organismo. A esse respeito, numere a coluna da direita
04. o pericndrio, formado por tecido conjuntivo denso, responsvel pela com base nas informaes da coluna da esquerda.
nutrio e pela regenerao das cartilagens.
1.Sntese de Fibras Colgenas
( ) Macrfagos
05. a cartilagem hialina ricamente vascularizada e, por isso, menos 2.Capacidade Fagocitria
( ) Plaquetas
sujeita a processos degenerativos.
3.Produo de Anticorpos
( ) Fibroblastos
4.Coagulao Sangnea
( ) Plasmcitos
12. Na coluna I, constam alguns tecidos constituintes do corpo dos animais, 5.Percepo de Cores
( ) Clulas da Glia
e, na coluna II, esto listadas algumas de suas caractersticas. Estabelea a 6.Sustentao dos Neurnios
( ) Cones
correta associao entre as colunas.
Assinale a alternativa que apresenta a seqncia correta da coluna da
I
direita, de cima para baixo.
(1) Tecido cartilaginoso
a) 2, 4, 1, 3, 6, 5.
(2) Tecido conjuntivo
b) 1, 4, 6, 2, 3, 5.
(3) Tecido sanguneo
c) 3, 2, 1, 4, 5, 6.
(4) Tecido sseo
d) 4, 2, 3, 5, 6, 1.
e) 1, 3, 4, 2, 5, 6.
II
(a) Caracteriza-se por ser um tecido rgido, especializado na sustentao do 16. Um jovem apresenta cicatriz na coxa, provocada por mordida de co
organismo, no suporte de partes moles e na proteo de rgos vitais. feroz. A cicatriz no local da leso deve-se:
constitudo por clulas denominadas de ostecitos, osteoblastos e a) ao acmulo de plaquetas que tm substncias ativas no processo de
osteoclastos e por uma matriz calcificada.
converso de fibrinognio em fibrina.
(b) Caracteriza-se por ser um tecido malevel, constitudo por uma
b) substituio do tecido muscular por tecido conjuntivo, por meio de
substncia intercelular denominada de matriz e por poucas clulas
fibroblastos e substncias intercelulares.
denominadas de condrcitos e condroblastos.
c) regenerao do tecido epitelial pseudoestratificado por meio de suas
(c) Caracteriza-se por apresentar riqueza de tipos celulares, como os
clulas totipotentes indiferenciadas.
fibroblastos e os macrfagos, e abundante material intercelular.
d) organizao de fibras reticulares que atuam como uma trama de
(d) Caracteriza-se por apresentar uma parte lquida, o plasma, e uma parte
sustentao das clulas.
slida, os elementos figurados: hemcias, leuccitos e plaquetas. Sua e) interao entre filamentos de actina e miosina nos sarcmeros.
principal funo de transportar substncias pelo corpo.
Assinale a alternativa que apresenta a associao correta..
17. O tecido conjuntivo propriamente dito um verdadeiro campo de batalha
a) 1b / 2c / 3d / 4a.
b) 1a / 2b / 3d / 4c.
de nosso corpo, pois nele que se estabelecem vrios mecanismos de defesa
c) 1c / 2a / 3b / 4d.
d) 1d / 2b / 3a / 4c.
para a manuteno da integridade do organismo. Sobre estes mecanismos de
e) 1c / 2d / 3a / 4b.
defesa, considere as afirmativas a seguir.
13. Graas a uma mandbula feita sob medida, um homem de 56 anos de
idade, que teve cncer na face, conseguiu fazer sua primeira refeio slida,
depois de 9 anos. Para isso, foi criado um molde na forma de U (Figura A),
preenchido com hidroxiapatita, sangue e protenas, que ajudaram a formar a
estrutura ssea. O prottipo foi colocado na musculatura das costas do
paciente (Figura B, setas) e, sete semanas depois, foi retirado e implantado
na face. Aps quatro semanas da cirurgia de implantao, o paciente comeu
salsicha e po.
Cincia Hoje, vol. 35, n 209, 2004 [adapt.].

I. Os anticorpos so produzidos pelos plasmcitos, clulas do tecido


conjuntivo originadas a partir de linfcitos B.
II. Os fibroblastos so as clulas mais importantes no processo de remoo
de restos celulares do tecido conjuntivo, atravs de mecanismos de
fagocitose.
III. Os neutrfilos passam da corrente circulatria para o tecido conjuntivo
atravs de movimentos de diapedese e, nesse tecido, exercem funes
fagocitrias.
IV. Os linfcitos B so essencialmente responsveis pela defesa de base
celular, destruindo clulas estranhas e so os principais responsveis
pela rejeio de enxertos.
Esto corretas apenas as afirmativas:
a) I e II.
b) I e III.
c) II e IV.
d) I, III e IV.
e) II, III e IV.

18. Um estudante recebeu de seu professor quatro fichas com as seguintes


Com base no texto e em seus conhecimentos, assinale a alternativa informaes sobre diferentes tecidos:
INCORRETA:
Ficha I
a) As clulas do tecido no qual foi colocado o molde para a formao da
Clulas localizadas em colunas
mandbula so ricas em filamentos contrteis, especializadas em
Matriz percorrida por canalculos
contrao.
Matriz constituda por substncias orgnicas e minerais
b) O procedimento descrito acima s foi possvel pelo fato de o tecido sseo
Canais que contm vasos sanguneos
ser um tipo de tecido conjuntivo, assim como o tecido muscular.
Ficha II
c) As clulas do tecido sseo se localizam em espaos (lacunas) do material
Clulas grandes e globosas
intercelular,
comunicando-se
entre
si
pelos
prolongamentos
Constitui reserva de material energtico
citoplasmticos.
Encontra-se em permanente renovao
Desempenha funo de proteo contra a perda de calor

167

Ficha III
Apresenta numerosos tipos celulares
Aspecto de massa esbranquiada, mais ou menos rgida e fibrosa
Muito difundido no organismo
Ficha IV
Clulas fusiformes com ncleo central
Contrao lenta e involuntria
Que alternativa apresenta os nomes dos tecidos correspondentes s
informaes das respectivas fichas?
a) I - sseo; II adiposo; III - conjuntivo propriamente dito; IV - muscular
liso
b) I - cartilaginoso; II - adiposo; III - sseo; IV - muscular esqueltico
c) I - sseo; II - cartilaginoso; III - conjuntivo frouxo; IV - muscular liso
d) I - hemocitopotico; II - cartilaginoso; III - sseo; IV - muscular cardaco
e) I - sseo; II - adiposo; III - cartilaginoso; IV - muscular liso
19. ... Deus me deu mos de veludo pra fazer carcia/ Deus me deu muitas
saudades e muita preguia/ Deus me deu pernas compridas e muita malcia/
Pra correr atrs de bola e fugir da polcia/ Um dia ainda sou notcia(...) Deus
me fez um cara fraco, desdentado e feio/ Pele e osso simplesmente quase
sem recheio/...
(Trecho da msica Partido Alto, de Chico Buarque).
Analisando alguns trechos da cano sob o ponto de vista da histologia,
correto afirmar:
01. Considerando que se trate no texto de um indivduo adulto com idade
entre 25 e 30 anos, a utilizao do termo pernas compridas pode
indicar que os ossos das pernas j efetuaram o crescimento longitudinal
e que a cartilagem de conjugao j foi totalmente substituda por osso.
02. Para evitar a fragilidade ssea, faz-se necessria, dentre outros fatores,
uma dieta alimentar rica em clcio e protenas.
04. Os nutrientes chegam a todas as clulas do organismo pelos vasos
sanguneos. Os nutrientes absorvidos pelo epitlio intestinal penetram
nos vasos sanguneos do tecido conjuntivo adjacente, sendo distribudos
para os outros tecidos vascularizados, como o adiposo, o sseo e os
tecidos musculares.
08. A ao de fazer carcias e de correr atrs da bola ocorre da seguinte
maneira: para contrair o msculo liso (contrao voluntria), o qual se
encontra ligado ao esqueleto sseo e proporciona movimentos teis,
necessrio haver impulso do Sistema Nervoso Central.
16. A epiderme uma das camadas da pele que reveste externamente o
organismo humano. responsvel pela formao das glndulas
sudorparas, que executam a secreo do suor.
32. A falta de vitamina C pode ser responsvel pelo fato de o eu lrico da
cano ser desdentado: essa vitamina atua na sntese do colgeno,
fundamental na sustentao dos tecidos conjuntivos atravs das fibras
colgenas.
20. ... os punhos e os pulsos cortados e o resto do meu corpo inteiro/ h
flores cobrindo o telhado e embaixo do meu travesseiro/ h flores por todos
os lados/ h flores em tudo que vejo/ a dor vai curar estas lstimas o soro
tem gosto de lgrimas/ as flores tm cheiro de morte/ a dor vai fechar esses
cortes/ flores, flores, as flores de plstico no morrem...
(Trecho da letra da msica Flores, dos integrantes do grupo Tits Charles
Gavin, Tony Bellotto, Paulo Miklos e Srgio Britto).
Analisando histologicamente alguns trechos, correto afirmar:
01. esperado que aps a coagulao do sangue na rea da leso ocorrida
("os punhos e os pulsos cortados") concentre-se a grande nmero de
macrfagos, fibroblastos e plasmcitos.
02. Se uma cartilagem hialina for lesada em um indivduo adulto, a sua
regenerao ocorre facilmente, j que ela um tecido ricamente
vascularizado.
04. Quando ocorre um corte profundo na pele, os vasos sanguneos so
lesados e isso ocasiona o extravasamento do plasma, juntamente com
clulas de sangue.
08. O msculo estriado esqueltico um tipo especializado de tecido
conjuntivo.
16. Protenas colgenas so constituintes importantes na matriz extracelular
do tecido conjuntivo, sendo necessrias nos processos de reparo e
cicatrizao. Elas podem ser sintetizadas por osteoblastos, condroblastos
e fibroblastos.
32. O tecido sseo muito resistente, sem plasticidade e com pouca irrigao
sangunea.
21. As lminas I, II e III representam o aspecto de trs tipos de tecido
muscular de ces, quando analisados sob microscpio.

As fibras observadas nas lminas


respectivamente, dos msculos
a) do estmago, do corao e da pata.
b) do corao, da pata e do estmago.
c) da pata, do estmago e do corao.
d) do corao, do estmago e da pata.
e) do estmago, da pata e do corao.

I,

II

III

foram

retiradas,

22. As responsveis pelo encurtamento da fibra muscular esqueltica e,


consequentemente, pela sua contrao so as miofibrilas, constitudas pelas
protenas actinas e miosina. A unidade estrutural e funcional da miofibrila
o:
a) ADP
b) Fosfocreatina
c) ATP
d) Sarcmero
e) ons clcio
23. O alimento passa do esfago para o estmago como resultado de uma
onda peristltica. Assinale a alternativa que mostra o tecido responsvel pela
peristalse do sistema digestrio.
a) Tecido muscular esqueltico
b) Tecido muscular liso
c) Tecido conjuntivo
d) Tecido adiposo
e) Tecido epitelial
24. Com relao ao tecido muscular humano, assinale a alternativa correta.
a) O retculo sarcoplasmtico e o sistema T liberam ons Ca++ e K+ para o
citoplasma. Em presena desses dois ons, a actina adquire propriedade
ATPsica.
clulas adiposas e sem contato com axnios.
c) Na fibra muscular, a fonte primria de energia para contrao a glicose.
Assim, tanto a gliclise quanto o ciclo de Krebs e a cadeia respiratria
produzem ATP necessrio contrao.
d) As fibras lisas so sinccios, sem antenas, com discos intercalares, que so
finas membranas celulares, que separam fibras vizinhas.
25. A energia para contrao muscular suprida por molculas de ATP
(produzidas durante a respirao celular). O ATP atua na ligao de miosina
actina, o que resulta na contrao muscular. A principal reserva de energia
nas clulas musculares onde seus grupos de fosfatos, ricos em energia, so
transferidos o ADP na produo do ATP necessrio :
a) Hemoglobina
b) Glicognio
c) Fosfocreatina
d) Lactato
e) Piruvato
26. Experincias com ratos [...] indicam que a mistura de bebidas alcolicas
e energticos pode provocar doenas degenerativas do sistema nervoso. A
pesquisa verificou que o etanol acelera a morte de clulas do sistema
nervoso central e esse efeito potencializado pela cafena, principal
ingrediente dos energticos. Altas doses destes componentes aceleram um
mecanismo natural de renovao das clulas, chamado apoptose, fazendo
com que elas se autodestruam.
(CINCIA HOJE. Rio de Janeiro: SBPC, v. 32, n. 192, p. 55, abr. 2003).
Com relao aos vrios conceitos abordados no texto, CORRETO afirmar:
01. Como o sistema nervoso formado por clulas permanentes, os efeitos
sobre ele podero ser revertidos com alguns poucos cuidados, j que
uma das principais caractersticas desse tipo de clulas sua capacidade
mdia de regenerao.

168

02. Experincias com ratos no devem ser estendidas aos humanos uma
vez que essas duas espcies no fazem parte sequer da mesma Classe
Zoolgica.
04. A apoptose ocorre, por exemplo, quando as membranas entre os dedos
do feto so destrudas.
08. Se o descrito no texto ocorresse com clulas epiteliais, o problema seria
ainda mais grave, dado que este tipo de clula tem uma baixa
capacidade de regenerao.
16. Um neurnio, em geral, transmite seu potencial de ao para outro
atravs de sinapses geradas pela presena de molculas neurotransmissoras.
32. Os neurnios so formados por um corpo celular, pelo axnio e por
dendritos.
27. Alguns anestsicos locais agem impedindo a despolarizao da
membrana plasmtica dos neurnios. A sensao de dor , ento, eliminada
em funo da:
a) modificao da fenda sinptica.
b) ruptura da bainha de mielina dos neurnios.
c) diminuio do nmero de sinapses.
d) inibio da secreo do neurotransmissor.
e) inverso no sentido do impulso nervoso.
28. Foi Galvani (1780) quem constatou, pela primeira vez, a natureza eltrica
da atividade nervosa. Sabe-se que, em repouso, a membrana do axnio tem
carga eltrica positiva externamente e, negativa, internamente. Na figura
abaixo mostra-se:

a) o impulso nervoso em um axnio mielinizado.


b) o impulso nervoso em um axnio amielnico.
c) a conduo saltatria do impulso nervoso.
d) a ao de neurotransmissores.
e) o limiar de excitao neuronal.
29. Em relao conduo do impulso nervoso e considerando os desenhos
abaixo, assinale a(s) proposio(es) CORRETA(S).

01. As regies I, II e III do desenho representam, respectivamente, o axnio,


o corpo celular e o dendrito.
02. A regio V do desenho o local onde ocorre a Sinapse.
04. A regio VI do desenho indica que aquela parte da fibra est polarizada.
08. A regio VII do desenho mostra que aquela parte da fibra est em
potencial de ao.
16. A regio IV do desenho representa um ndulo de Ranvier. Tais ndulos
so vistos somente nos neurnios mielinizados e so responsveis pelo
aumento da velocidade do impulso nos mesmos. Como a inverso da
polaridade na fibra ocorre somente nesses ndulos, o impulso se
propagar saltando de ndulo em ndulo e aumentando sua
velocidade na fibra.
32. A propagao do impulso nervoso em um neurnio ocorre sempre no
sentido III, II, I.
30. Analise a figura sobre as clulas do tecido nervoso.

No que se refere s caractersticas e funes desempenhadas por estas


clulas, assinale a alternativa correta.
a) Figura 1 Astrcito clula cujos prolongamentos se enrolam sobre as
neurofibras presentes no sistema nervoso central, envolvendo-as com
camadas concntricas de sua membrana plasmtica e constitui a bainha
de mielina, que protege e auxilia o desempenho funcional dos neurnios.
b) Figura 2 Micrglia macrfago especializado, cuja funo fagocitar
detritos e restos celulares presentes no tecido nervoso. uma clula
grande, com muitos prolongamentos longos e pouco ramificados.
c) Figura 3 Corpo celular do neurnio consiste no centro metablico do
neurnio, contm o ncleo e a maioria das organelas da clula, tais
como retculo endoplasmtico liso, denominado corpsculo de Nissl, e se
relaciona sntese de neurotransmissores.
d) Figura 4 Dendritos consistem em prolongamentos ramificados e
especializados na recepo de estmulos provenientes de outros
neurnios ou de clulas sensoriais. Esses prolongamentos aumentam a
superfcie dos neurnios, o que lhes permite captar grande variedade de
estmulos.
e) Figura 5 Oligodendrcito apresenta grande nmero de prolongamentos
citoplasmticos, alguns destes se ligam aos neurnios, enquanto outros
se ligam a capilares sangneos por meio de expanses denominadas
ps-vasculares, estabelecendo uma ponte nutritiva entre o sangue e os
neurnios.
31. Sobre os vrus, podemos afirmar que
a) a dengue, o sarampo, a hepatite, a clera e a gripe, so doenas causadas
por eles.
b) so seres vivos celulares, podendo apresentar DNA ou RNA.
c) alguns apresentam um envoltrio externo ao capsdeo chamado de
envelope, geralmente composto por lipdios e glicoprotenas sintetizados
pelo prprio vrus.
d) aqueles que apresentam em sua estrutura a transcriptase reversa, enzima
capaz de realizar sntese de DNA utilizando como molde o RNA viral, so
chamados de retrovrus, a exemplo do HIV, o vrus da gripe e o do
sarampo.
e) so estruturalmente simples, apresentando uma capa protica o
capsdeo, que envolve a molcula de cido nuclico, que pode ser DNA
ou RNA.
32. A expanso da populao humana, com a degradao do ambiente
natural e conseqentes distrbios no equilbrio biolgico, so os principais
fatores que nos expem aos tipos de doenas por agentes biolgicos. A
dengue causada por Flavivrus (arbovrus), um tipo de vrus de RNA de
cadeia simples (+), sendo o mosquito Aedes aegypti o principal vetor da
doena no Brasil. A forma mais grave da doena, conhecida como dengue
hemorrgica pode levar morte.
I) A fmea infectada capaz, durante a picada e a alimentao, de
transmitir o vrus a indivduos suscetveis.
II) As fmeas infectadas podem transmitir o vrus para sua prole atravs
da transmisso transovariana.
III) No h tratamento especfico para a dengue; combatem-se os
sintomas com hidratao e antitrmicos base de cido acetilsaliclico.
IV) A dengue pode ser controlada e eventualmente erradicada com
eliminao
dos
mosquitos
vetores.
V) Se uma pessoa for infectada por uma das formas de vrus adquire
imunidade para as outras.
VI) Os vrus de RNA de cadeia simples (+) tambm so conhecidos como
retrovrus, que produzem DNA a partir do RNA viral.
Analise as afirmativas e assinale a alternativa correta.
a) I, III e V.
b) I, II e IV .
c) II, III e VI.
d) II, IV e VI.
e) III, V e VI.
33. Associe a doena com o seu modo de infeco.
Doenas:
I. Febre amarela.
II. Caxumba.
III. Meningite.
IV. Sarampo.
V. Hepatite.
VI. AIDS.

169

Modo de Infeco:
( ) O vrus penetra pela mucosa das vias respiratrias, cai na corrente
sangnea e se dissemina nas diversas artes do corpo.
( ) O vrus introduzido juntamente com a saliva do mosquito e instala-se no
bao, rins, medula ssea, fgado e gnglios linfticos.
( ) O vrus ataca normalmente as glndulas salivares partidas, podendo
localizar-se nos ovros, testculos, crebro e pncreas.
( ) O vrus pode atingir as clulas nervosas, destruindo- as, o que causa
paralisa e atrofia da musculatura esqueltica.
A seqncia correta entre doenas e modo de infeco
a) IV, I, II, III.
b) I, II, VI, III.
c) I, II, III, IV.
d) V, VI, I, II.
e) III, II, VI, I.
34. Brasil entra em alerta contra surto de rubola. Uma doena que poderia
estar erradicada volta a assustar o pas.
Segundo o Ministrio da Sade, nove estados e o Distrito
Federal registram surtos de rubola. As secretarias esto em estado de alerta
para a deteco de casos da doena. (Globo.com/notcias, 27.08.2007)
A rubola causada por
a) bactria e transmitida a partir do contato com gua e alimentos
contaminados. A preveno se d com saneamento bsico e o
tratamento feito base de antibiticos.
b) bactria e transmitida a partir de relaes sexuais. Pode ser transmitida de
me para filho durante a gestao. O tratamento feito base de
antibiticos.
c) vrus e transmitida por contato direto e pela saliva. Pode ser transmitida
de me para filho durante a gestao. A preveno realizada pela
vacinao.
d) vrus e transmitida a partir da picada de mosquitos que j tenham picado
uma pessoa doente. A preveno se d pelo combate ao mosquito vetor
e pela vacinao.
e) protozorio e transmitida por contato com fezes de animais domsticos, as
quais podem conter cistos do parasita. Em mulheres grvidas pode
atingir o feto. O tratamento feito base de antibiticos.
35. A encefalopatia espongiforme bovina, mais conhecida como doena da
vaca louca, faz parte de um grupo de doenas que tm como agente
causador
a) um lipdio.
b) uma protena.
c) um RNA viral.
d) um DNA bacteriano.
e) um glicdio.
36. A figura abaixo representa o desenho esquemtico de uma clula
bacteriana. Como todo ser vivo, este tambm se reproduz e transmite as
informaes genticas sua descendncia, atravs do seu DNA. A alternativa
que cita os dois componentes celulares bacterianos que contm DNA :

a) nucleide e mesossomo.
b) parede celular e plasmdio.
c) plasmdio e nucleide.
d) plo sexual e ribossomo.
e) membrana plasmtica e mesossomo.
37. As bactrias recebem nomes diferentes quando se apresentam
isoladamente: cocos, bacilos, e espirilos, de acordo com a forma. Mas elas
tambm podem ser vistas em grupos e, neste caso, so chamadas:
a) estreptococos, quando aparecem de duas em duas; estafilococos, quando
apresentam forma semelhante a um cacho de uvas; e diplococos,
quando se organizam em cadeias.
b) estreptococos, quando aparecem em forma de bacilo; estafilococos,
quando apresentam forma semelhante a um cacho de uvas; e,
diplococos quando se organizam em cadeias.
c) diplococos, quando aparecem de duas em duas; estafilococos, quando
apresentam forma semelhante a um cacho de uvas; e estreptococos,
quando se organizam em cadeias.

d) diplococos, quando aparecem isoladas; estafilococos, quando apresentam


forma semelhante a um cacho de uvas; e estreptococos, quando se
organizam em cadeias.
e) diplococos, quando aparecem de duas em duas; espirilo, quando
apresentam forma semelhante a um cacho de uvas; e estreptococos,
quando se organizam duas em duas.
38. Em bactrias, h 3 mecanismos conhecidos de reproduo sexuada,
que se constituem em meios de ampliao da variabilidade gentica. Qual a
denominao do mecanismo ilustrado na figura?

a) transformao
c) bipartio
e) transduo

b) conjugao
d) transcrio

39. Considere as afirmaes:


I. As cianobactrias (algas azuis ou cianofceas) formam um grupo especial
dentre as eubactrias, que so auttrofas, fotossintetizantes, possuem
uma distribuio ampla, ocorrendo no mar, em gua doce, em locais
midos sobre rochas, no solo e em troncos de rvores.
II. As cianobactrias apresentam flagelos e fmbrias, locomovemse por
batimentos dos flagelos e no por deslizamento.
III. As arqueobactrias diferem em muitos aspectos das eubactrias.
Algumas dessas diferenas so: a parede celular das arqueobactrias
possui peptidoglicano e os mesmos tipos de fosfolipdios que os das
eubactrias.
IV. A bactria Bacilus anthracis, que provoca feridas negras como o carvo,
denominada de Antraz (do grego anthracis) uma das doenas de
animais mais antigas j registradas, conhecida como carbnculo.
Podemos afirma que:
a) I e IV so corretas b) I e II so corretas
c) II e III so corretas d) III e IV so corretas
e) I e III so correta
40. Para a espcie humana, o parasitismo constitui um fator limitante,
principalmente em regies subdesenvolvidas. De fato, so inmeros os casos
de protozooses, viroses, infeces bacterianas e verminoses, cujos efeitos
so bem conhecidos, e ocorrem em todas as faixas de idade. Assinale a
alternativa que apresenta exclusivamente exemplos de doenas humanas
causadas por bactrias.
a) AIDS, rubola e sarampo.
b) caxumba, ttano e sfilis.
c) febre amarela, meningite e peste bubnica.
d) poliomielite, tracoma e coqueluche.
e) hansenase, clera e tuberculose.
41. O fenmeno denominado mar vermelha ocorre devido proliferao
de algumas espcies de dinoflagelados, que, sob determinadas condies,
formam populaes extraordinariamente grandes. Essa concentrao pode
liberar toxinas e provocar a contaminao e a mortalidade em organismos
filtradores, peixes e outros vertebrados marinhos. Essa contaminao e
mortalidade se devem
a) variedade de toxinas produzidas no vero por essas algas.
b) proliferao excessiva de certas macroalgas.
c) ao tipo de pigmento de algumas espcies de dinoflagelados.
d) elevada toxicidade da tetraodontoxina provocada pelos dinoflagelados.
e) elevada toxicidade da neurotoxina produzida por esses dinoflagelados.
42. Preencha as lacunas do texto a seguir.
O sushi um prato tpico da culinria japonesa e, no seu preparo, certas
espcies de __________________ so usadas, como o caso da nori
(Porphyra). Por ser rico em _________________, esse organismo auxilia no
combate a uma doena carencial, denominada escorbuto. Esse organismo
constitudo por clulas que possuem, envolvendo a membrana plasmtica,
uma ______________, formada por uma camada mais interna e rgida de
_________________, e outra mais externa, mucilaginosa, composta dos
polissacardeos gar e carrageano (ou carragenina). Dentre as caractersticas
consideradas importantes para a classificao desses organismos, incluem-se
os diferentes tipos de _______________.

170

43. No que se refere ao Reino Protista, assinale a(s) alternativa(s) correta(s).


01. Todos os protozorios atuam como parasitas do homem e de outros
seres vivos.
02. Os protozorios so eucariontes unicelulares que vivem isolados ou
formando colnias, nos mais variados tipos de habitat.
04. Os protozorios podem ser fixos ou se deslocar atravs de clios, flagelos
ou pseudpodes. De acordo com o tipo e a presena ou no dessas
organelas locomotoras, os protozorios so classificados em rizpodes,
mastigforos, ciliados e esporozorios.
08. A malria e a disenteria amebiana so doenas ocasionadas por
protozorios que so inoculados no homem atravs da picada das
fmeas de um mosquito.
16. As algas euglenofceas, pirrofceas e as crisofceas so exemplos de
organismos pertencentes ao Reino Protista.
32. Os protozorios de ambiente marinho apresentam um vacolo pulstil
que recolhe o excesso de gua que penetra na clula e, atravs de
movimentos de pulsao, elimina essa gua para o meio externo.
44. Em um experimento realizado com duas espcies de protozorios do
mesmo nvel trfico, o crescimento das populaes de cada uma das
espcies, em um meio de cultura, foi registrado em duas situaes
diferentes: com as duas espcies em separado e com as duas espcies
juntas. Os resultados obtidos so apresentados nos grficos abaixo.
Crescimento das Populaes em separado
Em relao a esse experimento, julgue os seguintes itens.
00. A capacidade de suporte de espcie I maior que a da espcie II.
01. As duas espcies no coexistem, no experimento, porque possuem
tamanhos de populao diferentes quando crescem no meio em
separado.
02. A populao da espcie II diminuiu, quando junto com a espcie I,
porque esgotaram-se os recursos do meio de cultura.
45. ...L vai o quatro olho! Os portadores de defeitos visuais geralmente
so discriminados. O perfeito funcionamento dos rgos dos sentidos
imprescindvel para o bom desempenho do indivduo. Com relao ao sistema
sensorial correto afirmar que:
01. a percepo do mundo exterior feita atravs da viso, do olfato, do
tato, da audio e da gustao que interagem com o sistema nervoso e
glandular;
02. a incidncia dos raios luminosos, que estimulam os bastonetes e cones na
retina, regulada pela pupila, que se relaxa ou se contrai para permitir a
passagem de maior ou menor quantidade de luz;
04. o som provoca vibraes no tmpano que causa modificaes na presso
do lquido da cclea, o que estimula as clulas auditivas;
08. alguns animais possuem um maior desenvolvimento em alguns rgos
dos sentidos, como a viso nas aves de rapina;
16. a lngua do homem contm diferentes grupos de clulas sensoriais
especializados em perceber diferentes sensaes olfativas;
32. os mamferos apresentam mecanorreceptores estimulveis pela luz
incidente na base de cada plo;
64. o nariz importante na digesto, pois o cheiro dos alimentos estimula as
secrees gstricas e salivares e a motilidade gastrintestinal.

a) ficomiceto.
b) ascomiceto.
c) basidiomiceto.
d) deuteromiceto.
e) zigomiceto.
48. Sobre os fungos, assinale o que for correto.
01. Os fungos so organismos uni ou pluricelulares, aclorofilados, com
parede celular e sua reproduo normalmente envolve esporos, como
ocorre entre algumas plantas. Armazenam glicognio e, como os
animais, apresentam nutrio hetertrofa. Em virtude de suas
peculiaridades, so enquadrados num reino exclusivo: o reino Fungi.
02. Na agricultura, so bastante reconhecidas as doenas causadas por
fungos em plantas cultivadas como: arroz, milho, feijo, batata, tomate,
caf, algodo e outras. Entre tantos exemplos pode-se citar o fungo
Hemileia vastatrix, causador da ferrugem do caf, doena que dizimou
grande parte dos cafezais brasileiros na dcada de 70.
04. Na fabricao do lcool e de bebidas alcolicas como o vinho e a cerveja,
fundamental a participao dos fungos da espcie Aspergillus flavus,
que realizam fermentao alcolica, convertendo acar em lcool
etlico. Esses fungos, conhecidos como leveduras, so anaerbios que
realizam apenas fermentao como forma de obteno de energia.
08. Na espcie humana so conhecidas diversas micoses, que so doenas
causadas por fungos. Entre elas consideram-se o "sapinho" ou a
candidase, causada pelo fungo Candida albicans e a "frieira" ou p-deatleta, provocada pelo fungo Tinea pedis.
16. Existem fungos aliados dos interesses humanos na agricultura. o caso
do Metarhizuim anisopliae, um fungo usado como controle biolgico no
combate aos seres nocivos s plantaes, como besouros, cigarrinhas e
outros insetos.

49. Marque a alternativa que identifica os processos assexuados de


reproduo:
a) diviso binria; esporulao; heterogamia; fragmentao; gemulao;
partenognese.
46. O mofo que ataca os alimentos, os cogumelos comestveis e o fermento
b) diviso binria; isogamia; brotamento; fragmentao; gemulao;
de fazer o po so formados por organismos que pertencem ao reino Fungi.
partenognese.
Com relao a esse grupo assinale a(s) proposio(es) VERDADEIRA(S).
01. So organismos eucariontes, unicelulares ou pluricelulares, autotrficos c) diviso binria; esporulao; brotamento; fragmentao; gemulao;
partenognese.
facultativos.
d) diviso binria; esporulao; brotamento; hermafroditas; gemulao;
02. O material nutritivo de reserva o glicognio.
partenognese.
04. Em funo da nutrio hetertrofa, esses seres podem viver em
e) diviso binria; esporulao; brotamento; fragmentao; organismos
mutualismo, em saprobiose ou em parasitismo.
diicos; partenognese.
08. Alguns fungos so utilizados na obteno de medicamentos.
16. Nutrem-se por digesto extracorprea, isto , liberam enzimas digestivas
no ambiente, que fragmentam macromolculas em molculas menores, 50. Assinale a(s) alternativa(s) correta(s) em relao ao tema Reproduo na
Espcie Humana.
permitindo sua absoro pelo organismo.
32. Na alimentao humana so utilizados, por exemplo, na fabricao de 01. O capuz acrossmico, presente no espermatozide, o local onde esto
situadas as enzimas que iro "digerir" a membrana do gameta feminino
queijos, como o roquefort e o gorgonzola.
na fecundao.
64. Reproduzem-se, apenas, assexuadamente por meio de esporos, formados
02. Na cauda do espermatozide esto dispostas as mitocndrias,
em estruturas denominadas esporngios, ascos e basdios.
responsveis pela liberao da energia necessria para que os
espermatozides possam movimentar-se.
47. No esquema abaixo, est ilustrada, de forma simplificada, parte do ciclo
04. No homem, uma elevada concentrao do hormnio testosterona no
de vida de um fungo denominado:
plasma sanguneo inibe a secreo dos hormnios FSH (Folculo
Estimulante) e LH (Luteinizante), tambm chamado ICSH (Estimulador
das Clulas Intersticiais) na hipfise.
08. Na ovognese ou oognese, tanto na meiose I quanto na II, formam-se
clulas de tamanhos diferentes, o que no acontece na
espermatognese.
16. Assim que o espermatozide finaliza a penetrao no gameta feminino, a
meiose II inicia-se, formando um corpsculo polar e o vulo.

171

32. A suspenso das menstruaes pode ser um sinal de incio de gravidez,


uma vez que o hormnio estrgeno inibe a secreo do FSH (Folculo
Estimulante) responsvel pelo despreendimento do endomtrio.
51. A perpetuao da vida ocorre por meio da reproduo. O sistema
reprodutor humano composto de rgos e estruturas marcadamente
diferentes no sexo masculino e no feminino. Diversos hormnios participam
do processo de reproduo, favorecendo o desenvolvimento dos rgos
genitais, o impulso sexual e a manuteno da gravidez, entre outras funes.
Sobre a reproduo humana, falso/verdadeiro afirmar que:
01. os espermatozides formam-se a partir de clulas da parede interna dos
tbulos seminferos, onde se localizam tambm as clulas intersticiais ou
de Leydig, cuja a funo produzir testosterona, o hormnio sexual
masculino;
02. o vulo em formao encontra-se recoberto por uma pelcula geltinosa
espessa, a zona pelcida, ao redor da qual as clulas foliculares se
multiplicam ativamente e secretam um lquido, que faz o folculo crescer.
Nesse estgio, ela passa a ser chamado de folculo de Graaf;
03. a produo dos hormnios sexuais masculinos e femininos est sob o
controle dos hormnios folculoestimulante (FSH) e luteinizante (LH),
genericamente chamados gonadotrofinas. Esses hormnios so
sintetizados pelo lobo posterior da hipfise;
04. o interior do tero revestido de um tecido ricamente vascularizado
chamado endomtrio. A partir da puberdade, mensalmente, o
endomtrio prepara-se para uma possvel gravidez.

52. A figura abaixo ilustra um esquema comparativo da formao de


gametas masculino e feminino.Com o auxlio da figura, julgue os seguintes
itens.

a) No ovrio, cada ovcito est contido em um folculo (A) e, a cada ciclo


menstrual, um desses folculos sofre maturao (B).
b) O folculo, depois da eliminao do ovcito (C), transforma-se no corpo
lteo, ou corpo amarelo (D).
c) O ovcito II (C) eliminado do ovrio aps ter iniciado a 2 diviso
meitica, com esse processo interrompido em metfase II.
d) Na espcie humana, aps a ovulao (E), o ovcito completa a 2 diviso
meitica ao penetrar na tuba uterina, dando o sinal qumico para que a
fecundao ocorra.
e) Na ovulognese, que ocorre no ovrio (F), no h um perodo de
diferenciao, ps-diviso meitica, como ocorre na espermatognese.
54. O processo de formao de espermatozides, conhecido como
espermatognese, ocorre em quatro perodos: germinativo, de crescimento,
de maturao e de diferenciao. Sabendo-se que o espermatcito I
encontrado no perodo de crescimento e que a espermtide encontrada ao
final do perodo de maturao, pergunta-se: quantos espermatozides sero
formados a partir de 80 espermatcitos I e de 80 espermtides?
a) 40 e 20
b) 160 e 320
c) 320 e 80
d) 320 e 160
55. A funo do vitelo nutrir o embrio durante o desenvolvimento
embrionrio, ao menos em suas primeiras fases. No reino animal, a
quantidade e distribuio do vitelo no ovo determinam as diferenas na
segmentao, podendo esta ocorrer em todo o ovo ou s em parte dele. O
tamanho das clulas formadas (Blastmeros) durante a segmentao pode
ser igual ou diferente.
A tabela abaixo indica o tipo de ovo, conforme o teor e distribuio do vitelo,
o tipo de segmentao e d exemplos de animais que apresentam esses tipos
de ovos e segmentao.
Tipo de ovo

Tipo de segmentao

Telolcito

Exemplos

A
Meroblstica discoidal
B

Heterolcito

Anfbios

Heroblstica igual
C
01. Da figura, infere-se que tanto os espermatcitos secundrios quanto as
espermtides so celulas haplides.
02. Em um indivduo vasectomizado, a diferenciao de espermatognias em
espermatozides impedia devido seco do canal deferente.
03. No final da meiose, os quatro espermatozides formados so
geneticamente idnticos.
04. As espermatognias so clulas germinativas que realizam a primeira
diviso meitica ainda antes do nascimento do indivduo.
53. Abaixo mostra-se esquematicamente um ovrio humano e so dadas
algumas indicaes sobre o processo da ovulognese. Com relao a esse
assunto, incorreto afirmar que:

Seres humanos
D

Centrolcito

Antrpodes

Analise as afirmativas:
I. A letra A corresponde a animais como as aves e rpteis, pois seus ovos
possuem alto teor de vitelo, e a segmentao ocorre apenas em um dos
plos.
II. A letra B corresponde a um tipo de segmentao em que todos os
blastmeros formados possuem o mesmo tamanho.
III. A letra C corresponde a ovos que possuem muito vitelo, ficando este
distribudo igualmente no citoplasma.
IV. A letra D corresponde a um tipo de segmentao em que as clulas
embrionrias ficam dispostas na superfcie do ovo.
Com base nos textos e em seus conhecimentos, quais afirmativas esto
corretas:
a) I, II e III.
b) I, III e IV.

172

c) II e IV.
d) I e IV.
e) II e III.
56. Entre os cerca de 75 trilhes de clulas existentes em um homem
adulto so encontrados em torno de 200 tipos celulares distintos. Todos eles
derivam de clulas precursoras, denominadas 'clulas-tronco'. A clula-tronco
prototpica o vulo fertilizado (zigoto). Essa nica clula capaz de gerar
todos os tipos celulares existentes em um organismo adulto. [...] As clulastronco embrionrias so estudadas desde o sculo XIX, mas h 20 anos dois
grupos independentes de pesquisadores conseguiram imortaliz-las, ou seja,
cultiv-las indefinidamente em laboratrio. Para isso, utilizaram clulas
retiradas da massa celular interna de blastocistos (um dos estgios iniciais
dos embries de mamferos) de camundongos.
(CARVALHO, A. C. C. de. Clulas-tronco. A medicina do futuro. Cincia Hoje,
v. 29, n. 172, jun. 2001. p. 26-31.)
Com base nas informaes do texto e nos conhecimentos sobre o assunto,
correto afirmar:
01. O zigoto ou ovo resultante da fertilizao deve sofrer uma srie de
divises celulares, que iniciam no processo de clivagem para originar um
organismo multicelular complexo.
02. O blastocisto corresponde fase de blstula no mamfero e contm
clulas capazes de originar diferentes tipos celulares.
04. Uma clula-tronco embrionria aquela que pode se diferenciar em um
nico tipo celular durante o processo de desenvolvimento.
08. O zigoto pode ser considerado uma clula totipotente, pois pode dar
origem a todos os tipos celulares de um organismo, inclusive os
gametas.
57. Complete: No anfioxo a gstrula formada por duas camadas celulares.
A camada mais externa denominada _______, e a mais interna
denominada _____, origina mais tarde, duas outras camadas celulares,
denominadas de __________ e _________.
Marque a alternativa correspondente:
a) arqunio, blastocela, endoderma e blastporo.
b) ectoderma, mesentoderma , mesoderma e endoderma.
c) mesentoderma, blastocela, arquntero e blastporo.
d) arqunio, mesoderma, arquntero e blastporo.
e) arqunio, blastocela, ectoderma e blastporo.
58. Associe a 2a coluna de acordo com a 1a.
(1) Acelomado
(2) Pseudocelomado
(3) Celomado
( ) aneldeos
( ) platelmintos
( ) equinodermos
( ) nematelmintos
( ) artrpodos
Assinale a seqncia correta.
a) 1, 2, 3, 1, 3
b) 3, 1, 3, 2, 3
c) 2, 1, 2, 1, 3
d) 3, 2, 3, 1, 2
e) 2, 3, 3, 2, 1

e) Em I est representado o celoma, em II est representada a mesoderme e


III dar origem aos sistemas esqueltico e nervoso.
60. Baseando-se na embriologia do Anfioxo, assinale a alternativa que
apresenta a seqncia correta das fases.
a) Mrula Gstrula Blstula Nurula
b) Mrula Blstula Gstrula Nurula
c) Blstula Mrula Nurula Gstrula
d) Blstula Nurula Mrula Gstrula
61. Aps a fecundao, o zigoto, clula totipotente, sofre sucessivas divises
at formar os tecidos embrionrios. Os animais triblsticos apresentam, na
sua fase embrionria, ectoderme, mesoderme e endoderme, que originaro
todos os tecidos definitivos do ser. Os tecidos que apresentam a mesma
origem embrionria so:
a) Epiderme, receptores sensitivos, glndulas anexas ao tubo digestrio e
vasos.
b) Sangue, esqueleto apendicular, derme e corao.
c) Medula espinhal, anexos da epiderme, esqueleto axial e peritnio.
d) Crebro, medula espinhal, corao e crnio.
e) Revestimento da bexiga urinria, revestimento do sistema respiratrio,
epiderme e esmalte dos dentes.
62. Os organismos pluricelulares animais podem ser classificados de acordo
com alguns critrios embriolgicos e que envolvem, por exemplo, nmero de
folhetos germinativos, celoma e anexos embrionrios.
Considerando esses critrios, assinale a(s) afirmativa(s) correta(s).
01. Animais anamniotas so os vertebrados que no possuem o mnion,
como os anfbios e rpteis.
02. Animais alantoidianos so os animais que possuem o anexo embrionrio
denominado alantide, que auxilia na respirao e na excreo do
embrio.
04. Os animais prototrios, da mesma forma que os metatrios, tm placenta
primitiva, completando-se o desenvolvimento embrionrio na bolsa
marsupial.
08. O saco vitelnico o nico anexo embrionrio que ocorre em todos os
vertebrados, sendo pouco desenvolvido nos mamferos.
16. Os animais triblsticos (e.g. celenterados, moluscos e artrpodos)
possuem trs folhetos germinativos: o ectoderma, o mesoderma e o
endoderma.
32. Os peixes so vertebrados triblsticos, celomados, amniotas e
analantoidianos.
63. A figura mostra um feto humano em desenvolvimento.

59. Observe o esquema de corte transversal de embrio de anfbios e


escolha a opo correta:
correto afirmar, EXCETO:
a) o feto recebe nutrientes e gases atravs do cordo umbilical, que o liga
placenta.
b) o saco amnitico protege o feto em desenvolvimento e, em seu interior,
podem ser coletadas clulas fetais para a cariotipagem.
c) hormnios produzidos por clulas embrionrias podem afetar a produo
hormonal materna.
d) atravs da placenta, o sangue materno passa normalmente para o feto
fornecendo-lhe defesa imunolgica.

a) Em I est representado o celoma, em II est representada a endoderme


III caracteriza os animais cordados.
b) Em I est representado o pseudoceloma, em II est representada
mesoderme e III dar origem ao tubo digestrio.
c) Em I est representado o celoma, em II est representada a endoderme
III dar origem ao sistema nervoso.
d) Em I est representado o pseudoceloma, em II est representada
endoderme e III dar origem pele e seus derivados.

e
a

64. A digesto, nos seres humanos, envolve processos fsicos e qumicos. O


fsico envolve a triturao e o transporte dos alimentos e o qumico, a
transformao dos alimentos em seus constituintes, por enzimas digestivas.
Ambos os processos trabalham em conjunto para que, no final da digesto,
possa ocorrer a absoro dos constituintes qumicos e sua conduo para os
diferentes rgos do nosso corpo, a fim de haver a manuteno do
metabolismo. O sistema digestrio envolve diversos rgos que podem
apresentar diferentes funes: triturao, transporte, quebra enzimtica,
absoro.

e
a

173

Com base nos textos e no esquema ao lado, INCORRETO afirmar que:

a) na boca (E) inicia o processo fsico, como a triturao do alimento pelos


dentes, e tambm o processo qumico, realizado pelas enzimas
produzidas pelas glndulas salivares (A).
b) o rgo B o fgado; ele produz a bile, que fica armazenada na vescula
biliar (C). A bile rica em cido clordrico, por isso, mantm o pH cido
do intestino delgado, fato muito importante para a atuao das enzimas
digestivas.
c) o rgo G o pncreas, que produz uma variedade muito grande de
enzimas as quais formam o suco pancretico. Entre essas enzimas, esto
a amilase, a protease e a lipase.
d) o rgo F, em humanos, o estmago; nos animais ruminantes, o
estmago
verdadeiro
(abomaso)

precedido
por
outros
compartimentos: o rmen, o retculo e o omaso.
e) nos rgos D e H, ocorre a etapa final da digesto. Esses rgos so
importantes na absoro de nutrientes e gua.

II. A respirao area realizada por insetos; a branquial, pelos peixes; a


cutnea, pelos aneldeos e a pulmonar, pelos mamferos.
III. A respirao celular, nos eucariotos aerbicos, se processa com a
participao da mitocndria. Nessa organela, ocorrem o ciclo de Krebs e
a cadeia respiratria, sendo que o oxignio participa diretamente apenas
da ltima etapa dessa cadeia.
IV. No ser humano, o sistema respiratrio composto pelas vias
respiratrias e pelos pulmes. Nesses rgos, as trocas gasosas ocorrem
nos alvolos, que so estruturas formadas por clulas epiteliais.
V. As hemcias humanas so anucleadas e contm, no seu interior, a
hemoglobina. Esta protena possui ferro, ao qual o oxignio se liga para
ser transportado pelo sangue. J o dixido de carbono, em sua maior
parte, transportado dissolvido no plasma sangneo, sob a forma de
ons bicarbonato.
Est (o) correta(s)
a) apenas II, III e V.
b) apenas I e IV.
c) apenas I, II e V.
d) I, III e V.
e) todas as afirmativas.

65. A evoluo do processo digestivo por fermentao, ocorrida muitas vezes


de forma independente entre os vertebrados, resultou em solues distintas
para os problemas apresentados no aproveitamento das plantas como
alimento.

68. Durante a queima de combustveis fsseis, como na combusto do


petrleo, a fumaa produzida pode levar morte por asfixia. correto
afirmar que a asfixia causada porque:
a) a fumaa estimula a produo de oxihemoglobina, aumentando a
afinidade do oxignio pela hemoglobina e tornando-o indisponvel para
os tecidos.
b) o cido carbnico liberado inibe o centro respiratrio, diminuindo a
ventilao pulmonar.
c) o dixido de carbono liberado tem baixa afinidade pela hemoglobina,
impedindo o transporte de oxignio para os tecidos.
d) o monxido de carbono liberado tem alta afinidade pela hemoglobina,
impedindo o transporte de oxignio para os tecidos.
e) o monxido de carbono inibe a anidrase carbnica, impedindo a liberao
de oxignio pela hemoglobina.

Considerando a figura acima, que mostra o sistema digestrio de um


ruminante com quatro cmaras no estmago, correto afirmar:
01. que, aps a ingesto, o alimento dirige-se imediatamente para o omaso.
02. que, no rume e no retculo, o alimento umedecido e misturado aos
microorganismos.
04. que, no retculo, ocorre a digesto de protenas, graas atividade dos
microorganismos.
08. que, periodicamente, o ruminante retorna o alimento boca para ser
mastigado mais eficientemente.
16. que os microorganismos so as principais fontes de aminocidos e
vitaminas para os ruminantes.

69 - As figuras I, II e III esquematizam trs modelos de sistemas


circulatrios encontrados em alguns grupos animais.

66. No homem, a freqncia respiratria de aproximadamente 15


movimentos por minuto. Durante uma atividade fsica intensa percebe-se que
a freqncia respiratria aumenta bastante. Quanto respirao no homem,
pode-se afirmar que:
00. a velocidade dos movimentos respiratrios aumenta quando a
concentrao de CO2 no sangue alta.
01. a concentrao de O2 no sangue tem um papel crucial no controle do pH
sangneo.
02. no hipotlamo, parte do sistema nervoso central, h um centro
controlador da respirao centro respiratrio que recebe informaes
sobre os parmetros respiratrios e os processa.
03. os sinais transmitidos pelo centro controlador da respirao so
conduzidos por nervos que controlam os msculos intercostais e o
diafragma, os principais msculos que participam dos movimentos
respiratrios.
04. enquanto praticamente todo o oxignio transportado pela hemoglobina,
cerca de 70% do total de CO2 no sangue transportado sob a forma de
ons bicarbonato no plasma sangneo.
67. Nos humanos, o processo de respirao do tipo pulmonar e envolve o
sistema circulatrio, pois os gases so transportados, atravs dos vasos
sangneos, dos pulmes para os tecidos e dos tecidos de volta para os
pulmes. Existem diferentes tipos de respirao para outros animais, como a
area, a branquial e a cutnea. No entanto, independente do animal e do
tipo de respirao, o oxignio, ao chegar s clulas dos tecidos, participa de
um processo chamado de respirao celular, ou seja, o processo de produo
de energia para a clula (ATP).
Analise as seguintes afirmativas.
I. No processo de respirao, ocorre a difuso de CO 2 dos tecidos para o
sangue e de O2 do sangue para os tecidos. O sangue, ao passar pelos
pulmes, faz a troca gasosa: deixa o CO2 e recebe O2. Em alguns
animais, porm, o sistema circulatrio no participa da conduo dos
gases nem das trocas gasosas.

A = trio
V = ventrculo
C
om relao s figuras acima e ao sistema circulatrio CORRETO afirmar
que:
01. As figuras I, II e III apresentam esquemas de sistemas circulatrios do
tipo fechado.
02. A figura I apresenta o esquema do sistema circulatrio dos peixes, no
qual se pode observar que pelo corao s circula o sangue venoso.
04. No esquema apresentado na figura II, o sangue arterial vindo dos
pulmes mistura-se ao sangue venoso. 08. Na figura III, o sangue
arterial fica completamente separado do sangue venoso.
16. Nenhum dos sistemas circulatrios apresentados encontrado em
Porferos, Cnidrios ou Platelmintos.
32. No sistema circulatrio apresentado na figura III, a artria Aorta a
responsvel pelo transporte do sangue venoso do ventrculo direito at
os pulmes.
64. Animais como os mamferos e as aves
apresentam o tipo de sistema circulatrio
mostrado na figura II.
70 - A figura abaixo representa a
hematose. Observe-a, a fim de responder
questo.

174

a)

00. Ao penetrar na hemcia, o gs carbnico reage com a gua e produz


cido carbnico. Essa reao acelerada pela enzima anidrase
carbnica.
01. Na circulao humana, os trios comunicam-se com os respectivos
ventrculos por meio de orifcios protegidos por valvas: a tricspide, no
lado esquerdo, e a bicspide, no lado direito.
02. Os sapos adultos realizam trocas gasosas com o ambiente, utilizando
brnquias e pulmes.
03. Os vasos linfticos de todo o corpo humano unem-se em dois grandes
vasos, que lanam a linfa nas veias prximas ao corao.
04. A circulao que leva sangue rico em oxignio aos tecidos e traz para o
corao sangue pobre em oxignio chamada circulao sistmica.
71 - Os animais desenvolveram, durante sua evoluo, adaptaes que lhes
permitem o desempenho das seguintes funes:

b)

c)

I. Eliminao de subprodutos do metabolismo celular.


II. Manuteno de diferentes ons, em concentraes adequadas, no
organismo.
III. Manuteno da gua corporal em quantidade adequada.
Considerando as funes acima, o Sistema Excretor est relacionado:
a) apenas com I;
b) apenas com I e III;
c) apenas com II e III;
d) com I, II e III.

d)

72 - O esquema abaixo representa um nfron com suas estruturas


enumeradas de I a VI.

e)

74. H uma relao direta entre a eliminao de urina e o volume de lquidos


corporais, tanto intersticiais quanto do prprio plasma. Quando a
concentrao do sangue circulante aumenta, como em caso de grande perda
de gua, correto afirmar que:
a) a urina torna-se mais diluda.
b) h aumento da produo do hormnio secretina.
c) a hipfise no libera o hormnio antidiurtico (ADH).
d) as clulas dos tbulos renais ficam menos permeveis gua.
e) h maior reabsoro de gua do filtrado glomerular.
(Frota-Pessoa, O. Os caminhos da vida: biologia no ensino mdio: estrutura e
ao. So Paulo: Scipione, 2001. Modificado.)
Assinale a(s) alternativa(s) correta(s).
01. A estrutura I corresponde cpsula de Bowman.
02. A estrutura II corresponde ao tbulo contornado distal.
04. A estrutura III corresponde ao tbulo contornado proximal.
08. A estrutura IV corresponde ala de Henle.
16. A estrutura V corresponde ao ducto coletor.
32. A estrutura VI corresponde ao glomrulo.
73. Os tbulos renais possuem a capacidade de reabsorver, por transporte
ativo, a glicose existente no plasma, desde que a glicemia mantenha-se em
torno de 125 mg%. A partir desse valor, o excesso de glicose totalmente
excretado pela urina.
Assinale a alternativa cujo grfico melhor representa essa situao.

75. Com relao a diferentes hormnios secretados pela hipfise, analise as


proposies abaixo.
00. Se a concentrao de hormnios tireoidianos no sangue est alta, a
adeno-hipfise secreta o hormnio tireotrofina ou TSH; nesse contexto,
ocorre, ento, a estimulao da tireide.
01. O hormnio adrenocorticotrfico ou ACTH, secretado pela adeno-hipfise,
estimula a secreo de hormnio pelo crtex das adrenais.
02. Os hormnios FSH e LH, secretados pela neuro-hipfise, alm de
estimularem a secreo de testosterona, participam da regulao da
diurese.
03. O hormnio HAD, antidiurtico ou vasopressina, secretado pela neurohipfise, estimula a reabsoro de gua pelos rins; a urina fica mais
concentrada.
04. O hormnio ocitocina, secretado pela neurohipfise, estimula as
contraes do tero, no parto, e tambm estimula a sada do leite na
lactao.

76. Analise este grfico, em que est representado o efeito da tiroxina sobre
o metabolismo basal:

175

Considerando-se as informaes desse grfico e outros conhecimentos sobre


o assunto, INCORRETO afirmar que, na condio de hipotireoidismo, ocorre
diminuio de
a) liplise no tecido adiposo.
b) peso corporal.
c) respirao celular.
d) sntese de protenas.
77. Considere as seguintes funes do sistema endcrino:
I. Estimula o crescimento;
II. Regula o metabolismo do clcio;
III. Glicogenlise no fgado;
IV. Facilita a ejeo de leite;
V. Aumenta a reabsoro de sdio no rim.
Os hormnios que correspondem a essas funes so, respectivamente,
a) paratormnio, somatotrofina, ocitocina, adrenalina e aldosterona.
b) aldosterona, paratormnio, adrenalina, ocitocina e somatotrofina.
c) somatotrofina, paratormnio, adrenalina, ocitocina e aldosterona.
d) aldosterona, somatotrofina, paratormnio, adrenalina e ocitocina.
e) somatotrofina, ocitocina, paratormnio, adrenalina e aldosterona.
78. Correlacione os nomes dos hormnios com os seus respectivos locais de
sntese e funo.
Hormnios Local de sntese / Funo
(1) Calcitonina
(2) Luteinizante
(3) Estrognio
(4) Oxitocina
(5) Glicocorticides
( ) Tireide / Atua no metabolismo de clcio.
( ) Supra-renal / Atua no metabolismo de carboidratos.
( ) Ovrio / Atua no desenvolvimento do endomtrio.
( ) Adeno-hipfise / Estimula as clulas intersticiais.
( ) Neuro-hipfise / Estimula a contrao do tero.
Assinale a seqncia CORRETA:
a) 1, 5, 3, 2, 4.
b) 2, 1, 3, 5, 4.
c) 2, 1, 5, 4, 3.
d) 1, 3, 2, 4, 5.
e) 5, 4, 3, 1, 2.
79. O sistema endcrino responsvel pela produo de hormnios,
mensageiros qumicos que fazem a comunicao entre as clulas do
organismo.
A figura abaixo indica algumas das principais glndulas de um homem:

b) Os testculos tambm so chamados de glndulas sexuais, pois eles


produzem os gametas, alm dos hormnios. O principal hormnio a
testosterona.
c) O rgo B corresponde s glndulas suprarenais, que recebem esse nome
devido sua localizao acima do rim. Um dos hormnios que elas
produzem a adrenalina, que provoca um rpido aumento na taxa
metablica basal.
d) Na tireidea, ocorre a produo de tri-iodotironina (T3) e tiroxina (T4). A
falta desse hormnio causa o hipotireoidismo, e o excesso, o
hipertireoidismo. Na infncia, se o nvel desses hormnios for baixo,
pode ocorrer o cretinismo.
e) O rgo A corresponde ao timo. Ele alm de ser uma glndula endcrina,
um rgo linfide, pois nele ocorre a formao dos linfcitos, clulas
envolvidas na defesa do corpo.
80. Numerando-se glndulas com algarismos romanos, hormnios com
algarismos arbicos e aes por letras maisculas:
GLNDULA
I. tireide
II. pncreas
III. supra renal
IV. hipfise
HORMNIO
1. calcitocina
2. adrenalina
3. insulina
4. tiroxina
5. glucagon
6. somatotropina
7. ocitocina
8. aldosterona
AO
A. dificulta a remoo de Ca++ nos ossos
B. reduz a concentrao de glicose no sangue
C. estimula a liberao de leite na amamentao
D. aumenta a reabsoro de Na+ pelos tbulos renais
E. estimula o crescimento dos tecidos
F. aumenta a concentrao de glicose no sangue
G. regula a presso arterial
H. acelera o metabolismo
Assinale
a(s)
alternativa(s)
em
glndula/hormnio/ao esto corretas.
01. I 1 A, II 3 B, III 2 G, IV 6 E
02. I 4 H, II 5 F, III 8 D, IV 7 C
04. I 3 F, II 4 E, III 7 A, IV 1 C
08. I 2 G, II 7 H, III 3 B, IV 4 D
16. I 1 A, II 5 F, III 2 G, IV 7 C
32. I 4 H, II 3 B, III 8 D, IV 6 E
64. I 5 B, II 6 G, III 4 C, IV 8 F

que

todas

as

associaes

81. A figura abaixo mostra rgos do sistema nervoso central de um


mamfero.

(Norma Maria Cleffi. Curso de Biologia. Estrutura-funo nos seres vivos. So


Paulo: Harbra, 1987. p. 29)
Os rgos que controlam a freqncia respiratria e o equilbrio so,
respectivamente,
a) I e II.
b) II e III.
c) II e IV. d) III e I.
e) III e II.

Com base na figura e em seus conhecimentos INCORRETO fazer a


seguinte afirmao.
a) O rgo C corresponde ao pncreas, uma glndula mista. A parte
endcrina produz dois hormnios: a insulina e o glucagon. A insulina
aumenta o nvel de glicose no sangue, e o glucagon o reduz.

82. O filme HANNIBAL (Direo de Ridley Scott) apresenta uma cena em que
o Dr. Lecter abre a caixa craniana de um homem com cuidado para no
danific-la internamente. Assim que ele retira o tampo sseo do crnio,
percebe que o encfalo est protegido por camadas sobrepostas de tecido
conjuntivo. Considerando o exposto, qual das afirmativas abaixo apresenta o
nome genrico das camadas teciduais e seus nomes especficos, da mais
externa a mais interna, respectivamente?
a) Medula; aracnide, piamter e duramter.
b) Meninge; duramter, piamter e aracnide.
c) Meninge; duramter, aracnide e pia-mter.
d) Meninge; aracnide, duramter e piamter.
e) Medula; aracnide, duramter e piamter.

176

83. Durante este ano, muito foi discutido sobre a crise de energia eltrica no
Brasil, e todos estivemos conscientes da possibilidade de haver interrupes
no fornecimento de energia em diversas cidades do pas. Suponha que um
indivduo A no soubesse desse fato (A poderia ser um estrangeiro recmchegado ao Brasil), que estivesse em uma dessas cidades e que subitamente
fosse surpreendido noite por um "apago". Com relao ao que ocorreria
ao Sistema Nervoso de A nessas circunstncias, correto afirmar:
01. Se A se assustasse e exibisse, em funo disso, um aumento de
freqncia cardaca, possvel afirmar que o Sistema Nervoso Autnomo
desse indivduo estaria envolvido nessa resposta.
02. Supondo que a ausncia de luz fosse um estmulo que desencadeasse a
propagao de um impulso nervoso em neurnios de A, para tanto
haveria necessariamente inverso da polaridade da membrana
plasmtica dessas clulas, com influxo de ons sdio para o meio
intracelular e sada posterior de ons potssio para o meio extracelular.
04. Supondo-se que os sinais visuais fossem importantes para a manuteno
do equilbrio e da postura de A, para que ele pudesse continuar
caminhando no escuro, o cerebelo seria a principal estrutura do Sistema
Nervoso a monitorar outras informaes que compensassem a ausncia
dos sinais visuais citados.
08. A capacidade do Sistema Nervoso de responder a uma ausncia total ou
parcial de luz est relacionada intensidade de resposta dos neurnios,
uma vez que, diferentemente dos demais neurnios, os neurnios
oculares no seguem o padro de resposta "tudo-ou-nada", segundo o
qual a intensidade de resposta sempre a mesma, desde que o estmulo
seja igual ou superior a um dado limiar.
16. Caso A casse, em funo de estar num local desconhecido no escuro,
todas as estruturas citadas a seguir seriam importantes para a proteo
do Sistema Nervoso desse indivduo: crnio, coluna vertebral, meninges
e lquido cefalorraquidiano.
32. A cabea de A corresponderia regio exploratria do "ambiente escuro",
assim como ocorreria com diversos outros animais, visto que a
centralizao e a anteriorizao das estruturas nervosas so tendncias
evolutivas que se verificam ao longo da escala zoolgica.
64. Se ao invs de ficar no escuro A tivesse posto a mo numa chapa quente,
a resposta de retirada rpida da mo constituir-se-a num ato reflexo e
envolveria obrigatria e unicamente as seguintes estruturas: receptor
sensorial, neurnio sensorial ou aferente, neurnio motor ou eferente e
rgo efetor da resposta.
84. Ao liberar acetilcolina, a fibra nervosa
a) simptica promove aumento do ritmo cardaco.
b) parassimptica promove diminuio do ritmo cardaco.
c) simptica promove diminuio do ritmo cardaco.
d) parassimptica promove aumento do ritmo cardaco.
e) simptica e a parassimptica no alteram o ritmo cardaco.

1) A

2) B

3) B

4) 18

5) C

6) D

7) A

8) D

9) C

10) A

11) V/V/F/V/F

12) A

13) B

14) V/V/V/F/F

15) A

16) B

17) B

18) A

19) V/V/V/F/V/V

20) V/F/V/F/V/F

21) B

22) D

23) B

24) C

25) C

26) 52

27) D

28) B

29) 18

30) D

31) E

32) B

33) A

34) C

35) B

36) C

37) C

38) E

39) A

40) E

41) E

43) 22

44) V/F/F

45) V/V/V/V/F/F/V

46) 62

47) C

48) 27

49) C

50) V/F/V/V/F/F

51) V/V/F/V

52) CEEE

53) D

54) C

55) D

56) V/V/F/V

57) B

58) B

59) A

60) B

61) B

62) 10

63) D

64) B

65) 26

66) V/F/F/V/V

67) E

68) D

69) 31

70) V/F/F/V/V

71) D

72) 33

73) E

74) E

75) F/V/F/V/V

76) B

77) C

78) A

79) A

80) 51

81) B

82) C

83) V/V/V/F/V/V/F

84) B

42) algas, vitamina C, parede

celular, celulose, pigmentos /


substncias de reserva

177

You might also like